Vous êtes sur la page 1sur 324

Organ

0hemist
Patt I
Sections I-IV
Section I
Structure, Bonding, and Reactivity
Section II
Structure Elucidation

Section III
Stereochemistry

$ection IV
Hydrocarbon Reactions

'll'te

K.lE. V. I. E. W @

Speci alizing in II{CAT Preparation


Nomenclature
a) IUPAC Nomenclature
b) General Nomenclature
Bonding and Molecular Orbitals
a) Lewis Dot Structures
Section I b) Bonding Model
c) Covalent tsonds
d) Molecular Orbitals and Bonds
i. Single Bonds
ii.
Structure, e)
iii.
Double Bonds
Triple Bonds
Molecular Structures
tsonding, f)
g)
Octet Rule (HONC Shortcut)
Charged Structures
and Hybridization
a) Hybridization of Atomic Orbitals
Reactivity i.
ii'
sp-Hybridzation
sP2-HYbridzation

"</
by Todd Bennett iii'
sP5-nybridzation
HH b) Common Shapes
Bond Energy
a) Bond Dissociation trnergy
/V"
HH
C-C.
b) Ionic Bonds
Intramolecular Features
HH a) Resonance
b) Inductive Effect
c) Steric Hindrance

7ffiV"
HH
d) Aromaticity
Fundamental Reactivity
a) Proton Transfer Reactions
b) Lewis Acid-Base Keactions
c) Acid and Base SLrength
i. Primary Effects
ii. Secondary Effects
iii.Values and Terminology
d) Electrophiles and Nucleophiles
Physical Properties
a) l-lydrogen-Bonding
b) Polarity
c) Van der Waals Forces
d) Solubility and Miscibility

RNKIIELEY
l)n.n-v.r)E.w'
Speciahztng in MCAT Preparation
Organic Chemistry Molecular Structure Introduction

The perfect place to start any review of organic chemistry is the basics of
molecular structure, which traditionally include bonding, hybridization, and
electronic distribution. We shall consider a chemical bond to be the result of
atomic orbitals overlapping to form molecular orbitais. We shall consider all
bonds involving carbon to be covalent in nature. A covalent bond is thought to
involve the sharing of electrons between two adjacent nuclei. According to the
rules of electrostatics, the region between two nuclei offers a highly favorable
environment for electrons, where they can be shared by neighboring atoms.
However, there are several other factors to consider in bonding. if bonding were
purely a matter of electrostatics, then all of the eiectrons would be found between
two neighboring nuclei, not just the bonding electrons. The sharing of electrons
may be either symmetric (when the two atoms of the bond are of equal
electronegativity) or asymmetric (when the two atoms of the bond are of unequal
electronegativity). Sharing of electrons occurs when the atoms of a bond lack a
compiete valence electron shell. By sharing electrons, each atom moves closer to
completing its shell This is the driving force behind the formation of stable
covalent bonds.
Having looked briefiy at electron distribution, we can introduce the idea of
electronic orbitals, which are three-dimensionai probability maps of the location
of an eiectron. They represent the region in space where an electron is found
95% of the time. We shall consider the orbitals and the overlap of orbitals to
describe the electronic distribution within a molecule. Once one has established
a foundation in bonding, the classification of molecules can be made based on
sinrilarities in their bonding of particular atoms, known as functional groups. Each
functional group shall be considered in terms of its unique electron distribution,
hvbridization, and nomenclature. Nomenclature, both that of the International
Union of Pure and Applied Chemists (IUPAC) and more general methods
describing the substitution of carbon within a functional group, shall be used to
describe a particuiar organic molecule. The review of nomenclature is
continuous throtrghout all sections of this book.
Then, we shall consider the factors that affect the distribution of eiectron density
within a molecule, including resonance, the inductive effect, steric hindrance,
aromaticity, and hybridization. The distribution of electron density can be used
to explain and predict chemical behavior. The simplest rule of reactivity in
organic chemistry is that regions of high electron density act as nucleophiles by
sharing their electron cloud with regions of low electron density, which act as
electrophiles. If you can correctly label a molecule in terms of the region that
caries a partially negative charge (the electron-rich environment) and the region
that carries a partially positive charge (the electron-poor environment), you can
understand chemical reactions better.
And so begins your review of organic chemistry. Fortunately, much of organic
chemistry is taught from the perspective of logic, which makes preparing for
organic chemistry on the MCAT easier. In organic chemistry courses you are
required to process information and reach conclusions based on observations,
which is also required on the MCAT. Reviewing and relearning this material
will help you develop criticai thinking skills, which will carry over into your
rerriew for other portions of the exam. Despite what you may have perceived
was a girth of information when you initially studied organi.c chemistry, you
don't need to review that much material to prepare successfully for the MCAT.

Copyright @ by The Berkeley Review Exclusive MCAT Preparation


Organic Chemistry Molecular Structure Nomenclature

IUPAC Nomenclature (Systematic Proper Naming)


IUPAC Nomenclature is an internationally used system for naming molecules.
Molecular names reflect the structural features (functional groups) and the
number of carbons in a molecule. In IUPAC nomenclature, the name is based on
the carbon chain length and the functional groups. The suffix indicates which
primary functional group is attached to the carbon chain. Table 1-1 lists prefixes
for carbon chains between one and twelve carbons in length. Table 1-2 lists the
suffices for various functional groups. Be aware that "R" stands for any generic
alkyl group. When R is used, it indicates that the carbon chain size is irrelevant
to the reaction. Table 1-3 summarizes the nomenclature process by listing several
four-carbon compounds.

Carbons Prefix Carbons Prefix Carbons Prefix


1 meth- 5 pent- 9 non-
2 eth- 6 hex- 10 dec-
J prop- 7 hept- 11 undec-
4 but- 8 oct- 12 dodec-
Table 1-1

Functionality Compound Name Bonding


R-CH3 Alkane C-C & C-H
R-O-R Ether C-O-C
o
R-CO-H Aldehyde il
C-C- H
R-CH2-OH Alcohol C-O-H
o
R-CO-R Ketone il
C_C-C
o
R-CO-OH Carboxylic acid il
C_C_ OH
Table 1-2

Formula IUPAC Name Structural Class


H3CCH2CH2CH3 Butane Alkane
H3CCH=CHCH3 Butene Alkene
H3CCH2CH2CHO Butanal Aldehyde
H3CCH2COCH3 Butanone Ketone
H3CCH2CH2CH2OH Butanol Alcohol
H3CCH2CH(oH)CH3 2-butanol Alcohol
H3CCH2CH2CH2NH2 Butanamine Amine
H3CCH2CH2CO2H Butanoic acid Carboxylic acid
Table 1-3

Copyright @ by The Berkeley Review The Berkeley Review

.ffi fr ::r'ii'*lllil+iji*i:il$i: :::l':: .:.


:.,

::ri:::;ni:iiliiii:ri:*iiiliffii
Organic Chemistry Molecular Structure Nomenclature

Figure 1-1 shows examples of IUPAC nomenclature for four organic compounds
with variable functional groups:

C1
3-methylpentanoic acid 4-chloro-S-methyl-3-heptanol
Longest chain: 5 carbons Longest chain: 7 carbons
Carboxylic acid group Alcohol group
Methyl substituent at C-3 Chloro substituent at C-4
Methyl substituent at C-5

3-ethylcyclopentanone 3,3-dibromobutanal
Ring of 5 carbons Longest chain: 4 carbons
Ketone group Aldehyde group
Ethyl substituent at C-3 2 Bromo substituents at C-3

Figure 1-1

General Nomenclature (Common Naming Based on Substitution)


In addition to the IUPAC naming system, there is a less rigorous method of
naming compounds by functional group and carbon type (based on carbon
substitution). Carbon type refers to the number of carbon atoms attached to the
central carbon atom (carbon atom of interest). A carbon with one other carbon
attached is referred to as a primary (1') carbon. A carbon with two other carbons
attached is secondary (2"). A carbon with three other carbons attached is tertiary
(3"). Figure 1-2 shows some sample structures.

H CH. HCH"HH
\ .s r"'r,iu.y carbon carbon \r)yn"t carbon
r_zt{ l}}i""oary
./-\
HgC CHg H3CH2C
./'\ oH -/-\ Cl
H3CH2C
Isobutane Sec-butanol n-Propyl chloride
(2-Methylpropane) (2-Butanol) (1-Chlropropane)

Figure 1-2

Nomenclature is an area of organic chemistry best learned through practice and


experience. We will deal with nomenclature throughout the course, as we
introduce each new functional group. Understanding nomenclature is especially
important in MCAT passages where names rather than structures are given. Be
sure to know the Greek prefixes for carbon chain lengths up to twelve carbons.

Copyright @ by The Berkeley Review Exclusive MCAT Preparation


Organic Chemistry Molecular Structure Bonding and Orbitals

Lewis Dot Structures (Two-Dimensional Depiction of Molecules)


Lewis dot structures represent the electrons in the valence shell of an atom or
bonding orbitals of a molecule. Typically, we consider the Lewis dot structures
of elements in the s-block and p-block of the periodic table. For every valence
electron, a dot is placed around the atom. Single bonds are represented by a pair
of dots in a line between the atoms, or by a line itself. A double bond is
represented by a double line (implying that four electrons are being shared.)
Likewise, a triple bond is represented by a triple line (implying that six electrons
are being shared.) Lewis dot structures are familiar to most chemistry students,
so recognize the exceptions to the rules, as they make good test questions.

Example 1.1
V\4rat is the Lewis dot structure for HcBF?

A. B.
:H-B-F: H- S-'F
t"
:
t" :

u .ry

C. D.
H-ri-r: H- B-F
t"
H H
t" :

Solution
Boron has only three valence electrons, hence it can make only three bonds.
There is no lone pair on the boron atom, eliminating choices A and C. Hydrogen
has only one electron, which is in the bond to boron, so there is never a lone pair
on a bonded hydrogen. This eliminates choices A (already eliminated) and B.
Fluorine has a completed octet, so it makes one bond and has three lone pairs, as
depicted in choice D, the best answer.

Bonding Model
Bonding is defined as the sharing of electron pairs between two atoms in either
an equal or unequal manner. As a general rule, a bond is the sharing of two
electrons between two adjacent nuclei. The region between two nuclei is the
most probable location for an electron. In most cases, with the exception ol ligand E.

bonds (kaown also as Leruis ttcid-base bonds), one electron from each atom goes into
forming the bond. When electrons are shared evenly between two atoms, the
bond is said to be a coualent bond. Ylh,en electrons are transferred from one atom
to another, the bond is said to be an ionic bond. The difference between a covalent
and ionic bond is measured in the degree of sharing of the electrons, which can
be determined from the dipole. The more evenly that the electrons are shared,
the less the polarity of the bond. The relative electronegativity of two atoms can
be determined by measuring the dipole of tl-re bond they form. When the 5
difference in electronegativity between two atoms is less than 1.5, then the bond
is said to be covalent. When the difference in electronegativity between two
atoms is greater than 2.0, then the bond is said to be ionic. When the difference
in electronegativity between two atoms is greater than 1.5 but less than 2.0, then l"t
the bond is said to be polar-covalent (or partially ionic). r

- :-,' l}.e Berkeley Review 6 The Berkeley Review


a
Organic Chemistry Molecular Structure Bonding and Orbitals

Example L.2
Which of the following bonds is MOST likely to be ionic?
A. C-O
B. N-F
C. Li-H
D. Li-F
Solution
A bond is ionic when the difference in electronegativity between the two atoms
exceeds 2.0. This means that the bond that is most likely to be ionic is the one
between the two atoms with the greatest difference in electronegativity. Lithium
is a metal and fluorine is a halide, so they exhibit the greatest electronegativity
difference of the choices listed. The best answer is therefore choice D.

Covalent Bonds
Bonds can be classified in one of three ways: ionic, polar-covalent, and covalent.
A covalent bond occurs when electrons are shared between two atoms having
Iittle to no difference in electronegativity. As the difference in electronegativity
decreases, the covalent nature of the bond increases. There are two types of
:ovalent bonds: sigma bonds (o), defined as having electron density shared
betvr'een the nuclei of the two atoms; and pi bonds (n), defined as having no
electron density shared between the nuclei of the two atoms, but instead only
above and below the internuclear region. Sigma bonds are made from many
types of atomic orbitals (including hybrids), while pi bonds are made exclusively
of parallel p-orbitals. In almost all cases, the sigma bond is stronger than the pi
bond, with molecular fluorine (F2) being a notable exception. Figure 1-3 shows a
generic sigma bond. You may notice that within a sigma bond, only about eighty
to ninety percent of the electron density lies between the nuclei, not all of it.

.Nuclei.

o@o Electron density

Figure L-3

Example 1.3
Which drawing depicts the electron density of a carbon-carbon sigma bond?

Solution
A sigma bond has its electron density between the two nuclei, which eliminates
choice D. The two atoms in the bond are identical, so the electron density should
be symmetrically displaced between the two nuclei. This eliminates choice B.
Most of electron density is between the nuclei, so choice A is a better answer than
choice C. These drawings are ugly, so focus on the concept, not the pictures.

Copyright @ by The Berkeley Review Exclusive MCAT Preparation


Organic Chemistry Molecular Structure Bonding and Orbitals

Figure 1-4 shows a generic n-bond. Within a n-bond, there is no electron density
between the two nuclei. The electron density in a ru-bond results from electrons
being shared between the adjacent lobes of parallel p-orbitals.

Nuclei

Electron Densitv
J

Figure 1-4

In organic chemistry, covalent bonds are viewed. in great detail, taking into
account hybridization and overlap. In alkanes, carbons have sp3-hybridiiation
and all of the bonds are sigma bonds. In alkanes, there are two typbs of bonds:
c H lorps-r bonds) and c- c (o'sp3-sp3 bonds). In alkenes, t-h-"." uru sigma
-
and pi bonds plesent. The n-bond Conslsts of p-orbitals side by side, und it,
carbons have sp2-hybridization. The C=Cbond is made up of aoroz-rozbond
and a n2r-2p bond. Bond length varies with the size of the orbitals ih ttre bond.
For instance, a sigma bond composed of an sp2-hybridized carbon and an sp3-
hybridized carbon is shorter than a sigma bond comprised of two sp3-hyb ridized,
carbons. Hydrogens use s-orbitals to form bonds. Figure 1-5 shows thiee sigma
bonds with their relative bond lengths. The longer bond is associated wittrthe
larger orbitals (bond radii: dz > dy > dil.

or-rp3 orp2-rp2 osp3_sp3

Figure 1-5

Figure 1-5 confirms that most of the electron density lies between the two nuclei
in sigma bonds, no matter what the orbitals are from which the sigma bond
originates. In pi bonds, electron density does not 1ie between the two nlclei. The
length of a bond is defined as the distance between the nuclei of the two atoms
making the bond. Figure 1-6 shows an example of a n-bond between two Zpr-
orbitals, which is typical for nearly all n-bonds encountered in organic chemistry,
because carbon, nitrogen, and oxygen have 2p-orbitals in their valence shells.

n ffi
eo
2p,-2p,

I
{
{
Figure 1-6 i
I

Copyright @ by The Berkeley Review The Berkeley Review- I


Organic Chemistry Molecular Structure Bonding and Orbitals

Pi bonds are found as the second bond present in double bonds and the second
and third bonds present in triple bonds. The first type of bond to form between
atoms is usually the sigma bond. Once a sigma bond exists between two carbon
atoms, then pi bonds can form between the atoms. Fluorine gas is an exception
to the "sigma bond first" rule. Molecular fluorine (F2) has only one rc-bond, with
no o-bond present. This is attributed to the small size of fluorine and the inter-
nuclear repulsion associated with a typical single bond. This is why the bond
dissociation energy of F2 is less than the bond dissociation energy of Cl2, even
though chlorine is below fluorine in the periodic table.

Molecular Orbitals
Molecular orbital is a fancy way of describing a bond or anti-bond that exists
between two atoms. An anti-bond is a molecular orbital that results in bond-
breaking when coupled with a bonding orbital. It is important to recognize the
shape and location of electron density in molecular orbitals. Figure 1-7 shows the
common bonding and anti-bonding orbitals associated with organic chemistry.

.@.@o + cffi'c
J orp3-rp3
sp sp3
Sigma bonding molecular orbital

D*.c@
,3 -3
+ D@c@ oxsp--sp-
.-3 .-i
,
sp"
sp" ' -

Sigma anti-bonding molecular orbital

I I +
p p frzp-zp

Pi bonding molecular orbital

fB n tr=
r ffic
5+6
P P
d6
lt*rr-r,
Pi anti-bonding molecular orbital

Figure 1-7

The shading of the lobes in each orbital represents the direction of spin for the
electron. h:r order for electron density to overlap, the electrons must have the
same spin. This is analogous to driving on the freeway. If you join a freeway in
the same direction as traffic is flowing, you can easily blend into traffic. This is a
favorable interaction. If you join a freeway in the opposite direction as traffic is
flowing, you cannot easily blend into traffic. This is an unfavorable interaction.

Copyright @by The Berkeley Review 9 Exclusive MCAT Preparation


Organic Chemistry Molecular Structure Bonding and Orbitals

l\lolecular Bonds
of
greater interest than the sigma-bonds and pi-bonds are the single, double, and
triple bonds present between atoms. single, double, and triple bond nature is
discussed more so than the sigma and pi nature of bonds. In trganic molecules,
there are only single, double, and triple bonds. Between like atoms, the
descending order of relative strengths of boncls is triple bond > double bond >
single bond. Another rule to consider is that for bonds between like atoms, the
ionger the bond, the less the electron density overlaps between nuclei, and thus
the weaker the bond. This is summarized as: longer ionds are rueaker bonds.

single Bonds: single bonds are composed of only one sigma bond between the
two atoms. single bonds are longer than double and triple bonds between two
atoms, even though fewe_r electrons are present. Ethane has sigma bonds only
and is shown in Figure 1-8 in both stick figures and with the rele')ant orbitals.

HH H H

HH
/v" C-C. OC ffi"p
orp3-5p3
/

\"
H

Figure 1-8

Double Bonds: Double bonds are composed of one sigma bond and one pi bond
between two adjacent atoms. Ethene (czH+) has f6ur sigma bonds between
carbon and hydrogen, a sigma bond between the two carf,ons, and a pi bond
present between the two carbons to compiete the carbon-carbon double bond.
Ethene is shown in Figure 1-9 in both stick figures and with the relevant orbitals.

ftp^-p^

Illtt,,,
H7:
..rr\H
\n %o="#
H
osp2-roz
@rffi%
Figure 1-9

Triple Bonds: Tripie bonds are composed of a sigma bond. and two pi bonds
between two adjacent atoms. Triple bonds are shorter than either iingte or
double bonds. Ethyne (CzHz) has two sigma bonds between carbon and
hydrogen, a sigma bond between the two carbons, and two pi bonds between the
two carbons to complete the carbon-carbon triple bond. Ethyne is shown in
Figure 1-10 in both stick figures and with the relevant orbitals.

fipv-pv

H @.- .P
Figure 1-10

Copyright O by The Berkeley Review lo The Berkeley Review


Organic Chemistry Molecular Structure Bonding and Orbitals

Example 1.4
\A/hat is the relative bond strength of carbon-carbon bonds in the molecule shown
below?
ooloo
bondaS\
,rr.. ,r,
l, /
C*C H
bond c _-_*/
\ .v z r
-\ ^CH^CH"
(H3.)2HC C

""Y T bond d
CHe

A. Bond a > Bondb > Bond c > Bond d


B. Bondb > Bond a > Bond c > Bond d
C. Bond d > Bond a > Bond c > Bondb
D. Bondb > Bond c > Bond a > Bond d

Solution
There strongest C-C bond is a double bond, bond b, so choices A and C are
-Bond
eliminated. d is the weakest, because it is between two sp3 carbons. Bond
c is stronger than bond a, despite both sharing an sp2 and an sp3 carbon, because
bond c contains the more highly substituted carbon. Choice D is the best answer.

Molecular Structures
We shall continue from the fundamental concept that a valence electron can be
shared between two nuclei rather than being isolated to just one nucleus, because
the attractive force of two positive sites is greater than the attractive force of one.
This is the basic, perhaps oversimplified, perspective of a chemical bond. The
sharing of electrons is what characterizes a covalent bond. One of the first rules
of organic chemistry that you must understand is the octet rule. It is valid for
carbon, nitrogen, and oxygen atoms. To understand organic chemistry, it is
important that you recall VSEPR theory, which applies to bonding (in particular,
to the subgroups of covalent bonding like single, double, and triple bonds and
their component o-bonds and n-bonds). Table 1-4 shows the skeletal structures
of molecules that contain carbon, nitrogen, oxygen, and hydrogen.

Valence To Complete Number of Bonds in Neutral


Atom Electrons Shell Compounds

Carbon (C) .c. n I

4. 4 e- needed
-tY' -L_

Nitrogen (N)
5.
.N. 3 e- needed - -''Y',
3 '";
-N' \
i5:

Oxygen (O) - .ct.


b .. 2 e- needed , _-6-_ :O.

Hydrogen (H) 1H. 1 e- needed 1H-


Table 1-4

Copyright @ by The Berkeley Review ll Exclusive MCAT Preparation


Organic Chemistry Molecular Structure Bonding and Orbitats (

Octet Rule and the HONC Shortcut:


C:
Every molecular structure should have atoms that obey the octet rule (eight
valence electrons for C, N, and o). The numbers of electrons needed to complete
-.
the shell in the Table 7-4 are derived from the electrons needed to obey the bctet {j
rule. All neutral structures have atomic arrangements as described inTable 1.4. :,:
To complete the octet valance shell, carbon requires four electron pairs in the ::
form of bonds, nitrogen requires one lone pair in addition to the thiee bonds it ::.
makes, and oxygen requires two lone pairs in addition to the two bonds it makes.
-a
You must be able to recognize valid structures by applying the bonding rules r-
(HONC-1234). In a neutral compound, hydrogen makes onebond., oxygen makes
truo bonds, nitrogen makes threebonds, and carbon makes
four bonds. Neutral '-'
)-l
structures always obey this rule. Figure 1-11 shows examples of valid and
invalid structures and a brief description of the bonding to the component atoms. T
tt H ,CHZ
/tnt \//
L-L L-L
^-A
/\ /\
HH HH
All carbons have 4 bonds. Most carbons have 4 bonds,
All hydrogens have 1 bond. but one carbon has 5 bonds.
All hydrogens have 1 bond.
Good Structure Bad Structure

/CH3
HsC- C- C- CH2CH2CH3 H"C-C:
cH2cH3
All carbons have 4 bonds.
A1l hydrogens have 1 bond. AII carbons have 4 bonds. Dn
Ali hydrogens have 1 bond,
Good Structure but oxygen has 3 bonds. :-..!i
Bad Strttctttre tl__ _

:_ _

'\H CH" tra, CHs :_r


/-"3
1\-L N-C
\
CHs H H
Ali carbons have 4 bonds. All carbons have 4 bonds.
A1l hydrogens have 1 bond. A11 hydrogens have 1 bond,
Nitrogen has 3 bonds. but nitrogen has 4 bonds.
Good Structure Bad Structure

Figure 1-11

You can validate molecular structures by seeing whether they satisfy bonding
rules (HoNC-7234) and conventions with regard to the number of bonds and
lone pairs. If a structure does not satisfy the rules, then there must be a charge
present. Generally, having too many bonds in a molecule results in a cation and
too few bonds results in an anion, except with carbon. For instance, if oxygen
makes three bonds and has one lone pair, it carries a positive charge. wnu.t
nitrogen makes two bonds and has two lone pairs, it carries a negative charge.
\Alhen carbon makes three bonds, the charge depends on the presence or absence
of a lone pair (presence yields an anion, r,t'hile absence yields a cation).

Copyright @ by The Berkeley Review t2 The Berkeley Review


-i!
Organic Chemistry Molecular Structure Bonding and Orbitals

Charged Structures
Formal charges (charged sites on a molecule) occur when there is an excess, or
shortage of electrons on an atom. For instance, an oxygen atom typically has six
valence electrons and wishes to have eight. This means that oxygen makes two
bonds to complete its valence shell (and thus satisfy the octet rule). However, if
an oxygen atom had only five valence electrons, it would be short one electron
from its original six and would consequently carry a positive charge. Having
only five valence electrons, the positively charged oxygen would need to make
three bonds (one more than its standard two) to complete its octet. We can
conclude that oxygen with three bonds carries a positive charge. Table 1-5 shows
somecommon organic ions to commit to memory:

Number of Bonds Number of Bonds to Number of Bonds to


to Neutral Atom Cationic Atom Anionic Atom
? ..
Carbon (C)
3l
4 .c. -/t\
-+
-
r+-
-'Y',-\
/--
-
1--

Nitrogen (N) 4 t
3 'N'
t'
z\':N\-5+= -t-
Oxygen (O)
'-ij\,:t :o*= t
2 .o. -p]'
Table 1-5

Drawing Molecular Structures


Drawing molecular structures from a given formula requires following the octet
rule for aII atoms except hydrogen. On occasion, thete will be charged atoms
within the compound, but the number of charged atoms within the structure
should be minimized. Figure 1-12 shows some samPle structures for a few
common molecules.

Molecular and 3D Structure


Lewis Structure
Structural Formula
HH H liu
U

ll \it
H_C_C- O- .C_C
c2H60
cH3cH2oH tl
H
\t\
H.; ^- rr
.
HH H .v II

HHH H H
c2H7N rtl C_
:
N'
cH3NHCH3 H_ C-N_ C_ H .(
t..l )Y
HtH
HH .C_H

HH H
"irH
c2H50* ll
H-C-C: O*- H .C-C
cH3cHo+H t.. \\
H .g*-H
"'*
Figure 1-12

Copyright @ by The Berkeley Review l5 Exclusive MCAT Preparation


Organic Chemistry Molecular Structure Ilybridization

Hybridization
Hybridization of Atomic Orbitals
Hybridization entaiis relocating electron density in atomic orbitals prior to
bonding, in order to minimize the repulsion between electron pairs and thereby
allow for bonding between atoms. There are three main types bi nyuria orbitals
to consider: sp,sp2, and sp3 hybrids. Hybrid orbitals are atomic orbitals that are
involved in making bonds between atoms. Listed in Table 1-6 are some pertinent :r
facts and structural features for each of the three types of hybridization. lable 1-6
represents general trends that are observed in neariy all molecules with
hybridized orbitals involved in their molecular orbitals.

Hybrid sp sp2 sp3


Atomic Orbitals s+p s+p+p s+P+p+p
Angle 180' 720" 109.5'
Shape linear trigonal pianar tetrahedral i!

o-bonds and e- pairs 2 3 4


r-bonds 2 1 0

Table 1-6

The number of n-bonds in Table 1-6 is for typicai molecules that obey the octet
rule' A class of molecules that is an exception to the features in Table 1-6 is the
boranes, such as BH3, BF3, and BR3. In boranes, the boron atom has only three
valence electrons, so a neutral boron cannot satisfy the octet rule. The result is
that boron has sp2-hybridization for its three sigma bonds, but no pi bond.

Structures with Orbitals s,4


j-
Lewis structures are used as shorthand representations of molecules. Flowever,
in organic chemistry, molecules should be visualized in three dimensions, which
hybridization helps to faciiitate. Determining the three-dimensional shape of a
molecule requires first assuming a shape based on hybridization of the central
atoms, then applying valence shell electron pair repulsion (vsEpR) theory.
Figure 1-13 shows molecular structures witl-r orbitals and three-dimensional
orientation. Structures should be drawn with and correct bond lengths and bond
angles should be based on hybridization, steric hindrance, and vsEpR rules.

,H
sp2 -hybrid,ized sp'-hybridized
S...'n,
BH: H-p-1 cH+
I

H-- \,,,*-,
0-n H
E

H
I
sp3-hybridized
,H L
sp'-hybridized
ql I

NHe r-Ut'un Hzo 5


H
"t :
*i
Figure 1-13

Copyright O by The Berkeley Review t4 The Berkeley Review


Organic Chemistry Molecular Structure Hybridization

The orbital shown for BH3 in Figure 1-13 is actually an empty p-orbital, while the
other orbitals depicted in Figure 1-13 are hybrid orbitals. The p2-orbital of BH3 is
devoid of electrons, so the hybridization is sp2. \Mhile an empty p-orbital does
not actually exist, we consider the region where an electron pair could be
accepted. The three hybrid orbitals are detailed in Figures I-1.4,1.-75, and 1-16.

sp-Hybridization: sp-hybtidization is the result of the mixing of the s-orbital and


the pporbital.

.&.o,
Two atomic orbitals
+
Hybridizes
to become
@&r.@
Two sphybrid orbitals

Figure 1-14

st'ayarialzation: sp2-hybrtdtzation is the result of the mixing of the s-orbital,


the pa-orbital, and the py orbital.

Py s^
.CJ@.O r)
o..

Hybridizes
three sp2-irybrid orbitals
Three atomic orbitals to become

Figure 1-15

tt'-nyAnalzation: sp3-hybtidi"ation is the result of the mixing of the s-orbital,


the pa-orbital, the py-orbital, and the p2-orbital.

i'rery. +@trb.e ,o3 ffi'I'^^, sp3

Hybridizes
Four atomic orbitals tobecome ,or.m
Figure L-16

Example 1.5
What is the hybridization of each carbon in propene (H2C=CH-CH3)?
A. sp, sp, and sp3^
B. sp/,sp, and spr
C. tp?,tp\, and sp3
D. sp', spr, and spJ
Solution
There are three carbons in propene. The first two carbons are involved in a rc-
bond, so they are each sp2-ltybridized. This makes the best answer choice C. The
last carbon is not involved in any n-bonds, so it has sp3-hybridization.

Copyright @ by The BerkeleY Review Exclusive MCAT PreParation


Organic Chemistry Molecular Structure Hybridization

Common Three Dimensional Shapes


Hybridization theory supports the notion that there are recurring molecular
shapes (tetrahedral, trigonal planar, and linear) that can be seen within different
molecules. This means that there is a electronic expianation for the structures
that are observed within various molecules. Hybridization is a theoretical
explanation to rationalize why electron pairs in the valence shells of bonding
atoms assume orientations as far from one another as possible. Hybridization is
used to explain bond lengths and bond angles. Figures r-r7,1.-rg, and 1-19 show
structures with their corresponding geometry and structural features.

Tetrahedral and sp3 -nybridization


A central atom with four electron pairs (any combination of bonds and lone
pairs) has tetrahedral orientation of the electron pairs about the central atom.
This does not mean that the shape is tetrahedral, but that the orientation of
electron pairs about the central atom (geometry) is tetrahedral. Figure 1-17
shows different structures with tetrahedral geometry about the central itom, but
different molecular shapes.

Tetrahedral Structure Trigonal pyramidal Structure


H
l-/- ------^..
------*
/ ..
t-t-'Y.tt
-Hlu
H
I H/NY,H
H/NY'H
4 atoms/0lone pairs I 3 atoms/1 lone pair
-symmetry, all bond
Because of uecause of rone pair repulsion, bond
lengths and bond angles
I
are equal./ angles decrease. N is r*ullu. than c, so
Oond length N-H is less than C-H.
I
C-H: 1.10A & < HCH: 109.5" / N-H: 1.00A & < HNH: 102.3"
I
l
I Bent Structure
\\ rr
\r'n'
1o-ru
2 atoms/21one pairs
Because of lone-pair repulsion, bond
angles decrease. O is smaller than N, so
bond length O-H is less than N-H.

O-FI: 0.96A & < HoH: 104.5'

Figure 1-17

Trigonal Planar an d sp2 -Hybridization


A central atom with three other atoms, two olher atoms and one lone pair, or one
-of
other atom and two lone pairs attached has trigonal p?ur,ut geometry th" th.""
substituents (or electron pairs) about the central atom. This does not mean that
the shape is trigonal planar, but that the orientation of electron pairs about the
central atom is trigonal planar. Figure 1-18 shows the planar structure and
spatial representation of the bonds in ethene. The stick and ball representation
shows the three-dimensional perspective for ethene.

C-opyright @ by The Berkeley Review l6 The Berkeley Review


Organic Chemistry Molecular Structure Ilybridization

Planar Structure Spatial Representation


sn2-hvbridization

\i \ /
H'/\'
t,- L
H

/\
HH
3 atoms/0 lone pairs
Each carbon in ethene
has sp2-hybridization.

Figure L-18

Line ar and sp-Hybridiz ation


A central atom with two other atoms or one other atom and one lone pair
attached has linear geometry of the two substituents (or electron pairs) about the
central atom, This does not mean that the shape is linear (although in most cases
it is), but that the orientation about the central atom is linear. Figure 1-19 shows
the linear structure and spatial representation of the bonds in ethyne. The stick
and ball representation shows the three-dimensional perspective for ethyne'

Linear Structure Spatial Representation


sr-hvbridization
' tt' 180.0"

C::
t\ /\
k\,
2 atoms/0lone pairs
Each carbon in ethyne
has sp-hybridization.

Figure L-19

Example 1.6
The hydrogen-carbon-hydrogen bond angle in formaldehyde (H2CO) is BEST
approximated by which of the following values?
A. 108.3'
B. 771.7"
c. 118.5'
D. 121.5'

Solution
The first feature to look at is the hybridization of carbon. Carbon is involved in
one n-bond, so the hybridization is sp2. The bond angle about an sp2-hybridized.
carbon is predicted to be 120". The question here is whether the angle is slightly
greater or slightly less than 120". Because there are two pairs of electrons on the
oxygen/ the electron density repels the electrons in the two carbon-hydrogen
bonds. This forces the two bonds closer together, which compresses the
hydrogen-carbon-hydrogen bond angle. According to valence shell electron pair
repulsion (vsEPR) theory, the angle shouid be slightly less than 120". The best
answer is thus choice C.
_:
Copyright @ by The Berkeley Review t7 Exclusive MCAT Preparation
Organic Chemistry llolecular Structure Bond Energies

Bond Energy
Bond Dissociation Energy
L:r organicchemistry, the energy required to cleave a bond in a homolytic fashion
is commonly used to compare reiative bond strengths. Homolytic cleivage refers
to the breaking of a chemicai bond into two fiee radical fiagments. This is
typically viewed in the gas phase or an aprotic, nonpolar solvenl, where ions are
too unstable to exist. It is important that you recall that energy is released when
a bond is formed and that energy must be absorbed by the ilolecule to break a
bond. By subtracting the energy released upon forming new bonds from the
energy required to break bonds, the enthaipy of a reaction can be determined.
This is shown in Equation 1.1.

AHReaction = lEnergy&onds broken) - Energy66nds (1.1)


formed)
If the enthalpy of a reaction is known, then the bond dissociation energies for
bonds that are formed and broken during the course of a reaction can be
determined. It is this method that allows for the comparison of bonds between
identical atoms within different morecules. For instance, the theory of
aromaticity is supported by the excess energy that is reieased. upon the formation
of a n-bond that completes the aromatic iing. The release of excess energy
implies that the molecule is more stable than expected from the standard bond
dissociation energies, so some other factor must be involved. Table I-T lists some
bond dissociation energies for typicat bonds in some common organic molecules.

Bond Dissociation Energies for A-B Bonds (Kcal/mole)


A B=H Me Et i-Pt t-Bu Ph OH NHz
Methyl 105 90 89 86 84 102 93 85
Ethyl 101 89 B8 6/ 85 101 95 85
n-Propyl 101 89 88 86 85 101 95 85
Isopropyl 98 89 87 B5 82 99 96 B5
f-Butyl 96 87 95 82 77 99 96 B5
Phenyl 111 702 100 99 96 115 111 702
Benzyl 88 76 75 /+ 73 90 81 77
Allyl 86 74 70 70 67 N/A 7B 69
Acetyl 86 81 79 77 75 93,5 107 96
Ethoxy L04 83 85 N/A N/A 101 44 N/A
Vinyl 1I2 102 101 100 95 105 N/A N/A
H 704.2 105 i01 98 96 111 IT9 1.07

Table 1-7

A greater value in Table 1-7 implies that the bond is stronger. you may note that
the weakest bond listed in Tabie 1-7 is an o-o single bond within a peroxide
molecule (Eto-oH). Because this bond is so r.r'eak, peroxides are highly reactive
species, often used to oxidize other compounds. The data in Table 7-7 also reveal
that the substitution of the carbon and the position of the bond within the
molecule affect the bond energy. The effect of hybridization can also be extracted
when comparing bond energies betl-een r-inr.l and methyl substituents.

Cop.,-r::: I :'. lie Berkeley Review la The Berkeley Review


Organic Chemistry Molecular Structure Bond Energies

Example 1.7
According to the data in TabIe 1.-7, which of the following carbon-carbon single
bonds is the MOST stable?
A. An sp2-carbon to a primary sf -carbon
B. An spt-carbon to a second.ary spr-carbon
C. A secondary sp3-carbon to a primary sf -carbon
D. A secon dary sp3-carbon to a iecondary sp3-carbon
Solution
The most stable bond is the strongest bond. The strongest bond has the greatest
bond dissociation energy, so to solve this question, the bond energies from Table
1-7 must be referenced. An spt-hybridized carbon is found in the double bond of
an alkene. This is described as a vinylic carbon, so the vinyl entry in Table 1-7 is
necessary for choices A and B. Considering that we are looking at carbon-carbon
bonds, a primary carbon (with only one bond to a carbon) would have to come
from a methyl group. This value is necessary for choices A and C. Likewise, a
secondary carbon would come from a group such as ethyl or n-propyl
Considering only Et is listed as a substituent in Table 7-7, the value for Et is
necessary in choices B, C, and D.
Choice A is found by looking at the entry for Vinyl-Me, which is 102 kcal/mole.
Choice B is found by looking at the entry for Vinyl-Et, which is 101 kcal/mole.
Choice C is found by looking at the entry for Et-Me, which is 89 kcal/mole.
Choice D is found by looking at the entry for Et-Et, which is 88 kcal/mole. The
most stable bond is the one that requires the greatest energy to break. The
greatest bond dissociation energy among these choices is 102 kcal/mole, so
choice A is the best answer.

Ionic Bonds
Ionic bonds are bonds formed between two oppositely charged ions. They are
common between metals and nonmetals. The strength of an ionic bond can be
determined using Coulomb's law, Equation 1.2. Coulomb's law states that the
force between two charged species is equal to a constant k times the charge on
each ion, divided by the square of distance between the two charges, which are
treated as point charges:

p - v t 1z -
t-^rt 7 q1q2
-4"% 0.2\
c
where F = force, q = charge, r = distance, andto=
" 8.85 x rcjP -C*.
N.m2

The greater the charge on the ion, the stronger the bond; and the closer the ions
are to one another, the stronger the bond. Ionic bonds are typically stronger than
covalent bonds. However, because ions can be solvated in a polar, protic solvent,
ionic bonds are often cleaved more readily than covalent bonds in a protic
environment. In other words, despite the strength of ionic bonds, they are easily
broken by adding water to the ionic lattice. This implies that the Coulombic
attraction of the ions to water is comparable to the attraction of the ions to one
another.

Copyright @ by The Berkeley Review l9 Exclusive MCAT Preparation


Organic Chemistry Molecular Structure Intramolecular Features g
|r
t
Intramolecular Features a
Intramolecular features encompass anything that affects the stability of a I
molecule and the sharing of electron density beyond the localized region {':
between two neighboring, bonding atoms. There are various factors that dictate
the chemical reactivity of a compound and explain the distribution of electron
fi
density within a molecule. I like to call them the "five excuses" to explain organic
chemistry. They are reslnence, the inductiae effect, steric interactions, aromnticity,
and hybridization. I/y'e have already examined hybridization and seen the effect it
q
has on the structure of a molecule in terms of bond angles. Besides considering
the three-dimensional position of the atoms within a molecule, we will consider
electron density and thus establish reactive sites within a molecule. We shall
start by considering the ever-so-loved resonance theory.
Resonance ,!
Resonance is an intramolecular phenomenon whereby electron density is shifted 5
through regions of the molecule via ru-bonds. Resonance is defined as the #'
s
delocalization of electrons through a continuous array of overlapping p-orbitals #
(n-bonds and adjacent lone pairs). Resonance theory can be used to determine
the stability of a structure. There are three rules to follow to determine the
stability of a resonance form prioritized according to importance from most to
least: fr
1. The resonance structure should contain atoms with filled octets
(excluding hydrogen).
2. The best structure minimizes the number of formal charges throughout
the molecule.
3a. if the molecule contains a negative charge, it is best placed on the most
electronegative atom.
3b. If the molecule contains a positive charge, it is best placed on the least
electronegative atom.
Figure 1-20 shows two resonance forms for an amide compound that obey the
octet rule, and a resonance hybrid that shows the composite effect. The
resonance hybrid is an average of all the major resonance contributors.

oo- o6-
t: .&-

,rr.A*rr.-
form
More stable
,r.4.**r,
Less stable form
,rr.A'r'?",
Resonance hybrid
*
.$r-
e

Figure 1-20
St
The resonance structure farthest to the left in Figure 1-20 is more stable than the
middle structure, because there is no separation of charge. You must be able to -
or
rank the stability of resonance structures and decide whether it is a major fl
contributor. Typical questions based on resonance include determining where *
certain molecules are most reactive. You should be able to apply resonance f,
theory to other features of chemical structure and reactivity. For instance, when *
viewing an amide, the electron-rich oxygen is the most nucleophilic site on the m
molecule. When protonating an amide, it is the oxygen that gets protonated. q
When amides form hydrogen bonds, the oxygen is the electron-donating site. fi
This has a significant impact on molecular structure in protein folding. I

Copyright @ by The Berkeley Review 20 The Berkeley Review fti


Organic Chemistry Molecular Structure Intramolecular Features

Figure 1-21 shows four examples of resonance structures and the arrow pushing
necessary to convert between resonance forms. To draw resonance structures
that are stable, it is often helpful to start with a lone pair and push those electrons
into a n-bond. The electrons from the adjacent n-bond turn into a new lone pair.

roi H H (ot :ij:o H


ll l^<-+ \lltq.z ts
I -* I
to \
I

H,C/t-
'lv'L'l g,Qfl*],. - nil,. - r- r"
HHH
o 6-
Resonance Hvbrid: T
/C1.6;2 Ct s-
HsC NH
f
H

..
ioi o o6-
I ti

6zt-'dl aoy'c:.oo-
Resonance Hybrid

t'
CH" CH" CH"
t"
",P'q8
-* 3r.-t\.,,,
t"
,.liz't'l3;,
Resonance Hybrid

t'
CH"

{-*
CH"
l" CH"
t"
Hrc?5l,', *rr3-c\c*r,
"lll"'13.,
Resonance Hybrid

Figure 1-21

In Examples 1",2, and 3, shown in Figure 7-2I, the negative charge moves every
other atom between resonance forms. The lone pair becomes a n-bond, and the
n-bond becomes a lone pair two atoms away. This is true when the number of
total charged sites remains constant. In Example 1, there is only one charged
atom in each of the resonance forms. You must look for the all-octet resonance
structures. A11 of the resonance forms except the carbocations in Example 4 are
all-octet resonance forms. This satisfies Rule 1 on the list of resonance rules.
Neither structure in Example 4 satisfies the octet rule. The resonance hybrid is a
composite of the individual resonance contributors. The most stable resonance
structures (major resonance contributors as they are called) have the greatest effect
on reactivity and structure for a compound exhibiting resonance.

Copyright @ by The Berkeley Review 2t Exclusive MCAT Preparation


Organic Chemistry Molecular Structure Intramolecular Features

Example L.8
The C-O bond length is LONGEST in the compound on the:

*
o o

5-"
A. left, because nitrogen donates electrons through resonance.
B. right, because nitrogen donates electrons through resonance,
C. left, because nitrogen withdraws electrons through resonance.
D. right, because nitrogen withdraws electrons through resonance.

Solution
For this question, the resonance forms of the lactam should be drawn first:
6-
Partially o
single bond

$-" +
The all-octet resonance form on the right has the carbonyl bond in single bond
form. The single-bond resonance is caused by the donation of a lone pair of
electrons by nitrogen. This means that the C-O bond is longer in the compound
on the left, because nitrogen donates electrons through resonance. The correct
answer is thus choice A. The effect of resonance outweighs the inductive effect.
The inductive effect predicts that the nitrogen would be electron-withdrawing,
because it is more electronegative than carbon.

Inductive Effect
The inductive effect, as the name implies, induces charge separation in a
molecule, just as induction in physics refers to the creation of charged sites
through induction. From a chemist's perspective, the inductive effect is the
delocalization of electrons induced by electronegative atoms. The inductive
effect involves the transfer of electron density through the sigma bonds. A
highly electronegative atom pulls electron density from its neighbor, which in
turn pulls electron density from its neighbor. The effect dissipates over distance,
but it can affect bonds between atoms up to three or four atoms away. We most
often consider the inductive effect when a molecule has a halogen.
The inductive effect increases with the electronegativity of the atom. Fluorine is
the most electronegative atom found in organic molecules, so it pulls electrons
from the carbon to which it is attached in a strong manner. This makes that
carbon electron-poor, so it in turn pulls electrons from its neighbor. Ultimately,
as we see with polarity, the electron density in the molecule is pulled towards the
most electronegative atom in the compound. An electronegative atom therefore
withdraws electron density and thus can increase a compound's acidity, increase
its electrophilicity, decrease its basicity, or decrease its nucleophilicity.

Copyright @ by The Berkeley Review 22 The Berkeley Review


Organic Chemistry Molecular Structure Intramolecular Features

For the relative electronegativity of common atoms in organic chemistry, the


followingrelationshipholds: F > O> N > Cl > Br > I > S > C > H. Justrecall
"Fonclbrisch" and you'll be in good shape. It may seem strange, but alkyl groups
are electron-donating by the inductive effect, because hydrogen is less
electronegative than carbon. Figure 1-22 shows an example of the inductive
effect as it applies to the nucleophilicity of amines reacting with an alkyl halide.

H H
t. t.
t:a"*-rt t-.a*l*,
I Z-roftate=L44 I Z- -log rate = 4.31
HH FF
Methylamine Trifluoromethylamine
Rate for H3CNH2 > rate for F3CNH2
Less nucleophilic due to the electronegative fluorine atoms

Figurel-22

The withdrawal of electron density by the fluorine atoms decreases the


nucleophilicity of the amine compound by pulling electrons away from the
nitrogen atom. As electron density is removed, the compound becomes electron-
poor and thus a worse electron donor. This can be verified by the reaction rate in
a substitution reaction. As the negative log of the rate increases, the rate of the
reaction decreases.

Example L.9
\\hich of the following compounds undergoes a nucleophilic substitution
reaction with ethyl chloride at the GREATEST rate?
-{. H3CCHFNH2
B. FH2CCH2NH2
C. H3CCHCINHz
D. CIHzCCH2NH2

Solution
The greatest reaction rate (the fastest reaction) is observed with the best
nucleophile. Each answer choice has one halogen, so all the choices have slower
rates than ethyl amine. The question asks which experiences the least inductive
"-cithdrawal. Chlorine is less electronegative than fluorine (Fonclbrisch), so
choices A and B are eliminated. The inductive effect diminishes with distance, so
tFre least electron withdrawal is observed with choice D. You must consider both
proximity and electronegativity when looking at the inductive effect.

-\lthough not applicable in Example !.9, you must also consider whether the
inductive effect involves electron donation or electron withdrawal. For instance,
methyl amine is more nucleophilic than ammonia (NHg), because the methyl
group is electron-donating. Varying the R-group changes the inductive effect. It
also changes the size of the molecule, so steric hindrance can affect the reaction.
For instance, trimethyl amine ((H3C)3N) is less nucleophilic than dimethyl amine
(H3C)2NH), because the electron donation by the additional methyl group does
not compensate for the increase in molecular size.

Copyright @ by The Berkeley Review 23 Exclusive MCAT Preparation


Organic Chemistry Molecular Structure Intramolecular Features

Steric Hindrance
Steric hindrance occurs any time two atoms attempt to be in the same place at the
sarne time. It is repulsive in nature and increases as the atoms draw closer. No
one is certain about the nature of the force, but it is believed to be electron cloud
repulsion. The effects are similar to what is observed in general chemistry with
VSEPR (valence shell electron pair repulsion) theory, except that it is considered
only when two separate atoms or functional groups interact. Electrons move to
be as far apart as possible, so lone pairs and bonds spread out to accommodate
the geometry that spaces the greatest distance between electrons. Figure 1-23
demonstrates the effects of steric hindrance on a couple of organic molecules.
Because the alkene is planar, the substituents on the alkene carbons have a
tendency to collide with one another.

H.C H.C CHs


/\ /\ //
I
I L- t
LJ
\,"/L-L_ \
^-^

'""/
HsC HsC CHe
Larger C-C-C bond angle Smaller C-C-C bond angle

HHH
/"t\
H"C CH.
\.-./ \'
{
\. \/
H"C
-\__/ CHN
H_H H
Reduced bond angle in dimethylbutene,
because the methyl group hydrogens repel.

Figure 1-23

Example 1.10
Which of the foilowing functional groups is MOST likely to be found in the
equatorial position on cyclohexane?
A.
B. -OCH3
C. -OCH2CH2CH2CH1
D. -OCH(CH3)CH2CH3
-oC(CH3)3
Solution
On cyclohexane, substituents with axial orientation experience greater steric
hindrance than substituents with equatorial orientation. Because of steric
hindrance, the substituent most likely to assume the equatorial orientation is the
bulkiest. The tert-butoxide substituent, choice D, has the most crowded alkyl
groups/ resulting in the greatest steric hindrance. This makes the best answer
choice D.

Copyright @ by The Berkeley Review The Berkeley Review


Organic Chemistry Molecular Structure Intramolecular Features

Aromaticity
Aromaticity is stability generated from having 4n + 2 ru electrons in a continuous,
overlapping ring of p-orbitals, where n is any integer including 0. This is known
as the Hilckel rule. The stability is rooted in the molecular orbital model, where
an energy level is completely filled when there are 4n + 2 n-electrons in the cyclic
n-network. Figure 1-24 lists experimental values for the enthalpy of
hydrogenation of a series of alkenes. The large deviation associated with

o#o
benzene is attributed to its aromatic stability.

AH = -2g
ktul/.ol.

o#o AH = -56
ktul/*ol"

o#o AH = -54
ktul/-n,"

o#* o LH = -49
ktul/r',or"

Despite the presence of three n-bonds, the 1,3,5-cyclohexatriene


(benzene) yields far less heat from hydrogenation than expected
due to its aromatic stability.

Figure 1-24

The first two entries show that the enthalpy of hydrogenation of an alkene is -28
kcal/mole per n-bond. The third entry shows that conjugation results in
stability, reducing the amount of heat released upon hydrogenation, but only by
about 2 kcals/mole. Based on -28 kcal/mole for each n-bond, benzene should be
expected to have a AH of approximately -84 kcal/mole. The difference of 35
kcal/mole (84 - 49) cannot be attributed to conjugation alone, hence it is said to
be due to aromatic stability.
)'lot all cyclic, conjugated polyenes show such a large deviation from the
expected value for the enthalpy of reaction. 1,3,5,7-Cyclooctatetraene (CgHg)
shows a deviation of only 8 kcal/mole from its expected value of -112 kcal/mole.
This implies that conjugation is useful for only a small fraction of the 35
kcal/mole difference observed with benzene between its expected and actual
va-lues. Because benzene has 6 n-electrons in a continuous n-cycle, it obeys the
Huckel rule (it has 4n + 2 n-electrons where n = 1), whiIe1",3,5,7-cyclooctatetraene
(CsHs) has 8 n-electrons in a continuous n-cycle and does not obey the Hrickel
rule. This lack of aromaticity results in a less stable reactant, so more heat is
generated in the hydrogenation reaction.

Copyright O by The Berkeley Review Exclusive MCAT Preparation


Organic Chemistry Molecular Structure Intramolecular Features

Example 1.1L
The hydrogen-carbon-hydrogen bond angle about the terminal carbon in the
following alkene is BEST approximated by which of the following values?

" '\
H"CH"C
f-- a-
H
/aI
v-v
/\/
HsC H
A. 108.3"
B. 711..7"
c. 118.5"
D. 121.5"
Solution
First we must consider the hybridization of carbon. It is involved in one n-bond,
so the hybridization is sp2. The bond angle about an sp2-hybtidized carbon is
predicted to be 120". The question heri: is whether the angle is slightly greater or
slightly less than 120". Because there are two alkyl groups on the other carbon of
the alkene, the electron density repels the electrons in the two carbon-hydrogen
bonds. This forces the two C-H bonds closer together, which compresses the
hydrogen-carbon-hydrogen bond angle. According to steric repulsion theory,
the angle should be slightly less than 120'. The best answer is thus choice C.

Example 1.L2
Which of the following explanations accounts for the pKu of L,3-cyclopentadiene
being only 15, while the pKu for hydrogen on other spr-carbons is around 49?
A. The strain of the five-membered ring forces the proton off.
B. The proton is involved in resonance.
C. The conjugate base is aromatic.
D. The steric hindrance of the sp3-carbon weakens the C-H bond on that carbon.

Solution
The acidity of a compound capable of losing a proton ({+) can be determined by
the stability of its conjugate base. When an ordinary spr-hybridized carbon (one
that is not stabilized by resonance or the inductive effect) is deprotonated, the
carbanion that is formed is unstable. Carbon is not electropositive, so it does not
readily lose a proton. With 1,3-cyclopentadiene, however, the carbanion that is
formed upon deprotonation has both resonance and aromatic stabilization once it
loses the proton. The cyclopentadienyl anion that is formed is aromatic. This
makes choice C the best choice. The reaction is drawn below:

HH H

H'

Six conjugated n-electrons in a continuous


planar arrangement of p-orbitals is aromatic.
Because it includes the word tesonance, choice B may at first seem apPealing. But
a proton, having no electron pair, cannot be involved in resonance. Be careful of
wording like this, because it is easy to pick resonance without thinking about it.

Copyright @by The Berkeley Review The Berkeley Review


s Organic Chemistry Molecular Structure Fundamental Reactivity

F dffifitffi:xiRd#Ctffit$
Fundamental Reactions in Organic Chemistry
In organic chemistry, perhaps the most common class of reaction is nucleophilic
attack. In the simplest sense, a nucleophilic compound (one with an electron-rich
site) attacks an electrophilic compound (one with an electron-poor site) to form a
new bond. In some instances a leaving group is discarded, while in others a n-
bond is broken. No matter what the result, the reaction has the same
tundamental drive and mechanics. The reactions can be viewed as Lewis acid-
base reactions, so organic chemistry starts with a thorough look at Lewis acids
and bases. Prior to that, we shall review Bransted-Lowry acid-base chemistry.

Proton Transfer Reactions (Brsnsted-Lowry Acid-Base Reactions)


Brsnsted-Lowry acid-base reactions involve the transfer of a proton (H+) from
the acid (defined as the proton-donor) to the base (defined as the proton-
I,
acceptor). This means that to be a Brsnsted-Lowry acid, the compound must
have a hydrogen that can be lost as H+. A hydrogen like this is often referred to
is
as a protic hydrogen or proton. Throughout this section, we will be using the term
)r
grotonation to describe the gain of an H+. A hydrogen atom has one proton in its
rf
nucleus and one orbiting electron (in a 1s-orbital). when hydrogen loses an
n
electron to become H+, all that remains is a proton. This is to say that H+ is a
.e
Flroton, and thus the gain of H+ can be referred to as protonation. Deprotonation is
the loss of H+.
To be a Brsnsted-Lowry base, the compound must have electrons available that
can form a bond to H+. Because a lone pair of electrons is necessary to form a
bond to the proton, all Bronsted-Lowry bases are also Lewis bases. Figure 1-25
shows a proton-transfer reaction, a one-step reaction.

trt,

..'
,N:'
N:,a-\
Bond forming

+ Hf-.Cl '
+ =F-
"\o
H"C

... N- H + :cl:
..O
Hrc\\Y son;/;iins
Bond breaking Hrc'{,:*:,
-I formed
Bond
broken
"
H H
Base Acid Conjugate Conjugate
v (Proton-acceptor) (Proton-donor) Acid Base
te
re Figure 1-25
)t
is ln the reaction in Figure 1-25, you should note that an arrow going from a lone
it pair to an atom becomes a bond in the product, and an arrow going from a bond
is to an atom becomes a lone pair on that atom in the product. This is a standard
convention in drawing mechanisms. The reaction shown in Figure 1-25 is very
favorable, as indicated by the asymmetric equilibrium arrow. The favorability is
attributed to the fact that HCI is a strong acid. Proton-transfer reactions proceed
favorably (AG < 0) from the side with the stronger acid and stronger base to the
side with the weaker acid and weaker base. This is to say that a favorable
chemical reaction proceeds from the less stable species to the more stable species.
There are five strong acids used in organic chemistry that you should recognize:
H2SO4, HNO3, HCl, HBr, and HL An important fact to know is that as a
Bronsted-Lowry acid gets stronger, it loses a proton more readily, so its
lrt conjugate base is less willing to gain a proton. The result is:
of \Alhen comparing two conjugate pairs, the pair with
the stronger acid has the weaker conjugate base.

w Copyright O by The Berkeley Review Exclusive MCAT Preparation


Organic Chemistry Molecular Structure Fundamental Reactivity
T
Lewis Acid-Base Reactions I
Lewis acid-base reactions involve the transfer of an electron pair from the base n
(defined as the electron-pair donor) to the acid (defined as the electron-pair fl
acceptor). This means that for a compound to be a Lewis base, it must have il
electrons available that can form a bond to an electron deficient atom (such as, W
but not exclusively, H*). A Lewis acid can have a protic hydrogen, but a Lewis ,t
acid may have an empty valence shell capable of accepting electrons. Typical d
Lewis acids include BF3, AlCl3, FeBr3, and SOCI2. Figure 1-26 shows a Lewis q
acid-base reaction, where ammonia is the Lewis base and BF3 is the Lewis acid. il

"'\o
ril
H.C Bond forming H^C E: m

.N: eti:' l[
tr!

H,CV
H "r."'.?'13,"#;\..
H .F:
Base Acid
(Electron Pair Donor) (Electron Pair Acceptor)
n
rrrt
Figure 1,-26
4
.&.
The role of a base is essentially the same in both the Lewis and Bronsted-Lowry
4&
definitions. A base donates a lone pair of electrons to form a bond to an acid, &t.
whether the acid is a Brsnsted-Lowry acid or a Lewis acid. In organic chemistry,
the terminology varies, and Lewis bases are frequently referred to as nucleophiles.
Nucleophile means "nucleus loving", which implies that nucleophiles seek out
positively charged sites (referred to as electrophiles). The simple guide to
organic chemistry is that negative charges seek out and bond to positive charges.

Acidity
Acidity is defined by three definitions: the Arrhenius definition, the Brsnsted-
Lowry definition, and the Lewis definition. The Arrhenius definition is that an
acid yields HgO* when added to water. The Bronsted-Lowry definition is that
an acid is a proton (H+) donor. The Lewis definition is that an acid is an electron
pair acceptor. The strength of an acid depends on the effects of intramolecular
forces on the bond to the acidic proton. These are the electronic forces within a
molecule. They are responsible for the distribution of valance electrons, which
accounts for the chemical behavior (such as acidity) of the molecule. An acid is
stronger when an electron-withdrawing group is attached to the backbone of the
acid, because the molecule is electron-poor, and thus a better electron-pair
acceptor. An acid is weaker when an electron-donating group is attached to its
backbone, because the molecule is electron-rich, and thus a wolse electron-pair
acceptor. The primary task associated with evaluating organic acid strength is to
decide which groups are electron-donating and which are electron-withdrawing.
Figure 1-27 shows some common organic acids and their pKu values.
OH
PKu =10 H
o o*'=e-10 o
I PKa = 15

.A';il" o-*Vt H
R- OH |x:=17'20
Carboxylic acid Phenol Alkyl ammonium cation Alkoxide Carbonyl cr-proton

Figure L-27

Copyright Q by The Berkeley Review The Berkeley Review

.ffii;'...:
Organic Chemistry Molecular Structure Fundamental Reactivity
3
Basicity
lSE Basicity is most easily thought of as the opposite of acidity. Basicity is also
air defined by three definitions: the Arrhenius definition, th-e Bronsted-Lowry
rve definition, and the Lewis definition. The Arrhenius definition is that a bas-e
?S, vields OH- when added to water. The Brsnsted-Lowry definition is that a base is
vis a proton (H+) acceptor. The Lewis definition is that a base is an electron-pair
cal donor. The rules that you use for acidity can be applied to basicity, but with the
r'\i iS opposite effect. Electron-donating groups increase basicity (while they decrease
acidity) and electron-withdrawing groups decrease the basicity. As a result, the
strength of a base or acid can be determined from the stability of its conjugate.
The more stable the conjugate, the weaker the conjugate and the strongei the
respective compound (either acid or base). Figure 1-28 shows some common
organic bases.

d
=4
o PKa= 4
Jl prn =e-71 PKu = -1
? PKu=-6--a
-/\
RO- o/*Y,' H
R-O-: RACH,-
Carboxylate Phenoxide Alkyl amine Alkoxide Anionic g-carbon
Iry
id, Figure 1-28
ry,
Ies. AJthough you are not required to memorize exact pK6 values, it is a good idea to
)ut know the "-5-10-75-20 general rule" for organic acids. The pKn for i carboxylic
to acid is about 5, for a phenol it's about 10, for an alcohol it's about 15, and for a
es. proton alpha to a carbonyl it's about 20. These are close enough for good
guessing.

3d-
an Example 1.13
hat tr\hat is the pK6 for p-nitrophenoi and the pK6 for its conjugate base?
:on A. O2NC6H4OH has pKa = 7.2; O2NC6H4O- has pKU = t2.8
ilar B. O2NC6HaOH has pKa = 11.6; O2NC6H4O- has pKU = 8.4
na C. O2NC6HaOH has pKa = 7.2; O2NC6H4O- has pKb = 6.8
ich D. O2NC6H4OH has pKa = 11.6; O2NC6H4O- has pKb=2.4
lis
the Solution
rair The nitro group is electron-withdrawing, which makes nitrophenol a stronger
its acid than phenol. As an acid becomes stronger, its pKu value decreur"r. Thit
rair means that the pKu for nitrophenol is likely tobe7.2 rather than 11.6. This
;to eliminates choices B and D. The pKu and pK6 sum to L4, so choice C is the best
ng. answer. This question could also be answered from a base perspective. The nitro
group is electron-withdrawing, which makes nitrophenoxide a weaker base than
phenoxide. As a base becomes weaker, its pK5 value increases. This means that
T" prr, for nitrophenoxide is greater than 4,0. This eliminates choice D. Again,
the pKu and pK5 sum to 14, so choice C is the best answer. pick C andlain
-20 rncredible satisfaction doing what you should do.
Pick the perspective (acid or base perspective) that works best for you, and use it
rvith these questions.

rew Copyright O by The Berkeley Review Exclusive MCAT Preparation


Organic Chemistry Molecular Structure Fundamental Reactivity

Choosing Where to Protonate and Deprotonate


A common question in organic acid-base chemistry is, "Which is more acidic?"
This question can refer to two separate compounds, or two sites within the same
compound. There is also the complementary question, "Which is more basic?" In
Figure I-29,we are dealing with two sites on the same molecule, so the question
is, "Which lone pair on the molecule is most basic?" It shows the two possible
products when acetic acid (CH3CO2H) is treated with a strong acid. Acetic acid
(a carboxylic acid) has two sites with lone pairs (two oxygen atoms). Deciding
which oxygen gets protonated (which forms a more stable protonated species)
requires thai you consider many different factors, including the resonance
stability associated with each protonated product. Resonance can help to
stabilize the excess positive charge on the compound, and thus make the species
more stable.

at
oxygen
^r,o} tt oxygen u :p- H
protonation
o*ysJl';"'
uuc-h!/) <+ H3c-c
f
\i6-11 \
oxygena [. .g^-H
// All-octet resonance form, #aking
H.C-
"\ C this the more stable product.

Oxygenb i9-11 o: :'6:o


//) /
H1C-u
" <-'> H"C-
" CO
Protonation at \^ \^
Oxygen b Oxygen t !g-- g Oxygen bi6;'- 11
/ /
H H
Non-octet resonance form with separation of
charge, making this the less stable product.

Figure 1-29

Because there is resonance stabilization when the compound is protonated at


Oxygen a, and there is no resonance stabilization when the compound is
protonated at Oxygen b, it can be concluded that carboxylic acids are protonated
at the carboxyl oxygen (C=O), not the hydroxyl oxygen (OH). This means that
the carbonyl oxygen is more basic than the alcohol oxygen. In reality, we rarely
see carboxylic acids acting as bases, but amides are similar in structure, and it is
critical in biochemistry that we know where they are protonated and thus where
they exhibit hydrogen-bonding.

Example 1.14
Which of the following statements BEST explains why amides are protonated at
oxygen rather than nitrogen?
A. The oxygen is less electronegative than nitrogen, so it donates electrons more
readily"
B. The oxygen is larger than nitrogen, so its electron cloud attracts protons more
readily.
C. Oxygen carries a partial positive charge due to resonance withdrawal from
the nitrogen.
D. Oxygen caries a partial negative charge due to resonance donation from the
nitrogen.

Copyright @ by The Berkeley Review 30 The Berkeley Review


Organic Chemistry Molecular Structure Fundamental Reactivity

Solution
As shown below, the nitrogen of the amide donates electron density to the
carbonyl oxygen through resonance. This places a partial negative charge on
oxygen (increasing its basicity), and a partial positive charge on nitrogen
'decreasing its basicity). Choice A is eliminated, because oxygen is more
electronegative than nitrogen. Choice B is eliminated, because oxygen is smaller
Jran nitrogen, not iarger. Because of resonance, oxygen carries a partial negative
:harge, while nitrogen carries a partial positive charge. This means that choice C
-: false and choice D is true. This explains the basicity of amides.

: O : 9
O ' *ot" basic due to negative charge'

ll-l
^Ar,- NHz
R Aa -NHzlessbasicduetoabsenceofelec{rons.
R-

{cid Strength Factors


affecting the strength of an acid or base can be broken down into primary
- ,'-ctors
=:iects and secondary effects. Primary effects depend on the bond directly to the
',-cic proton. The weaker the bond to the acidic proton, the more readily it
:::aks, and consequently more acidic the acid. With acids, the bond breaks in a
--:rerolytic fashion, forming ions. Primary effects include size, electronegativity,
.:.i hybridization of the atom directly attached to the acidic proton. Secondary
,.-.;ls depencl on the effect of the molecule on the atom bonded to the acidic
:.,rtor-r. The more electron-rich that atom, the less acidic the proton. The more
= .:tron-poor that atom, the more acidic the proton. secondary effects include
-:Surflorc (the delocalization of electrons within a molecule through ru-bonds),
-: - nductive effect (the delocalization of electrons within a molecule
through o-
: -::.1s), and electron cloud repulsion (deformations of a molecule and elongation
:: londs within the molecule). There is also aromaticity to consider, but that will
:= :-Cdressed as a special case. Secondary effects involve intramolecular forces,
-.-:h dictate where the electron density within a molecule lies, and thus they
,:,::atethereactivityof acompound. weshalllookathowthesefeaturesaffect
it --- electron distribution within a molecule, starting with the primary effects.
;- l,:r-e 1-B shows common acids and bases in organic chemistry, listed according
d - :elative strength (in both the acids and the bases).
rt
Strong Acids H2SOa > HI > HBr > HCI > HNO3
)'
rs HF > HCO2H > H3CCO2H > H2CO3 > H3CSH > H3CNH3CI
i\-eak Acids
:e H5C6OH > H3COH
: trong Bases CH3(CH2)3Li > NaNH2 > KH > NaOCH2CH3 > NaOH = KOH
l\'eak Bases H3CNH2 > NaHCO3 > H3CCO2Na > HCO2Na > H3COH
at
Table 1-8

Fimary Acid Strength Factors


: ,::-.arv factors directly affect the strength of the bond to the protic hydrogen.
--. ,'. i.'ond weakens (homolytic and heterolytic bond dissociation energy lessens),
, -. j strength increases and conjugate base strength decreases. The three primary
:::ors to consider are atomic size (when comparing acids involving atoms
-:rin the same column of the periodic table), electronegativity (when
- --.raring acids involving atoms within the same row of the periodic table), and
' ::r;lization (when comparing acids where hydrogen is on the same atom).

- :,, right @ by The Berkeley Review 3l Exclusive MCAT Preparation


Organic Chemistry Molecular Structure Fundamental Reactivity

Strcngth Dependence on Atomic Size


When comparing acids within a column of the periodic table, the strength of the
acid is dictated by the size of the atom to which hydrogen is bonded. Smaller
atoms form shorter bonds to hydrogen than their larger counterparts. Given that
longer bonds are weaker bonds (in both a homolytic and heterolytic fashion), the
acidity of a compound directly depends on the length and strength of the bond'
A significant difference in atomic size ultimately determines the relative strength
of acids, because larger atoms make stronger acids.
A prime example of this involves proton exchange (acid-base) chemistry between
thiols and alcohols, where thiols are stronger acids than alcohols. This is because
sulfur (found in a thiol) is a larger atom than oxygen (found in an alcohol). in
acid-base chemistry, you may also consider the reactivity from the base
perspective. To apply this atomic size theory to bases, compare the distribution
of electron density about a small atom velsus a larger atom. A conjugate base is
more stable if electron density is spread out over more space. Because larger
atoms stabilize negative charge more readily, they are not as reactive, and thus
not as basic. This is often referred as polarizability. Il you keep in mind that the
more the negative charge is spread out,.the harder it is for a proton to find the
negative charge, then you see that the compound is not as basic. The thiol-
alcohol example is shown in Figure 1-30.

HH
\r
,A-
HS CH: HO CHg
PKa = 10'4 PKu = 15.7
Stronger Acid Weaker Acid

Figure 1-30

You should understand that atomic size (polarizability of the conjugate base) can
be applied only when the protic hydrogen is directly bonded to the larger atom.
The atomic size argument is used to explain relative acidity for haloacids.
Relative haloacid strength for haloacids is: HI > HBr > HCI > HF, confirming that
size is more important than electronegativity for elements within the same
column of the periodic table. This contradicts what you would expect if you
were to apply the rules of electronegativity. The relative strength in the case of
haloacids is dictated by atomic size of the halogen. As mentioned before, it is
because the bond is longest between hydrogen and the largest halogen, thus
making it the weakest and most readily broken hydrogen-to-halogen bond. It
can also be considered that the conjugate base (halide) is more stable as it gets
larger, because of the greater space over which the negative charge is distributed.
In the halogen case, the less concentrated (more diffuse) negative charge on
iodide (i-) is not as readily shared as the negative charge on bromide (Br-),
chloride (Cl-), or fluoride (F-). The weaker the conjugate base, the stronger the
conjugate acid, This is to say that because I- is the most stable anion of the
halides, it is the weakest base of the halides. Therefore, HI (the conjugate acid of
I-) is the strongest of the haloacids. This theory is also applied when looking at
the relative reactivity of halogen containing organic compounds in reactions
where the halide is a leaving gfouP. The relative acidity of haloacids should be
familiar to you. The mathematics of acidity and basicity is important in general
chemistry, but we shall consider only values such as pK6 to compare the relative
strengths of acids. In organic chemistry, by quantifying acidity with pKa values,
we can support or disprove relationships.

Copyright @ by The Berkeley Review The Berkeley Review


Organic Chemistry Molecular Structure Fundamental Reactivity

Str ength D ep en dence on El ectr oneg atia ity


e Within a row of the periodic table, the strength of the acid is dictated by the
r electronegativity of the atom to which hydrogen is bonded. Given that atoms
rt within the same row of the periodic table are approximately equal in size, the
e bond strength depends more on the distribution of electrons within the bond
t. than it does on the length of the bonds. Because the electronegativity of the atom
I bonded to hydrogen affects the distribution of electron density within a bond, the
electronegativity of the atom directly bonded to the acidic proton dictates the
acidity of the compound.
R

e The relationship is reasonable in that as an atom bonded to hydrogen has a


n greater electronegativity, the electrons are pul1ed more towards that atom, rather
e than toward hydrogen. This allows the bond to be cleaved in a heterolytic
n fashion rather easily. The result is that the more electronegative the atom, the
s stronger the acid.
r
s
Comparing electronegativity applies only for hydrogens
e
bonded to atoms in the same row of the periodic table,
e
not the same column of the periodic table.
t-
The greater acidity associated with alcohols than amines of equal alkyl
substitution can be explained by invoking the fact that the electronegativity of
oxygen is greater than that of nitrogen and that oxygen and nitrogen have
roughly equivalent atomic sizes. Figure L-31 shows the relative acidity when
comparing a hydrogen bonded to fluorine, oxygen/ nitrogen, and carbon.
Because fluorine is more electronegative than oxygen, which is in turn more
electronegative than nitrogen, the following relationship holds true:

HH HH HH
H-F t \r \l \r
n HO.4. CHs HzN CHg H CH:
L Strongest Acid Amphoteric Very Weak Acid Weakest Acid
i.
PKu = 3'2 pKa = t57 pKa = 33 PKo= 49
rt
e
tl Figure 1-31
f
s The same rationale used to explain relative acidity in Figure 1-31 can be used to
S
erplain relative basicity in the conjugate bases amines, alcohols, and hydrofluoric
tt
add. Because nitlogen is less electronegative than oxygen and fluorine, it more
s rcadiiy donates an electron pair to a proton, so the trend in basicity shown in
t.
Figure 1-32 holds true:

n
,), ui.io cH.,
e ir<o = -rl iro = -r.s PKu = 10'8
e
rf Figure L-32
It
s Amides (RZN-) are some of the strong bases that are used in organic chemistry.
e They are used to remove alpha protons (the protons bond to the carbon that is
il alpha to a carbonyl) and the terminal hydrogen of an alkyne, both with pKu
e values above 17.
s/

Copyright @ by The Berkeley Review .'.' Exclusive MCAT Preparation


Organic Chemistry Molecular Structure Fundamental Reactivity o
Strength D ependence on Hybridization >e:
The hybridization of an atom affects the bond length and the distribution of ir":
electron density within a bond, so the hybridization of an atom directly bonded
to the acidic proton affects the acidity of the compound. The relationship is not ::r
obvious in that it is not true that longer bonds lead to stronger acids, as is the case : :r:-
with most other acids. In fact, the relationship between length and acid strength :a
is exactly the opposite. As the hybrid orbital gets smaller, the electrons are held :*ii l
closer to the nucleus of the atom bonded to hydrogen, so the bond can be cleaved nt-:,:
in a heterolytic fashion more easily. r"ai:r:
*C*,
The result is that the more s-character in the hybrid orbital
of the atom bonded to hydrogen, the stronger the acid, : -*,
--.
re5
This results in the relative acidity being sp > sp2 > sp3. It is most commonly
observed with carbon acidity, but can also be observed with nitrogen and
oxygen. Figure 1-33 shows the comparison of acids where hybridization explains
the difference in acid strength. :,*rl"

-HCH"
sPL \ / '
pKa=26 H- C= C- CH3 ^CrC:spr
Ca

PKo=36 H CHs
Stronger Acid Weaker Acid L*

: ::-
-{ -

oK"=26 H PKo=33 H
I I
N tir
rr;N--.cu,
SP"
^"\r'cH3
SP.N
il-ril:

H
lz-
HH
Stronger Acid Weaker Acid

Figure 1-33 l:.


i:.:.
:-?;
Example 1.15
A compound with which of the following atoms would be the STRONGEST * i -!..-

base?
A. A weakly electronegative atom carrying a negative charge
B. A highly electronegative atom carrying a negative charge
C. A weakly electronegative atom carrying no charge
D. A highly electronegative atom carrying no charge

A base is a compound that donates electrons. The best electron-donating group


would be an atom that readily shares its electron density, so the atom should not
be very electronegative. The stronger base carries a negative charge, rather than
no charge, so it can more easily donate electrons to an H+. The best answer is
choice A,

Copvright O by The Berkeley Review 34 The Berkeley Review


Organic Chemistry Molecular Structure Flrndamental Keactivity

Secondary Acid Strength Factors


Secondary factors indirectly affect the strength of the bond to the protic
hydrogen. Secondary effects involve the electronic environment of the atom
bonded to the protic hydrogen. Simply put: If the atom has electron density
donated to it from the rest of the molecule, it will not pull away electrons,from
the bond to hydrogen as readily, so the compound is less acidic. if the atom has
electron density withdrawn from it by the rest of the molecule, it will pull away
electrons from the bond to hydrogen more readily, so the compound is more
acidic. Secondary factors to consider for now are resonance and the inductive
effect. Resonance is a more significant factor than the inductive effect.

Strength Dependence on Resonance


Resonance, being ihe delocalization of electrons through an overlapping string of
;-bonds, can be either an electron donating effect or an electron-withdrawing
effect. It is important to distinguish between electron-donating groups and
electron-withdrawing groups, because they affect acidity in opposite ways.

When looking at resonance, an adjacent atom making


a n-bond is electron-withdrawing, while an adjacent
atom with a lone pair is electron-donating.

Trpical examples of organic acids that are affected by resonance include phenols
and carboxylic acids. Figure 1-34 shows the resonance effect of an electron-
donating group (OCH3) on a carboxylic acid. It decreases the acidity, raising the
pKu only when it can delocalize n-electrons.

o
!\ ith no n-bond, there
can be no donation oH, uuco: Methoxy lone pair can
donate to acid group
*uough resonance FoH through the n-bond
Pf"=56
Figure 1-34

The methoxy substituent (OCH3) has a lone pair of electrons on the first atom
adjacent to the carboxylic acid. The oxygen atom of the methoxy group donates
eleckon density to the system, and thereby decreases the acidity of the carboxylic
acid. An electron withdrawing substituent has the opposite effect, because it
grakes hydrogen electron deficient, and thus more protic. Electron-withdrawing
substituents increase the acidity of a compound, as shown in Figure 1-35.

".$.{'"o_Q
o*Jo o pKa=10.0

Figure 1-35

It is important to know that electron-withdrawing groups make the compound a


5etter electron-pair acceptor (Lewis acid), so the acidity increases with electron-
-.'.ithdrawing substituents. Electron-donating groups make the compound a
lt orse electron-pair acceptor, so the acidity decreases with electron-donating
substituents. This holds true with all effects (not just resonance). It is easy to get
caught up in the idea that any resonance makes a compound more stable, but this
l-c not always true. Use the electron-donating or electron-withdrawing
;haracteristics to determine the effect on the acidity or basicity.

Copyright @ by The Berkeley Review t5 Exclusive MCAT Preparation


Organic Chemistry Molecular Structure Fundamental Reactivity

Strength Dependence on the lnductioe Effect


The inductive effect is the delocalization of electrons through the sigma bonds of
a molecule. Fluorine is the most electronegative atom found in organic
molecules. It withdraws electron density from the carbon to which it is attached,
which makes that carbon electron-poor. As a result, the electron-poor carbon
pulls electron density from its neighbor. Ultimately the electron density in the
molecule (as with polarity) is pulled towards the most electronegative atom and
away from the other side of the molecule. Hydrogen is often at the other end of
the molecule, so it becomes electron-deficient and becomes a better electron-pair
acceptor (more acidic). An electronegative atom therefore withdraws electron
denslty and thus increases an acidic compound's acidity or decreases a basic
compound's basicity. For common atoms in organic chemistry, the order of
electronegativityyou should recallis: F > O > N > Cl > Br > I > S > C > H' This
helps you predict the relative acidity due to the inductive effect. The carboxylic
acids in Figure 1-36 show the effect of electronegativity.
oo
Noz t cl>
HO ,"\"^o\,
HH HH HH HH HH HH
PKa = 1'58 PKo =2.59 pKo = 2.88 PKa=2.92 PKa = 3.17 PKa= 4.74

Figure 1"-36

The inductive effect depends not only on the electronegativity of the


withdrawing atoms, but also on their proximity. In Figure 1.-37, the proximity of
the chlorine atom to the acidic group affects the acidity of the carboxylic acid.
The effect on the compound's acidity drops with distance and is negligible when
they are over four atoms apart. The closer the electron-withdrawing substituent
to the acid functionality, the greater the increase in the acidity.

CHg HO CHe HO cH2cl Ho


Cl H HH HH HH
PKa = 2'88 PKa= 4'07 PKa = 4'51 PKa = 4'82

Figure 1-37

It may seem strange, but alkyl groups are electron-donating. This is attributed to
the fact that hydrogen is less electronegative than carbon. When the substituent
is electron-donating, the inductive effect decreases the acidity. As a result, alkyl
groups reduce thJ acidity of a compound. In Figure L-38, the compounds
become less acidic due to the electron-donating methyl group'

o o

HoA.r ' ,to 'no\'"nu


H] HH
PKa = 3'78 PKa = 434 PKa = 4'82

Figure 1-38

Copyright @ by The Berkeley Review The Berkeley Review


itv Organic Chemistry Molecular Structure Fundamental Reactivity

Values, Terminology, and Applications


;of Conceptuai questions in acid-base chemistry can often be reworded into: "\zVhich
ni.c acid is stronger?" The answer can tell you the sign of values and the nature of a
ed, leaction. The following terms are ways in which an increase in a compound's
ron acidity can be observed. As a compound becomes more acidic:
the
Ku of the acid increases acid dissociation increases
rrd
lof :Ku of the acid decreases pH of a 1.0 M solution decreases
rair -\G for reaction with a base decreases conjugate base strength decreases
ron fKA of the conjugate base increases the stability of the conjugate base increases
sic i ou must also be able to use the qualitative concepts. A proton transfer reaction
of
his ;"rcs favorably from the side with the stronger acid to the side with the weaker
:cid; a favorable reaction proceeds from stronger acid to weaker acid. To
rlic jetermine the K"O for a proton-transfer reaction, first decide which of the acids
:l conjugate bases) is stronger. From there, it is a matter of determining the
;rection of the equilibrium. If the equilibrium lies in favor of the reactants, the
K*" < 1. If the equilibrium lies in favor of the products, the K"q t 1.
.H

Example 1.16
r\irich of the following compounds is the STRONGEST acid?
-4- H3CCH2OH
m. H3CCO2H
C F3CCH2OH
the D. F3CCO2H
'of
id. S'olution
len -::boxylic acids are stronger acids than alcohols, due to the withdrawing of
ent .!e;ton density by the carbonyl oxygen through resonance. This eliminates
::--..ices A and C. Fluorine is highly electronegative, so it withdraws electrons
::n the acidic hydrogen via the inductive effect, thus increasing the acidity.
::,e skongest acid is the carboxylic acid with fluorines attached, choice D.

H"J
Etample 1.17
i: --r,' can the difference in aciditv between trifluoroacetic acid and trichloroacetic
r-: be explained?
".4. Fluorine is larger than chlorine, so trifluoroacetic acid is a stronger acid.
$" Chlorine is larger than fluorine, so trichloroacetic acid is a stronger acid.
C lirichloroacetic acid is a stronger acid, because chlorine is more
Ito eleckone gative than fluorine.
ent D. lirifluoroacetic acid is a stronger acid, because fluorine is more
kyl e.iectronegative than chlorine.
rds
iolution
F:-rcrine is smaller than chlorine, so choice A is eliminated. The acidic proton is
:rar :,onded to the halogen, so atomic size does not dictate the acid strength. This
*:::nates choice B. Fluorine is more electronegative than chlorine, so F
Cl, making trifluoroacetic acid more
,r-'.:-i.lrarvs electron density more than
*!"-:Ton poor and a stronger acid (better electron pair acceptor). The pKu of
:'iuoroacetic acid (F3CCO2H) is 0.18, while the pK3 of trichloroacetic acid
i-irCCQH) is 0.64. A lower pKu value confirms that trifluoroacetic acid
F:CCO2H) is the stronger of the two acids. Choice D is the best answer.

rew J:rrr, right O by The Berkeley Review Exclusive MCAT Preparation


Organic Chemistry Molecular Structure Flrndamental Reactivity

Example 1.18
Which of the following statements is true as it pertains to pKu values?
A. A functional group that is electron-withdrawing by resonance lowers the
pKu value, while a functional group that is electron-withdrawing by the
inductive effect raises the pKu value.
B. A functional group that is electron-withdrawing by resonance lowers the
pKu value, while a functional group that is electron-donating by the
inductive effect raises the pKu value.
C. A functional group that is electron-donating by resonance lowers the pKu
value, while a functional group that is electron-withdrawing by the inductive
effect raises the pKu value.
D. A functional group that is electron-donating by resonance lowers the pKu
value, while a functional group that is electron-donating by the inductive
effect raises the pKu value.

Solution
Regardless of the effect (whether it is resonance or the inductive effect), electron-
withdrawing groups increase acidity and lower the pKu while electron-donating
groups decrease acidity and raise the pKu. This eliminates choices A, C, and D
and leaves choice B as the best answer. The effect of an electron-withdrawing
group can be seen in the following trend: H3CCO2H (pKa = 4.74) is less acid than
H2CICCO2H (pKa = 2.85), which is less acidic than HCI2CCO2H (pKa = 1.26),
which in turn is less acidic than CI3CCO2H (pKu = 0.64). As the amount of
electron-withdrawal increases, the acidity increases.

Example 1.19
Which nitrogen atom in the following molecule is the MOST basic?

d
NHz

NHz O
A. Nitrogen a
B. Nitrogen b
C. Nitrogen c
D. Nitrogen d

Solution
The most basic nitrogen atom is the one most capable of sharing its lone pair with
a proton or electrophile, which means that the nitrogen where the lone pair is
least shared within the molecule is the most basic. Nitrogens a and b have
reduced basicity, because the electron pair on nitrogen is being donated to the
aromatic ring through resonance. Nitrogen d has reduced basicity, because the
electron pair is being donated to the carbonyl group through resonance. Only
Nitrogen c is free to share its electrons (which are not being delocalized
""*^"r" -t,^t^ -" *

Copyright @ by The Berkeley Review 5A The Berkeley Review


ty Organic Chemistry Molecular Structure F-undamental Reactivity

Example 1.20
l\hat is observed when histidine is protonated on its side chain?
le A. The imine nitrogen gets protonated, because it is more electronegative than
te the amine nitrogen.
B. The imine nitrogen gets protonated, because it is less electronegative than the
le amine nitrogen.
le C. The amine nitrogen gets protonated, because the imine nitrogen shares its
lone pair of electrons with the n-bonds in the ring through resonance/ thus it
\a cannot be protonated.
IE D. The imine nitrogen gets protonated, because the amine nitrogen shares its
lone pair of electrons with the n-bonds in the ring through resonance, so it
\a cannot be protonated.
/e
Solution
tsoth nitrogen atoms are equally electronegative, because neither carries a charge.
This eliminates choices A and B. One could argue that having different
n- :"'bridization makes the electronegativity different, but the goal here is to find
rg -ie best answer, and if it is necessary to stretch the definition of terms, you're
D :etter off finding a better answer choice. The histidine is protonated on the imine
r8 :rtuogen, because the lone pair on the amine nitrogen is being shared with the
an :;clic rc-system through resonance. This makes the ring aromatic, so the lone
6), :air on the amine nitrogen is not available to be donated as a base. The best
of ins\{,er is choice D, as shown below.

o @
HeN H.N
oor
H CH,, H CH"
t-
4;, H*

->
Ax
)^,J
r\-j
H

\:'
@
lmine nitrogen R"ronur,ce is still possibie, so
the n-system remains aromatic.

o
t'*#,r'
Oil @
HsN

Ir---
th H CH? H CH?
l-H
7;;"
is
ve
he
\ 7 Amine nitrogen
1x,^
he
Lly
NT-
.I\- \Jz
No resonance is possible, so the
ed
n-system is no longer aromatic.

:w Copvright @ by The Berkeley Review Exclusive MCAT Preparation


Organic Chemistry Molecular Structure Fundamental Reactivity

Summary of Acids and Bases Equation hfr


In acid-base chemistry, it is impossible to avoid equations, aithough general hu6
chemistry typically involves more math than organic chemistry. Some equations -{llrt
define concepts, while others help in caiculations. Equations 1.3 through 1.10 are tu;
defining equations, and each should be tattooed in your memory bank: miln$mfr
iMM
pH = -log[H3O+] (1.3) [HaO+] = 10-PH (1.4) sMl q
,m$Mfl
pOH = -log [OH-] (1.5) pH + pOH = 14 (at25"C) (1.6) ,rm&
Ku = 16-PKu (1.8)
dM
PKa = -1og Ku (1".7')
d!
(1.e) pKa + pKb = 74 (at25"C)
PKb = -1og K6

Given pKu values, the AG" and K"O for a proton transfer-reaction can be
(1.10)
ry
mhq$h'
calculated. Listed below are four equations that you should be able to work ffi
with. Equation 1.11 is the Henderson-Hasselbalch equation. It is used to ilbttrrd
determine the pH of buffered solutions and is derived from the fundamental acid ffi
ffi@
dissociation reaction. Equations 1.11 and 1.13 are common, and should be
committed to memory: dQu
ry'F
pH=pKu+logl{l ffi
- tHA]
(1.11)
flmr
mmm
1o(Pu-pKu;- [A-] (1.12\
ffi
IHA] 'm
@
ForHA + H2O -:i+ H3O+ + A- ro=[H:-O*]-[A]
tHAI
(1.13)
w
ffi@
r4
Keq = 10Pkn(product acid) - PKa(reactant acid)

Equation 1.14 is derived from the fundamental reaction:


(1.1,4)

ry
q0
HA + B- + A- + HB, wirere K". =
[HB][A-]
lHAttB-t
By definition:

Ka(HA) = = 10-PKa(HA) and Ku1Hs,=


t"i#it = 1o-pKa(HB)
'
Using equilibrium:
,, =#F
tHBltA-l tA l lHBl =[H3o+][A] x irll
tHAltB-l tHAl tB-l tHAl lH3o+llB-l

= Ka(HA) x 1 - Ka(HA)
Ka(HB) Ka(HB)
Thus: nffinW
d
Ko. =Ka(HA) = lO-pKa(HA) = 1g-pKa(HA) * 16+praluB) = 16(pKoen)-pKa111a)) rillld
Ka(HB) 1g-PKa(HB) rnffi
ffimr
HB is the product acid and HA is the reactant acid, so by strbstitution we derive il@iifrfl'fr
Equation 1.13: M
Keq = 1gpKa(product acid) - pKa(reactant acid)

Copyright O by The Berkeley Review The Berkeley Review

i#t,,
i$ii:r
Organic Chemistry Molecular Structure Fundamental Reactivity
1
Reaction Types
'al Lr organic chemistry, there are a few fundamental reactions that describe the
NS najority of reactivity in organic chemistry. In addition to acid-base chemistry,
re there are substitution and addition reactions. All of these reactions involve the
aCdition of an electron pair to an electron-poor site. If you are able to identify the
:lectron pair that can be shared and the electron-poor site within the reactants,
,4) '"-ou will be successful at predicting chemical reactivity. Addition and
:ubstitution reactions are often further classified according to their nucleophile
,6) ,rl eiectrophile, but for now, we shall consider only a generic addition reaction to
::,iroduce reaction pathways, mechanistic logic, transition states, intermediates,
,8)
'nd energy diagrams.
0)
Electrophilic Addition Reactions
be ;-ectrophilic addition reactions are common irr alkene chemistry. The alkene
rk ::cleophile donates its most available electron pair (the electrons of the n-bond)
to :r an electrophile. The reaction involves a strong acid, often a haloacid, as the
id =-ectrophiie. If the alkene is asymmetric, then you need to consider orientation
be :.:tors in the reaction in terms of minimizing steric hindrance in the transition
':ate and stability of the intermediate. When looking at an alkene, the term
.;:'-.:,tnrctric refers to a state where the two carbons have an unequal types of
:-'-:bons bonded to each. When the reactant alkene is asymmetric, the reaction
-:,-ion's Markovnikov addition rules. The electrophilic substituent adds to the
.=ss hindered carbon of the n-bond, according to the rules associated with
l, f:rkovnikov addition.

\lechanisms (Reaction Pathways)


l,le,chanisms are the step-by-step account of all species thought to exist as
:=--tants proceed to form products. Mechanisms are commonly referred to as
':-;--;irrr pathzoays. Mechanisms focus on intermediates, propose transition states,
4) ::rl break reactions into steps. Few reactions in organic chemistry occur in one
: -:. so mechanisms often involve many steps before reaching a product. You
:-.=.,' recall from general chemistry that the rate of a reaction depends on the rate-
:=:ermining step. Whenever there is more than one step in a reaction
:re;hanism, there exists a rate-determining step.
-l:: mechanism for a reaction is proposed based on the products that are formed,
: :;-,' fast the reaction proceeds, and which changes in conditions alter the rate. It
:::. be supported by monitoring isotopically labeled atoms on a molecule. A
:.=:hanism can never be proven, only disproved. Because reaction data are
::=ed on the rate-determining steps, rate data are critical in supporting a
:ri:-hanism. Mechanisms are the most probable pathway that accounts for the
-::a presented. Most reactions in organic chemistry involve a nucleophile
r-:-;krng an electrophile, which is often referred to as nucleophilic substitution.
ie:-.re any mechanistic studies are encountered, be sure that you have a strong
;35p of the definitions of the terms.
lL= reason for thoroughly addressing a reaction using arrow-pushing technique
r. :o learn themes in reactions that are repeated in other reactions. In other
,,'.':ls, understanding mechanisms allows you to predict products of unknown
:=::tlons. For instance, if a carbocation is formed during one reaction
:,:iranism, an analogous reaction is likely to form a carbocation as well. From
--:*. perspective, almost all organic chemistry reactions can be classified as Lewis
l::i-base reactions. These include all reactions that involve a nucleophile
:::"cking an electrophiie.

:w - irlright O bv The Berkeiey Review Exclusive MCAT Preparation


Organic Chemistry Molecular Structure Fundamental Reactivity

Example L.21
A carbocation can be classified as:
A. an intermediate.
B. a transition state.
C. a product.
D. a catalyst.

Solution
A carbocation is an intermediate, because it is not stable enough to be present at
the completion of a reaction, eliminating choice C. It is not present at the start of
a reaction, and it does not form an activation energy-lowering complex during
the reaction, so it is not a catalyst. This eliminates choice D. It is present for a
finite period of time, so it is not a transition state, meaning choice B is eliminated.
It has a finite lifetime and is semi-stable, so the best answer is choice A.

Arrow-Pushing in Mechanisms
When drawing mechanisms, the atrow starts with the electron pair destined to
form a bond. If a bond is broken, then the arrow starts at the bond being broken
and finishes as either a new bond being formed, or as a lone pair on another
atom. Figure !-39 explains the arrow-pushing associated with the electrophilic
addition of hydrobromic acid, HBr, to a symmetric alkene, E-2-butene.
Step 1: There is no lone pair in the alkene, so the arrow starts from the
most available electron pair (the rc-electrons) and goes to the partially
positive hydrogen of HBr. The bond between H and Br is broken.
a^
*a^
"").l.iYg.fil -- "Jj:"3>"
H"C Bond H
, .
..o
r,t.
,/n-Bond\ Bondbrbaking /\
f{ breaking CH3 H CHs
Nucleophile Electrophile Planar carbocation Bromide anion
(e- Pair Donor) (e- Pair Acceptor) (Electron poor) (Electron rich)

Step 2: A lone pair on bromine goes to the positive carbon of the


carbocation intermediate to form a sigma bond between C and
Br. The product is an alkane with bromine (alkyl bromide).

H.C
.i_
" \^ ;f H
n

H CHa
Planar carbocation Bromide anion
":',J-'t.",
(Carbon is electron-poor) (Bromine is electron-rich) - Alkyl bromide product

Figure 1-39

Copyright @ by The Berkeley Review The Berkeley Review


Organic Chemistry Molecular Structure Fundamental Keactivity
v
Competing Mechanisms
lVhen an asymmetric alkene reacts with hydrobromic acid, there are multiple
potential products. The reaction can produce either a secondary or tertiary alkyl
bromide product. The pathway leading to the tertiary alkyl bromide has a lower
activation energy, forms a more stable intermediate, and results in the more
stable product. The pathway leading to the more stable product yields the
lhermodynamic product. The pathway with the lower activation energy yields
fie kinetic product. Some reactions may involve a competition between the
pathway leading to the thermodynamic product versus the pathway leading to
at the kinetic product. Figure 1-40 shows the treatment of an asymmetric alkene
Df
rvith hydrobromic acid.
rg
Observed Reaction
a
(Leads to the More Stable Product by way of the More Stable Intermediate)
al.

H"C

!o HsC CH: H:C CHg


II Nucleophile Electrophile 3'Carbocation Bromideanion
er (Alkene ru-bond) (Strong haloacid) stability) (Leaving group)
(3" > 2" in
ic

H.CA
'J4.}H A '\^ ;f
H.C H
u
+ .I1f. ... L- L
/\ H.c\7 \ CH:
HeC CHe : Bi
3" Carbocation Bromide anion- - S' efiyf bromide product
(Electron-poor) (Electron-rich) (More stable than 2'product )

Minor Product Reaction


(Leads to the Less Stable Product by way of the Less Stable Intermediate)

H"C A
').L.1 \*s-;ry;t-
/\
H:C CHs HeC CHs
Nucleophile Electrophile 2'Carbocation Bromideanion
n-bond)
(Alkene (Strong haloacid) stability) (Leaving group)
(2' < 3" in

H"C H Hsc. \H ..
...c- /, ...+ \_
-..
lP'':
\,
H$t"/ \ H\t"'/ \
H:C CHs HsC CHs
2'Carbocation Bromide anion 2" Alkyl bromide product
(Electron-poor) (Electron-rich) (Less stable than 3" product )

Figure 1"-40

)w Copyright @ by The Berkeley Review Exclusive MCAT Preparation


Organic Chemistry Molecular Sllrrclrrre Fundamental Reactivity

Energy Diagrams
An energy diagram accounts for the energy of the system as a reaction proceeds.
As energy is added to the system (to break bonds or reorient the molecule to a
less stable structure), the energy goes up. As energy is released from the system
(upon making bonds or reorienting to a more stable structure), the energy goes
down. The overall difference from start to finish represents the energy change
for the reaction (either AG or AH.) The diagram reaches an apex with a transition
state, and a localized nadir in the middle with an intermediate. The start
represents the energy of the reactants, and the end represents the energy of the
products. Each apex represents a transition state in the reaction, which results in
a step in the reaction, Figure 1-41 shows the two energy diagrams associated
with the two possible reactions when adding HBr to an asymmetric alkene.

T
2(3')

Products
2": (H3C)2CHCHBTCH3
3': (H3C)2CBICH2CH3
>'
bo
rr Reactants
o /
(H3C)2C=CHCHa + HBr AG"12.; = -7.63kcar mole
aGr(3") = -8.77kcar /
AJ
O)
li mole
trr

Reaction coordinate

Figure 1-41

Energetics and Kinetics


Will a reaction proceed or not? This question deals with thermodynamics
(overall energy of the process) and kinetics (overall rate of the process).
Reactivity can be analyzed as follows:
1. Identify and label the reactants (in most cases as nucleophile and
electrophile).
2. Predict the products of the reaction. If the reaction occurs in multiple steps,
predict products for each step.
3. Show the flow of electrons in each of the steps. This is known as drawing the
mechanism.
4. Evaluate whether the reaction is overall thermodynamically favorable. AG"
is the standard free energy. When negative (AG" < 0), the reaction is
favorable in the forward direction. When AG' is negative, Ksq (the
equilibrium constant measuring the ratio of products to reactants) is greater
than 1 (K"q > 1). Because AG" = AH'- TAS", and most organic reactions take
place in sblution where entropy is relatively unchanging, AH" can be
approximated from AG". In conclusion, to determine whether a reaction is
thermodynamically feasible, look at O6', KeO, and AH".

Copyright @ by The Berkeley Review 44 The Berkeley Review


ty_ Organic Chemistry Molecular Structure Fundamental Reactivity

i. Evaluate whether the reaction can proceed at the given temperature. There
s. are favorable reactions that never take place, because the activation energy
a (E6s1) is too great. When looking at reactions, there is AGX to consider, the
n activation free energy for each step. There must be enough free energy
}S present that a reasonable number of reactants can overcome the activation
!e barrier. This factor is hidden in the rate constant (k.*), which takes into
rn account the frequency of collision, the temperature, and the orientation of
rt molecules during collision. In conclusion, look at: T (high temperature
te equals fast reaction), Eact (low Eagl equals fast reaction), and intermediates
n (stable intermediates equal fast reaction).
d
Effect of Temperature on Reaction Rate
The rate of a reaction increases as temperatute increases, because there is more
=ee energy available in solution for reaction. Temperature is part the rate
:onstant, kr*, mathematically expressed Equation 1.15.

k.* = A (1.1s)
"-Eut'/RT
-,q-here A is the Arrhenius constant and E661 is the activation energy.

The energy diagrams in Figure 1-42 show the change in energy level as the
:eaction proceeds and Figure 1-43 shows the molecular energy distribution
iroughout the entire system at two different temperatures (T1 and T2).
state (f)

>'
bD
t+
q.)

rl]
Reaction coordiante

Figure 1,-42

TztTr Minium Eo.1 for reaction

s Kinetic energy
e
tr Figure 1-43
e
e At T2, the average kinetic energy of the molecules is greater than it is at T1. Thus,
6 at T2, more molecules have enough energy to overcome the activation energy
'rarrier
of the reaction.

Copyright @ by The Berkeley Review 4s Exclusive MCAT Preparation


Organic Chemistry Molecular Structure Physical Properties j
Physical,, Propefties,
Physical Properties and Intermolecular Forces
It is important to have an understanding of how molecules interact with one
another. By understanding these interactions, it is easier to predict what will
take place in a chemical reaction or physical change, CommonMCAT questions
involving intermolecular forces include determining the boiling points of two
related compounds, as well as a compound's solubility propeitils or melting
point. The rule is simple: the greater the forces, the higher the boiling point. we
will use relative boiling points to verify the relative intermoleCular forces
between compounds. The following are intermolecular forces that affect the
boiling points,listed in descending order of strength.
Hydrogen bonding
Hydrogen bonding is a weak bond (approximately 4 to g kcals/mole) that exists
between a lone pair of electrons and a hydrogen that carries a substantial partial
positive charge. A hydrogen has a substantial partial positive charge when it is
bonded to a small electronegative atom such ai nitrogen, o*yg".r,-o, fluorine.
There are no hydrogen bonds involving hydrogens thai are covalently bonded to
carbon! You should be aware that not all hydrogen-bonds have the same
strength. For instance, an amine lone pair binds the protic hydrogen of an
alcohol more tightly than an alcohol lone pair binds the protic hydro-gen of an
amine. The strength of a hydrogen bond can be estimated from thu bur"
properties of the lone pair donor and the acid properties of the hydrogen donor.
PoIar lnteractions
Polar interactions are the Coulombic interaction between partially charged
particles (approximately 1 to 3 kcals/mole). Negatively chirged sites attiact
positively charged sites. The greater the partial charge on the site of the
molecule, the stronger the force between opposite chargei. The strength of the
force also increases as the distance between oppositely charged sites d"creases.
A typical example of a polar interaction is the dissolving of ions and polar
species into water.
Van der Waals
van der waals forces exist between all compounds. van der waals forces are
considered only when no other forces exist to any extent. They are the result of t
the attraction between temporary dipoles (a very weak force). They are the ft
weakest of the three intermolecular forces between molecules that we shall m
consider. They are considered to be less than 1 kcal/mole.
W
The intermoiecular forces are the primary consideration when approximating
O!
physical properties, when forces are not enough to determine ihe physical ,*
properties such as boiling and melting point, then structural features such as
molecular mass and molecular rigidity become the determining factors. The
heavier the compound, the harder it is to remove it from a lower energy phase
and place it into a higher energy phase (i.e., liquid to gas). what is meJnt by
"molecular flexibility" is the ability to twist and conform to allow for more
surface area, and thus more intermolecular interactions. Van der Waals forces
are rarely used to explain anything except why there is not a complete absence of
intermolecular force. Figure L-44 shows the different forces.

Copyright @ by The Berkeley Review 46 The Berkeley Review


Organic Chemistry Molecular Structure Physical Properties

Hydrogen-bonding:

gu
H
-rr.
HeC -?o
\
I

CHs H

Dip ole - Dip ole I nte ractio n :

F_ kry
T.)a%'"
H3CV'06- H
6+l
HaC

Van der Waals interactions:

Tr--$T,{,
H
:-.$e?.
H

Figure L-44

F-xample 1.22
'ir{-hv is methanol (CH3OH) a liquid at room temperature, while ethane
CII3CH3) is a gas?
3" Ethane is more polar than methanol.
E- Methanol is significantly heavier than ethane.
C Methanol has stronger van der Waals interactions than ethane.
D. Methanol has hydrogen-bonding, while ethane does not.

Solution
!fiethanol (CH3OH) is a liquid at room temperature, while ethane (CH3CH3) is a
Sas, so methanol has the greater boiling point. Ethane is a nonpolar molecule, so
Cpice A is eliminated. Methanol has a molecular mass of 32 grams/mole, while
e$lane has a molecular mass of 30 grams f mole, so methanol is only slightly (not
significantly) heavier than ethane, so choice B is eliminated. The van der Waals
rnEractions are roughly equal for all molecules, so choice C is eliminated.
hfiethanol has hydrogen-bonding, while ethane has no protic hydrogen and
Sr,erefore no hydrogen-bonding. The higher boiling point is due to the
hr-drogen-bonding of CH3OH, so choice D is all yours!

Copvright O by The Berkeley Review Exclusive MCAT Preparation


Organic Chemistry Molecular Structure Physical Properties

Hydrogen-Bonding
Hydrogen-bonding is the strongest of the common intermolecular forces. It can
be thought of as a weak covalent bond between a hydrogen that carries a partial
positive charge and the lone pair on a nearby atom. The strength of a hydrogen
bond varies between 4 and 8 kcals per mole. Hydrogen bonds are similar to the
interaction of a base with an acidic proton in the transition state of a proton
transfer reaction. This is where the term protic comes from, as a hydrogen
capable of hydrogen-bonding is also slightly acidic. The partial positive on
hydrogen is strong enough to form hydrogen bonds when the hydrogen is
bonded to either fluorine, oxygerL or nitrogen (highly electronegative atoms).
The strength of the hydrogen bond is best approximated by the acidity of the
proton and the basicity of the lone pair donor. The strongest hydrogen bond
exists between hydrogen on fluorine and a lone pair from nitrogen. Hydrogen
bonding in alcohols is stronger than in amines, as supported by the greater
boiling points of alcohols relative to amines with comparable mass. The best
explanation for this is the greater acidity of the protic hydrogen of an alcohol
than an amine. compounds that form hydrogen bonds are polar, so when a
compound has hydrogen-bonding, it also has dipole-dipole interactions. This
means that when you are asked to compare boiling points of compounds, you
should first look for hydrogen-bonding.
Figure 1-45 shows the structures and boiling points of butanol and butanal.
Butanol is capable of hydrogen-bonding, while butanal is not. For this reason,
the forces between butanol molecules are stronger than the forces between
butanal molecules. The result is that butanol molecules bind one another more
tightly, resulting in a higher boiling point. This comparison of the two is
reasonable, because the two molecules are of roughly equal mass.

OH o

/ \//
I

,,^-'At
Butanol (b.p. = 717.4' C) Butanal (b.p. = 76.1,'C)

Figure 1-45

Example 1.23
Which of the following compounds has the HIGHEST boiling point?
A. (H3C)3N
B. (H3C)2NH
C. (H3C)3CH
D. H3COCH3
Solution
This question centers on intermolecular forces, particularly hydrogen-bonding.
As a rule, the compound with the greatest intermolecular forces has the highest
boiling point. Hydrogen-bonding is the strongest of the intermolecular forces,
and if a compound has hydrogen-bonding, it also has dipole-dipole interactions,
so the compound with hydrogen-bonding has the greatest intermolecular forces.
To form a hydrogen bond, both a lone pair of electrons and a hydrogen on a
highly electronegative atom (N, O, or F) are required. Choice C does not contain
a lone pair, so it is eliminated. Choices A and D have all of their hydrogens on
carbon, so they are both eliminated. This leaves choice B as the only molecule
that forms hydrogen bonds.

Copyright @ by The Berkeley Review The Berkeley Review


gs Organic Chemistry Molecular Structure Physical Properties

Folarity
ln ?olarity is defined as an asymmetric distribution of electron density within a
ai :roiecule. Perhaps an easier way to think of this is as a tug-o-war for the
ln =-ectrons between atoms. The more electronegative atoms pull the electrons
1e ::ore tightly. If the molecule has more of one atom on one side of the molecule
)n -:ra-n another--that is, if it is asymmetric about a ceniral point-- then it is polar.
tn :-:ure 1-46 shows a series of chlorinated methane derivatives that demonstrate
)n ::-arity (or lack of polarity) based on structure.
is
I
H CI H H CI

:IE I I I I I

rd
ln
er
st
l'--.npoiar Polar Polar
'/Y,.'ct/V.
Polar Nonpolar
ol
Figure 1-46
a
ris
-. : --iar compound has a dipole, which for a1l intents and purposes is a line
)u
:::,1 r'r form the positive side of the molecule to the negative side of the molecule
ri- : i".-av that sums up the polarity vectors of each bond in the molecule. The line
il. rL'-:ates the direction in which the electron density shifts. Figure 7-47 shows the
n/ m: from Figure 7-46, wlth dipoles now drawn in.
ln 'e;ules
re
is F{ H C1

I f' I T I

,r'"{'Jn H .ztyrn H CI/IV:, ct o./Y.,


'',,: l:O1af Polar Polar Nonpolar

Figure 1,-47

-r,e :,:elaction of the dipoles between two nearby molecules accounts for a weak
r'ur,:: l-- shown in Figure 1-44. The alignment of dipoles is best when the partial
J,"$itr-- e of one molecule aligns near the partial negative of another molecule.

{x.mrnie 1.24
:i the following molecules has a dipole moment of zero?
rL,r,'tu::

4.lr* -::ion monoxide


il" -l::*oromethane
: -:-Dchloropropane
I l:':r.-1,1-dichlorocyclohexane
$uluruc.ou-r
1r :a ^: a dipole moment of zero, the molecule must be symmetric. Carbon
:ru n r,:, :ie i C=O), dichloromethane (see Fi gure 1,- 47 ), and 2,2- dichlor oprop ane are
r:r-;-=h'x; and thus polar. The trvo carbon-chlorine bonds of trans-1,4-
.sil$r

lLr:-:r!:::.:i'clohexane are on opposite sides of the molecule, symmetrically


lrsrJ:,i-n,i. about a central point in the molecule, so their dipoles cancel out.
-]ilu "L':t D rs the best answer.

:w l*um-nr,,::.=-- C by The Berkeley Review 49 Exclusive MCAT Preparation


Organic Chemistry Molecular Structure Physical Properties trg
Van der Waals Interactions Solub
Van der Waals forces are weak intermolecular forces that exist between all Sneubd
molecules' These weak attractive forces account for the minimal attraction brmscift
between hydrocarbons. In biochemical discussions of hydrophobic interactions, ln*e s[
it is in fact van der Waals forces that are being considered. maFrfff,
Figure 1-48 shows why lard (a saturated fat) has a higher melting point than hu'}"e0
vegetable oil (an unsaturated fat). Because the molecules in laid are more *oto,.{ Src

flexible, they are better able to interact with another molecule (tie knots around mrMl
another molecule, if you will) than is vegetable oil. Perhaps it is easier to picture lmaru
lard as a pile of strings that can tie knots around themselves, while vegetible oil he$p
is like a pile of straws. If you were to build two separate piles, one a pile of rwmnmrhi

strings and the other a pile of straws, then intertwine each within itself, from lx.{ryd
which would it be easier to remove a piece? It would be far easier to remove a dmpdl
straw from the straw pile, because the straws are not tangled up. Their rigidity ffir.mmm

prevents interactions between the straws. This is why saturited fats 1*hose
molecules are flexible like strings) are solid at room temperature, while
unsaturated fats (whose molecules are rigid like straws) are llquid at room
temperature. The greater the number of rc-bonds in a compound, the lower its
tr
f,'*rmt
il
melting point, and the greater the odds it is a liquid at room temperature. The n-
bonds in the fatty acids in a phospholipid bilayer affect the fiuidity of a cell
membrane.
ill*
ffi-
t" ffig"-
Vegetable OiI (More rigid structure) Lard (More flexible structure)
,IMffi
Figure 1-48 @fu
s
Example 1.25 @

Cell membranes are composed of many molecules, including phospholipids. A {a


phospholipid is a molecule with a glycerol backbone (HocH2-cH(oH)CH2oH), s
plus two fatty acids, and a phosphate attached to the oxygen atoms of glyceroi.
*"n
A cell membrane would be most rigid if both of its fatty acids were:
A. completely saturated and short molecules. il
B. completely saturated and long molecules. 0

C. uqsaturated and short molecules. mhi


D. unsaturated and long molecules. mm
Solution
For the membrane to be rigid, there must be many interactions between the
d
lipid
chains. The maximum degree of interaction occurs with long, saturated fitty
acids. Pick B and feel good yet again.

Copyright O by The Berkeley Review 50 The Berkeley Review


Organic Chemistry Molecular Structure Physical Properties

Solubility and Miscibility


Solubility is defined as the ability of a solute (solid) to dissolve into solution.
Miscibility describes the ability of a liquid to mix (dissolve) into another liquid.
We shall look at both of these abilities in terms of physical properties and
intermolecular forces. The intermolecular forces of greatest concern here are
hydrogen-bonding and polarity. The basic rule of polarity governing miscibility
and solubility is that like dissolaes like. This means that a polar molecule dissolves
most readily in a polar solvent, and a nonpolar molecule dissolves most readily
in a nonpolar solvent. Miscibility and solubility can be used as diagnostic tests to
help determine the identity of an unknown substance. There are three
combinations of properties that a solvent may have. It may be polar and protic
(capable of hydrogen-bonding), polar and aprotic (no hydrogen-bonding, but has
dipole-dipole interactions), and nonpolar and aprotic (weak intermolecular
forces). Types of solvents and their properties are described in the Table 1-9.

Type Intermolecular Forces Examples


H-bonding, dipole-dipole,
Polar, Protic Water. Alcohols
and van der Waals
Dipole-dipole and van
Polar, Aprotic Ketones, Ethers, Alkyl halides
der Waals

Nonpolar, Aprotic van der Waals Oils, Petroleum

Table 1-9

The followrng three solubility observations can be explained by the solubility


rules in Table 1-9.
Salts dissociate into water, because ions are stabilized by the protic
nature of water.
Sugar dissolves into alcohol, because of the large amount of hydrogen
bonding.
Wax dissolves into oil, because it is entropically favorable to randomize.
Individual van der Waals interactions are weak; but over a long
molecule, they quickly become significant.

It is important that you have a good understanding of which common solvents


have which properties. The solvent properties come into play when dealing with
chromatography and extraction. In chromato graphy, the greater the solubility of
a solute in the solvent, the greater the affinity of the particle for the mobile phase,
meaning it can travel farther and faster. In extraction, solutes are separated from
one another by their relative solubility (or miscibility) in two solvents. The
difference in solubility is attributed to functional groups on both the solvent and
soiute. Micelles can be employed to increase the apparent solubility of a solute in
a solvent in which it is said to be insoluble.

How can a nonpolar particle dissolve into water? Soaps help to make a
nonpolar, aprotic species dissolve into water. For a soap (surfactant) to do this, it
must be both hydrophilic (water-soluble) and hydrophobic (water-insoluble)
simultaneously. Such molecules contain a polar (or charged) end (referred to as
the head) and a alkyl chain (referred to as the tnil). Originally, soaps were made
by treating animal fats with strong base to convert the ester to a carboxylic acid,
and then further convert the carboxyiic acid to carboxylate (its conjugate base), a

Copyright @ by The Berkeley Review 5l Exclusive MCAT Preparation


Organic Chemistry Molecular Structure Physical Properties o
species with an organic tail and a charged head. soap molecules form micelles
when placed into water. Micelles are little pockets (spherical in shape) with an IUo
organic core and polar heads sticking out from the core to interact with the
water. They are shuttle pods for nonpolar, aprotic species through water. m_

How do candles burn? This question requires an understanding of how phase


changes with temperature. Is it the solid form or the liquid form of the wax that
burns? what is the melting point of the wax in a candle? Is heat distributed
evenly in a candle (i.e., is it a thermal conductor or a thermal insulator?) These
questions all ask about the physical properties of wax, a conglomerate of carbon
chains usually containing between 31 and 50 carbons. As heat is applied, wax
melts to form an oil. The melting point for wax i.s greater than room
temperature, but less than the temperature of the flame. The oil and vapor that f,o
form burn when exposed to heat from the flame and to oxygen. only the oil near 1*
the flame burns, so heat is not evenly distributed in the system. The workings of
a candle can be explained using fundamental principles in science. euite often,
the simplest things in life are organic chemistry in action,

Example 1..26
\Alhich of the foiiowing compounds would make the BEST micelle?
A. H3C(CHz)gCOzH
B. H3C(CH2)3CO2-
C. H3C(CH2)1aCO2H
D. H3C(CHz)tqCOz-
Solution
The best micelle has an ionic (charged) head and a long carbon chain for the
organic tail. Choices A and c are eliminated, because they have uncharged
heads. Although a carboxylic acid group is polar and protic, a charged site is
better, because it is more hydrophilic when charged. Choice D is better than
choice B, because it has a longer organic tail. Pick D to score big on this question.

At
L

E{
tn"-

Copyright O by The Berkeley Review 52 The Berkeley Review C{


Organic Chemistry Molecular Structure Section Summary

Key Points for Molecular Structure (Section 1)


-\omenclature
1. IUPAC Nomenclature (Names are assigned systematically based on functional
groups and carbon chain length)
a) Name Root (Assigned according to the longest chain)
b) Suffices (Assigned according to functional group--generally, the most
oxidized functional group gets top priority)
c) Prefixes (Assigned to note stereochemistry; i.e., R or S, E or Z, and a or B)
d) Common Nomenclature (Prefixes based on substitution and relative
positions of functional groups; i.e., geminal diol and secondary alcohol)

Bonding and Orbitals


1. Bonding (An attractive interaction between neighboring atoms)
a) Covalent Bond (The sharing of electrons between atoms; carbon makes
covalent bonds)
i.Single bonds are made of a sigma-bond; they are weaker than both
double and triple bonds.
ii. Double bonds are made of one sigma-bond and one n-bond; they are
weaker than triple bonds, but stringer than single bonds.
iii. Triple bonds are made of one sigma-bond and two n-bonds; they are
stronger than both single and double bonds.
iv. Sigma-bonds share electron density between nuclei while n-bonds
share electron density in the plane above and below the nuclei.
b) Molecular Orbitals
i. Like atoms, bonds have electrons in regions of high probability, so
there are sigma and pi orbitals to describe molecular bonds.
ii. Sigma bonding orbitals are more stable than pi bonding orbitals, while
sigma anti-bonding orbitals are less stable than pi anti-bonding
orbitals. They fill 62n4v*46*2
iii. Anti-bonding orbitals have no overlap between atoms
c) Structural Rules (Atoms obey predictable behavior when making bonds)
i, Octet Rule (Atoms in the second row of the periodic table seek to
complete their valence shell by obtaining eight electrons.)
ii. HONC Rule (In neutral molecules, H makes one bond, O makes two
bonds, N makes three bonds, and C makes four bonds. If an atom
deviates from these values, it carries a charge.)

Hybridization
1. The mixing of atomic orbitals (s and p in organic chemistry) to form hybrid
orbitals capable of combining to make molecular orbitals.
a) sp-hybridrzation results in linear compounds, often with two ru-bonds, a
180" bond angle, and the shortest of all hybrid orbitals.
b) sp2-hybridi"ation results in trigonal planar compounds, often with one n-
bond, a 120" bond angle, and a medium sized hybrid orbital.
c) sp3-hybridlzation results in tetrahedral compounds, with no n-bond.s, a
109.5'bond angle, and the longest of all hybrid orbitals.

Bond Dissociation Energy


1. The energy required to break a bond in a homolytic fashion (into free radicals)
is the bond dissociation energy. Higher BE refers to a stronger bond.
a) BDE depends on the atoms, the substitution, and electron distribution
b) Ionic bonds, rare in organic chemistry, break in a heterolytic fashion.

Copyright O by The Berkeley Review 5t Exclusive MCAT Preparation


Organic Chemistry Molecular Structure Section Summary

Intramolecular Features
1. Forces Affecting Electron Distribution within a Molecule
a) Resonance (Sharing of n-electrons through an array of p-orbitals)
i, Most stable resonance structure has an octet for all atoms but
hydrogen, minimal charged sites, and if there must be a charge, it sits
on an atom of appropriate electronegativity.
ii. Also know as conjugation when dealing with just n-bonds
iii. Atoms with lone pairs are generally electron-donating while atoms
with a n-bond and no lone pair are generally electron-withdrawing.
b) Inductive Effect (The pull of electron density through sigma bonds)
i. Depends on electronegativity of atoms
ii. Diminishes over distance, becoming negligible after four carbons.
c) Steric Hindrance (Repulsion of two atoms at the same location)
d) Aromaticity (Stability for cyclic systems with a set number of n-electrons)
i. Must contain a continuous cycle of overlapping p-orbitals
ii. Must have 4n + 2 n-electrons, where n is 0, 1, 2, etc... (Hrickel's Rule)
Fundamental Reactivity
1. Organic Chemistry at it foundation is Lewis Acid-Base Chemistry
a) Acid-Base Chemistry
i. Bronsted-Lowry deals with proton transfer while Lewis deals with the
accepting and donating of electron pairs.
ii. Electron pair donors are Lewis bases and nucleophiles, while electron
pair acceptors are Lewis acids and electrophiles.
b) Determining Acid Strength (stronger acids have lower pKu values)
i. Primary factors affecting acid strength are the size, hybridization and
electronegativity of the atom to which H is bonded.
ii. Secondary factors affecting acid strength are resonance and induction.
iii. Base strength goes in the opposite fashion as an acid.
c) Acid and Base Terminology and Facts
t. Acids: pH = -log[H3O+], [H3O+] = 10-PH, pKa = -log Ka, K6 = lQ-pK2
t1, As acid strength increases: 1) acid dissociation increases, 2) Ku
increases, 3) pKu decreases, 4) pH in an aqueous solution decreases,
and 5) conjugate base strength decreases and stability increases
111. Bases: pOH = -log [OH-], [OH-] = 10-POH, pKb = -Iog K6, K6 = 1g-pK6
iv. As base strength increases: 1) base hydrolysis increases, 2) KU
increases, 3) pKU decreases, ) pH in aqueous solution increases, and 5)
conjugate acid strength decreases and stability increases
d) Mechanisms involve tracking the pathway of electron transfer

Physical Properties
1. The physical properties of a compound are affected by intermolecular forces
a) Hydrogen Bonding (Occurs between H on N, O, or F and a lone pair)
i. Increases boiling point and melting point by increasing attraction.
b) Polarity (Interaction between dipoles of adjacent compounds
i.
Polar interactions are weaker than a hydrogen bonds
c) Van der Waals Forces (Weak interaction between temporary dipoles)
i. Small impact on physical properties
d) Solubility and Miscibility (Ability to dissolve into a solvent)
i. Based on the idea that "Like dissolves like."
ii. Solid into solvent is solubility while liquid into solvent is miscibility
Copyright @ by The Berkeley Review 54 The Berkeley Review
Stmcture, Bonding,
and Keactivity
Passages
15 Passages
I OO Questions

Suggested Structure, Bonding, and Keactivity Passage Schedule:


I: After reading this section and attending lecture: Passages I - III & VI - VIII
Grade passages immediately after completion and log your mistakes.
II: Following Task I: Passages IV V & IX, (20 questions in 26 minutes)
Time yourself accurately, grade your answers, and review mistakes.
III: Review: Passages X - XIII & Questions 92 - IOO
Focus on reviewing the concepts. Do not worry about timing.

,1fu
REKI{ELEY
L)R E.v.r.n.w'
Speciahzrng in MCAT Preparation
betn
engt
is to
and
I. Bond Dissociation Energies (l -8) carb
ofb
II. Structure of Caffeine (e - 14) inao

I
bood

III. Solubility of Dyes and Soap (rs - 2L)


IV. Amide Protonation (22 - 28) It'c
V. l"i
VI.
Physical Properties and Intermolecular Forces
Molecular Structure and Polarity
(2e - 34)
(55 - 42)
E
lnrc
fut
VII. Micelles (43 - 4e) lEtr
l'"r.
VIII.
IX.
IR Determination of 1", 2', and 5"Alcohols
PetroleumDistillation
(5o - 56)
(57 - 63)
E
I,nrc
t**.
X. Acidity and Hybridization (64 - 70) l'"r.
XI. Ksq and Acidity (7r - 77) @
XII. Alkoxides and Alkyl Sulfides (78 - 84) T
guury
XIII. Nucleophilicity and Basicity (85 - el) Thcrt
@
Questions not Based on a Descriptive Passage (92 - lOO) ThErc
wrqE
TanilE
Structure, Bonding, and Reactivity Scoring Scale 0fbil
Kaw Score MCAT Score rm a&

84 - 100 l5 l5 lb'@
ilm ilE
66-43 10 l2 &s,
47 -6s 7 -9
34 46 4-6
t-53 t-3
Fassage I (Questions 1 - 8) The presence of an alkyl group on an alkene strengthens
its rc-bond. Alkyl groups on vinylic carbons are considered
I ,\s an approximation when determining the enthalpy of to be electron donating, so fi-bonds must be electron
**n;tion fiom bond energies, it is assumed
that a bond acceptors.
rri'aeen two atoms has a fixed value for its bond dissociation
=
I
sme'rsv, regardless of the substituents on the molecule. This I. What bond dissociation energy would you expect for the
$ :,J say that one assumes a covalent bond between carbon bond between carbon-l and hydrogen and the only
nr,i iodine always has the same bond energy, whether the carbon-carbon single bond in H-C:C-CH:?
;*rron is a teftiary or primary carbon. A more critical view
rr :ond energies, however, shows this assumption to be
A. C1-H 92 kcal/mole; CZ-CZ 86 kcal/mole
l:rlru;urate. Table I lists a series of energies for common
B. C1-H 1 16 kcal/mole; C2-C3 86 kcal/mole
C. C1-H 92 kcallmole; C2-C3 110 kcal/mole
rmml: in a wide range of organic molecules.
D. C1-H I 16 kcal/mole; C2-C3 1 10 kcal/mole
BE 1-keal-; Bond BElleel;
mole mole
H3C-CH3 88
H5C2-CH3 85 2. Bromine would make the STRONGEST bond with
(H3C)2CH-CH3 84 which type of carbon?
(H3C)3C-CH3 81 A. Methyl
H2C=CH-CH3 91
B. Primary
HrC-H C. Secondary
t04 D. Tertiary
H5C2-H 98
(H:C)zCH-H 95
(H:C)rC-H 91
ffilJ:C-I H2C=CH-H 108 3. What can be concluded about the relationship between
H3CO-H t02 atomic size and bonding?
H5C2O-H 103 A. Smaller atoms form longer, stronger bonds than
larger atoms.
Table 1
B. Smaller atoms form longer, weaker bonds than
T-l: r'alues in Table i
demonstrate the effect of alkyl larger atoms.
g{lrum$ -rn neighboring atoms and the bonds that they fbrm. C. Smaller atoms form shorter, stronger bonds than
^i'brmry s a correlation between carbon substitution and bond larger atoms.
1mruilu:- ls rvell as between atomic size and bond strength. D. Smaller atoms form shorter, weaker bonds than
lllhmm ,s also a correlation between hybridization and bond larger atoms.
itimmrsfir- L'ut it is not substantiated by the limited data in
Tliutulre :" nhich presents too few examples of varying degrees
mi m.uqrrir;lzation to reach a solid conclusion about the effect of
fllilril$mflnliilriiion on bond strength. The effect of substitution on
,1111p rtiil[,,rrrc on bond strength can also be evaluated using bond
4. The GREATEST amount of energy is released by the
oxidative cleavage of an alkene that is:
rflrilllfiHrHmr-i. Table 2 lists the enthalpy of reaction for the
iltltlltrNflrurgfl:r.tion reactions of various alkenes. The dilferences A. unsubstituted.
irilttil t'r,fri r!611-ts5 tbr the variOus hydrogenation reactions are due B. monosubstituted.
rui' lm* {re.Jt of alkyl groups on the strength of a n-bond. C. disubstituted.
D. trisubstituted.
Alkene AH lteal;
H2C=CH2 -32.6
RHC=CHz -30.2
5. The difference in enthalpy ofhydrogenation between the
cis-RHC=CHR -28.s cis and trans alkenes can be attributed to a difference in:
R2C=CH2 -28.3
A. resonance.
trans-RHC=CHR -27.4 B. hybridization.
R7C=CHR -26.1 C. the electronegativity of carbon.
R2C=CR2 -26.4 D. steric hindrance.

Table 2

J;nrlght @ by The Berkeley Review@ GO ON TO THE NEXT PAGE


6. The hybridization of the carbons in H2C=CH-CH: Passage ll (Questions I - 14)
can best be described as:
Caffeine, a drug extracted from tea leaves, coffee beans,
A. sp, sp, and sp2.
and cacao plants, exists in two forms, depending on the pH at
B. sp, sp,andsp3.
which it is processed: the neutral (freebase) form or the
C. tp3, sp3, and sp2.
protonated (acid salt) form. Caffeine has medicinal uses as a
D, tp2, sp2, and sp3.
stimulant and as a coagulant. For years, beverages
containing this drug have been popular for their stimulant
effects and in some cases flavor. Caffeine can be extracted
from tea leaves by employing acid-base extraction using
standard extraction techniques on a pulverized conglomerate of
7 . Which of the following single bonds is the strongest? the leaves. Caffeine isolated from the leaves in this way is
A. H3C-I relatively pure.
B. H3C-CI Once extracted, either the acidic form or basic form of
C. (H3C)2CH-I caffeine can be isolated. Both forms are air-stable in their
D. (H3C)2CH-C1 crystalline solid state and in solution, but they exhibit
different physical properties due to the ionic nature ofthe acid
salt form. The acid salt form is more water-soluble and has a
higher melting point than the free-base form. The acid salt
form of caffeine can be converted into the free-base form by
treating the acid salt with a strong base. Equally, the free-
8 . The GREATEST amount of energy is required to break
base form can be converted into the acid salt form by treating
which of the following carbon-carbon bonds?
the free base with a strong acid. Drawn in Figure 1 are the
A. H3C-CH3 free-base and hydrochloric acid salt forms of caffeine.
B. (H3C)3C-C(CH3)3
Acid salt
C. H2C=CH2 Freebase

D. (HgC)zC=C(CHl)z .\
HrC o
-\
HrC o cl-
N '\*--CHr N \*z cH:
( (
N *Ao +N
I *Ao
I H I
CH: cHr
Figure L Freebase and acid salt forms of caffeine

Because the reactivity of caffeine varies with its form,


predictions about its reactivity must be based on the pH of its
environment. The conjugate acid form of caffeine has a pK6
value that is slightly higher than 6, making caffeine a weak
acid. It exists primarily in its deprotonated form at a pH of 7
in an aqueous solution. The extraction of caffeine from tea
leaves can be carried out using vinegar to lower the pH of
aqueous layer and using an ether solvent (organic layer) as the
second layer of the biphasic system. The caffeine cation
dissolves into the aqueous layer, while the other orgi
components of the leaves dissolve preferentially into
organic layer.

9 , The acid salt form of caffeine can be converted to


free-base form most readily by adding which of
following reagents?
A. HCI
B. NaCl
C. CaCO3
D. NaOH

Copyright O by The Berkeley Review@ GO ON TO THE NEXT PAGE


I 0. What is the hybridization of the imine nitrogen of the 14. The compound formed by replacing the N-CH3 group
purine ring that is protonated in the acid salt form of between the two carbonyl carbons in caffeine with an O
beans, caffeine? atom is classified as an:
rpHat A. rp
or the A. acidanhydride.
9SASA
B. tp2 B. acid ester.
)rages c. tp3 C. ester.
nulant D. Nitrogen atoms do not exhibit hybridization. D. lactone.
racted
using
rrate of
way is

rrm of
n their i 1. Which of the following strucrural descriptions BEST
:xhibit describes the relationship between the four nitrogens in
re acid caffeine?
Jhasa A . Perpendicular planar
rid salt B. Coplanar
rrm by C. Tetrahedral
e free-
D . Inverted planar
reating
ue the

cl-

- CH3 I L The length of the carbonyl bonds (C=O) in the caffeine


molecule are BEST described by which of the following
sLatements?
'o
A . Both C=O bonds in caffeine are longer than the
I
C=O bond in cyclohexanone. ll

e
B. Both C=O bonds in caffeine are shorter than the :'

i
C=O bond in formaldehyde.
;form, C . Both C=O bonds in caffeine are shorter than the
C=O bond in carbon dioxide.
{ of its
a PKa D . All C=O bonds are of equal length, regardless of the
compound.
r weak
rHofT
om tea
tof the
) as the
cation
rrganic
rto the Compared to the acid salt form of caffeine, the melting
point of the free-base form is:
A. higher, because it is the more polar form.
l to the
B. lower, because it is the more polar form.
of the
C. higher, because it is the less ionic form.
D. lower, because it is the less ionic form.

IAGE O by The Berkeley Review@ GO ON TO THE NEXT PAGE


Passage lll (Questions 15 - 21) 1 6. Which of the following would be the BEST solvent to
remove an ink dye that has hydroxyl (OH) groups
Many common household items are the products of basic attached to a carbon backbone?
organic chemistry. Dissolving one or more colored dyes into
A. Propanone
a volatile organic solvent, such as isopropanol, for instance,
makes ink. Paint, like ink, is the combination of a dye and a
B. Propanol
solvent. When ink is applied to a porous surface such as C. Propanal
paper, the pores of the material absorb the solution. Then as D. Propanoic acid
the volatile organic solvent evaporates away, the solid dye is
left bound to the pores of the material. This is why ink can
smear when initially applied, but once it has dried (once the
solvent has evaporated away), the ink does not smear.
17. What is the IUPAC name for the following compound?

It is possible to remove dried ink from paper by treating CH 3 CH 2 C H 2CH 2CH 2CH2CH2CH2CH 2C O 2H
it with organic solvent. A problem with this method is that A. Nonionic acid
the solvent diffuses radially across the paper, taking the B. Decanoic acid
dissolved dye with it as it travels. This is commonly referred
C. Undecanoic acid
to as running and is the basis of paper chromatography. Inks
D. Dodecanoic acid
that run when water is spilled onto the paper to which they
are bound are made out of water-soluble dyes. The eraser
capable of removing erasable ink has a surface to which the
dye in the ink adheres more tightly than it adheres to the I 8. Some kinds if ink run when water is spilled on
paper. paper to which they adhere. This can best be explai
Another common household product derived from organic by which of the following reasons?
compounds is soap. Each soap molecule has a hydrophilic A. The organic solvent of the ink is miscible in water.
(waterJoving) end and a hydrophobic (water-fearing) end. In B. The organic solvent of the ink is immiscible i
water, the hydrophobic portions of several soap molecules water.
form an aggregate pore in which nonpolar, hydrophobic C . The dye of the ink is soluble in water.
species (dirt and oil) can gather. This pore or nticelle (the D . The dye of the ink is insoluble in water.
spherical cell formed by several aligned and coagulated soap
molecules) is water-soluble, because the hydrophilic end of
each molecule composing it solvates in the water. A micelle
is lemoved by continuous exposure to running water, into 19. Which of the following would be the BEST soap?
which it dissolves and migrates.
A. CH3CH2CO2H
One of the most common industrial soaps is sodium B. CH3CH2CO2Na
dodecyl sulfate (SDS), found in many commercial shampoos
C. CH3 CH2CH2CH2CH 2CH2CH2CO 2H
and hand soaps. Soaps can be made by treating animal lard
(fatty-acid triglycerides) with a strong base (such as NaOH). D. CH3CH2CH2CH2CH2CH2CH2CO2Na
This forms glycerol (HOCH2CH(OH)CHZOH) and
carboxylate anions (fatty-acid carboxylates) by a reaction
referred to as s(tponification. Carboxylic acids once
deprotonated form carboxylates (the conjugate base of the
20. Which of the following compounds is MOST sol
in water?
acid). The organic chain of a soap molecule is most useful
when it contains at least eight carbons. Longer carbon chains A. CHqCHzCO2H
are common in soaps that are used to remove oils having B. CH3CH2CO2K
longer carbon chains. C. CH3CH2CH2CH2CH2CO2H
D . CH3CH 2CH2CH2CH2CO 2K
15. A11 of the following would be ideal properties for a
solvent used to dissolve a dye within an ink EXCEPT:
A. exerting a high vapor pressure at room temperature.
B. containing functional groups similar to the dye. 21. Which of the following reactions forms CH3CO2Na?
C. being highly reactive with cellulose. A. CH3CO2H + CH3MgCl
D. having a boiling point slightly above room B. HCO2H + CH3MgCl
femperature.
C. Ethanoic acid + NaOH
D . Propanoic acid + NaOH

Copyright @ by The Berkeley Review@ GO ON TO THE NEXT PA


Passage lV (Questions 22 - 28) 23. We know that amides are protonated at oxygen rather
than nitrogen, because oxygen:
For years, chemists pondered whether amides were
;rotonated on the nitrogen or oxygen atom. The amide is an
A. is less electronegative than nitrogen, so it donates
electrons more readily.
.nalog to peptide linkages, so the root of this question is
-runded in the chemistry of proteins. By determining the site
B. is larger than nitrogen, so it's electron cloud attracts
protons more readily.
-: protonation, conclusions about hydrogen-bonding in the
C. carries a partial positive charge due to resonance
';;ondary structure of proteins can be made. Before the withdrawal of n-electrons by the nitrogen.
.jvent and advancement of x-ray crystallography, protein
!-iucture could only be hypothesized. Due to the importance
D. carries a partial negative charge due to resonance
donation of n-electrons fr:om the nitrogen.
. - nydrogen-bonding in protein structure, detemination of the
ound? ::rtonation site was critical. Figure 1 shows the structural
,2H :-:3cts of protonation on the oxygen atom of the amide.

o ao.n o.H 4. Which of the following statements correctly describes


Ll H-- ii <+ i
2
the geometry of the molecule shown below?

--c\N-R R,,\N-R *-c\p.,R' o


ttl lt
HHH
Figure 1 Protonation of amide on oxygen
HT
-C-. /H
on the H
rlained . =rre 2 shows the structural effects of protonation on the A. The nitrogen has trigonal pyramidal geometry, so
r.:osen atom of the amide. the two hydrogens are outside of the plane created
water. o o by the other four atoms.
No resonance
ible in il lt B. The nitrogen has tetrahedral geometry, so the two

*- \g,,R'
/R' _>
H+ hydrogens are outside of the plane created by the
-c\
"N other four atoms.
I /\ C. The carbon has tetrahedral geometry, so the carbon
H HH hydrogen is outside ofthe plane created by the other
Figure 2 Protonation of amide on nitrogen five atoms.
D. The six atoms are coplanar.
Frotonation at the oxygen site is favored because of the
'rri.-:rlnce stabilization of the protonated product, similar to
,;":;- is observed when protonating esters. Despite the greater
'r ':.:-t)' of nitrogen relative to oxygen (oxygen is less basic,
'f ir;:-se it is rnore electronegative), the resonance stability is 25. What is the MOST basic site on the following
-r::i:: -nough to favor O-protonation. This manifests itself in molecule?
i':.:.n structure through the formation of hydrogen-bonds b c
!1,- :hs carbonyl oxygen (lone-pair donor) to the nitrogen o o
r-::-r (partially positive proton). Support for this
soluble 'r: :sion is found in the planar B-pleated sheets and helices
rr,::. ed in the secondary structure of proteins. nbUr$sn'
l:;ause of the resonanca structures with O-protonation,
A. Site a

:ir :.{ atoms of the amide are all coplanar. This is due to B. Site b
:: r :-
:-h1'bridization of carbon, oxygen, and nitrogen. C. Site c
D. Site d
L l . Alich of the following statements CANNOT be true?
I The C=O bond of an amide is shorter than the C=O
bond ol a ketone.

2Na? I The C-N bond of an amide is shorter than the C-N 26. The MOST stable hydrogen-bond between amides
bond ol a plimary amine.
extends from the:
fi. Amides are more basic that aldehvdes. A. carbonyl oxygen to the H on nitrogen.
\. I only B. the carbonyl oxygen to the H on carbon.
B. III only C. amide nitrogen to the H on another nitrogen.
C . I and II only D. amide nitrogen to the H on carbon.
D. I and III only

PAGE -.,:'"-risht @ by The Berkeley Review@ GO ON TO THE NEXT PAGE


2 7. Which of the following is NOT a resonance structure of Passage V (Questions 29 - 34)
an amide?

A. o B. o-
The boiling point of a compound is defined
temperature at which the vapor pressure of the compound
as the
il I
equals the atmospheric pressure. It is also thought of as the
C--**. R'
*,.c-*.,R' *, highest temperature at which a compound may still be
I I observed in a liquid state. Boiling points vary with
H H atmospheric pressure, so when comparing the boiling points
c. o- D. o- of different compounds, a standard pressure is referenced.
I I Under standard pressure, a compound's boiling point
C-*,,R' corelates with the heat energy required to vaporize a molecule
R,,clN,,R **.. of it from solution. As the heat energy of vaporization
H
I I
H
(AHuaporirulion) increases, the boiling point for the
compound increases.
Two chemists speculate about the reasons for the
differences they observe in the boiling points of various
organic compounds.
28. Which arrangement accurately relates the boiling points
of acetamide (H3CCONH2), acetone (H3CCOCH3), and Chemist l
propane to each other in descending order? Chemist I proposes that the difference in boiling poin
A. BPnsslsmi6s ) BPgsslene ) BPpropane for two similar organic compounds is related to the
B. BPsssl6ng ) BPssslam;6. > BPpropane differences in their molecular masses. The heavier
C. BPpropans ) BPas6l6mide ) BPu."ion" molecule, the more energy that must be required to liberate
D. BPpropans ) BP6ss16ng ) BPsssl2mi6s the compound into the gas phase from the liquid phase.
liberate the molecules into the gas phase, heat energy m
be added to the solution, which increases the temperature
the solution. Chemist 1 concludes that heavier molecul
have higher boiling points than lighter molecules.

Chemist 2
Chemist 2 proposes that the boiling point of
compound depends primarily on the strength of the attracti
intermolecular forces between molecules in solution.
stronger the attractive intermolecular forces bet
molecules, the harder it must be to remove a molecule f
the solution to the gas phase. As it becomes more diffic
to liberate a molecule from its liquid phase into its gas
more heat energy is required to carry the process out.
result is that the boiling point of a compound increases as
molecules bind to each other more tightly in solution. W
the intermolecular forces are greater, fewer molecu
vaporize, so the boiling point of the compound increases,
the vapor pressure of the compound decreases.

The hierarchy in attractive intermolecular forces


hydrogen-bonding first, polarity is second, and van der W
forces rank third in strengths. Hydrogen bonds contain
greatest amount of energy of these three forces, but not
hydrogen-bonds are equal in strength. For instance, alco
have stronger hydrogen bonds than amines, because t
hydrogen of the alcohol is more acidic than the hydrogen
the amine. The greater acidity allows the hydrogen to
electron density more readily. Any compound capable
forming hydrogen bonds is also polar. Polar attractions
stronger than van der Waals forces, the weakest of
intermolecul ar forces.

Copyright @ by The Berkeley Review@ GO ON TO THE NEXT PA


19. All of the following observations support Chemist l's 3 3. As you climb higher in the mountains, the amount of
theory EXCEPT: gases in the atmosphere decreases. This affects the
the
A. (H3C)2CHOH has a higher boiling poinr rhan boiling point of propanol such that it:
rund
s the
H3CCH2OH. A. decreases, because the amount ofhydrogen-bonding
B. H3CCH2OH has a higher vapor pressure than
lbe decreases.

with
H3CCO2H. B. decreases, because the amount ofhydrogen-bonding
C . H3CCH2CH2OH has a higher boiling point than rncreases.
oints
H3CCH2OCH2CH3. C. decreases, because the atmospheric pressure
rced. D . H 3 COH has a higher vapor pressure than increases.
roint
xule
H3C(CH2)6CH3. D. decreases, because the atmospheric pressure
decreases.
ltion
the

- the 3tt. 11sw would Chemist 2 rank the following compounds


rious according to their boiling points? 34. How do the boiling points of the following three
L H3CCH2OH chlorohydrocarbons compare with each other?

tr. H3COCH3 HsQ cH3


Itr. H3CCH2NH2
'ornts
r the A. H3CCH2OH> H3CCH2NH2 > H3COCH3
r the B. H3CCH2OH> H3COCH3 > H3CCH2NH2 cl
terate C. H3CCH2NHz > H3CCH2OH> H3COCH3
r. To D. H3CCHzNHz > H3COCH3 > H3CCH2OH II.
ct
must
rre of
cules
ct
: ll. The hydrogenation of an eight-carbon diene has which
of the following effects on the physical properties of the m.
ct-t
ofa compound?
3

rctive
A. Both the molecular mass and the melting point
The
increase.
ween
B. The molecular mass increases, while the melting CI GI
from
point decreases. A. Compound I > Compound fII > Compound II
Ticult
C. The molecular mass decreases, while the melting B. Compound I > Compound II > Compound III
rhase,
point increases. C. Compound II > Compound I > Compound III
The
D. Both the molecular mass and the melting point D. Compound III > Compound I > Compound II
as the
decrease.
When
:cules
s, and

:es is According to Chemist 2, as intermolecular hydrogen-


Waals bonding increases, which of the following trends should
.in the be observed?

rot all A. Both the boiling point and the vapor pressure
:ohols increase.
se the B. The boiling point increases, while the vapor
gen of pressure decreases.
accept C . The boiling point decreases, while the vapor
ble of pressure increases.
Ins arg D. Both the boiling point and the vapor pressure
of the decrease.

,AGE 63 GO ON TO THE NEXT PAGE


Passage Vl (Questions 35 - 42) 3 8. Infrared spectroscopy involves radiating a compound
with electromagnetic radiation of a known wavelength
The dipole of a bond can be fbund by considering the and observing any changes in the lengths of bonds
distribution of charge within the bond and the length of the within that molecule as they stretch. When the bonds
bond. The larger the difference in electronegativity between are stretched, the dipole moment changes and thus can
be detected. Which of the following would show the
the two atoms forming the bond, the greater the magnitude of
LEAST change in dipole moment?
the partial charges on each atom, resulting in a larger overall
dipole. When considering the dipole associated with A. Stretching a carbonyl bond in an asymmetric
molecules, the electron density of the entire structure is molecule
determined by the symmetry of the structure. Each bond is B. Bending a carbonyl bond in an asymmetilc
treated individually, and the sum of the component vectors is molecule
the approximate dipole. The estimated dipole is good enough C. Stretching a carbonyl bond in a symmetric
to predict chemical behavior. Figure 1 shows examples of a molecule
polar and a nonpolar cyclohexane derivative. D . Bending a carbonyl bond in a symmeric molecule

3 9. Which is the BEST description of the nonpolar structu


of Fe(NH3)aC12?
Nonpolar Polar A. Octahedral shape with the two Cl ligands cis
Figure 1 Polar and nonpolar disubstituted cyclohexanes B. Octahedral shape with the two Cl ligands trans
C. Tetrahedral shape with the two Cl ligands cis
The magnitude of a dipole is measured by placing the D. Tetrahedral shape with the two Cl ligands trans
compound between the two charged plates and observing the
drop in voltage. A large voltage drop in the capacitor implies
that the dielectric constant for the compound is large, so the
molecule has a large dipoie. This technique works because of
4 0. Which of the following compounds shows a dielect
constant of zero when placed in a capacitor?
the ability of a neutral polar compound when added to an
electric field to align with the fleld. If there is a net charge A. 1,1-dichloroethane
on the molecule, it migrates toward the capacitor plate with B. cis-1,2-dichloroethene
the opposite charge. C. trans-1,2-dichloroethene
D. E-1-chloro-2-fluoroethene
35. Which of the ibllowing compounds, when added to the
gap between the two plates of a capacitor, produces the
GREATEST reduction in voltage?
41. Which compound has the LARGEST dipole moment?
A. ArHF
B. C6H6 A . I,1,2,z-tetrafluoroethane
c. Nz B . 1,1,2,2-tetrafluoropropane
D. CO,I C . 1,1-difluoro-2,2-dichloroethane
D . I ,I -difluoro-2,2-dichloropropane

3 6. In which of the following reactions is it possible to


form a nonpolar organic product? 42. Which of the following statements CANNOT be true?

A. Hydrolysis ofan alkene I. For a tetrahedral structure, if any of the four li


are not equivalent to the others, the molecule
B. Halogenation of an alkane
polar.
C. Hydrogenation of an alkene
D. Reduction of an amide II. A11 1,4-disubstituted cyclohexane molecules
polar.
il. A11 optically active molecules are polar.

3 7. Which change does NOT result in an increased dipole A. I only


moment? B. II only
C. I and II only
A. Replacing iodine with brornine on an alkyl halide D. II and III only
B. Oxidizing a primary alcohol into a carboxylic acid
C. Replacing iluorine with chlorine on an alkyl halide
D. Adding HBr anti-Markovnikov to an alkene

Copyright O by The Berkeley Review@ GO ON TO THE NEXT PA


pound Passage Vll (Questions 43 - 49) 4 3. Which of the following compounds should be used in
rlength order to make a dication more soluble in an organic
bonds A common problem facing pharmacists is developing solvent?
bonds -rugs in a form that can be easily ingested by human beings, A. H3C(CH)nCO2H
ius can :.rticularly the problem of getting organic compounds to
rw the :-ssolve in water. As a general rule, organic compounds are B. H3C(CH)nCOz'
rt water-soluble, so it is difficult for them to migrate C. H3C(CHz)nNHz
metric ,.rough the bloodstream. Because most organic compounds D. H3C(CHz)nNH:+
:..aibit little to no hydrogen-bonding, they are referred to as
metric '.^Trophobic (Greek for "water fearing"). To overcome the
- . Jrophobic nature of organic compounds, one of two
.;hniques can be employed. 4 4. How would a micelle appear in an organic solvent?

.,lnique I
iecule
Technique 1 involves the use of micelles, three-
: rensional bulbs composed of compounds that are partly
.:rc and partly hydrophobic (organic). A prime example of
mlcture - r.mpound that forms a micelle in water is the conjugate
::.: of a fatty acid (H3C(CH2)nCO2- Na+). The micelle is a
. *erical membrane that forms when the organic tails
;
,-ire-qate as shown in Figure l.
NS
5 Pol Hydrophobic tails
lns

lil:l:l:
4 5. Which of the following compounds could MOST likely
be taken into the body through respiration?
A. (H3C)2CHOCH3
B. (H3C)2CHCH2OH

Fizure 1 Aqueous arrangement of molecules in a micelle


C. (H:C)zCHNHCH3
D. (HrC)zCHCH2NH2
.\n organic compound (such as an antibiotic) prefers the
, : of the micelle over the aqueous solution. Overall, the
-:;l1e is water-soluble due to the polar heads of the 4 6. Which of the following compounds would be MOST
r- "idual fatty-acid carboxylate anions. After migrating soluble in water?
- : an aqueous environment to a hydrophobic environment
rrl B.
..1), a micelle turns itself inside out and releases the H
,lecule i ::'iric compound in its core. This mechanism is what
:-::les water-insoluble drugs to be transported through the
- , jstream (an aqueous environment) to hydrophobic target
j rxs of the body (such as lipid membranes).
"\y::
-
-. ::'.r:ique 2

Technique 2 involves converting the compound into a D.


,'..:r-soluble derivative that decomposes into its active,
"..:r-insoluble form once inside the body. This often
" ,lves converting neutral organic compounds into ions by
: .:.lr protolotion or deprotonation or altering a functional
""-. -:. such as converting an alcohol into an ester by reacting
;.:h an acyl group. The drug returns to its original active
, -:r it is exposed to physiological conditions.

- -:r right @ by The Berkeley Review@ b5 GO ON TO THE NEXT PAGE


4 7. Which of the following compounds would make the Passage Vlll (Questions 50 - 56)
BEST micelle?
A. H3C(CH2\CO2H A hydrogen bond is formed between an atom able to
donate a lone pair of electrons and an electropositive hydrogen
B. H3C(CH)zCoz' (an H covalently bonded to nitrogen, oxygen, or fluorine). A
C. H3C(CH)13CO2H hydrogen capable of forming a hydrogen bond is said to be
D. H3C(CH)nCOz- protic. A protic hydrogen can form one covalent bond and
one hydrogen bond. As the hydrogen bond becomes stronger,
the covalent bond becomes weaker. This is to say that as a
lone pair is donated to a protic hydrogen, the original
covalent bond to hydrogen weakens.
Covalent bonds can be studied using infrared
4 8. Which of the following compounds would require a
spectroscopy. Different bonds have different characteristic
micelle to make it water-soluble?
absorbances based on their bond strength and atomic masses.
A. An alcohol (RCH2OH) Because the degree of hydrogen-bonding affects the strength of
B. A carboxylic acid (RCOZH) the covalent bond, a hydrogen bond can be seen indirectly in
C. An amine (RCHZNHZ) the IR stretch of the hydroxyl peak. Figure I shows the IR
D. absorbances associated with four different hydroxyl groups.

VV
An alkene (R2C=CR2)
3'Alcohol 2'Alcohol 1" Alcohol

4 9. What force holds the organic tails of a micelle together?


'Van
A. der Waals forces
B.
C.
Polar attractions
Hydrogen-bonding ,01",, ,oL.*,
t
3396crnl
D. Covalent bonding
Carboxylic acid

Figure 1 IR signals for hydroxyl functional groups


53.
Because the covalent bond is weakened by hydrogen-
bonding, the IR signal of a covalent bond between atoms
involved in hydrogen-bonding broadens as the degree of
hydrogen-bonding increases. Not all hydrogen bonds are
equivalent, so the signal becomes a range of absorbances that
appear as one broad band. The wave number of the
absorbance lowers, because the energy decreases. The
absorbances in Figure 1 show that as hydrogen-bonding
increases, the IR signal broadens and the maximum
absorbance occurs at a lower wave number.

50 . According to the IR absorbances in Figure 1, which of


the following compounds exhibits the GREATEST
amount of hydrogen-bondin g?

A. The tertiary alcohol


B. The secondary alcohol
C. The primary alcohol
D. The carboxylic acid

Copyright O by The Berkeley Review@ 66 GO ON TO THE NEXT PAGE


Which of the following amine compounds should show 5 5. As dimethyl sulfide is mixed into a pure sample of an
the BROADEST signal above 3000 cm-l? alcohol, the O-H absorbance:
-{. Ammonia A. broadens and shifts to a lower value on the wave
B. Propylamine number scale.
C. Dipropylamine B. broadens and shifts to a higher value on the wave
D. Tripropylamine number scale.
C. sharpens and shifts to a lower value on the wave
number scale.
D. sharpens and shifts to a higher value on the wave
number scale.
How is the absorbance value in the IR for a covalent
bond between oxygen and hydrogen affected by the bond
trength and hydrogen-bonding to other atoms?

.\ . As the bond length increases, the wave number


(cm-l) ofthe absorbance decreases; so as the degree 5 6. Which of the following statements CANNOT be true?
of hydrogen-bonding increases, the bond length
increases and the wave number (cm-l) of the
I. The IR absorbance of a covalent bond involving an
atom engaged in hydrogen-bonding is not affected
absorbance decreases. by the hydrogen-bonding.
B. As the bond length increases, the wave number II. The bond length of the covalent bond to the protic
(cm-l; of the absorbance increases, so as the degree hydrogen increases with hydrogen-bonding.
of hydrogen-bonding increases, the bond length
increases and the wave number (cm-1) of the
m. The acidity of a proton is increased by hydrogen-
bonding.
absorbance increases.
C. As the bond length increases, the wave number A. I only
(cm-l) ofthe absorbance decreases; so as the degree B. II only
of hydrogen-bonding increases, the bond length C. I and II only
decreases and the wave number (cm-l) of the D. II and III only
absorbance increases.
D. As the bond length increases, the wave number
(cm-l; of the absorbance increases; so as the degree
of hydrogen-bonding increases, the bond length
decreases and the wave number (cm-1) of the
absorbance decreases.

Hydrogen-bonding occurs within which of the following


compounds?

.{. Aldehydes
B. Esters
C. Ketones
D. Primary amines

The STRONGEST hydrogen bond is formed between:


A. the lone pair of O and a hydrogen bonded to O.
B. the lone pair of N and a hydrogen bonded to O.
C. the lone pair of O and a hydrogen bonded to N.
D. the lone pair ofN and a hydrogen bonded to N.

@ by The Berkeiey Review@ ol GO ON TO THE NEXT PAGE


Passage lX (Questions 57 - 63) 6'1"
Hydrocarbon Octane Rating Boilins Point
The petrolcurn industry provides roughly forty percent of 2-Methvlbutane 93 28'C
the annual cnergy needs of the United States. Crude oil is a Benzene r06 80'c
mixture of hydrocarbons that is refined to produce tuels, n-Hexane 25 69"C
including heating oil and petroleum. Many lightweight,
Toluene 120 104'c
alkene by-products fron.r the refinement of crude oil are used
as raw materials in making polylners. The industrial process n-Heptane 0 98'C
fbr refining crude oil into useful components is relerred to as 2-Methvlhexane 12 88'C
cracking and is sirnilar to fractional distillation. Figure 1 2.2.3-Trimethvlbutane t25 82'C
shows a schematic lepresentation of the cracking column used 2,2, 4 -T r imethy I pen tane 100 104"C
to refine crudc oil and the fi'agrnents collected at different
levels of refinement.
Table 1

5 7. Which of the following eight-carbon hydrocarbons has


the GREATEST octane rating?
A. 2-Methylheptane
Vapor B. n-Octane
- C. 2,2-Dimethylhexane
Petroleum D. 2,2,4-Trimethylpentane
25"C-t]5'C
----+-

5 8. Which of the fbllowing components is MOST likely a


fr}..
component of kerosene?
Kerosene
175'C-280'C
A. n-Octane
B. n-Decane
___> Heating oil C. 2,2-Dimethyloctane
250'C-350"C D, 2,2,4,4-Tetramethyldecane

_+ Lubricating oil
300'c-375"C

5 9. Which is NOT an effect of branching in a hydrocarbor


chai n?

-------->Tar A. An increase in octane rating


B. A decrease in boiling point
Figure 1 Cracking column used to refine crude oil C. A increase in density
D. An increase in hydrogen-bonding
Petroleum distillaie is sold as gasoline, the luel most
commonly used in internal combustion engines. 'Ihe best
air-petroleum mixture for such engines is the one that
produces the most unilbrm distribution o1' heat over the
pcriod of timc that the piston is doing work. This allows for 6 0. The cracking (refining) column operates according to the
an even expansion of the gas in the piston, which results in principle that:
more useful wolk. The result is a smooth lifting of the A. more dense hydrocarbons rise higher than less dense
piston, rather than an cxplosive jerk. Engine efficiency hydrocarbons.
depends on tl-rc unilbrmity ol heat distribution within it, so B. hydrocarbons with lower boiling points rise highe:
the choice of fLrel influenccs engine etTiciency. than hydrocarbons with higher boiling points.

Gasoline is given an octane rating that is based on its C . hydrocarbons with higher boiling points rise higher
than hydrocarbons with lower boiling points.
combr-rstion rate. An octiine rating is e measule of a fuel's
tendcncy to caLtse knocking (non-unifbnrr combustion.) The
D . aromatic hydrocarbons rise higher than non-
aromatic hydrocarbons.
scale is set using 2,2,4-tlimethylpentane, wl-rich is assigned
an octane rating of 100, and n-heptanc, which is assigned an
octane rating of zcro. A higher octane rating implies a better
fuel. Table 1 lists the octane ratings and boiling points lbr
some compollents of pctroleurn distillate.

Copyright O by Tire Berkeley Review@ GO ON TO THE NEXT PAGE


6 1. The efficiency (octane rating) ofa fuel depends on the: Passage X (Questions 64 - 70)
A. enthalpy of combustion.
B. The acidity, bond strengths, and bond lengths of
entropy of combustion.
hydrocarbons depend on the hybridization of the carbons
C. ratio of CO2 to water in the exhaust.
within the compound. Hybridization is defined as the mixing
D. r'ate o1'combustion.
of atomic orbitals to fbrm new hybrid orbitals that are
correctly aligned to make up the covalent bonds. Hybrid
orbitals are oriented to align the atoms within the molecule
into the least sterically hindered position fbr bonding. The
2. orientation of electrons allows the molecule to form
6 Which of the following statements must be true?
structures with a central carbon that has either tetrahedral
I. Aromaticity incleases octane rating.
1sp3-hybridized), tri gonal planar 1sp2 -hybridized), or linear
II. Ethylbenzene has an octane rating of less than 100. (sp-hybridized) geometry. Although the geometry dictates the
il. 2,2,3-Trintethylbutane is a good fuel additive to hybridization, the hybridization oI a carbon within a molecule
incrcase fuel eff iciency. can be used to predict the structure of the molecule.
A. I only The more p-character thele is in the hybrid, the longer
B. III only the hybrid orbital is, and thus the further away the electrons
C. I and II only ale f}om the nucleus. This variation in length can be used to
D. I and III only explain differences in chemical reactivity and physical
properties. When estimating properties of a bond, one must
consider that acidity results frorn heterolytic cleavage, while
bond energies are determined from homolytic cleavage,
Figure 1 shows both heterolytic and homoiytic cleavagc Ibr
oly a
63 . The hybridization of carbon in the aerobic cornbustion the C-H bond of a telminal alkyne.
of 2,2,4-trimethylpentane changes from :
Hetreolytic Cleavage (into ions)
A. rp3 to sp2.
B.
C.
rp2 to sp3.
sp to sp3.
R-C:n' __ .o +
R-C:C: ..@
H

D. sp3 to sp.

Homolytic Cleavage (into free radicals)

-_^E^:Lr o-^E^. a u.
=:
Figure I Heterolytic and homolytic cleavage of a C-H bond

Acidity can be explained in tenns of heterolytic cleavage.


The closer the electrons of a carbon-hydrogen bond are to the
carbon nucleus, the more acidic the hydrogen on that carbon.
This is to say that as the electrons in the bond get closer to
the carbon nucleus, the bond is easier to break heterolytically,
and thus the acidity of the hydrogen increases. Electrons get
closer to the nucleus of carbon when the bond is shorter-
to the However, as the bond gets shorter, it becomes more difficult
to break the bond in a homolytic fashion. It is more difficult
dense fbr hydrogen to remove one bonding electron from the bond
to carbon. This means that as the hydrogen becomes more
righer acidic, the homolytic bond energy increases.
The less s-character in the carbon hybrid, the longer the
righer length of the bond between carbon and the atom to which it
is bonded. As the bond becomes longer, it becomes weaker
non- in a homolytic sense.

AGE Copyright @ by The Berkeley Review@ GO ON TO THE NEXT PAGE


64. The MOST acidic hydrogen on 3-methyl-1-penryne is
on which carbon?
6 9. Which of the following compounds has the WEAKEST fuc
carbon-carbon single bond?
A. Carbon-l A' H-c:c-H B. Hsc-cEc-cr-rs il
B. Carbon-2 otrErt
C. Carbon-3
qiffiEr
g. D'
^.- Hs
D. H:.. ThEB
Carbon-4
^_ t- \^-^.zH
t- t- fli:t$ fi
H/u K^- t-
('v- ^/ H
isau
' , gnhflGf,
from ,

6 5. The LARGEST Ku is associated with which of the wmsm


mufu
following compounds?

"Go. ur\o, 70. The LOWEST pK6 is associated with which of the tr lwt

oG*,,
following nitrogen containing compounds?
B.
-NH
Tr
rerirj i
uquilil
.\*.\ (,\*, &][!DeEl|
*sasti
ilfum
C. D.

G," Eq
lu0msh
66. Which of the following organic compounds is the
STRONGEST base? Kq"
A. CH3CH2CH2CH2Na
B. CH3CH2CH=CHNa
C. CH3CH2CNa=CH2
C. CH3CH2C=CNa ,l\ r
cl'ick$

6 7. NaNH2 is a base strong enough to deprotonate the first


hydrogen on a terminal alkyne. Which of the following
hydrogens could it also deprotonate?
A. H on carbon-l of 2-methyl-1-butene
B. H on carbon-2 of 2-methyl-1-butene
C. H on carbon-l of 2-methyl-1-butanol
D. H on oxygen of 2-methyl-l-butanol

6 8. The LONGEST carbon-carbon bond can be found in Th


which of the following compounds? i.r dEE
A. H- CE C- Ct-ls B. H-C= c- H WurftlE
mesrmt
fuErudl
D' H:^- md rce
"'*..-r n-n./u3 t\
^//H diiffieru
,/" '- o, ryid cm
,/u: n
cqldliihi
mfirm* f
ffiuc rtpr

Copyright O by The Berkeiey Review@ GO ON TO THE NEXT PAGE


Passage Xl (Questions 71 - 77) Table 1 lists the theoretical equilibrium constants for the
six acid-base reactions patterned after Reaction 2.
Most reactions using organic reagents require a solvent
:.her than water, so acid-base chemistry must be viewed frour Organic Acid (HA) Equilibrium Constant (Ksq)
::irer the Brpnsted-Lowry definition or the Lewis definition.
lie Brgnsted-Lowry definition of an acid is a compound that H3CCOCH3 2.Ox1O4
.!;:s as a proton donor, while the Lewis definition of an acid H3CCOCH2COCH3 1.2 x 106
i a compound that accepts electron pairs. In an organic H3COH 3.9 x 10-i
:'::t'ent, acid-base reactions involve the transfer of a proton H3CCH25H x
5.2 104
::rn one reactant to another. There is an equilibrium
;,nstanl associated with this process that is predictable based
CI3CH 8.0 x 10-9

- r the pKz values of the two acids in the reaction (one acid is H3CNO2 8.2 x I04
i :eactant, and the other acid is a product).
Table 1
To determine the Ku value for organic acids, an organic
:;r,l is added quantitatively to an organic base. The 71. Which of the following reactions has an equilibrium
l:rilibrium constant (K"q) is determined from the constant greater than 1?
--ncentration of each species, once equilibrium is reached. A. CI3CH + H3CCH2S- C13C'+ H3CCH2SH
-r-;;ction 1 is a generic reaction between an organic acid and -+
.:,e conjugate base of a second organic acid
B. H3COH + H2CNO2- + H3CO- + H3CNO2
C. H3CCOCH3 + H3CO- *
HA + B- A- + HB
==::\ H3CCOCH2-+H3COH
Reaction 1 D. H3CCOCH2COCH3 + C13C- +
H3CCOCHCOCH3- + CI3CH
Equation 1 can be used to determine the equilibrium
: , rstant for Reaction I .

Ka (acid HA)
H* = tA-llFIBl - = tg(pKa (HB) - pKa (HA))
' tHAltBl Ka (acid HB) 72. Which of the following compounds can deprotonate
c5H6?
Equation I
A. H3CCOCH3
A series of six different organic acids are treated with 1,3- B. H3CCH2SH
-r;lopentadienyl anion, as shown in Reaction 2. C. H3CO-
HA+CsHs-5+A-+C5H6 D. HzCNOz-
Reaction 2

The structur"e of 1,3,-cyclopentadienyl anion is shown in


L:ure 1,
7 3. For the following reaction:
o H3CCH2SH + H3Co- -- H3CCH2S- + H3COH
what is true about the relative concentrations of each
species at equilibrium, if the reactants are mixed in
equal molar portions?
A. [H3CO-] > [H3CCHzS-]; [H:Co-] > [H:CoH];
Figure 1 1,3-Cyclopentadienyl anion
[H3CCH2SH] > [H:CO-]
The concentration of each organic species at equilibrium B. [H3CCH2S-] > [H:CO-]; [H:COH] > [H:CO-];
. determined using UV-visible spectroscopy whenever [H3CCH2SH] > [H:CO-]
:,:ssible. In cases where no n-bond is present in both the C. [H3Co-] > [H:CCHzS-]; [H:CO-] > [H:COH];
::actant acid and the product acid, the concentrations are [H3CCH2SH] = [H3CO-1
::termined using gas chromatography. The conjugate base
D. [H3CCH2S-] > [H3CO-]; [H:COH] > [H:CO-];
trd reactant base concentlations are determined by the
[H3CCH2SH] = [H3CO-J
:rference between initial acid concentration and equilibrium
::id concentration. The concentrations are used to determine
rquilibrium constants. The calculated values are compared to
,:lues found using pKu numbers in Equation 1. It is found
iat the error is greatest when Kst is greater than 104.

Jopyright O by The Berkeley Review@ 7l GO ON TO THE NEXT PAGE


74, The acidity of the C5H6 is abnormally high for Passage Xll (Questions 78 - 84) 79. !
hydrocarbons, because:
I
A. it is aromatic. The effect of atomic size on reactivity is perhaps most
B. pronounced when comparing the reactivity of thiols and ^l
it has an aromatic conjugate base. I
alcohols. In protic solvents, such as water, alkoxides (RO-)
C. its hydrogens withdraw electron density through the
are less nucleophilic than alkyl sulfides (RS-), because (
inductive effect.
alkoxides are capable of forming hydrogen bonds. In aprotic I
D. in the conjugate base, the hydrogens withdraw
solvents, alkyl sulfides are less nucleophilic than alkoxides,
electron density through the inductive effect.
because they are less basic and thus less able to donate their
lone pair of electrons to an electrophile. To compare the
nucleophilicity of alkoxides and alkyl sulfides, Reaction 1 is -9" T
carried out with a range of combinations of one solvent and
d
one nucleophile.
&
7 5. Which of the following relationships accurately shows tlrf
the relative pKa values for the given acids?
.l
Nuc- I B
A. pKalct3cH) > pKa(cH3oH) > pKa(HqcNoz)
B. pKagr3cNoz) > pKalgu3oH) > pKalgr3cH) H:C
Tffi n.c4ff. +r
D
C

C. pKalcn3oH) >pKalgl3CH) > pKalu3cNo2;


Reaction I
D. pKalcu3oH) > pKalg3cNo2) >pKalcr3cH)
Table I lists the negative logs of the reaction rates for
Reaction 1 observed in a series of solvents reacting with a it fr. tf
series of nucleophiles. In each case, the reaction is carried :F
out at 30'C, and with an initial concentrarion of 0.10 M for ni{
the nucleophile and of 0.1 1 M for the electrophile.
A
7 6. Which of the following compounds is NOT an example
of a Lewis acid? Nucleophilic Solvent -Log rate a
C
A. CI3CH H3CO- Ether t.44 D
B. H3CNO2 H3CS- Ether 1 9',7

C. NaCH3 HO' Ether 1.03


D. BFr HS- Ether 1.16
H3CO- Ethanol 3.19 s 1. -\,r
H3CS- Ethanol 2.12 fol
HO- Ethanol 3.35 A.
HS- Ethanol r.96
B.
77. Which of the following acids has a pKa value close C.
10.0, given that the pKa for C5H6 is 15.0? H3CO Water 4.22
.t
D.
A. H3CCOCH3 H:CS Water ^'7
B. CI3CH HO- 'Water 5.62
C. H3COH HS Water 2.14
D. H3CNO2
Table I ft'c
Se
Because the value in Table 1 is the negative log of the
rate, the magnitude of the effect of nucleophile and solvent on
-{.
the reaction rate is not immediately apparent. The smaller
B.
the negative 1og of the rate, the greater the rate. The
difference between the rate of SH- and the rate of RS- is
C.
attributable to differences in their molecular size. The
difference in reaction rates btween the various solvents is
attributable to a change in the mechanism from Sy2-like to
D.
Syl-like (as the solvent changes from ether to water). The
more the solvent binds the nucleophile, the less rapidly the
nucleophile can attack the electrophile and thus the slower the
rate of the nucieophilic substitution reaction. This affects the
reaction rate of nucleophilic substitution in a protic solvent.

Copyright O by The Berkeley Review@ GO ON TO THE NEXT PAGE


- 8. Which of the following sets of conditions results in the 8 3. Which of the following changes results in an increase in
FASTEST reaction rate? the reaction rate?
)st
A. An alkoxide in an aprotic solvent A. Changing the nucleophile from HS- to H3CS-
nd
B. An alkoxide in a protic solvent B. Changing the solvent from ether to water
I)
C. An alkyi sulfide in an aprotic solvent C. Changing the reaction temperature from 30'C to
ise
tic D. An alkyl sulfide in a protic solvent 25"C

3S,
D. Changing the solvent from water to ethanol
eir
he
is - 9. What value should be expected for the negative log of
nd
the reaction rate, if ethyl sulfide (CHjCH2S-) were 8 4. In comparing the properties of alkoxides and alkyl
added to 2-iodopropane in ether solvent? sulfides dissolved in ether, one notes that alkoxides have
A. 1.05
B. 2.04 A. higher pK6 and react faster than alkyl sulfides.
c. 3.09 B. higher pK6 and react slower than alkyl sulfides.
D. 4.52 C. lower pK6 and react faster than alkyl sulfides.
D. lower pK6 and react slower than alkyl sulfides.

for
na \ill If iodine was replaced with bromine in Reaction 1, one
ied should expect that lhe negative log of the reaction rate
for would:
A. increase, and the reaction rate would increase.
B. decrease, while the reaction rate would increase.
C. increase, while the reaction rate would decrease.
D. decrease, and thc reaction rate would decrease.

r 1. According to the data in Table 1, which of the


lbllowing bonds is the WEAKEST?
.{. c-H
B. C-I
C. C-O
D. C-S

From the data in Table 1, what can be concluded about


the effect of hydrogen-bonding on reaction rate?
the A. Hydrogen bonding hindels nucleophilic attack and
ton thus lowers the reaction rate.
rller B. Hydrogen bonding enhances nucleophilic attack and
The thus lowers the reaction rate.
)lS
C. Hydrogen bonding hinders nucleophilic attack and
The thus increases the reaction rate.
rs is D. Hydrogen bonding enhances nucleophilic attack and
eto thus increases the reaction rate.
The
the
r the
; the
:nt.

GE
-..pvright O by The Berkeley Review@ GO ON TO THE NEXT PAGE
Passage Xlll (Questions 85 - 91) 86. The nucleophilicity ofeach reagent in aqueous solution:

Esters are semi-reactive carbonyl compounds that


A. decreases as the pH is increased.

undergo substitution chenristry at the carbonyl carbon.


B. is best when the species is a cation.

Several biological reactions (including transesterification and


C, is best when the species is neutral. illi
transamination) proceed through standard carbonyl chemistry. D. depends only on the size of the nucleophile.
The reactivity of the carbonyl depends on both the
nucleophile and the leaving group. A researcher set out to
determine the reactivity of three dil'f'erent nucleophiles in a 87. Which of the following compounds would be the
standard substitution reaction using an ester electrophilc. For MOST reactive nucleophile at pH = 9.0?
the reaction in Figure 1, three diff'erent compounds A. H3CCH2CH2OCH3
(Compound A, Compound B, and Compound C) were used. B. H3CCH2CCO2CH3
ol,cur C. H3CCH2CH2CONH2
D. H3CCH2CH2CH2NH2
o
.-------->

N-
88. If the pKu fbr H3CNH3+ is 10.3, which of the
I following is the BEST approximation for the pK6 for
CH 1 Cl3CNH3+?
Figure I Deacylation of an ester A. 17.3
B. 12.3
Contpound A has the forniula C13H24S, Compound B c. 8.3
has the lormula C9H1gO, ancl Cornpound C has the folrnula D. 1.0
C7H9N. Figure 2 shows three graphs ciepicting the change
in reaction rate of the deacyltrtion reaction as a function of the
solution pH fbr each of the thlee separate compounds. 89. If the pKu for NH4+ is 9.25, then pK6 for NH3 must
be equal to which of the fbllowing?
Compound Cornpound Cornpound C
A. 9.25
B. 7.00
c. 5.75
D. 4.15

6.0 1.0 8.0 6 0 7.0 8.0 6 0 7.0 8.0


90. How does the hybridization of the carbonyl carbon
pH pH pH
change during the reaction'J
Figure 2 Reaction rate as a lunction ol'solution pH
A, It changes fiom tp2 to tp3 and back to sp2.
Each reaction obeys standard mechanistic behavior fbr B. It changes from tp3 to t1t2 and back to spj.
carbonyl substitution. They arc believed to proceed throLrgh a C. It rernains sp2 throughout the reaction.
mechanism where the nucleopl-rile attacks the calbonyl D. It remains spJ throughout the reaction.
carbon, breaking the C=O n-boncl and forming a tetrahedral
intermediate. A lone pair ol clectrons on oxygen then
reforms the n-bond, ejecting lhe leaving group. Although 9 1. Aniline and benzylamine, drawn below, are both:
the nucleophilicity of the dil'fcrent compounds is not equal, NHz
the similar mechanisms make the reactions comparable. At
low pH, the carbonyl compound can be pl'otonated, making it
a better electrophile. This ncgates the effbct of decreasing
nucleophilicity of alcohols and thiols, beceruse they remain Aniline Benzylamine
uncharged at low pH values.
A. primary amines.
85 . Which of the tbllowing types ol conipounds is tl.re B. aromatic amines.
\lOST basic'l C. conjugated amines.
.{, Primary alcohols D. nonalkyl amines.
m

B. Esters
C. Secondary amines :

D. Teltiary thiols
I

Coprright O by The Berkeley Reviewo GO ON TO THE NEXT PAGE @irqplifi


97. Which of the following isomers has the HIGHEST
Questions 92 through
boiling point?
descriptive passage.
A. H3CCH2OCH2CH3
B. H3CCH2CH2OCH3
4 2. Which of the following compounds has the HIGHEST C. (H3C)2CHoCH3
boiling point? D. H3CCH2CH2CH2OH
A. 2-pentene
B. Diethyl ether
C. Heptanal
D. Cyclohexanol 9 8. Which of the following molecules is NOT polar?
A. cis-1,3-dichlorocyclopentane
B. trans-1 ,3-dichlorocyclopentane

C. E-1,4-dichloro-2-butene
I ,1 . The correct IUPAC name for the following molecule is:
D. 1,2,2,3-tetrabromopropane

9 9. Which of the following compounds releases the


GREATEST amount of heat upon combustion?
A.\ -\
A. 2,4-diethyl-3,5-dimethylheptane
B. 4-ethyl-3,5,6-trimethyloctane
C. 3-ethyl-5,6-dimethylnonane
D. 3,5,6-trimethyldecane

C. D.

o
I
r 4. Which of the following compounds exhibits
conjugation?
I. 1,4-cyclohexadiene
II. 3-ethylcyclohexene
m. 2-methyl-1,3-cyclopentadiene
A. I only
B. II only 100. Which of the following functional groups is found in
C . III only C2H5 CH(oCH: )C (o) CH( CH:) z?
D . I and III only
A. Aldehyde
B. Ester
C. Ketone
'r 5. Which of the following compounds is MOST stable?
D. Oxirane
A. 2-methyl-I,4-pentadiene
l.D 2.A 3.C 4.A 5.D 6.D
B. 3-methyl-1,4-pentadiene '7.8 8D 9.D 10.8 11.8 12.A
C. 2-methyl-l,3-pentadiene 13. D 14. A 15. C 16. B r1. B 18. C
D. 1.S-hexadiene 19. D 20. B 2t. C 22. A 23. D 24. D
25. C 26. A 21. D 28. A 29. C 30. A
31. A 32. B 33. D 34. A 35. A 36. C
31. C 38. C 39. B 40. C 41. D 42. B
i 6. Which of the following molecules would have a dipole 43. B 44. A 45. A 46. B 47. D 48. D
moment NOT equal to zero? 49. A 50. D 51. A 52. A 53. D 54. B
L Z-1,4-dichloro-2-butene 55, D 56. A 51. D 58. D s9. D 60. B
61. D 62. D 63. D 64. A 65. D 66. A
II. E-1,4-dichloro-2-butene 61. D 68. C 69. C 70. A 11. D ',72. C
m. cis-1,2-dichlorocyclopentane 13. D 14. B 15. A 16. C 17. D 78. A
A. Compound I only 79. B 80. C 81. B 82. A 83. D 84. C
85. C 86. C 87. D 88. C 89. D 90. A
B. Compound II only 91. A 92. D 93" B 94. C 95. C 96. D
C. Compound III only 91. D 98. C 99. B 100. C
D . Compounds I and III only

Copyright @ by The Berkeley Review@ /5 iCHEMICA ES FIN!


Structure, Bonding, and Reactivity Passage Answers

Choice D is correct. Table 1 does not list any aikynes, so the bond energies must be estimated from trends in the
data. According to data in Table 1, a bond between two spJ-hybidized carbons has a bond dissociation energy,
BDE, between 81 and 88 kcals/mole. A bond between an sp2-hybridized carbon and an sp3-hybtidi"ed carbon
has a BDE ol 97 kcals/mole. This means that the bond between C2 and C3, a bond between an sp-hybridized
carbon and an sp3-hybridired carbon, should have a BDE greater than97 kcals/mole. This eliminates choices
A and B. A bond between a hydrogen and an sp3-hybtidi"ed carbon has a BDE between 91 and 104 kcals/mole.
A bond between a hydrogen and an sp2-hybtidited carbon has a BDE of 108 kcals,imole. This means that the
bond between hydrogeir and an sp-hybridized carbon should have a BDE greater than 108 kcals/moie. This
eliminates choice C and makes choice D the best answer.

) Choice A is correct. According to the bond dissociation energies listed in Table 1, iodine and chlorine both make
their strongest bonds to methyi carbons. It thus can be assumed that bromine would exhibit the same behavior
as these other halides, and that the strongest bond to bromine is formed by a methyl carbon. Choice A is best.

J. Choice C is correct. Table 1 shows an increase in bond strength for bonds formed between equivalent carbons and
atoms of decreasing size (I, C1, ar-rd O). From this, it can be concluded that shorter bonds are generally stronger
than longer bonds, and that as atomic size decreases, the bond length to a neighboring atom decreases. No bond-
length data are provided in the table, but this can be inferred from the passage. Choice C is best.

4. Choice A is correct. The hydrogenation of an unsubstituted alkene yields the greatest energy (according to the
heats of reaction listed in Table 2). A less stable reactant yields a greater amount of heat upon reaction, so the
n-bond must be weakest in an unsubstituted alkene. The energy generated from oxidative cleavage, or any
reaction that breaks the n-bond, is greatest when the alkene is unsubstituted. The best answer is choice A.

5. Choice D is correct. The difference in reactivity between the cis and trans geometrical isomers of an alkene is
attributecl to i.ntramolecular steric hindrance in the cis compound, because the substituents are on the same side
of the molecule. The resonance, hybridization, and electronegativity of carbon are the same in both
geometrical isomers of the alkene. This eliminates choices A, B, and C and makes the best answer choice D.

6. Choice D is correct. This question should be a lvelcome freebie, relative to other questions in the passage.
Alkene carbons have sp2-hybridization, and alkane carbons have sp3-hybridization. Two of the three carbons
in the compound are alkene carbons, while tire other carbon is an alkane carbon. The best answer is choice D,
two sp2-hybriclized carbons and one sp3-hybtidized carbon. Pick choice D to get your point for correctness.

7. Choice B is correct. Table 1 lists the bond dissociation energy for various bonds, which is the energy required to
break the bond in a homoiytic fashion. This in essence means that Table 1 lists the bond strength. The strongest
bond, according to that data listed in Table 1, is the bond with the highest bond energy. The highest value
among the answer choices is the bond between the methyl carbon and chlorine. The best answer is choice B.

Choice D is correct. Double bonds are stronger than single bonds, meaning that more energy is required to break
a double bond than a single bond. Choices A and B are thus eliminated. The lower heat of hydrogenation in
the second chart implies that the reactant alkene molecuie is more stable. The more stable the alkene
compound, the stronger its n-bond. This means that the double bond in the tetrasubstituted aikene is stronger
than the double bond in the unsubstituted alkene. The best answer is choice D.

L Choice D is correct. To convert the acid form of a nitrogen-containilg compound (in this case, a protonated imine
species) into a neutral species, you must add a strong base (strong enough to deprotonate the iminium cation).
Choice A is eliminated, because it is a strong acid, which protonates rather than deprotonates the compound.
Choice B is eliminated, because it is inert and has no effect on caffeile. Choice C and choice D are both bases,
but the stronger base is NaOH, so choice D is the better choice. The carbonate base is not strong enough to fu111'
deprotonate ihu i*ir-ri.rrrr cation. For best results in a case like this, pick choice D.

Copyright C bv The Berkcley Reviewo 76 Section I Detailed Dxplanations


Choice B is correct. We know that the nitrogen in questions has sp2-hybridization for three reasons: It is
involved in one rc-bond, it has three substituents attached and no lone pairs, and it is pianar with respect to the
adjacent atoms bonded to it, with bond angles of approximately 120'. For so many good chemistry reasons, the
E nitrogen in question is sp2-hybridized,, so why choose anything but choice B?

r the Choice B is correct. Because of the delocalization of electron density throughout the n-network of the caffeine
tW' molecule (achieved by the resonance between all adjacent non-hydrogen atoms), the compound must be planar to
'bon
allow the p-orbitals to overlap correctly. This means that all of the atoms in a caffeine molecule, and thus all
ized of the nitrogens, must be coplanar with respect to one another. Pick choice B for a whopping good answer.
rices
role. Choice A is correct. Because of the resonance donation of the lone pair of electrons on nitrogen to the carbonyl
:the carbon, the C=O bond takes on some single-bond properti"es, which makes it longer than a standard spr-
Ihis hybridized C=O bond that lacks any resonance effect. The best choice is therefore choice A. |ust as a side note,
the C=O bond in formaldehyde is longer than the C=O bond in carbon dioxide because of the varying
hybridization of carbon in the two compounds. In formaldehyde the hybridization of carbon is sp2,while in
rake carbon dioxide it is sp. The more s-character in the hybrid orbital, the shorter the orbital and thus the shorter
vior the bond. The stable resonance structures for an amide are shown below:
-t
,1,
..o
and
:9: longer than a stanc{ard
llger lt lF C=Obond
:nd-
<----->
\r.-/t\
N- \gZL\
N-
tl
r the
r the Choice D is correct. Ionic forces are stronger than polar forces, so the ionic compound (acid salt form) should
any have both a higher melting and a higher boiling point than the polar uncharged compound (free-base form).
This makes choice D correct, and you want that which is conect. Follow society's infiuence and pick D.

Choice A is correct. If the N-CH3 group is replaced by an oxygen atom, the compound that remains has an
oxygen between two carbonyls. This is referred to as an acid nnhydride (from the fact that the compounds forms
when two acids combine in a dehydration reaction). The best answer is choice A. There is no such term as an
"acid ester", so choice B is elin-rinated. An ester inl,olves just one carbonyl, so choice C is eliminated. A lactone
is a cyclic ester, and given that the compound is not an ester at all, it can't be a lactone. Choice D is eliminated.

-5. Choice C is correct. A good soivent for dissolving a dye to form an ink is one that is a liquid at roorn
temperature, evaporates quickly, and exhibits a high degree of dye solubility. Having a high vapor pressure
impiies that it evaporates readily, so choice A is eliminated. If it has functional groups that are similar to
the dye, then the dye is like1y to be highly soluble in the solvent, so choice B is eiiminated. If the boiling
point is slightly above room temperature, then it is a volatile liquid, so choice D is eliminated. The solvent
should not react with paper (celluiose), so the correct answer is choice C.

reak -:" Choice B is correct. Becar-rse like dissolves like, the best solvent for dissolving the dye should also have
rn in hydroxyl groups attached to it, just as the dye does. The best choice is therefore the alcohol, The carboxylic
(ene acid is not a good choice, because carboxylic acids are not as volatile as alcohols. If you have melting points
nger memorized, then yor-l may be aware that carboxylic acids that are three carbons or greater in length are solids
at room temperature. If yor-r don't have them memori.zed, like 99.999"/0 of us, that's okay too. Aldehydes and

q :-.
ketones may work, but not as well as the aicohol. The best choice is B.

Choice B is correct. Determining the IUPAC name for the compound requires that you count the longest
nrne continuous chain of carbon atoms in this straight chain compound, which yields a total of ten. Next, you must
ion). identify all functional groups on the molecu1e, including alkyl groups that are not a part of the carbon backbone.
und. The only functional group on this compound is a carboxylic acid. If there were multiple functional groups, the
ases/ more oxidized functior-ral gror,rp gets higher priority in the name of the compound. For instance, if there were
rully also an OH group, then it would be namecl a hydroxy substituent, rather than an alcohol. Having ten carbons
and a carboxylic acid group makes the compound decanoic acid. Choice B is the best answer.

-:ryright @ by The Berkeley Review@ 77 Section I Detaited Explanations


18. Choice C is correct. When water is spilled on paper, it diffuses across the surface of the paper. If the ink bound
to the paper is soluble in water, it dissolves into the water and spreads out, or runs. So if an ink runs, it must be
soluble in water. The colored portion of the ink is the dye, not the solvent. Running ink means ihat the ink dye
is water soluble. Pick choice C for best results.

19. Choice D is correct. As stated in the last paragraph of the passage, a compound must contain at least eight
carbons in its chain to be a good soap. The best soap has a polar and nonpolar end associated with the molecule.
The negatively charged carboxylate is at one end and an organic tail is at the other end. Molecules with
charged and organic ends are optimal for making a soap. Choice D has both eight carbons and a charged end.

20. Choice B is correct. To be soluble in water, a compound must be either charged or polar. Because choices B and D
are ionic, they are better in this regard than choices A or C. The organic tail is smaller in choice B, so it
dissolves into water more readily than choice D. Pick choice B, and feel the sensation of correctness.

21. Choice C is correct. Carboxylates are formed when a carboxylic acid is deprotonated. The Grignard reagent in
choice A deprotonates the carboxylic acid to form the carboxylate, but the cation is not sodium, so choice A is
eliminated. Choice B is invalid, because the Grignard reagent deprotonates the carboxylic acid (formic acid),
and the carboxylic acid does not have enough carbons to make sodium acetate. Choice D is invalid, because it
has too many carbons (propanoic acid has three carbons). The H3CCO2Na molecule results from the
deprotonation of acetic acid by a base with a counterion of Na+. The best choice is therefore choice C.

Choice A is correct. Because of the resonance donation from nitrogen, the carbonyl bond (C=O) of an amide has
some single-bond character. Since a single bond is longer than a double bond, the single-bond character of the
amide carbonyi bond results in a longer carbonyl bond than the unconjr-rgated carbonyl (as observed with the
ketone). This makes statement I a false (not true) statement. Because of the previously mentioned resonance,
the carbon-nitrogen bond has some double-bond character, making it shorter than a standard carbon-nitrogen
single bond (as seen with a primary amine). This makes statement II a true statement. Because of the
resonance, the carbonyl oxygen carries a partial negative charge. This makes the oxygen more basic than
typical carbonyl oxygens (such as the one in an aldehyde). Statement III is also a true statement. Only
statement I is ttot true, so choice A is the best answer.

23. Choice D is correct. As emphasized in the passage, nitrogen donates electron density to oxygen through
resonance. This places a partial negative charge on oxygen (increasing its basicity) and a partial positive
charge on nitrogen (decreasing its basicity). Choice C is thus eliminated, and choice D is correct. Choice A
should be eliminated, because oxygen is more electronegative than nitrogen. Choice B should be eliminated, =fll
because oxygen is smaller than nitrogen. If you want to do what you should do, pick D and gain incredible
satisfaction doing what you should do.

Choice D is correct. Because of the resonance donation from nitrogen, the nitrogen has sp2-hybridization.
Having sp2-hybridization results in trigonal planar geometry. The carbonyl carbon aiso has trigonal planar
geometry, so the central two atoms force the three hydrogens and one oxygen to assume a coplanar orientation.
Choices A, B, and C therefore ail must be eliminated as incorrect geometric descriptions, making choice D the
best answer. The two resonance forms and the resonance hvbrid are drawn below:
o o- o6
I'
il I
sp2-Hybridizati,n li
,.'C\ /H -7 Cf. ./H
H/ \N*-H
Trigonal planar
H N n-----------* H a]T---._ sp2-Hybridization
I I
Trigonal planar
H H H

25. Choice C is correct. From the passage, we know that amides are protonated at the carbonyl oxygen, so choice D
is eliminated. Because nitrogen is less electronegative than oxygen, it donates more electron density to the
carbonyl oxygen (through resonance) than the ester oxygen donates to the ester carbonyl oxygen. This places a
larger partial negative charge on the amide carbonyl oxygen than or-r the ester carbonyl oxygen. The larger
negative charge makes Site c the most basic site. Choice C is therefore the best answer.

Copyright O by The BerkeJey Review@ 7a Section I Detailed Explanations


Choice A is correct. The most stable hydrogen bond forms between the best lone-pair donor (most basic site) and
the hydrogen with the greatest partial positive charge. Because amides are protonated at the carbonyl oxygen
(as stated and drawn in the passage), the carbonyl oxygen is most basic and thus donates one lone pair of its
electrons. This eliminates both choice C and choice D, Of the answer choices remaining, the hydrogen on
nitrogen carries the partial positive charge, not the hydrogen on carbon. This means that the hydrogen bond
forms between the carbonyl oxygen and the hydrogen bonded to nitrogen, thus the best answer is choice A. If you
are unsure, think of the hydrogen-bonding in B-pleated sheets.

Choice D is correct. Choice A is the most stable resonance structure of the amide (all octet and no formal
charges are present). When nitrogen donates electron density to oxygen, choice B becomes the resonance
structure. This is a minor contributor due to the formation of charges on the molecule. The fact that it is an all-
octet resonance structure is favorable. If the nitrogen were to pull its n-electrons back from the carbon in answer
choice B, the resonance structure represented by answer choice C would be formed. Because carbon does not have
a complete octet in this resonance structure, it is a very minor contributor, but it is none-the-less a resonance
structure of the amide. It is not possible to form a double bond to the R-group, because that would require five
bonds to carbon. In order for carbon to donate in that manner (and have only four bonds), it must have had a lone
pair (and thus a negative charge) in the original structure. R was not drawn as having a lone pair, so it is
assumed that the R represents a standard alkyl group. The best answer is therefore choice D.
/o
\il o- o-

,. c5-R' ..-
I I

r\ 1\ o.r\*,R' -* o-tlN.R
I
H
lt
HH
Choice A (major) Choice B (minor) Choice C (very minor)

Choice A is correct. Given that the molecular masses of the three compounds are roughly equal (59 g/mol, 58
g/llrol, and 44 g/mol), the top consideration for determining their boiling points is the intermolecular forces. An
amide has hydrogen-bonding, while a ketone and a hydrocarbon do not. At room temperature, most amides are
solids, acetone is a volatile liquid, and propane is a gas. Based strictly on the phases, the best answer (and the
only answer that lists acetamide as the highest) is choice A, BP'sslnmide > BPacetor.,u > BPpropane. Acetone has
a higher boiiing point than propane, because it is polar and more massive.

Choice C is correct. Chemist 1 considers molecular mass to determine the relative boiling points of compounds.
We are looking for the exception, so the correct answer is the choice where the lighter compound has the
higher boiling point (or lower vapor pressure). A higher vapor pressure at room temperature corresponds with
a lower boiling point. In choice A, the heavier compound of the two has the higher boiling point, so choice A is
not an exception to Chemist 1's general rule. In choice B, the heavier compound of the two has the lower vapor
pressure (and thus higher boiling point), so choice B also follows the rule. In choice D, the heavier compound of
the two has the lower vapor pressure (and thus higher boiling point), so choice D follows the rule too. In choice
C, the heavier compound of the two has the lower boiling point, so choice C contradicts Chemist 1's theory.

Choice A is correct. Chemist 2 considers intermolecular forces to determine the boiling point of a compound.
The strongest intermolecular forces correspond to the highest boiling point. The passage states that alcohols
have stronger hydrogen-bonding than amines. This means that the alcohol (I) has the highest boiling point
(and thus is listed first), because ethanol has the strongest H-bonds of the three compounds. The ether (II) has
the lowest boiling point, because it cannot form hydrogen bonds with itself (due to its lack of an electropositive
hydrogen). The order of the boiling points is therefore: I > III > II, making choice A the best answer.

Choice A is correct. Hydrogenation is the addition of H2 gas to an alkene to break the n-bond and reduce the
compound to an alkane. For every n-bond that is lost by the alkene molecule, two hydrogens are gained. This
increases both the molecular weight of the compound and the molecular flexibility of the compound (the
product is both more massive and more flexible than the reactant). Both of these effects increase the melting
point of the compound, making the melting point of the product greater than the melting point of the reactant.
This makes choice A the correct answer. Pick A and you'il be an MCAT supernova.

- :vright O by The Berkeley Review@ 79 Section I Detailed Explanations


32. Choice B is correct. According to Chemist 2 (and as a general rule), the stronger the intermolecular forces, the
greater the boiling point for a compound. The greater the boiling point for a compound, the less of it there is
that evaporates, thus the lower its vapor pressure. Pick choice B, to correctly interpret the logic of Chemist 2.

33. Choice D is correct. Higher elevation means fewer molecules of gas per volume of air, and thus a lower
atmospheric pressure. The elevation and atmospheric pressure have no effect on the intermolecular forces
between molecules. Flowever, the lower atmospheric pressure means that less energy (heat) is required to
rgach a temperature at which the vapor pressure (Prrupo.) is equal to the atmospheric p."rs.tte (Patmosph"ric),
the definition of the boiling point. The boiling point is therefore lowered as elevation increases. This'makes
choice D correct.

34. Choice A is correct. The boiling points of Compounds I and II are directly comparable, because they are
geometrical isomers. Compound I (the cis isomer) is poiar, while Compound II (the trans isomer) is nonpolar.
This means that the boiling point of Compound I is greater than the boiling point of Compound II, which
eliminates choice C. Because Compound III is an alkane, it is flexible (whereas Compounds i and II are rigid,
due to the n-bond), so Compound III is able to rotate between conformers. The most stable conformation of
Compound III is nonpolar, but because it can assume polar conformations on occasion, the compound is slightly
polar. The boiling point of Compound III is less than the boiling point of Compound L Compound III shbuld
have the second highest boiling point, because it is slightly polar, while Compound II is non-polar. Thus, the
correct order is I > IiI > II, making choice A the best answer.

J5. Choice A is correct. The greatest reduction in voltage is caused by the compound with the greatest dielectric
constant. The greatest dielectric constant is associated with the most polar compound. Choices B, C, and D are
all symmetric, so they are a1l nonpolar. This eliminates choices B, C, and D. Only ArHF (choice A) is polar,
meaning that ArHF has the greatest dielectric constant. Choice A is a fine choice in a situation like this.

36. Choice C is correct. The hydrolysis of an alkene forms an alcohol. An alcohol is polar, so choice A is
eliminated. The halogenation of an alkane forms an alkyl halide. An alkyl halide is polar, so choice B is
eliminated. The hydrogenation of an alkene forms an alkane. An alkane is most often nonpolar, so the best
answer is choice C. Reduction of an amide forms a primary amine. An amine is polar, so choice D is eliminated.

J/. Choice C is correct. Assuming that an alkyl iodide is polar to begin with, then replacing iodine with bromine
results in a more polar compound, because bromine is more electronegative than iodine, so that the difference in
electronegativity between the halogen and carbon has increased. A carboxylic acid is more polar than a
primary alcohol (or any alcohol, for that matter), so choice B results in a more polar compound. Alkenes are
typically nonpolar, so the addition of HBr forms an alkyl bromide, which increases the polarity, so choice D is
eliminated. Because fluorine is more electronegative than chlorine, replacing a fh,rorine substituent with a
chiorine substituent results in a compound that is less polar, making choice C the choice that does nof result in
increased poiarity. Pick choice C to be a star of chemistry.

38. Choice C is correct. The dipole moment changes only when a compound's bonds are either stretched or bent, if
the compound is asymmetric. This makes choices A and B less likely to exhibit the least change in dipole
moment. The dipole moment does not change drastically (if at al1), when the chemical bonds of a symmetric
compound are either bent or stretched. Therefore, the least change in dipole is observed in a symmetric
molecule. Stretching a symmetric molecule often balances out, meaning that the electron density is shifted
uniformly in opposing directions, The result is that the dipole of the molecule does not change. The best answer
is choice C. Bending a symmetric molecule can make it asymmetric, so choice D is not as good as choice C.

39. Choice B is correct. To be nonpolar, all of the ligands must pu1l in such a way that the vectors of each
individual bond cancel out. Tetrahedral structures are not possible with six ligands, so choices C and D are
eliminated. It is only when the two chlorine ligands are trans to one another that they cancel out one another
in terms of polarity. The best answer is therefore choice B.

40. Choice C is correct. A dielectric constant of zero results from a nonpolar molecule. The only nonpolar molecule
among the answer choices is trans dichloroethene. Cis alkenes are polar, so choice B is eliminated. Choice D is
trans, but there are different substituents on each carbon, so it is polar. The best answer is choice C.

Copyright @ by The Berkeley Review@ 80 Section I Detailed Dxplanations


1e :1, Choice D is correct. Choice A is nonpolar, because the vectors expressing the electron withdrawal of the
15 fiuorines cancel out, so choice A is eliminated. Fluorine is more electronegative than chlorine, so the
asymrnetric electron distribution is found n'ith choices C and D, which rules out choice B. In choices C and D,
the fluorine atoms withdraw the electron der-rsity, making the molecule asymmetric. In the propane molecule
CI (choice D), the methyl substituent donates eiectron density to the electron-poor central carbon, placing a
ES partial positive charge on the methyl groupi therefore, it increases the dipole moment. This makes the
to propane molecule more polar than the ethane molecule. The structures are shown below:
:),
ES

re
rr.
,h C-F vectors cancel, br-rt methyl donates
All vectors cancel .'. nonpolar
.1,
.'. slightly polar
of
lr.
td
1e

I Vectors almost cancel .'. slightly polar C-F vectors almost cancel C-Cl vectors,
]C but methyl donates .'. polar

.t,
Choice B is correct. For a tetrahedral structure, if the four ligands are not all equivalent, then the structure is
asymrnetric. If the compound is asymmetric, then it must be polar (have an asymmetric distribution of electron
Lc
der-rsity). This makes statement I true. Figure 1 shows an example of a 1,4-disubstituted cyclohexane molecuie
is that is tlot polat, which means that statement Ii is rzol true. A11 optically active compounds must be
St
asymmetric in order to be optically active, so they must be at least slightly polar, This makes statement III
j trtre. Only statement lI is not true, so choice B is the best answer.

'.e
tn
a
Choice B is correct. A dication carries a +2 charge, so it must be coupled with an anion that is organically
:e soluble. The only organic anion among the answer choices is choice B.
is
Choice A is correct. A micelle turns inside out from its aqueous structure rt'hen it is added to an organic solvent
(as stated in the passage). Figure 1 shows a micelle as it appears in water, where the polar heads are exposed
to the liquid, and the organic tails are protected in the core. In a hydrophobic (organic) solvent, the organic
tails are exposecl, and the polar heads form a protective core. This is best illustrated in choice A. Choice C
may look familiar, in tl-Lat cell membranes arrange themselves in such a manner. Choice D is a "throw-away"
answer, because the tail and the head of the compound exhibit no attractive forces.

Choice A is correct. To be absorbed through respiration, a cornpouncl must be a gas or a vapor, because only gases
are absorbeci through respiration. This means that any compound intended to be taken into the lung must be
either a gas or a liquid r,r,ith a low boiiing (one with a high vapor pressure). There is hydrogen-bonding in
choices B, C, ancl D, but not in choice A. Hydrogen-bonding increases the boiling point and thus lowers the
vapor pressure. All of the compounds have roughly comparable masses (either 73 or 74 grams per mole). The
_:1
only factor to consider in approximating the relative boiting points is hydroger-r-bonding. The best answer is the
ether, choice A. As a point of trivia, it is estimated ihat the average human adult takes in approximately
3500 gallons of air a day. Just thought you might like to know.

Choice B is correct. To be water-soluble, the compound should be able to form hydrogen bonds. Choices C and D
are eliminated immediately, because they are hydrophobic. Although choice A has an alcohol group, it is
1S
primaril;z organic. Choice B has two hydroxyl groups and an amide group. All of these functional groups form
hvdrogen bonds, so choice B exhibits the greatest amount of hydrogen-bonding. The best answer is choice B.

'-'vright O by The Berkeley Revien'@ 8l Section I Detailed Explanations


Choice D is correct. The best micelle has an ionic (charged) head and a long carbon chain for its organic tail.
47.
Choices A and C are eliminated, because they have uncharged heads. Choice D is a better answer than choice
B, because it has a longer organic tail.
E
48. Choice D is correct. A micelle enhances the water solubility of a compound that is normally insoluble in water.
This question therefore is asking for the least watersoluble compound. An alcohol, a carboxylic acid, and an
amine all exhibit hydrogen-bonding (although within a tertiary amine, there is no protic hydrogen for
hydrogen-bonding), so they should all be water-soluble to some degree. Because an alkene has no hydrogen-
bonding (it has neither a lone pair nor an electropositive hydrogen), it is unlikely that it would be water-
soluble at all. The best answer of the given choices is therefore choice D.

49. Choice A is correct. The organic tails of micelles are held together by the weak attraction associated with van
der Waals forces (choice A). The organic tails are alkyl-based, so they are nonpolar, and they contain neither
nitrogen, oxygen/ nor fluorine. This means that choices B and C are both eliminated, because to form hydrogen
bonds, a compound must have an electropositive hydrogen bonded to either nitrogen, oxygen, or fluorine. Choice
D is eliminated, because covalent bonds are formed in chemicai reactions, and the organic tails in micelles
exhibit only attractive forces, nothing as strong as covalent bonding. The best answer, and thus choice to make,
is choice A.

50. Choice D is correct. Hydrogen-bonding weakens the covalent bond to hydrogen and thus makes the bond easier
to vibrate. This means that as the degree of hydrogen-bonding to a protic hydrogen increases, the IR
absorbance for the bond decreases in energy (in terms of wave numbers) and the peak broadens (showing a
variety of strengths associated with the hydrogen-oxygen covalent bond). The greatest amount of hydrogen-
bonding is found with the carboxylic acid, as shown by the smallest wave number and broadest absorbance in
the IR. The best answer is choice D.

51. Choice A is correct. The broadest peak is associated with the compound having the greatest amount of
hydrogen-bonding. As is observed in alcohols, the amine with the least steric hindrance exhibits the greatest
amount of hydrogen-bonding. The least steric hindrance is found in ammonia. The best answer is choice A. As a
point of interest, the tertiary amine has no N-H covalent bonds, so it has no hydrogen-bonding.

\) Choice A is correct. The relationship between bond. strength and IR absorbance is that the lower the absorbance
value in the IR (as measured in cm-1), the lower the energy associated with the stretching vibration of the
bond. The lower the energy necessary to stretch a bond, the lower the energy necessary to break the bond, and
thus the weaker the bond. Longer bonds are usually weaker bonds. Thus, as bond length increases, the wave
number of IR absorbance decreases. This eliminates choices B and D. An increase in the degree of hydrogen-
bonding weakens and thus lengthens the bond. This eliminates choice C and makes choice A the best answer.

53. Choice D is correct. Of the choices, only primary amines have a protic hydrogen, which means that only
primary amines exhibit hydrogen-bonding. The best answer is choice D.

54. Choice B is correct. The strongest hydrogen bond comes from the more basic lone-pair donor (found on the
nitrogen atom, which is less electronegative than oxygen) being donated to the most protic hydrogen (found
covalently bonded to the oxygen). This makes choice B the best choice.

55. Choice D is correct. The addition of dimethyl sulfide to solution reduces the degree of hydrogen bonding
exhibited by the alcohol, because less alcohols will be adjacent to one another to form hydrogen bonds. The
absorbance associated with a hydroxyl peak sharpens with the reduced hydrogen bonding. Associated with
reduced hydrogen bonding is a stronger covalent bond and thus an IR absorbance with a higher wave number.
Pick choice D for optimum correctness satisfaction.

56. Choice A is correct. The IR absorbance of a covalent bond is affected by hydrogen-bonding as stated in the
passage/ so Statementl is not true. As hydrogen-bonding increases, the covalent bond lengthens, so Statement II
is true. The acidity of a proton increases with hydrogen-bonding, because the covalent bond to hydrogen is
weakened. This is why acidity is higher in water than in other solvents. This makes Statement III true. The
only not true statement is Statement I. The best answer is thus choice A.

Copyright @ by The Berkeley Review@ a2 Section I Detailed Explanations


57. Choice D is correct. From the data in Table 1, it can be seen that branching in a hydrocarbon increases its octane
rating. The greatest amount of branching is observed with 2,2,4-trtnethylpentane, choice D, which can be
inferred from reading that 2,2,4-trimethylpentane has an octane rating of 100, higher than the straight-chain
hydrocarbons. Don't be a dodo, pick D.

58. Choice D is correct. According to Figure 1, kerosene has a boiling-point range of 175"C to 280"C, so the
component most likely to be found in kerosene should have a boiling point in that range. The four answer
choices are saturated hydrocarbons of eight, ten, ten, and fourteen carbons. The eight-carbon compound should
be found in the petroleum range, and the ten-carbon compounds are probably found in the petroleum-to-naphtha
1 range. You should use test-taking iogic to eliminate choices B and C, because their boiling points are simiiar,
r given that their molecular masses are identical and their structures are similar. The best answer is choice D,
,r with the highest molecular mass (and thus the higher boiling point) of the choices. To make an estimate of
e the boiling points for both n-octane and n-decane, you can use the trend in other straight-chain hydrocarbons,
S where n-hexane has a boiling point of 69"C and n-heptane has a boiling point of 9B'C. Following this trend
t, predicts that n-octane has an approximate boiling point of I25"C - 130"C and n-decane an approximate boiling
point of 775'C - 180"C. The branching of Z,2-dimethyloctane reduces the boiling point from that of n-decane
(the straight-chain, ten-carbon alkane) to somewhere around 165"C to I70'C. The2,2,4,4-tetramethyldecane is
most likely to have a boiling point in the 175"C to 280"C range. Your job, should you accept it, is to pick D,
T
tr 59. Choice D is correct. From the data in Table 1, it can be seen that branching increases the octane rating of a
R hydrocarbon. For example, as branching increases, so does octane rating for the seven-carbon aliphatic
a hydrocarbons 2,2,3-trtmethylbutane > 2-methylhexane > n-heptane. This makes choice A a valid statement,
thus eliminating choice A. From the data in Table 1, it can be seen that as branching increases, the boiling
point decreases (for hydrocarbons of comparable mass). This can also be seen with the boiling points of the
seven carbon aliphatic hydrocarbons, which have relative boiling points of n-heptane > 2-methyihexane >
2,2,3-trrrnethyibutane. Choice B is a valid statement, so it is also eliminated. Due to branching, the
rf hydrocarbon with the greatest number of alkyl substituents has the greatest mass of compound occupying the
)L smallest volurne. This results in an increase in density with branching. Choice C is a valid staternent, so it is
a also eliminated. Hydrocarbons have no hydrogen-bonding, so regardless of the arnount of branching, hydrogen-
bonding neither increases nor decreases from hydrocarbon to hydrocarbon, This makes choice D an invalid
statement as to the effect of branching. You should smile brightly when you pick choice D.
:e
IC 60. Choice B is correct. As density increases for a hydrocarbon (or any gas), it does not rise as easily. This means
Ld that as density decreases, the ability of the vapor to rise (ascend the cracking column) increases. This
,-e eliminates choice A. Choice D is eliminated, because as shown in the apparatus in Figure 1, the aromatic
n- hydrocarbons are not collected in the highest chamber of the cracking column. You could have immediately
deduced that the correct answer is either choice B or C, because they are opposites and the boiling point is
listed in the diagram. As indicated by the picture in Figure 1, the hydrocarbons with the lower boiling points
are collected towards the top of the cracking column, which makes choice B the best choice. You'd be sad if you
were to choose anything except choice B.

61. Choice D is correct. Octane rating is based on the ability of a compound to distribute heat uniformly as it
rd combusts. This ability is found in compounds that are capable of reieasing their heat energy steadily over an
extentied period of tirne. The best answer is therefore choice D. Do what is best, and pick choice D. The octane
rating does not depend on the enthalpy or entropy of combustion, although the favorability of the combustion
irg reaction does. The ratio of carbon dioxide to water depends only on the number of carbons and hydrogens in the
he fuel"
rth
er. 62. Choice D is correct. Because toluene and benzene have octane ratings higher than the other six- and seven-
carbon saturated hydrocarbons, it can be inferred that aromaticity increases octane rating. Statement I is
therefore a true statement. Because toluene (methylbenzene) has an octane rating of 120 and benzene has an
he octane rating of 106, it is assumed that ethylbenzene should also have an octane rating in excess of 100.
III Statement II is therefore a false statement. Because of the brar-rching associated wrth 2,2,3-trimethylbutane, it
.is has a high octane rating. A high octane rating is a quality associated with a good fuel additive, so a branched
he hydrocarbon such as 2,2,3-trirnethylbutane is a good fuel additive. Statement IIi is therefore a true statement.
Because statements I and III are both true statements, the best answer is choice D.

Copyright O by The Berkeley Review@ 83 Section I Detaited Explanations


63. Choice D is correct. Because 2,2,4-ttimelhylpentane is a saturated hydrocarbon, all of its carbons have a
hybridization of sp3. In the final product, the carbons are all pr"r"r-,i in the form of carbon dioxide. The T
hybridization of^ carbon in carbon dioxide (CO2) is sp. This means that in this reaction, the hybridization Tt-
changes from sp3 to sp. The best answer, and on- we highly recommend to all parties interested in success, is
choice D.

64. Choice A is correct. The start of the third paragraph states that the closer the electrons within a carbon-
hydrogen bond are to the carbon nucleus, the more acidic the compound is. To determine the relative acidity,
you must make a decision about how close the electrons are to the nucleus. The passage also states that the
more p-character there is in the hybrid, the longer the bond is. Connecting the two concepts, you should reach
the conclusion that the shorter the bond, the closer the electrons are to the nucleus. This means that the less p-
character there is in the hybrid, the more acidic the hydrogen. The most acidic hydrogen is thus found on an
sp-carbon. In 3-methyl-1-pentyne, carbons 1 and 2 are sp-hybridized, but only carbon t has a hydrogen
attached. Pick choice A for optimal results. Make a note from the conclusions that sp > sp2 > sp3 for uiiaity.
sphybridized

i\
H- C: C- Ct - CH 2- CH3
I

cll3

55. Choice D is correct. The largest Ku is associated with the strongest acid. All of the choices are hydrocarbons,
so the most acidic proton is the one on an sp-hybridized carbon, as opposed to either an sp2-I.rybridized o, sp3-
hybridized carbon. Of the four answer choices, only choice D has a hydrogen bonded to an sp-hybridized
carbon, so choice D is the best answer.

66. Choice A is correct. Al1 of the compounds are depro^tonated hydrocarbons (with a lone pair on carbon), so the
strongest base is the one with the lone pair on an spJ-hybridized carbon. The only choice with a lone pair of
electrons ot-t an sp3-hybridized carbon is choice A. The iation is irrelevant to the pioblem, because it is sodium
in each answer choice.

67. Choice D is correct. NaNH2 is a base strong enough to deprotonate a hydrogen on an sp-hybridized carbon.
Although this is true, it is not critical information in solving this question. Only one answer choice can be
correct, so the correct choice must be the compound with the most acidic hydrogen. This means that this
question is reduced to asking "Which compound, of the choices listed, has the most acidic proton?" The most
acidic hydrogen is attached to the oxygen, so you had better pick D.

68. Choice C is correct. The longest carbon-carbol bond is a single bond between the two largest orbitals. The
largest of the three possible hybrid orbitals is sp3 , so the longest carbon-carbon bond is formed between an sp3-
hybridized carbon and an spJ-hybridized carbon. Choices B and D are eliminated immed-iately, because they
contain no C-C single bond. Choice A is a bond between an sp-hybridized carbon and an sp3-ltybridized carbon,
while choice C is between an sp2-hybtidized carbon and an sp3-hybridlzed carbon. An sp2-6t6rid orbital is
longer than an sp-hybrid orbitai, so choice C is the best answer.

69. Choice C is correct. The weakest carbon-carbon bond is associated with the longest carbon-carbon bond. Choice
A is eliminated, because the C-C bond is a triple bond, and triple bonds are the shortest of carbon-carbon bonds.
Choice B is between an sp-hybridized carbon and an sp3-hybridized carbon, choice C is between an sp2-
hybridized carbon and an sp3-hybtidi"ed carbon, and choice D is between an sp2-hybridized carbon and an sp2-
hybridized carbon. Choice C is the longest, so it would terrific if you would pick choice C.

70" Choice A is correct. The iowest pK6 is associated with the strongest base. Because the most acidic proton is
found on an sp-hybridized atom, the strongest base must be a lone pair on un sp3-hybtidrzed atom. Choices C
and D are spz-hybridized nitrogens, so they are both eliminated. Choice A is better than choice B, because the
lone pair of electrons on nitrogen in choice B is tied into resonance r,r'ith the adjacent alkene n-bond. Electron-
withdrawing resonance reduces a compound's basicity.

Copyright O by The Berkeley Reviewo a4 Section I Detailed Explanations


Choice D is correct. For a reaction to have an equiiibrium constant greater than 1.0, the reaction must be
favorable in the forward direction as written. A favorable acid-base reaction proceeds from stronger acid to
weaker acid in the forward direction as written. The larger the equilibrium constant, the more favorable the
reaction, so the strength of each acid can be inferred from the K"O values in the table. In choice A, the reaction
proceeds from the weaker acid (C13CH with KuO of 8.0 x 10-9) to the stronger acid (H3CCH2SH with K"O of 5.2
x t0a). This means that this reaction is unfavorable and thus has u Keg < 1. Choice A is therefore eliminated'
In choice B, the reaction proceeds from the weaker acid (H3COH witfi K"O of g.g x 10-1) to the stronger acid
(H3CNO2 with K"O of 8.2 x 104). This means that this reaction is unfavorable and th.y_s \a1 l Sgq < 1. Choice
B is therefore eliminated. In choice C, the reaction proceeds Jrom the weaker acid (H3CCOCH3 with Kg* of
2.0 x 10-4) to the stronger acid (H3COH with K"O of e.g x 10-1). This means that this reaction is unfavorable
and thus has a K"O < 1. Choice C is therefore elimlnated, It is only in choice D that the reaction proceeds from
a stronger acia 6IiCCOCH2COCH3 with Keq = 1.2 x 106) to a weaker acid (CI3CH with Kuo = 8.0 x tg-v;. The
correct answer is choice D.

Choice C is correct. Reaction 2, the experimental reaction from the passage, involves the protonation of CSHS-
to form CSHO. This question asks for the reverse reaction. This means that any acid that shows an equilibrium
constant less than 1.0 has a conjugate base that is strong enough to deprotonate C5H6. Choices A and B are
eliminated, because they are acids, not bases. Because only methanol (CH3OH) shows an equilibrium constant
less than 1.0, only methoxide anion (CH3O-) is strong enough to deprotonate C5H6. The best answer is choice C.

Choice D is correct. The reaction as drawn proceeds from the stronger acid to the weaker acid, therefore the
equilibrium constant is greater than 1.0. When the equilibrium constant is greater than 1.0, the products are in
higher concentration at equilibrium than the reactants. This means that H3CCH2S- is in higher concentration
than H3CO-. This eliminates choices A and C. To distinguish choice B from choice D, the initial
concentrations must be known. Because H3CCH2SH and H3CO- are mixed equally initially, they must be
equally concentrated at equilibrium, The best answer therefore is choice D'

Choice B is correct. The conjugate base of the 1,3-cyclopentadiene species has six n-electrons in a continuous
cyclic planat affay of p-orbitals. These conditions result in aromatic stability. The best explanation for the
relative ease with which the 1,3-cyclopentadiene loses its proton is the aromaticity associated with the
conjugate base ( 1,3-cyclopentadienyl anion). The more stable that the conjugate base is, the stronger the acid
L-
is. Pick choice B and be satisfied.
e
.S
Choice A is correct. The weaker of the two acids has the larger of the two pKn values. This question is asking
for the weakest acid relative to the strongest acid. As the acid gets weaker, the reaction with CSHS- becomes
less favorable, so the equilibrium constant for the reaction gets smaller. C13CH shows the Iowest equilibrium
constant of the answer choices, so it is the weakest acid and thus has the highest pKo value. It is only in choice
A that CI3CH is listed as having the highest pK3 value, which makes choice A correct.

Choice C is correct. A Lewis acid is an electron-pair acceptor. The classic example of a Lewis acid is choice D,
BF3, with highly electronegative fluorine atoms and an empty p-orbital that can readily accept electrons.
'i\ This makes the boron severely eiectron-deficient. Both CH3CI and CH3N 02 are listed as acids in the table, so
choices A and B are not good choices. NaCH3 cannot accept a lone pair, but instead readily donates a lone pair.
This means that choice C is not a Lewis acid, and in fact is a Lewis base. Pick C to be terrific.

Choice D is correct. Using Equation 1, the K"q for a reaction is found by taking L0 to the power of the product
acid pKu minus the reactant acid pKu. In the itandard reaction, C5H6 is the product acid andits^pKuvalue is
10 0)
given as fS.0" If the pKu of the reactant acid is 10.0, then the equilibri.tm cor-rsiar-,t wouid be 10(i5'0-- = 195'
ihe question is thereiore: "Which acid in Table t has an equilibrium constant of roughly 105?" The best answer
is choice D, CH3NO2, with an equilibrium constant of 8.2. x 104 when it reacts with C5H5-.
.ls
iC
ire -i.
rn- Choice A is correct. Reading from Table 1, the fastest reaction rate corresponds to the lowest negative log
value. Of the answer choices, the slowest reaction is observed with an alkoxide in an aprotic solvent. The 1.44
makes it the fastest reaction rate of the choices offered to you. The best answer is therefore choice A.

- -pyright @ by The Berkeley Review@ a5 Section I Detailed Explanations


79. Choice B is correct. Ethyl sulfide would react with 2-iodopropane just slightly more slowly than methyl
sulfide, so the negative log of the reaction rate would be slightly higher tnai lsir for the ethyl sulfide. The
best answer is choice B, 2.04. Following the trend for ether s6lvent silows a negative 1og value for HS-
of I.76
and for H3CS- of 7.97. It can be conciuded from this that the negative log of-rate for "H3CCH2S- should be
greater than 7.97, but no iarger than about 2.2. The only choice in this .utlgJi, 2.04. The tact of linearity in the
trend is due to the fact that the change in steric hindrance from H3CS- to u3CCu2S- is less drastic than the
change from HS- to H3CS-. The difference il the change in steric hindrance can be seen in the smaller change
i1
negative log value for the reaction rate.

80. Choice C is correct. Because bromine is smaller than iodine, bromine forms a stronger bond with carbon than
iodine, so bromine is a worse ieaving group than iodine. Tl-re rate of the reaction dJpends on the electrophile.
The worse the leaving group, the worse the electrophile, and the slower the nucleophilic substitution reaction.
The bromine leaving group wotild yield a slower reaction (a decrease in the ,"u.tior-, rate) than the iodine
leaving group, and consequer-rtly a larger negative log of the reaction rate. The best answer is thus choice C.

81. Choice B is correct. The weakest bond is the one that would be broken in a nucleophilic substitution reaction,
such as Reaction 1. In Reaction 1, the bond that is broken is the one between carbon und iodi.,", thus the
C-I bond
must be the weakest of the choices. Pick choice B. Choice A should be eliminated, because the H is not the
leaving group in Reaction 1. Because a C-O bond and C-S bond are formed when a C-I bond is broken in
separate reactions listed in Table 1, it can be inferred that both the C-O and C-S bonds are stronger
than the
C-I bond. This eliminates choices C and D and further supports choice B as the best answer.
82. Choice A is correct- As the solvent is changed from ether to ethanol and finally water, the degree of hydrogen
bonding in solution increases. It can be observed from the data in Table 1 that the reaction rate decreases. The
conclusion must be that hydrogen bonding decreases the reaction rate. Choices C and D are eliminated. The

83.
rate must decrease due to hindrance of the nucleophile, not enhancement. Your answer is choice A"

Choice D is correct. According to the data in Table 1, regardless of the solvent, the negative 1og of the reaction
iE
r!9lti
rate is greater with H3CS- than HS-. This means that changing the nucleophile from HS- to H3CS- decreases
the reaction rate. Choice A is therefore eliminated. the negitive log values of the rate are lower with the
ether solvent than the water solvent, therefore water solvent must decrease the reaction rate. This eliminates
choice B. Decreasing the temperature always produces a clecrease in the reaction rate, so choice C is eliminated
too. The negative log values of the rate are greater with the water solvent than the ethanol solvent, so
ethanol solvent must increase the reaction rate. That makes choice D the best answer. This question requires
determining the relationship between the reaction rate ancl negative 1og of the rate.

84. Choice C is correct. In ether, aikoxides have lower negative 1og values for their reaction rates than alkyl
sulfides, so the alkoxides must react faster. This eliminates choices B and D. The passage states that the
difference in reactivity can be attributed to alkoxides being better bases than alkyl sulfides. The stronger the
base, the lower the pK6 value. Pick C and feel jovial for just a moment, at least until the next question stirts.

85. Choice C is correct. The most basic species is the compound containi-ng nitrogen. In general, nitrogen compounds
are more basic than oxygell- and sulfur-containing co-mpounds of eqtr"at hybiidization. This eliminates choices A
and D. The degree of substitution is irrelevant. Esters have no lone pair of electrons that can be readih-
donated to a proton, so choice B is not correct. The best answer is an amine, independent of whether it is
primary, secondary, or tertiary. This means that you really should pick C for the sensation of correctness.

86' Choice C is correct. The rate referred to in Figure 2 is for a nucleop}rilic substitution reaction at a carbonyl site.
There is a direct corelation between nucleophilicity and the rate of reaction. The graphs show that above a
pH of 7, as the pH increases, so does the reaction rate. This mea11s that ihe nucleoptritcity increases. As pH
incteases, compounds are no longer cationic. This eliminates choices A ancl B. Size ls noi applicable here, so
choice D is eliminated. Afier eliminating the wrong choices, vou shor,rld settle for choice C as the best answer.
(
dt

L
87. Choice D is correct. At pH = 9.0, all of the compounds should be neutral (although the amine in choice D mar-
.il1
have a small fraciion that rernains protonated). Tl-re most reactive compound is the best nucleophile. For
nucleophilicity, an arnine is better than an ether, an ester, or an arnide. Foi this reason, pick D.
Copyright @ by The Berkeley Review@ Section I Detailed Dxplanations
'-l
Choice C is correct. The chlorine atoms are electron-withdrawing by the inductive effect (chlorine is more
IE electronegative than carbon). Electron-withdrawing groups make the compound more acidic and thus lower its
-bt
pKu value. Both choice C and choice D are lower than 10.3. Choice C is the better choice, because the
te inductive effect is not so substantial that it will make the ammonium cation that acidic. For the pKu to drop
1e down to 1.0 would mean that the three chlorine atoms on the methyl group increased the acidity by a factor of
',e 109'3 = 2 x 709 = 2,000,000,000 times. That is too much. Be conservative and pick C.
in
Choice D is correct. The sum of pKu + pKU for a conjugate pair in water is equai to 14.0 al25"C. This means that
pK6 for NH3 (the conjugate base of NH+*) is equal to 14.0 - 9.25 = 4.75. Pick D and score with the best of them
(whoever they are).

Choice A is correct. The hybridization of carbon in a carbonyl compound, such as an ester (which contains one 7[-
bond), must be sp2 1th" n-bond requires one p-orbital, so only two p-orbitals remain for hybridization). This can
also be deduced from the trigonal planar structure of the carbonyl compound. The hybridization of carbon in the
tetrahedral intermediate (which contains no lr-bonds) is spJ. The final product again has the carbonyl
functionaiity, only now with the nucleophile attached. Tl." carbonyl product still has trigonal planar
geometry. The hybridization therefore changes from spt to spJ and back to spt in the overall reaction, making
choice A your choice. Make that choice today!

Choice A is correct. In each case, there is a nitrogen with one alkyl group and two hydrogens. This defines a
primary amine , so both compounds are primary amines. It just so happens that the R-groups are aromatic rings/
but they are not aromatic amines per se, because the nitrogen atom is not a part of the aromatic system. The best
answer is choice A. Of the two amines, only one is conjugated (aniline), so choice C does not describe both
structures.

Choice D is correct. Alcohois exhibit hydrogen-bonding, which increases their intermolecular forces. The
stronger forces make it harder to move a molecule from the liquid phase into the gas phase. This raises the
boiling point of an alcohol compared to a molecule of comparable size, so choice D has the highest boiling
point. Molecular mass is of concern as well, but choice D is also the heaviest of the choices.

Choice B is correct. The longest chain is eight carbons, so based on that alone, you know that the best answer is
choice B (octane).All you need to do is find the longest chain to decipher the correct answer choice.

Choice C is correct. Conjugation is defined as consecutive, alternating n-bonds. The structures are drawn below.
L,. OnIy Compound III has conjugation, so choice C is correct.
.i._
:he

il
^ i-
1,,4-cyclohexadiene 3-methylcyclohexene 2-methyl-1,3-cyclopentadiene

Choice C is correct. Choice C is the most stable compound, because it is the only diene that has conjugation.
Note that the structures are straight chains and not rings. It is easy to insert the word "cyclo" inadvertently
rl.)' into the name. Avoid careless mistakes and choose C. Drawn below are the structures of all four choices:
_:-
- 1:
2-methyl-1,4- 3-methyl-1,4-pentadiene 2-methyl-1,3-pentadiene 1,5-hexadiene

'ite,
\4,/ \,. ,.^
I
';H
Choice D is correct. Saying that a compound has a dipole that is not equal to zero is equivalent to saying that
the compound is polar. Cis compounds (both alkenes and cyclic structures) are aln'ays polar. This makes both
Compound I and Compound III polar. You need not even examine Compound II, because no answer choice includes
all three compounds. Pick choice D to score more MCAT points.
F..l

LOns - :vright @ by The Berkeley Review@ Section I Detailed Explanations


97. Choice D is correct. All alcohols have hydrogen-bonding, which increases their intermolecular forces and thus
increases their boiling points, so choice D has the highest boiling point. Note that all of the compounds have
exactly the same formula (and thus the same molecular mass). This eliminates the need to accbunt for any
differences in molecular mass (which would also affect the boiling point). Because of the linear nature of
choice B and its asymmetry (which makes it more polar than the two remaining choices), it should have the
second-highest boiling point.

98. Choice C is correct. For a compound not to be polar, it must be s).'rnmetric. Cis compounds are asynunetric about a
point (although they may have mirror-plane symmetry, rather than point symmetry), and thus are always
polar, so choice A is eliminated. This leaves choices B, C and D as possible answers. An odd-numbered ring
must be polar when it has two substituents, so choice B is polar and thus ruled out. In choice D, the middle
carbon has varying substituents attached (not all four groups are identical), so it cannot be symmetric, thus it is
polar, too. By eliminating three choices, choice C must be the correct answer. Drawing choice C out shows that
the individual dipoles for the bonds cancel each other out, making the compound nonpolar.

E-1,4-dichloro-2-butene

The individual vectors cancel out, so there is


no net vector. The compound is nonpolar.

99. Choice B is correct. The most heat is generated by the least stable compound; thus finding the least stable
compound is the task at hand. All of the choices have the same formula (C6Hg) so it comes down to structural
features. The four-membered ring is unstable, so choices A and B are good. Choice B has no conjugation, while
choice A does (conjugation is a stabilizing feature), so this makes B the least stable compound a*ot g the answer
choices. The bond angles are not the optimal 109.5", whether the n-bond is in the ring or not.

100. Choice C is correct. Translating from the chemical formula into the structure yields the compound below:

H3CH2C

H3CO H H CFI3
There is no aldehyde group (which would have been represented as CHO), so choice A is eliminated. There is
no ester group (which would have been represented as CO2R), so choice B is eliminated. There is a carbonyl
adjacent to two alkyl groups, so the compound has a ketone functionality. This eliminates choice D and makes
choice C the best answer.

Copyright @ by The Berkeley Review@ Section I Detailed Bxplanations


lsomerism
a) Isomers

Section II i. Constitutional Isomers


ii. Stereoisomers
iii. Configurational lsomers
iv. Optical and Geometrical Isomers
Structure b) Conformational Isomers
i. Eclipsed vs. Staggered Conformation
ii. Oauche vs. Anti position
Elucidation Newmanprojections
!ii. Cycloalkanes
iv.
by Todd Bennelt v. Cyclohexane
vi. Chair and Boat Conformations
vii. trquatorial vs. Axial Orientation
Structural trnsights
a) Structural Symmetry
H:C
b) Units of Unsaturation
Spectroscopy and Analysis
a) Qeneral Spectroscopy
b) Infrared Spectroscopy
i. Theclry and Key Signals
ii. CommonAppplications
iil Hydrogen-Bonding
c) Ultraviolet Spectroscopy
l. I heory
ii" Common Appplications
d) NMR Spectroscopy
i. Theory
ii. Structural Symmetry
iii. Proton NIVIR
iv. Shift Values
v. Splitting Pattern
oi: Signal Integration
vii. Structural Features

RERI{EIEY
L)K.E'v.r.-b.wt
Speci ahzing in MCAT Preparation
l
Structure Elucidation
Section Goals
Be able to identifv isomers from both their structure and their name.
There are several types of isomers. Be familiar with structural isomers (identified by different
connectivity), stereoisomers (identified bv asymmetry), optical isome-{s (and their ability to rotate
plane-polaijzed lieht), geometrical isom'ers (found #ith rings and alkenes), and conformational
isomers (identified"by ro-tation about bonds or ring-flips). You should know how the different types
of isomers are related to one another.

Be able to identify the more stable chair conformation for six-membered rings.
Cvclohexane and pyranose sugars involve three-dimensional ring structures. The most stable
conformation result3 in the leaEt steric hindrance. As a general rule, axial orientation results in
greater steric hindrance than equatorial orientation. Know the difference in stability between axial
Srientation and equatorial oriehtation. Recognize the steric repulsion associated *ith 7,2-diaxial,
1,3-diaxial, and 1,4-diaxial orientation.

Be able to identify the more stable Newmann proiection for a compound.


@3 Structures can orient themselves in a staggered conformation, an eclipsed conformation, or some
conformation between eclipsed and staggered. You must be able to identify the most stable orientation *t
for a structure and distinguish between gauche and anti positions.
,-11-

Be able to use the molecular formula to determine the units of unsaturation. *I-

@3 Some auestions reouire vou to determine the potential functional groups of a molecule. The presence -:ii
of eithbr a n-bond'and 1 rine within a strucfure results in a uniFof dnsaturation, which manifests rM':lT

itself as two fewer hvdroeefrs in the formula. A fullv saturated hvdrocarbon or carbohydrate has
a total of 2n + 2 hydrdgen itoms in the compouLnd, where n is the nurhber of carbons in the fompouLnd. :tr:I,

Jtll

Be able to translate structures from two dimensions into three dimensions.


I'L

[ht*
Know what the terms staggered, eclipsed, gauche, and anti mean, and be able to draw structures
in the Newmann projectidi to show ihe orlentation of substituents in these structures. Be able to arLi:r:

rotate about o-bonds.


ll l:-i,

t? Be able to deduce structural features using IK spectroscopy.


You should have a basic understanding of the operations of an infrared spectrophotometer. You
f,
Jllli.ru
JIl

must know the IR stretches for a carboiyl and a hydroxyl bond. You shou-ld be able to determine l8,itr
which structural features correspond to #hich IR absorbairces. You must be able to eliminate and/or lll [.i ri

confirm possible structures, using IR data. You must be able to decipher IR spectroscopy graphs slru
and identify the key peaks.
ItI l[

Be able to deduce structural features using NMR sPectroscopy. ffir*


You must know the NMR shifts for carbonyl compor-rnds, alkene compounds, and aromaLic compounds.
You should be able to determine the struch-re of in unknown compor.rnd using the spechal information ;r Ii|]U
from the NMR. Most structures you will encounter on the MeAT are sm"all and symmetrical, so
thev are easilv solved. You mustbe able to eliminate incorrect structures based onNMR data. Be
s'.rri: to underitand. what the shift value (measured in ppm) tells you, what the integration tells you, ryr*
and what the peak shaoe and couplins constants tell vou. Each piece of information can be used rux
to help deterniine the s'tructure of in u"nknowt-, compound. Use these data in coniunction with the rllietu
units of unsaturation. 1ll[ir'u

]L
LJ u
4$r
W,ot!,r
$mfigll
Organic Chemistry Structure Elucidation Introduction

structure elucidation involves applying ail available information, from


spectroscopic data to chemical reactivity, to ascertain the three-dimensional
shape of a molecule. It entails determining the atoms within the molecule,
the
functional groups present on the molecule, and in advancecl cases,
the three-
dimensional folding of the structure. Structure elucidation involves determining
the number of isomers that fits a molecular formula and then systematically
eliminati.g isomers that do not fit the data untit, only one structure remains.
In this section, lve shall address the concept of isomerism and the many
classes of
isomers. Isomers have the same atoms within the molecule, but they differ
in
some manner, so that the molecules are not superimposable on one
another. The
difference could result from different bonding *il-,ir-, the molecules,
similar
bonding but different three-dimensional distribution about a stereogenic
center,
or the same bonds and stereogenic syrnmetry with different conformational
orientation. A significant part of structure elucidation is determining the
exact
isomers that are formed in a chemical reaction.
Other structure elucidation tools shall be discussed.
Questions that involve
structure elucidation are often made easier by first determining the
units of
unsaturation from the molecular formula of a compound. Thii information
provides hints as to the presence of n-bonds and./or iings within the
structure.
Chemical tests can be carried out to determine the nuriber of rc-bonds,
which
h'hen combined with the units of unsaturation, can specify the exact
number of
rings and ru-bonds within a molecule.
In this section we shall also address spectroscopy a.d the information
about
structure it can provide. I'frared (IR) spectros.opy ir typically used
to determine
the functional groups within u .o*por'rd. It iin atso give some information
about the of the molecule, the hybridizatiJn of carbon, and the
_symmetry
presence of groups capable of forming hydrogen bonds. Ultraviolet-visible
(uv-
Yis) sPectroscopy tells us information about the n-bonds and
conjugation within
a molecule' Although all molectiles absorb ultraviolet radiation,
for practical
irurposes/ we use it only to detect n-bonds. Nuclear magnetic ,"ror-rur.,.u (NMR)
spectroscopy describes the connectivity of a molecule and its specific
structural
reatures. In its simplest application, NMR can show the carbon skeleton
of a
molecule. In its more sophisticated application, NMR can show the presence
of
stereoisomers and the exact positions of functional groups. we shall address
:roth carbon-13 and proton NMR. combining xvn data with uv-vis
spectroscopy and IR spectroscopy data allows for precise determination
of three-
limensional molecular structure.
-t is best to review NMR with symmetry as your focus. The question ',How can
' ou distinguish compounds by NMR?" .ur-r b" reduced to ,,How many different
:r'pes of hydroge^s are there in each compound?" Multiple-choice NMR
questions can be answered easilv by predicting the spectra from
possible
:tructures. For instance, if you can narrow down the potential struciures to
(etones, then it's just a matter of systematically eliminating
ketones that do not fit
:he spectral data. This is the perspective from which w.-e will approach
NMR.
i he abiiity to predict spectra from structures is best attained thiough practice.
-\s you do the multiple-choice qtiestions il the spectroscopy sections,-pr"di.t th"
spectra for the structures in this same manner. the cliiference belween the
:pectra in each answer choice (A,8, c, or D) is what often answers the question.

Copyright O by The Berkeley Review Exclusive MCAT Preparation


Organic Chemistry Structure Elucidation Isomerism

Isomerism
Isomers
Isomers are structures with the same formula, meaning they are made of the
exact same atoms, but they differ in the location of each atom. The difference in
position can be the result of different connectivity (bonds), different spatiai
arrangement because of asymmetry in the structure, or different orientation
about a bond. The result is that there are several different types of isomers.
Figure 2-1 shows a flow chart for determining the type of isomers.

ISOMERS
Structures made of the same atoms

Constitutional (Structural) Stereoisomers


Differ in connectivity (bonds) Differ in spatial arrangement of atoms

Conformers Configurational
Di f fer by orieniaion in space Differ by orientation in space
Identical after rotation about o-bond Can't rotate to become identicai

Optical Geometrical
Differ by orientation in space Differ by orientation in space (
Can't rotate to become identical due to Can't rotate to become identical due to (
asymmetry in the structure the presence of a ring or n-bond m
qn

rt
Diastereomers Enantiomers m
Nonsuperimposable and not Nonsuperimposable mirror images &
mirror images ffi
@
Figure 2-1 ffi
m
Constitutional isonrcrs, which have different bonding, ate more commonly
referred to as structtLrsl isomers. Structural isomers are most easily recognized by
their difference in IUPAC name. The difference may arise from the functional
groups (like an alcohol versus an ether, or a ketone versus an aldehyde) or it may
arise from the connectivity of the carbon backbone (like 2-methyihexane versus
3-methylhexane). Structural isomers can be further divided tnto functional group
islnrcrs, positionnl isonrcrs, and skeletal isonters.
Stereoisorners have exactly the same bonds (and therefore the same connectivity),
but they differ in the spatial arlangement of their atoms. On a more general note,
stereoisomers can be categorized as either configurationnl isomers (which differ in
spatial arrangement and cannot be converted into the other isomer without
breaking a bond) or conformational isomers (which differ in spatial arrangement
but can i:e converted into the other isomer by rotation without breaking a bond.)
Within configurational isomers, there are optical isomers (isomers that rotate
plane-poiarized light differently) , geometrical isomers (isomers that vary in
orientalion about a n-bond), ensntiorners (nonsuperimposable mirror images), and
diastereomers (nonsuperimposable and not mirror images). Configurational
isomers are most easily distinguished by their IUPAC prefix. The IUPAC prefix
contains either R or S, if the isomers differ in chirality at a stereocenter, or E and
Z, if the isomers differ in their arrangement about a n-bond. We shall address
stereoisomers in detaii in later sectrons'

Copyright @ by The Berkeley Review 92 The Berkeley Review


Organic Chemistry Structure Elucidation Isomerism

\vhat types of isomers are 2-methyl-3-pentanol and 3-methyl-2-pentanol?


A. Conformational isomers
B. Geometrical isomers
C. Structural isomers
D. Stereoisomers
S ol uti on

The two compounds have different IUPAC names, so they are structural isomers.
The two structures vary in the position of their alcohol and side chain methyl, so
:hey are also positional isomers. The question was not that specific, so the best
.rnswer is choice C. The two structures are drawn below:

3-methyl-2-pentanol 2-methyl-3-pentanol

-
Constitutional Isomers
constitutional isomers (also referred to as structural isomers) are unique
molecules that have the same formula, but different connectivity. In other
rgords, they have the same atoms, but the atoms have different bonding. For
tlstance, 3-metllylhexanal and 3-methyl-2-hexanone are constitutional isomers.
They each have the formr-rla c7H140, but they have a different sequence of
oonds. They can also be referred to as positional isomers. Using nomer-rclature
l'relps to determine whether two structures are consiitutional isomers, because
:onstitutional isomers must have different IUPAC names. Figure 2-2 shows
three pairs of structural isomers, one set of functional group isomers, one set of
positional isomers, and one set of skeletal isomers.

stntcturnl: Different arrangement of atoms (i.e. different bonds)

HoM Ho- cH2cH2cH2cH3 & H3cH2c- o- cH2cH3 , o .


1-butanol diethyl ether

,
LI
I

=/\
'l
I{1C- CH-
cl
CH2CH2CH3

2-chloropentane
&
HICCH2 - CH CH2CHT
I

LI
3-chloropentane
m C1

CHq H3C- CH- CH2CH2CH3 H:C- CH- CH- CHI CHr


I-
-l-.-^-. CHa &
lt
CHs CHa
2-methyipentane 2,3-dimethylbutane CHs

Structural isomers have different IUPAC names.

Figure 2-2

Copyright @ by The Berkeley Review Exclusive MCAT Preparation


Organic Chemistry Structure Elucidation Isomerism
(

Example 2.2
How many possible constitutional isomers exist for a molecule with the
molecular formula CaHlg?
A. 1
8.2
c.3
D.4
Solution
The maximum number of hydrogen atoms possible on a four-carbon alkane is
ten, so there are no units of unsaturation in C4H1g. This means that there are no
n-bonds or rings in the molecule. To solve this question, chains of varying
carbon connectivity (skeletons) must be considered. There is always one longest
chain structure (C-C-C-C). There is also the possibility of a three-carbon
chain with a methyl group ofi of the second carbon (if the methyl were on the
first carbon, it is still butane). This means that there are two constitutional
isomers for C4H1g, butane and 2-methyipropane. Pick choice B for the smile that
a correct answer brings.

Example 2.3
Which of the following pairs of molecules is NOT a set of constitutional isomers?
A. 2-Methylpentane and 3-methyipentane
B. Cyclobutanol and tetrahydrofuran
C. 1-Chlorobutane and 2-chiorobutane
D. 4-Ethylchlorocyclohexane and 3-methylchlorocyclopentane
Solution
In choice A, both compounds have the formula COH1+ and different IUPAC
names, so they are constitutional isomers. In choice B, both compounds have the
formula CaHgO and different IUPAC names, so they are constifutional isomers.
In choice C, both compounds have the formula CaHgCl and different IUPAC
names, so they are constitutional isomers. In choice D, the first compound has
Ttu$
the formula C3H15C1, while the second compound has the formula C6H11Cl, so Shfls
they are not even isomers, let alone constitutional isomers. This makes choice D 0m!um
the correct answer. dnr:mrln

lrMffi
htw un
For a given formula, tirere is a finite number of possible structural isomers. The
Will-
number of possible structural isomers depends on the molecular formula. fieffiD
Saturated aliphatic compounds (linear alkanes) are the simplest case. For each mfrems
extra carbon, the number of structural isomers increases. For instance, C3Hg has mr{
only one structural isomer, while C6H14 has five different structural isomers. It dtfimd
is important to realize that both formulae (CgHs and C6H1a) are for structures ru[
that are fully saturated (have no units of unsaturation). There is no easy formula -;.*-
@!-'0-
for determining the number of structural isomers possibie for a given formula, illifirrfnt
but there is a systematic way to determine the number. Figure 2-3 shows all of J;-
ut.E -
the structural isomers for C3Hg, C+HfO, CSHfZ, and C6H14, and lists them in nery:l
terms of charn length and substituent location. pirffiii
* lllcmr
au]l I
$riffwe
nUemr:

Copyright @ by The Berkeley Review The Berkeley Review i[r_,Mplilu


Organic Chemistry Structure Dlucidation Isomerism

C3H8 (1 total): CaHro (2 total): HsC


I
CH.
./ '\
H:rC' CHg Hr./cHt\ ,r{"'
4-Carbon chain "ra/tt-a",
3-Carbon chain

CsHtz (3 total):

"rf HqC
"\/ CHr
-\.f
CHr -CHz'-.ur,
_CH
HrC/ -CH3
-zc\
Hrc/ -."i HSC CH:
S-Carbon chain 4-Carbon chain 3-Carbon chain

CeHr+ (5 total):

ur./cH'\.t'ttt'' .r{"t
5-Carbon chain

CHr
-- -CHz
"l
HrC/ an- - cH, "rf
.zCH'
HsC
-zcH\-cH,
Hzc' -c",
S-Carbon chain S-Carbon chain

"rl
t"\
,t"' "l
"..-t"-
,r('^u
*C/t-.t'tt'
4-Carbon
CHs
chain 4-Carbon chain
e

Figure 2-3
C
IS
This procedure of determining the number of structural isomers is systematic.
;o
First, start with the longest continuous chain of carbons (equal to the total
D
number of carbons in the formula). In the case of CoHt+, the longest possible
chain is six carbons. After drawing the longest chain, draw a carbon chain of one
Iess carbon (five carbons) and systematically deduce all of the possible isomers
by moving the methyl group across the chain one carbon at a time. In the case of
\e
C6HI4, the next chain down from six carbons is five carbons and the extra (sixth)
a.
carbon is attached to one of the interior carbons in the chain. If the extra carbon
*r
were attached to a terminal carbon, then the longest chain would be six carbons,
AS
not five. In the case of C6H14, it is not possible to have 1-methylpentane, because
It that is really n-hexane. A guideline to follow as you deduce isomers is that
ES
structural isomers must have different IUPAC names. If you are ever in doubt
1a
about whether or not two compounds are structural isomers of one another,
,4,
name them using IUPAC conventions. To complete the process of determining
of
the isomers, systematically count isomers for each possible chain length,
in
reducing the length by one carbon each time. When you are finished with eich
possible chain length, sum all of the structures and that's your answer. For
alkanes with functional groups attached, the procedure is the same except once
all of the skeletal structures are determined, there is an additional itep of
systematically placing the functional group at all unique carbons. Example 2.4
demonstrates this procedure.

:w Copyright @ by The Berkeley Review 95 Exclusive MCAT Preparation


Organic Chemistry Structure Elucidation Isomerism (

=
Example 2.4 I
How many structural isomers are possible for the formula CaH9C1? I
A.3 I
8.4 I
c.5 (
D.6 T

Solution t
For a problem of this type, the possibilities for the carbon skeleton must be T
determined first. The four carbons can either be aligned four in a row or three in u
a row with the fourth carbon coming off of the second carbon of the three-carbon ft
chain.
STEP 1: 2 possible carbon skeletons
fii
C-C-C-C C-C-C ni
I
o!
C
4-Carbon chain 3-Carbon chain ffi
q
The second step is to determine how many unique carbons each chain contains. I
STEP 2: Each skeleton has two unique carbons /i

Co- Cu- Cu- Co Co- Cu Cu I


I
-
ca
2 Unique carbons 2 Unique carbons

The last step requires placing a chlorine on each unique carbon one structure at a
time and verifying your answer by checking to see if each structure has a
different IUPAC name.
STEP 3: 4 structural isomers total

C_C-C-C C-C-C-C
I I

C1 C1

1-chlorobutane 2-chlorobutane

C_C-C C-C-C
tl /\
CIC CCl
1-chloro-2-methylpropane 2-chloro-2-methylpropane

The best choice is answer B, because there are four possible structural isomers.
This systematic procedure works every time. It is assumed that isomer problems
much beyond this example in terms of difficulty will be avoided on the MCAT
because of time constraints. The skills employed when deducing the number of
structural isomers can also be used when deducing structure from spectral data,
such as IR and NMR information.

Copyright @ by The Berkeley Review The Berkeley Review


Organic Chemistry Structure Dlucidation Isomerism

Example 2.5
How many possible constitutional isomers have the molecular formula CaHg?
A.4
8.5
c.6
D.7
Solution
These questions are time-consuming, but unfortunately, there is not a convenient
rvay around it. To start, you must determine the units of unsaturation (also
l<rrown as degrees of unsafuration).

Degreesof unsaturation =z(+)+?-(8) =8+2-8=10-8


2222 -2 - 1

\Vith one unit of unsaturation, the structure must contain either a n-bond or a
ring. To get the correct answer, you must systematically consider each linear
connectivity and each cyclic connectivity. Be sure not to count stereoisomers
i'including geometrical isomers). The alkenes are listed first, followed by the
cvclic structures.

Possible alkene structures:

4-carbon chain (2 total) 3-carbon chain (1 total)


HzC: CHCH2CH3 H3CHC: CHCH3 H. ,CH'
1-Butene C,,
2-Butene ,C=
Note that there are two possible H CHs
geometrical isomers for 2-butene.
Methylpropene

Possible cyclic alkane structures:

4-carbon ring (1 total) 3-carbon ring (1 total)

-tt
H?C- CH,
/\'
CH"

H2C- CH2 H?C-


-\ CH
Cyclobutane CHg
Methylcyclopropane
Because there are five total constitutional isomers in all, the best answer is choice
B.

Copyright O by The Berkeley Review 97 Exclusive MCAT Preparation


Organic Chemistry Structure Elucidation Isomerism {

Stereoisomers I
Stereoisomers are molecules of the same formula that have the same bonds (
(connectivity), but a different spatial arrangement of the atoms. included in fi
stereoisomers are configurational isomers (molecules that cannot be converted ru

into one another through rotation about a o'-bond) and conformational isomers &
(caused by rotation and ring-flipping). Configurational isomers can be broken d[
down further into either geometrical isomers (associated with nonrotating m[

structures, such as rings and alkenes) and optical isomers (isomers that rotate ,0u

piane-polarized light differently). Not all configurational isomers rotate plane-


polarized light, as you have seen with meso compounds, but opticai isomers
differ in the magnitude and possible direction in which they rotate incident
plane-polarized light. Stereoisomers of all types are not superimposable (they
cannot be superposed onto one another.) Figure 2-4 shows the types of
stereoisomers. Rotamers are conformational isomers that vary in orientation in
space because of rotation about a sigma bond.

Stereoisomers; Same bonds, but a different spatial arrangement of the atoms


HOH
\s--(-
&
HOH

.z
\sC:.
H^ccHz/ at, H:C CH2CHl
(S)-2-butanol (R)-2-butanol
Configurational isomers
,Q
(Optical isomers) @
,{F

CH: & .'rtl CH3

(cis)-4-methylcyclohexanol (trans)-4-methylcyclohexanol
Configurational isomers
(Geometrical isomers)
Note: Configurational isomers have different prefixes in their IUPAC names

-z
C:.
\r
HH
& H"CCH'
isC:.H

H3CCH2- CH2OH H-z CH2OH


1-butanol 1-butanol
Conformational isomers
(Rotamers)

Figure 2-4

Configurational Isomers
Configurational isomers are a subgroup of stereoisomers that have the same
bonds, but a different arrangement of their atoms in space, no matter how the
structures are twisted and rotated. Common examples with which you are
familiar include optical isomers and geometrical isomers.

Copyright @ by The Berkeley Review 9a The Berkeley Review


Organic Chemistry Structure Elucidation Isomerism

Optical Isomers
optical isomers are molecules of the same formula and the exact same bonds
'connectivity), but a different spatial arrangement of the atoms due to asymmetry
-'r'ithin the structure. An optical isomer cannot
be rotated or manipulated tb
assume the structure of another isomer. They cannot be converted into another
cptical isomer without breaking a bond. Optical isomers rotate plane-polarized
light differently from one another. Figure 2-5 shows an example of a pair of
..ptical isomers.

optical Isomers: Identical bonds with a different spatial arrangement about


an asymmetric carbon that rotate plane-polarized light differently.

H H
\
cH2cH2cH3 & Ctrrrtt- CH2CH2CHI
",?l7t- HaC-
(R)-2-chloropentane (S)-2-chloropentane

Optical Isomers

Figure 2-5

Jptical isomers are a class of configuratisnal isomers. Configurational isomers


:an also be classified as enantiomers and diastereomers, so some optical isomers
::l also be referred to as enantiomers or diastereomers.

E.rample 2.6
:{orv can the relationship between the following two molecular structures BEST
:: described?
H CIH,CH?CH?C..\
\
CH2CH2CH2CI &
CH2C\IZ - .rut7t- CH2CI

H
.4,. Identicalmolecules
B. Optical isomers
Ll. Skeletal isomers
f. Structural isomers
Solution
',-e can start by naming each of the structures. Both have three chlorine atoms on
. iive-carbon chain. The chlorine atoms are on carbons 7, 2, and 5, so each
: rlecule has the IUPAC name 1,2,5-trichloropentane. To be skeletal or structural
-:-rmers requires that the two compounds have different IUPAC names, so
-:-oices C and D are eliminated. No matter how you rotate the first structure, it
:::rnot be superposed onto the second structure. This implies that they are not
:entical molecules, so choice A is eliminated. The left structure has s-
':ereochemistry at the second carbon, while the right structure has R-
:::reochemistry at the second carbon. This confirms that the two structures are
.:rical isomers, so the best answer is choice B.

-".pyright @ by The Berkeley Review 99 Exclusive MCAT Preparation


Organic Chemistry Structure Elucidation Isomerism

Geometrical Isomers
Geometrical isomers, simply put, are the cis and trans forms of a rigid compound
(where rigid implies that it is not free to rotate between conformations). They are
sometimes referred to as cis/trans isomers, which applies to rings and alkenes.
They are nonsuperimposable, because they are locked into an orientation by the
cyclic structure or n-bond. If two substituents on a cyclic compound are on the
same side of the plane or if two substituents on alkene are on the same side of the
carbon-carbon n-bond, then they are said to be cls to one another. If two
substituents on a cyclic compound are on opposite sides of the plane or if two
substituents on alkene are on opposite sides of the carbon-carbon n-bond, then
they are said to be trans to one another. IUPAC convention does not use cis or
trans in the naming of alkenes; instead, the letters E and Z are employed. In
general nomenclature, the terms cis and trans are common. For geometrical
isomers, which have different spatial arrangement about a n-bond, the prefix of E
is given for trans orientation of the two highest priority groups, while Z is given
for cis orientation of the two highest priority groups. Figure 2-6 shows several
examples of pairs of geometrical isomers, both alkenes and cyclic structures.

Geometrical Isomers: Identical bonds with a different spatial


arrangement (found with double bonds and rings).
Alkenes

H CH,CHTCH3 HH
\/
c:c
\/LL

& L-L
/\ /\
HsC H HsC CH2CH2CH3
Trans (E) Cis (Z)

CHg H?CO H
&

HsC H CHe
Z ("Zis")

Cyclic Systems
(uP)
cH"
(down)
(up) (up) /YcHs
HrC-H-) & HzC-_/--/
Cis Trans

(up) (t.tp) (up) (down)


Hrc\rA.rCH" H3C\.n,.,rt\CH3
\J&
Cis
U Trans

Figure 2-6

Copyright @ by The Berkeley Review loo The Berkeley Kevieu


Organic Chemistry Structure Elucidation Isomerism

The term E is derived from the German word entgegen meaning "across from"
and refers to a compound where the two highest priority groups on each
respective alkene carbon are trans to one another. The term Z is derived from the
cerman word zusnmmen meanine "together" and refers to a compound where the
two highest priority groups on each respective alkene carbon are cis to one
another. Think of cis as "Zis", and you will always remember which is which.

Example 2.7
All of the following are true about geometrical isomers EXCEpT that:
A. the E designation for an alkene refers to the highest priority groups on each
alkene carbon in trans orientation.
B. in both the E and Z isomers of an alkene, the atoms directly bonded to the
alkene carbons are all coplanar.
c. molecuies capable of forming geometrical isomers have greater entropy than
linear alkanes of equal carbon chain length
D. geometrical isomers have relatively static structural features, such as polarity
and solubility.

Solution
To determine the geometry of an alkene, first locate the two carbons that
constitute the alkene. Determine the highest priority substituent on each of the
two alkene carbons, using the Cahn-Ingold-Prelog rules for assigning priorities to
substituents by sequentialiy looking at the atoms attached to the site of interest.
If the two highest priority groups are across from one another with respect to the
louble bond, then the compound is trans and thus is assigned the letter E. If the
:rvo highest priority groups are both on the same side of the double bond, then
'&e compound is cis and thus is assigned the letter Z. This makes choice A a true
,qtatement, so it is eliminated. Because the two p-orbitals of the rc-bond are
coplanar with an orientation perpendicular to the substituents on the alkene
carbons, the four atoms bonded to the two carbons of the alkene must be
.oplanar. It is not possible to rotate around the doubie bond, because the p-
.rrbitals would no longer be coplanar, breaking the n-bond. This makes choice B
a true statement, which eliminates it. There are only two geometrical isomers
possible for an alkene. Because it is not possible to rotate about a double
bond
the ru-bond would have to be broken), it is not possible to convert between the
cis and trans geometrical isomers without adding a great deal of energy. The
.onsequence is that alkenes are rigid and thus have less entropy than alkanes of a
lomparable carbon chain length. This makes choice C a false statement, and thus
lhe best answer. The structures of both an aikene and cyclic molecule are
relatively static and do not change drastically. The ring may flip-flop a littie, but
rotation is observed only for the substituents on the ring, and not observed for
Jee bonds in the ring. Because the structure is static, the molecular features are
constant. This makes choice D a true statement, eliminating it.

lVe shall discuss stereoisomers oniy at this superficial 1evel for the time being. In
the stereochemistry and carbohydrate sections, stereoisomers will be discussed
in
greater detail.

Copyright O by The Berkeley Review lol Exclusive MCAT Preparation


Organic Chemistry Structure Elucidation lsomerism

Conf ormational Isomers


Conformational isomer s, ot conformers, are molecules with identical connectivity
(bonds) that are nonsuperimposable because of rotation about a bond or
The most
iontortion (often referr"d to u, iing-flipping) of the molecular structure'
stable conformation of tne motlculai structure is predictable based on
hybridization,stericrepulsion,andVSEPR(ValenceShellElectronPair
nlpurcion) theory. Understanding conformational isomerism is vital,
because
anl of compound. Figure 2-7 shows
strlcture ii.tut", the reactivity stability a
two pairs of conformational isomers'

Conformationnl Isomers: Identical bonds with different


spatial orientations
of ring structures'
carrs"d by either rotation about sigma bonds or contortion

t\
OH CH"
|| .._ t" oH
H-4
Hzc-=L--,/ -

OH OH

H:C H\/t/cwzc
"--/--" H3

cH2cH3 CHa

Figure 2-7

Conformational isomers are different orientations of the same


molecule' Figure
orientations) for butane'
2-8 shor"s six possible conformations (three-dimensional
Conformational isomers are in dynamic equilibrium at room temperature,
because rotation about sigma bonds is possible with the
energy available'
not remain in jusi one of
Because molecules are consiantly rotating, butane does
time or another'
the conformations, but rather urrn*", all confirmations
at one
frequently' This means that there
It assumes the most stable conformation most
is a most favored and least favored conformation'

nT1.,, CH: CHs

nrr/ Y;" V^H


Structure #2 Structure #3
Structure #1

CHe CHa H
HeC HsC

H\ul
H
v^ H
H\9)
HsC

#4
Structure #4 Structure #5 btructure #6
Structure ffo

Figure 2-8

from one conformer into


Rotation about a carbon-carbon bond converts butane
another.Forinstance,al80"rotationabouttheCz-Czbondofbutaneconverts60'
Structure #1 in Figure 2-8 into Structure #4. The
six structures represent
incrementalrotationsabouttheC2_.Cgbond,acrossafull360"rotation.

t02 The BerkeleY Keview


Copyright @ bY The BerkeleY Review
Organic Chemistry Structure Elucidation Isomerism

Of the conformers in Figure 2-8, Structure #4 is the least stable, due to steric
hindrance between the two methyl substituents (the largest groups). Structure
#4 has hydrogens on carbon 1 and carbon 4 colliding with one another. Structure
#1 is the most stable, because repulsion is minimized. To maximize stability, the
largest groups on each carbon (CH3, in this case) need to be as far apart from one
another as possible (tn anti orientation), if they repel. This minimizes the steric
repulsion between the two groups. The least stable conformation (eclipsed, as it is
cailed) has the two largest groups colliding into one another. The different
orientations are referred to as confonnational isomers. Figure 2-9 shows a side-
r.iew of steric repulsion for the fully eclipsed butane molecule in Structure #4.

'.i. H.
/\'H Steric repulsion between
HzC.. ,CHz hydrogens makes this
\-/ structure less stable.
H\YZ
HH
VN
Figure 2-9

A major part of learning about conformational isomers is learning the


terminology. Listed below is a giossary of terms that apply to conformational
isomers based on the structures shown in Figure 2-8.

Eclipsed: Eclipsed refers to the orientation where from a front view of the
molecule, you are prevented from seeing all of the substituents on the rear
tarbon, because the substituents on the front carbon block the view of the rear.
Structures #2, #4, and #6 are exampies of the eclipsed orientation of butane,
Because of steric hindrance, the eclipsed conformations are less stable than the
staggered conformations. Figure 2-10 shows the eclipsed conformation of ethane.

Eclipsed Drawings of Ethane


HH

"Jy'-'Y';,.
Figure 2-10

Staggered: Staggered refers to the orientation where from a front view of the
rrolecule, you can see all of the substituents on the front and rear carbons. There
,s a 60" dihedral angle between neighboring substituents. Structures #1, #3, and
=5 are examples of the staggered orientation of butane. Staggered is the most
.:able conformation. Figure 2-11 shows the staggered conformation of ethane.

Staggered Drawings of Ethane

n H H

c-c
H
v"
H
Figure 2-11

-opyright @ by The Berkeiey Review l03 Dxclusive MCAT Preparation


Organic Chemistry Structure Elucidation Isomerism
(

Within staggered conformation, there are two terms used to describe the relative m

position of substituents on adjacent atoms. These terms arc gauche and anti, and L0

jj
they refer to the position of a substituent on one atom relative to the position of a
substituent on an adjacent atom. In the case of ethane, where each carbon has
three hydrogen atoms attached, each hydrogen has two hydrogens gauche to it
and one hvdrogen anti to it. Butane, having a methyl group on both carbon-2 il\

:[
and carbon-3, is a simplistic molecule to consider gauche and anti orientation.
5
Rotation about the bond between carbon-2 and carbon-3 of butane generates
conformational isomers where there is a methyl group on each carbon. We shall
T
demonstrate gauche and anti orientation by referencing the methyl substituent
on carbon-2 of butane relative to the methyl substituent on carbon-3 of butane.
The terms are defined belorv.

Gsttclrc: Gauche refers to a staggered conformation of the molecule, where the


two groups of interest (often the iargest groups) have a dihedral angle of 60".
Structures #3 and #5 have the CH3 groups gauche to one another. Three
different perspectives of the staggered conformation of butane, with carbons 1
and 4 gauche to one another, are shown in Figure 2-12.

Staggered Conformation of Butane with CH, groups Gauche

CH, H.C
Hfi!-cHr ,/tn.
C-
v"
C..

'>(" n'/ H l
H :
Figure 2-12 :.
Anti: Anti refers to a staggered conformation of the molecule, where the two
t-
groups of interest have a dihedral angle of 180". Structure #1 has the CH3 groups !,

anti to one another. This is the most stable structurel Three different
perspectives of the staggered conformation of butane, with carbons 1 and 4 anti
to one another, are shown in Figure 2-13. Er
^1
ar
Staggered Conformation of Butane with CH, groups Anti IC
-{

+
H B.
C.

c-c. ,/'n'
H:C \,"
H Sc

CH: B
L<
B"
br,
Figure 2-13
ar
st:

Copyright O by The Berkeley Review The Berkeley Review Cr


Organic Chemistry Structure Elucidation Isomerism

Because atoms within a molecule are in constant rotational motion, a structure


loes not exist in one fixed conformation. Figure 2-14 shows the energy diagram
that corresponds to a complete 360' rotation about the CZ-CS carbon-carbon bond
rf butane, starting and finishing with Structure #L from Figure 2-8. The energy
:iagram starts with the lowest energy structure at 0' (which is the conformation
carbons 1 and 4 anti to one another) and rotates 360" about the C2-C3 bond
".-ith
:o return to the same orientation. Note that the highest energy structure is
eractly 180" apart from the lowest energy structure, and that a 60" rotation takes
-"-ou from an apex (a localized
energy maximum) to a nadir (a localized energy
ninimum) on the energy diagram.

io
:!

0" 60" 1.20' 180' 240' 300' 360'


Degrees of rotation

Figure 2-14

ihe energy diagram for butane is symmetrical, because butane is a symmetrical


inolecule. When the compound is chiral (contains stereogenic centers), the
energy diagram is not symmetric. The energy axis of the diagram is not
;uantified in Figure 2-74, so it demonstrates only a conceptual relationship. If
any calculations are required on the MCAT, values will be provided for the
energy of the eclipsed and staggered conformations, and for the gauche and anti
::'rteractions of various substituents.

Example 2.8
Ilow does the energy diagram for the complete rotation about a central bond of
an asymmetric (chiral) compound compare to the energy diagram for its
complete rotation about the CZ-CS bond of butane?
A. The energy diagram is symmetric like butane, but with higher energy values.
B. The energy diagram is symmetric like butane, but with lower energy values.
C. The energy diagram is asymmetric like butane, but wiih greater differences
in energy values.
D. The energy diagram is asymmetric, with different energy values than butane.

Solution
Because the structure is asymmetric, the energy diagram depicting the rotation
associated with the molecule must also be asymmetric, eliminating choices A and
B. The energy diagram for the complete rotation about the C2--4.g bond of
butane is symmetric, so choice C is eliminated. Choice D must be the correct
answer. The three staggered conformations are of unequal energy. The
staggered conformational isomers of l1-2,3-dimethylpentane are:

Copyright @ by The Berkeley Review l05 Exclusive MCAT Preparation


Organic Chemistry Structure Elucidation Isomerism I

H,. Hz9 .cH.cH, H"C. =


H3c<.^
^/
.cH2cH3
r&.- _/tn"n' r-r.'c'32.^
^,/tn,t', I
L

HsC
-(.
V"CH: n,c/'- \:,f n)'- \:,t =
i

$
CH2CH?
HsC CHa HgC\. CH"J

HsC HgC H H:C H


CHs CHs H
2 methyl/methyl and 1 methyl/methyl and 1 methyl,/methyl and
1 ethyl/methyl interactions I ethyl/methyl interactions 2 ethyl/methyl interactions
Lowest Energy Conformation

Figure 2-15 shows the exact orientation of the atoms at various times during the
first 120" of rotation of ethane. The graph shows that at 20" and 100" the
structures are symmetric and thus of equal energy. This is also observed for the
conformers at 40" and 80".

Rotation about the


C-C bond of ethane

0 20 40 60 80 100 720
Degree of rotational displacement from staggered conformation

Figure 2-15

If the substituents repel one another, then the anti orientation is lowest on the
energy diagram. However, when the force between neighboring substituents is
attractive in nature, tl-re gauche orientation is most favorable. A good example is
a vicinal diol, where the two hydroxyls on adjacent carbons exhibit hydrogen-
bonding with one another.

Copyright @ by The Berkeley Review The Berkeley Keview


Organic Chemistry Structure Elucidation Isomerism

Erample 2.9
.he best explanation for why the most stable orientation about the C2-C3 bond of
:i.e amino group and the carboxylic acid group in 3-aminopropanoic acid is
:auche is which of the following?
\. Gauche is the most stable, because it minimizes the steric hindrance between
the amino group and the carboxylic acid group.
B. The gauche conformation is always more stable than the eclipsed
conformation.
C. The carboxylic acid and amino groups form a hydrogen bond best from
gauche orientation.
D. The carboxylic acid and amino groups form a hydrogen bond best from anti
orientation.

Solution
il the substituents attract one another, then the most stable conformation is
s:aggered with the two attracting groups gauche with respect to one another.
I-re best choice is C, because gauche has the amino and hydroxyl groups close
.nough to form a hydrogen bond. Hydrogen-bonding is a stabilizing force. The
:lrongest hydrogen bond is formed by the lone pair of nitrogen with the acidic
:,r'drogen of the carboxylic acid group.

n&J*n,
H

,o,-./ \:r:"
ll"
o
Anti orientation o?c -
zHvdrogen bond------- - ,,H-
o- ta
(.
I ^.'.4 u^xr.
U-
"&_J \Hz

,'/ Y3"
Gauche orientation
Hydrogen-bonding is possible when the two groups
are only 60' apart, but not when they are 180" apart.

Copyright @ by The Berkeley Review r07 Exclusive MCAT Preparation


Organic Chemistry Structure Elucidation Isomerism
T
Newman Projections r,n
Understanding the nuances of conformational isomers requires good three- str
dimensional viewing skills, so you may wish to dig out your molecular models if ery
you still have them. Be able to recognize structures from the stick figure view drit
(with dashed and bold wedges), as well as from Newman projections. Newman wT
projections are front views of a molecuie. In drafting, three views are given to trmr

see the whole. It is no different in organic chemistry. The side view is a dashed- I[tuP

and-bold wedge representation, the front view is a Newman projection, and a MW

top view is a Fischer projection. Figure 2-16 shows a pictorial explanation of the od
conversion from a dashed-and-bold wedge drawing to a Newman projection,
while Figure 2-77 shows a pictorial explanation of the conversion from a
Newman projection to a dashed-and-bold wedge drawing.
When viewed from the right, substituents pointing out of the plane in a dashed-
I
and-bold wedge drawing are on the left side in a Newman projection. mft
Substituents behind the plane in a dashed-and-bold wedge drawing are on the
right side in a Newman projection. Substituents on the left side in the Newman
projection end up pointing out of the plane in the dashed-and-bold wedge
drawing. Substituents on the right side in the Newman projection end up behind
the plane in the dashed-and-bold wedge drawrng.

Conversion from dashed-and-bold wedge drawing to Newman projection:


H CJ
q,)

o. HsC H

,/
d
o 9.
6
Y;,.", o
L H CHs
-o

Figure 2-16

Conversion from Newman projection to dashed-and-bold wedge drawing:


behind
OH
HgC H
+) :f&1"'
in front
of plane ,/ L-\,
behind

H CHs
,/
H Y,::*,"
ln frontH
of piane

Figure 2-17

Cycloalkanes
Cyclic alkanes which contain only one ring have the chemical formula C.,H2.,
and contain no n-bonds. The stability of a given cycloalkane is rooted in its
ability to form bond angies of approximately 109.5', the norm for sp3-hybridized
carbons. The farther from 109.5" the angle is, the greater the reactivity of the
cycloalkane. For this reason, three- and four-membered rings are reactive, while
five- and six-membered rings are stable. When treated with hydrogen gas (H2),
cyclopropane and cyclobutane readily form straight chain alkanes (propane and
butane). Cyclopentane and cyclohexane do not undergo hydrogenation.

Copyright @ by The Berkeley Review loa The Berkeley Keview


Organic Chemistry Structure Elucidation Isomerism

The reactivity of three- and four-membered rings is attributed to ring strain. Rtng
strain is defined as the energy difference between the linear and cyclic alkanes of
T
equal carbon iength. Because the bond angle in cyclopropane is 60", vastly
different than 109.5' associated with a normal spr-hybridrzed carbon, there is a
:I
great deal of ring strain. To relieve this angle problem, cyclopropane forms what
o
are referred to as bent bonds (sigma bonds in which the electron density does not
t-
lie between the two nuclei). Because the electron density is not between the two
a
nuclei, the bond is much weaker and thus easier to break. Figure 2-18 shows the
e
orbital and bonding pictures for various cyclic alkanes.
I,
a Cyclopropane

t-
I. Ring strain 27.4 kcal / mole
e
n
e
The carbon-carbon bonds are bent (not coilinear), making
d
them weaker than standard carbon-carbon singie bonds.

Cyclobutane

Ring strain 26.2 kcal / nole

Cyclopentane
H2C- cH2
uri-."r-)rr,
Rrng strain 6.8 kcai/mole

Cyclohexane
H

HH

H H H H

Figure 2-18

ln lecause of their stability, most cyclic organic and bio-organic molecules are
its
ed =rther five-membered or six-membered rings. Six-membered rings are slightly
rore stable than five-membered rings. Given their frequent presence in
:iological molecules, five-membered and six-membered rings have a high
:le
:robability of appearing on the MCAT.
t!.
-J

3I'{ -opyright @ by The Berkeley Review l09 Dxclusive MCAT Preparation


Organic Chemistry Structure Elucidation Isomerism j
Cyclopentane \
Cyclopentane does not require much^distortion of its bonds and shape to (
accommodate the 109.5" angle for the sf -hybrid. A perfect pentagon has angles
of 108', so there is only a small discrepancy from 109.5". This angle difference m
;
does not account for the small ring strain of 6.8 kcals per mo1e. To achieve the ui"

correct angle and alleviate this torsional strain, cyclopentane forms what is
referred to as an enaelope shape, where one of the carbons is not coplanar with the lh

other four. The major problem with cyclopentane is not the ring bond angles, but LU

the substituents on the ring that are in an eclipsed conformation as a result of the c
near-planar ring structure. This eclipsing of hydrogen atoms causes further g
contortion of the structure, which accounts for the ring strain energy. Still,
cyclopentane structures are relatively stable, Their stability makes them common lllln

in such biological structures as ribose, deoxyribose and the purine ring of the m

DNA bases adenine and guanine. Figure 2-19 shows a few other common five- il

membered rings frequently encountered in the biological sciences. rilm

COr-
t-
"l
*H"N-

t'
CH"

OHH
B-D-Fructofuranose
n
HN\---zN
L-Histidine

OH HO OHH
B-D-Ribofuranose 2-Deoxy-B-D-ribofuranose

Figure 2-19

Copyright O by The Berkeley Review lIo The Berkeley Review L-l


Organic Chemistry Structure Elucidation Isomerism

Cyclohexane
Cyclohexane has the most stable ring structure of all of the cycloalkanes. This is
evident in biological molecules, which have multiple rings of six atoms. The
most stable form of cyclohexane is the chsir conformation. There are two
different chair conformations for cyclohexane. The two conformational isomers
can interconvert through a process referred to as ring-flipping. In interconverting,
the structure passes through the boat conformation. Because the two chair
conformations are equally stable, AGr* - 0. The interconversion between the two
chair conformations of cyclohexane requires 10.8 kcals/mole in activation
energy. The chair conformation offers two substituent positions: equatorial
(named for its orientation around the equatorial plane of the ring) and axial
(named for its vertical alignment like an axis). Equatorial is more stable than
axial, so the most stable conformation of a cyclohexane compound has the largest
substituents in the equatorial positions. Figure 2-20 shows chair conformations
of cyclohexane with detailed positions.

H
Cyclohexane showing equatorial Hs Cyclohexane showing axial Hs

axial (up) axiai (up)


equatorial (up)

(up)
eq (down)
eq (down)
eq (up)

eq (up) axial (down)

equatorial (down)
axial (down)
axial (down)
Cyclohexane showing all substituents

Figure 2-20

Copyright @ by The Berkeley Review lll Exclusive MCAT Preparation


Organic Chemistry Structure Elucidation Isomerism

Interconversion between Cyclohexane Conformers


The interconversion between the two chair conformations of cyclohexane
requires activation energy, but the free energy (AGr*) is zero, because there are
no substituents on the cyclohexane ring. As cyclohexane undergoes ring-flip
between chair conformations, it passes through intermediate structures. The two
most significant intermediate structures are tLle twist form and the boat form. The
twist form is of slightly lower energy than the boat form, because the hydrogens
are not eclipsed in the twist form. The ring-flip conversion of cyclohexane from
chair to boat and on to the other chair conformation is shown in Figure 2-21.

il
m
H + ry
dffi
H H tu
p1 H ro
Twist ffi
mh

tu
@ry

@
ffi
H ffim

il[m

HH @M

Figure 2-21

Because molecules can readiiy rotate and contort at room temperature, an


equilibrium exists between the two chair conformations of the cyclohexane
molecule. Either at low temperatures or'in cases where a substituent is too bulky
to orient itself in an axiai manner, one conformer is present exclusiveiy. The
proton NMR can be used to determine the more stable structure by focusing on
the different signals for the two orientations of hydrogen. You should be able to
determine the most stable conformation. The boat conformation is not as stable
as the chair conformation, because its substituents encounter high energy
eclipsed interactions its substituents encounter, while the substituents in the
chair conformation are staggered with lower energy gauche and anti interactions.

Monosubstituted Cyclohexane
The ring-flip process is the same when there is a substituent on the ring as it is for
cyclohexane, but the energetics are different. When a substituent is present, the
activation energy for interconversion increases, and the two chair conformations
differ in stability. Because the two chair conforrnations are no longer of equal
energy, the two twist forrns are also no longer of equal energy. The most stable
chair conformation is the structure r,r'ith the least steric repulsion. Substituents
with axial orientation on the same side of the ring are close enough to repel
(known as 1,3-diaxinl interactions), so axial orientation is less favorable than
equatorial orientation. This is indicatecl in Figure 2-22, where the two chair
conformations of both methylcyclohexane ar-rd cvclohexanol are shown. The two
chair conformations of methylcyclohexane differ in stabiliiy by 7.69 kcals,/mole.
Because a hydroxyl grorlp is smaller than a methyl group, the difference in
energy for the two chair conformations of cr-clohexanol is only 1.04 kcals/mole.

Copyright O by T'he Berkeley Review tt2 The Berkeley Review


Organic Chemistry Structure Elucidation Isomerism

oH

Methylcyclohexane
H I

Cyclohexanol
Equatorial > Axial by 7.69 kcals/mole Equatorial > Axial by 7.04 kcals/mole

Figwre 2-22

Disubstituted Cyclohexane
Disubstituted cyclohexane exhibits different dynamics than monosubstituted
cyclohexane. The greatest energy difference is observed when comparingT,3-
diaxial to 1,3-diequatorial, because the steric repulsion of 1,3-diaxial substituents
is the strongest repulsion encountered in cyclohexane. This is shown in Figure 2-
22 for dimethylcyclohexane. When comparing 7,2-diaxial to 1,2-diequatorial,
there are no diaxial interactions of the bulkiest substituents, because the two
bulky substituents are trans to one another. This makes the energy difference
between chair conformations less than what would be observed with the 1,3-
cyclohexane. The energy difference between the 1,2-diaxial and 1,2-diequatorial
orientations is also less than the energy difference between 1,4-diaxial and 1.,4-
diequatorial (which is another compound having the two bulkiest substituents
trans to one another), because the 1,2-diequatorialcyclohexane species has gauche
interactions between the bulkiest substituents. The energetics of the
conformational isomers of dimethylcyclohexane are shown in Figure 2-23.

CH"
l"
L;-/ : "'[.\1-\
I

/-=--/4 H.c/114
I

CH.
"
Trans-7,2-dimethylcyclohexane
1,z-Diequatorial > 1,2-Dtaxialby 2.8 kcals/mole

N
| .r,
:- "''H*"
CHs Cis-1,3-dimethylcyclohexane
1,3-Diequatorial > 1,3-Diaxial by 8.4 kcals/mo1e

9H:

F/Fcrr.
H:c-\/-\ r

CH, Traus-7,4-dimethylcyclohexane
" t,+-Diequatorial > 1,4-Diaxialby 3.4 kcals/mo1e

Figure 2-23

Copyright O by The Berkeley Review ll3 Exclusive MCAT Preparation


Organic Chemistry Structure Elucidation Isomerism (

Table 2-1 lists the possible orientations for disubstituted cyclohexane. Trans
refers to substituents on opposite sides of the ring, equating to either up/down :
or down/up. Cis refers to substituents on the same side of the ring, equating to
either up/up or down/down. Cis and trans do not refer to equatoriil or axial :
orientations, but rather to whether the substituents are above or below the ring. si
J:,

t.
Possible Orientations for Disubstituted Cyclohexanes

7,2-trans (upldown) or (down/up) C-1 axial C-2 axial or C-1 equatorial C-2 equatorial
1,2-cis (uplup) or (down/down) <+ C-1 axial C-2 equatoriai or C-1 equatorial C-2 axial
7,3-cis (uplup) or (down/down) e; C-1 axial C-3 axial or C,1 equatorial C-3 equatorial
7,3-trans (upldown) or (down/up) e> C-1 axial C-3 equatorial or C-1 equatorial C-3 axial
7, -trans (upldown) or (down/up) e C-L axial C-4 axial or C-1 equatorial C-4 equatorial
\, -cis (up/up) or (down/down) ++ C-1 axial C-4 equatorial or C-1 equatorial C-4 axial

Table 2-1
-:
It is essential that you be able to translate from nomenclature to the most stable rr.x
conformation. For instance, trnns-3-methylethylcyclohexane is a 1,,3-trans llhr
compound. A compound with 7,3-trans orientation has both an axial and an
equatorial substituent. The ethyl group is larger than the methyl group, so the
ethyl group occupies the equatorial orientation in the most stable conformation tl!
of trans-3-methylethylcyclohexane. This is shown in Figwe 2-24. I"}ff
:lhu
ilIJI,

f__4 IIIIIII

b--_LcH:
:ilh,r

dlilrru

[]rgft

I I[!r!
cH2cH3

Figure 2-24

You should take note that when you convert from one chair conformer to the
other, the axial substituents become equatorial (as seen with the ethyl group),
and the equatoriai substituents become axial (as seen with the methyl group).
The most stable conformation has the least steric repulsion.

Example 2.10
The most stable conformation of cis-L,2,4-trimethylcyclohexane has which of the
following orientations for the three methyl groups?

CHg

A. The chair conformation witl-r 3 equatorial methyls and 0 axial methyls


B. The chair conformation with 2 equatorial methyls and 1 axial methyl
C. The chair conformation with 1 equatorial methyl and 2 axiai methyls
D. The chair conformation with 0 equatorial methyls and 3 axial methyls

Copyright @ by The Berkeley Review ll4 The Berkeley Review


Organic Chemistry Structure Elucidation Isomerism

Solution
The most stable conformation has as many methyls equatorial as possible. 7,2-cis
has one axial and one equatorial substituent, so there must be at least one axial
substituent. Answer B is best. Drawn below are the two chair conformations of
cis-7,2,4-trimethylcyclohexane (or (1R,2S,4R)-1,2,4-trimethylcyclohexane as
correctly name using IUPAC nomenclature rules.)

CH. CH, CH.


l" l'
H:.cd
r-)
/ / \-\4
1"".

WcH.,
\

1 equatorial;2axial - 2 equatorial; 1 axial


-\
This covers the topics associated with isomerism. These topics are applicable in
both the physical and biological sciences areas, so know them well. To implant
this section as a working knowledge base, there are many passages with which to
work. From the very beginning, you want to emphasize the logic behind your
answers. The MCAT may not have passages that are verbatim duplicates of
what you see in here, but if you answer these questions using sound logic and
fundamental concepts, then you will slowly get acclimated to the MCAT way of
thinking. At this point, passages may seem like an absurd form of asking
questions, but hopefully you wiil take a liking to the style. Passages present
information that you must incorporate into your background knowledge, and
then using all the information you have, you must reach a conclusion concerning
their questions. Multiple-choice tests require that you find the best, most
reasonable answer. You are not required to solve detailed questions or derive
fundamental concepts. Just find the best answer, as fast as you can.

Copyright @ by The Berkeley Review I l5 Exclusive MCAT Preparation


Organic Chemistry Structure Elucidation Structural Insights

# ffi#ffiilffit
Structural Symmetry
\A/hen deducing the molecular structure for an organic molecule, it helps to know
something about the symmetry of the compound and its units of unsaturation.
symmetry can be broken into plane symmetry and point symmetry. In plane
symmetry, the compound has two halves that are evenly displaced about an
imaginary mirror in the middle of the molecule. In a structure with point
symmetry, there is an inversion point at the center of the molecule such that if
two lines are drawn in opposite directions along the same axis, then both line
segments intercept identical atoms at the same diJtance from the inversion point.
This may not seem clear in words, but looking at a structure helps illustrate the
concept. Figure 2-25 shows one compound with mirror plane symmetry and
another with inversion symmetry. Molecules with inversion points are nonpolar,
because all of the individual bond dipole vectors cancel each other out.

cH20H HOH"C H
'\r.-o1rY*'
HgC *--*oof'- "\..
' -,H CH2Q.H
Molecule with mirror symmetry Molecule with an inversion point

Figure 2-25

Symmetry within a molecule affects its NMR and IR spectra. As symmetry


increases, the number of signals in a spectroscopic study decreases. coupling
symmetry information with units of unsaturation helps to deduce the structural
feafures and connectivity of a molecule.

Units of Unsaturation
Units of unssturation are calculated from the molecular formula. The units of
unsaturation give us information about the number of rings and/or n-bonds
present within a molecule. There is some minimum number of bonds needed to
hold the atoms in a molecule together, and any additional bonds beyond the
minimum are the units of unsaturation. To hold two atoms together, it takes one
bond (Atom1-Atom2). To hold three atoms together, it takes two bonds
(Atom1-Atom2-Atom3). The minimum number of bonds required to hold a
molecule together is always one less than the number of atoms. The minimum
number of bonding electrons is two times the minimum number of bonds. Any
electrons beyond the bare minimum needed to hold the molecule together can be
used to form additional bonds. For every extra pair of electrons, there is a unit of
unsaturation. To determine the units of unsaturation, the strategy is to
determine the number of excess bonding electrons. There are a few different
methods for doing this.
1) C3H6 contains eleven atoms, which requires at minimum ten bonds (and
thus twenty bonding electrons). There are three carbons with four bonding
electrons each. There are eight hydrogens with one bonding electron each.
This means that propane (C3Hg) has exactly the twenty bonding electrons
needed. There are no extra bonding electrons, so propane has a linear
structure with no n-bonds. This is to say that propane has no units of
unsaturation.

Copyright O by The Berkeley Review rr6 The Berkeley Review


Organic Chemistry Structure Elucidation Structural Insights

The units of unsaturation for hydrocarbons and carbohydrates can be


derived from the formula for aliphatic alkanes, Cr,H2', 1 2. "Aliphatic" refers
to a structure that has no rings or n-bonds. An aliphatic alkane has the bare
minimum number of bonds, so there are no units of unsaturation. For every
'1. ur-rit of unsaturation, there are two fewer hydrogen atoms than the
ie maximum. Thus, the units of unsaturation can be obtained by comparing the
n actual formula to the fully saturated formula. For instance, C5H6 has four
rt hydrogens less than the fully saturated formula for five carbons, CSH1Z.
iJ Because it has four fewer hydrogens, it has two units of unsaturation.
re
L-
The units of unsaturation depend on the surplus of bonding electrons. To
re keep any chain propa gated, every member of it must make two connections.
d ln a molecule, each atom must make two bonds to keep it intact. This means
rl that every atom needs a minimum of two bonding electrons. Using this
perspective, we can determine the number of excess electrons per atom.
Hydrogen makes just one bond, so you subtract one for each hydrogen in the
molecule. Oxygen atoms are ignored, because they make the minimum two
bonds that are needed. Carbons are multiplied by two, because carbons
make four bonds, two beyond the minimum to propagate the chain. There
are two ends to every chain, so two is added to the total. The units of
unsaturation refer to bonds, rather than bonding electrons, so the sum of
excess electrons must be divided by two. This is summarized in Equation
2.7.

2(#C)+2-(#H)
Units of unsaturation = (2.1)
2

l"lethod 3 works with other atoms, too. Nitrogen makes three bonds, which is
:\ ::-e more than the minimum needed, so you add 1 per nitrogen atom. Halogens
)g
'D :-':ke one bond, which is one less than the minimum, so you subtract 1 per
a1 . -:-,de. Equation 2.2 includes nitrogen and halogens.

2(#c) + (#N) - (#H) - (#x) + 2


Units of unsaturation = (2.2)
2
0t
ls
to Erample 2.11
:IC :::",r' many units of unsaturation are present in a compound with the molecular
1e ::::nula C7H9N3O2Cl2?
ls {. 1
ta 8,2
II1 :.3
-t\' 1.4
be
of t rlution
to -:,s question is solved by applying Equation 2.2.
nt
-. )+ (3)-(9)-(2)+2 =r4+3-9-2+2 =72 -9 - B - 4Unitsof unsaturation
2222
.,J :::ause there are four units of unsaturation, choice D is the best answer. With
ng : -rr units of unsatuiation, the compound could contain three n-bonds and one
ln. ::ng, meaning it is potentially a benzene derivative. When there are four units of
'l:s ,:.saturation, you should immediately consider the possibility that the
lar . --mpound contains an aromatic ring. While there are other combinations of four
oi -::ts of unsaturation, there is a high probability of having an aromatic ring.

:*' -:p.r'1i*1't, @ by The Berkeley Review tt7 Exclusive MCAT Preparation


Organic Chemistry Structure Elucidation Structural Insights
l
Spectro3bop$ an Andysis 0
,il
@
Spectroscopy W
The MCAT togics include infrared absorption spectroscopy, uitravioiet-visible il
spectroscopy, 1H (proton) nuclear rnugtl"ti. ,"ronun." spectroscopy, and 13C
(carbon) nuclear magnetic resonance spectroscopy. There is nothing for you to
d
fear, however. At just over one minute per problem, if you have to deduce a
d
structure from spectroscopic information, then it will likely be easy or symmetric. m
For example, on a previous exam, there was a proton NMR of ethanol that many
students said was very easy. Besides just having to determine the structure of an
Jffi
unknown, you may also have to assign signals and peaks to an existing d
compound. The spectra should be interpreted using the typical features you m
have learned. To date, the test has emphasized only a few features that have
tffi
been stressed in course work. We shall start by reviewing the basic operations of
mh
the IR and its applications to structure elucidation. From there we sha1l consider
l@G
ultraviolet,/visible spectroscopy, proton NMR, and carbon-13 NMR.
m
Infrared Spectroscopy
h
Er,'ery molecule produces a unique IR spectrum. Infrared spectroscopy starts by
ffi
adding a monochromatic beam of IR photons to either a thin oil suipension (if
the compound is a solid) or a neat solution (if the compound is a liquid) between
salt plates. The molecule absorbs electromagnetic radiation that causes
transitions between vibrational energy levels within it, meaning that the
molecule vibrates more or less frequently as energy is absorbed or emitted.
when a molecule soaks up the EM radiation and vibrates at a higher energy.
This change in stretching (vibrating) between atoms within the molecul" .u.6ei'a
change in the dipole moment, which can then be monitored. An infrared
spectrometer uses light of wavelength 2,500 nanometers to 17,000 nanometers
(recorded as 4000 cm-1 to 600 cm-1 on the graph). \A/hat we record is the change
in the intensity of the EM radiation from when it enters the molecule to when it
exits the molecule. This is compared to a reference beam that traverses a path of
identicai length but does not pass through the compound itself. If the compound.
absorbs a given wavelength of light (corresponding to some transition), then we
observe an absence of light exiting the sample tube. This is known as absorption
spectroscopy. The graph records transmittance as a function of wave number (cm-
r), so absorbances are represented by drops rn intensity.

The frequency at which light is absorbed is specific for each type of bond. As
you may have learned in physics, the frequency of light is directly proportional
to the masses of the two atoms in the bond and the bond strength. This is to say
that the potential energy in a resonating system (such as a spring that obeys
Hooke's law) is described by Equation 2.3

P.E. = 1 kx2 (2.3)


2

The k-term is the spring constant, which we can say describes the bond strength.
The x-term describes the distance from equillbrium that the bond has stretched.
The absorbance can be thought of as increasing the potential energy of the bond,
so the absorbance is proportional to the energy of the bond. As a resuit, the bond
dissociation is directly proportional to the energy that is absorbed. This is not
exactly true, but close enough to help approximate spectra. Because a wave
number is measured in cm-l, it is an inverse of the wavelength. The inverse of
the wavelength is directly proportional to the energy of the photon. This means
that the higher the wave number is, the greater its energy.

Copyright @ bv The Berkeley Review rr8 The Berkeley Review


Organic Chemistry Structure Dlucidation Spectroscopy and Analysis

For instance, a C=O boncl absorbs around 1700 cm -1, while a C-O bond absorbs
around 1300 cm -1. this is because a C=O bond is stronger than a C-O bond.
Carbonyl functional groups a-re common, so you should know the absorbance
value for a C=O bond. An sp3-C-to-H bond absorbs just below 3000 cm-1, while
an sp2-C-to-H bond absorbs just above 3000 cm-l, because an sf -C-to-H bond is
stronger than an sp3-C-to-H bond. This is because the sp2-hybrid, having more s-
character, is smaller than the sp3-hybtid. The result is that an sp2-C-to-H bond is
shorter and thus stronger than an sp3-C-to-H bond. The stronger bond, having a
higher bond dissociation energy, has a higher energy absorbance.

Although the molecule as a whole absorbs the EM radiation, we can use the
absorbances we measure to fingerprint particular functional groups and bonds
rvithin the molecule. The skill needed to make IR useful is an active process.
Scientists use IR not only to confirm the presence of certain functional groups,
but to also to help decide which functional groups are not there. IR is most
useful as a supplement to the moiecular formula and the NMR spectra for
molecules. Table 2-2lists several useful IR absorbances. The values are listed in
terms of wave numbers. Note that the absorbance of a given bond varies with
ihe compound in which the bond exists.

Bond type Stretching (cm-1) Bending (cm-1)


O-H alcohol (no H-bonding) 3640 - 3580 v)
O-H aicohol (H-bonding) 3600 - 3200 (s, broad) 1620 - 1590 (v)
N-H amides 3500 - 3350 m)
N-H amines 3450 - 3200 m)
C-H alkvnes 3300 - 3220 s)
C-H aromatic 3100 - 3000 v) 880 660 (v)
C-H alkenes 3060 - 3020 m) 1000 - 700 (s)
C-H alkanes 2980 - 2860 s) 1470 - 7320 (s)
C-H aldehyde 2900+,2700+ (m, 2 bands)
O-H acids (H-bonding) 3000 2500 (s, broad) 1655 - 1510 (s)

C=C alkvnes 2260 -2720 (v)

C=N nitrile 2260 - 2220 (v)

C=O ester 7750 - 1735 (s)

C=O aldehyde 7740 - 7720 (s)

C=O ketone 1725 - 1705 (s)

C=O acid 1725 - 7700 (s)

C=O aryl ketone 1700 - 1680 (s)


C=O amide 1690 - 1550 (s)
C=O cr,B-unsaturated ketone 1685 - 1665 (s)
C=C alkene 1680 - 1620 (v)
C=C aromatic 1600 - 1450 (v)

C-O alcohols, ethers, esters 1300 - 1000 (s)


C-N amines, alkyl 1220 - 1020 (w)
s = strong absorption w = weak absorption
m = medium absorption v = variable absorption

Table 2-2

Copyright @ by The Berkeley Review ll9 Exclusive MCAT Preparation


Organic Chemistry Structure Elucidation Spectroscopy and Analysis

using Table 2-2, you can evaluate IR data given in spectrum form to identify =
structures. As for memorizing peaks, according to the MCAT student Maru,Lal you
I
are required to "know the important ones," which is open to interpretation. You
fr
don't necessarily have to memorize all of the values, but if you do enough
$
problems, the values you repeatedly see should become second nature.
$

As a diagnostic tool, IR is used to detect certain functional groups. You have J


reached the pinnacle of utility when you use it to determine which functionalities I
are not present as well as which functional groups are present. Just as peaks (
confirm the presence of a certain bond, the absence of a peak supports the l
absence of that bond. Here is an example of how IR spectroscopy is used:
$
An unknown compound with formula C_aHgO is analyzed by IR spectroscopy. T
An intense band is detected at 1770 cm-1 (IR absorbances are listeh by energy oru

according to the wave number as measured in cm-1). By comparing the value to u


a chart of IR absorbances, this peak can be attributed to a C=O. The compound ,&
has one degree of unsaturation attributable to a C=O, which makes it possible to lflr,

narrow it down to a small number of isomers. The structure cannot be cyclic and efl
has a carbonyl. Given that the longest chain is four carbons, the carbonyl can be mm

only on carbon 1 or carbon 2. This narrows it down to only two butane @u


derivatives. The iongest chain could be only three carbons, with a methyl
substituent on carbon 2. In that particular structure, the carbonyl group has to be
on carbon one. This leaves only three possibilities, and they are:

1. H3CCOCH2CH3 (butanone)
2. H3CCH2CH2CHO (butanal)
3. H3CCH(CH3)CHO (2-methylpropanal)
Thus, we can reduce the choices from many types of compounds having one unit
of unsaturation and one oxygen (a cyclic ether, for example) to a few. Ketones
and aldehydes have different chemical reactivity and physical properties, so
when we combine IR information with chemical tests and the melting point of
the compound, we can eliminate two of the three structures. This is a structure
elucidation technique you have done many times in the past. Through examples TM
ar-rd practice, you can familiarize yourself with the peaks and become talented at
solving the problems using deductive reasoning. w
ffi
&
Example 2.12
h
tu
Which of the following compounds with the formula C5H19O cannot have an IR
absorbance peak between 1700 cm-1 and 1750 cm-1?
A. An aldehyde
B. A ketone
C. A cyclic ether
D. A11 of the above have an IR absorbance between 1700 cm-1 and 1750 cm-1.

Solution
An IR absorbance between 1700 cm-1 and 1750 cm-1 implies that the compound
has a C=O in its structure. Because it has no absorbance between 1700 cm-1 and
1750 cm-1, it does not have a C=O bond. Choices A and B have a C=O in their
structure, so they can be eliminated. The best answer is choice C. The one
degree of unsaturation associated wlth the formula is used in the ring. The one
oxygen in the formula is in the ether, which contains carbon-oxygen single
bonds. Choice D is also eliminated, because choices A and B are eliminated.

Copyright @ by The Berkeley Review The Berkeley Review


Organic Chemistry Structure Elucidation Spectroscopy and Analysis

Example 2.13
How many structural isomers of C4H3O2 are possible that have an IR absorbance
peak between 7735 cm-1 and 1750 cm-1, u p"ik around 1200 cm-1 and no broad
peaks above 2500 cm-1 ?

4.2
B.3
c.4
D.5
Solution
The absence of a broad peak above 2500 cm-1 indicates that the compound does
not have an o-H bond, which eliminates the possibility of it being an alcohol or
carboxylic acid. It is most likely an ester, although it could have both a carbonyl
and ether functionality. According to Table 2-2, the peak betweenIT35 cm-1 and
1750 cm-1 indicates ihat there ;s a c=o bond of an ester and not a ketone or
aldehyde. The compound must be an ester, so the question now becomes, "How
many esters are there that contain only four carbons in their structure?" There
are only four four-carbon esters, as drawn below, so the best answer is choice C.

1)o2) o
lt il
C

H,/-\ OCH"CH2CH3 H3CH2C/C- Oarr,

3)o 4)o
il il
t-
H,ZC\ OCH(CH3)2 nrC/ oarrra*ru
The two aldehyde structures (1 and 3) could be confirmed or eliminated by the
presence or absence of two peaks at 2900 cm-1 and2700 cm-1. If this were a real
iaboratory scenario, it would be far easier at this point to use proton NMR to
deduce the structure of the unknown compound. Structure t has four types of
hvdrogenin a 1:2:2:3 ratio. Structure2hasthree types of hydrogenina3:2:
i ratio. Structure 3 has three types of hydrogen in a 1 : 1 : 6 ratio. Structure 4 has
three types of hydrogen in a 3 : 2 : 3 ratio. The integration would be enough to
distinguish anything except Structure 2 from Structure 4. To distinguish these
trvo structures requires identifying the ppm shift value of each type of hydrogen.
iVe shall address NMR spectroscopy later in this chapter. IR spectroscopy
r""ta r"
"tttt"a

;
f

1r
te
te
ie

w Copyright O by The Berkeley Review t2l Exclusive MCAT Preparation


Organic Chemistry Structure Elucidation Spectroscopy and Analysis

Hydrogen-Bonding in Infrared Spectroscopy


Because the formation of hydrogen bonds affects the covalent bond between an
atom and a partially positive hydrogen involved in hydrogen-bonding, any
spectroscopy techniques focusing on the covalent bond to hydrogen, or the
hydrogen itself, is affected by hydrogen-bonding. The effect is a broadened peak
(observed in both the IR and NMR techniques). In the case of IR, the broadening
of the hydroxyl absorbance associated with hydrogen-bonding is caused by the
weakening of the covalent bond between the hydrogen and the atom (nitrogen,
oxygen, or fluorine) to which it is bonded. This lowers the energy of the covalent
bond and thus lowers the energy of absorption for the bond. As the hydrogen
bond increases in strength, the covalent bond weakens. Because not all of the
hydrogens have the same degree of hydrogen-bonding, their covalent bonds
exhibit many different absorptions, ranging from unaffected and therefore
standard covalent bonds to covalent bonds that are highly weakened by the
hydrogen bond. This range of covalent bonds gets grouped together into the one
broad peak. The same alcohol exhibiting two different degrees of hydrogen-
bonding is shown inFigure2-26.

Stronger hydrogen-bot-td f
l""o"Tro"* \""0I .$ HH \I
i ' '\
s
H

\ ts j"''-.'" *t \,.s c<H


o/t\ o/\n R
I
o/c\ o<n R
I

st.ong", cluut"r,t bond Weaker cJalent uona

Figure 2-26

Given that molecules are in continuous random motion within a liquid, some
alcohols have strong hydrogen bonds, while others have no hydrogen-bonding-
This means that the solution has a random distribution of hydrogen bonds and
therefore a random distribution of covalent bonds. The result is a distribution of
signals in infrared spectroscopy. To see each individual peak for each different
covalent bond requires a high resolution IR spectrophotometer. It is unlikely you
used such an instrument, so the signal with which your are familiar is a broad
composite signal covering the range of the individual signals. Figute 2-27 s
a high-resolution IR signal and the standard-resolution equivalent.

3440 3420 3400 3380 3360 3340 3320 3440 3420 3400 3380 3360 3340 3320

Wave number (cm-1) Wave number (cm-1)


High-resolution spectrum Low-resolution spectrum

Figure 2-27

Copyright @ by The Berkeley Review t22 The Berkeley


l5 0rganic Chemistry Structure Dlucidation Spectroscopy and Analysis

- Example 2.14
',"hat
is true for the compound associated with the following IR spectra?
te

1457 cm-l ll r:ii .,n '


& 736'7 cm-l
3317 cm-l
2903 cm1

2987 cn-l

-{. It exhibits no hydrogen-bonding.


B. It has a carbonyl group.
C, Ithas ahydroxyl group.
D. It has a molecular mass that is less than 30 grams per mole.

Solution
- he compound represe-nted by the lR spectrum above in the question has a broad
:-eak around 3300 cm-l and no peak near 1700 cm-]. These are the first areas to
:onsider when looking at IR spectra. The compound has a hydroxyl group, but
ro carbonyl group. This makes choice B incorrect and choice C correct. Because
:t has the hydroxyl group, it can exhibit hydrogen-bonding. This eliminates
;hoice A. Because of the peaks in the 1300-1400 cm-1 range, we know the
:ompound has a carbon-carbon bond, so it must have at least two carbons. The
Lightest compound with two carbons is ethyne (HC=CH), which has a molecular
nass of 26 grams per mole. However, because there is an oxygen present, the
compound must have a molecular mass greater than 30 grams per mole. Choice
D is eliminated.

Ultraviolet/Visible Spectroscopy
In addition to infrared spectroscop/, there is also ultraviolet/visible
spectroscopy. \A4rile infrared photons (in the 3-to-10 kcals/mole region) affect
the vibrational energies of a molecule, ultraviolet (in the 70-to-300 kcals/mo1e
region) and visible (in the 40-to-70 kcals/mole region) photons affect the
electronic energy levels. when uv or visible photons are absorbed by a
molecule, an electron is said to be excited from the ground state to an excited
state. Because o-bonds are so much stronger than n-bonds, the lowest energy
absorbance for alkanes is significantly higher than the lowest energy absorbance
for alkenes. To excite an electron from the o-level (sigma bonding orbital) to the
o*-level (sigma anti bonding orbital), photons of approximately 140 nm to 120 nm
are necessary. However, because molecules in the air readily absorb energy in
this region, the spectra must be obtained in a vacuum, Because this constraint is
rather impractical, uv-visible spectra typically range from 200 to 800 nm, where
air does not interfere. As a result, we typically use only UV-visible spectroscopy
to analyze molecules with n-bonds, especially conjugated systems. uv-visible
spectroscopy in organic chemistry focuses on transitions between the n and n*
energy levels. For systems with conjugation, there are several n-levels, but we
care about only the lowest energy transition.

3W Copyright @ by The Berkeley Review t23 Dxclusive MCAT Preparation


Organic Chemistry Structure Elucidation Spectroscopy and Analysis
I
The transition of interest is from fi to n*. The wavelength of highest absorbance, F[
known aslambda max (),^u*), changes with the amount of conjugation. The value J]iL J

of L,,'.u* depends on the amount of conjugation. As the conjugation increases, so dffi.

does the tnaaelength of L*o*. Figure 2-28 shows the l"*u* values associated with r@

the lowest energy n-to-n* transition of various conjugated hydrocarbons. M


mm

Il* _Tn fuil

l 'r3 ryr
iI -I.
Tn4 lrillt[

V\nrl
r75nm I 217nm z58nm I
{m
t/V\-l
t/\Arl
I
M
@M

l*ln3
n2 ,tE.
Md
TE
-r- ir1 rc1
'tlllllillF

ffie

- H,C:
-- CH ffiar

H"C: CH - -cn
H2C: CF{2 @
ll @m
CH: CHZ mry
/C' ,ilmhm

H2C: CH muil
mum
Figure 2-28 ,m
{ffrol
Because the energy gap between n and n* decreases as the conjugation increases, mru
the wavelength of maximum absorbance increases. 1,3,5,7-octatetraene has a rym
I*u* of 304 nm and 1,3,5,7,9-decaquintene has d Imax of 353 nm. When more
conjugation is added, the absorbance shifts into the visible range. Color resulF
from excessive conjugation within a molecule. For instance, B-carotene (with 1L
n-bonds) has absorbances at 483 nm and 453 nm. Substituted benzenes have a
number of peaks. Conjugated aldehydes and ketones have about the same n-:i*
absorbances as conjugated alkenes of the same number of n-bonds. However'
conjugated aldehydes and ketones have other, more intense absorbances (e >
10,000) that are of longer wavelength than their hydrocarbon counterparts. ThL"
is attributed to the n-to-fi* transition associated with aldehydes and ketones,
possible because of the lone pair of electrons on the carbonyl oxygen. Figwe2-29
shows the L,^u* values associated with the lowest energy n-to-fi* and n-to-n*
transitions of various ketones.

o o o
il

-\
HsC CHs H^cD ^r.W
n -+ 7tn I-u, = 270 nm n -t fi* I,,'u* = 324nm n --> 7t* I*u* = 368 nm

TE -) 1l* I-u, = 187 nm n -t Tl* I-u* = 219 nm Tl -+ 1l* l*u* = 267 nm

Figure 2-29

Unlike infrared spectroscopy, ultraviolet/visible spectroscopy can also


applied in a quantitative fashion. Ultraviolet/visible spectroscopy can be used
determine the yield of a reaction, if it involves a UV-visible active compound.
organic chemistry, a compound must have a n-bond to be UV-visible active'

Copyright @ by The Berkeley Review 124 The Berkeley


Organic Chemistry Structure Elucidation Spectroscopy and Analysis

Nuclear Magnetic Resonance


The fundamental principle behind nuclear magnetic resonance, NM& is the same
as for other forms of spectroscopy. Energy, in the form of electromagnetic
radiation in the radio frequency band, is added to the system and analyzed in
terms of what is absorbed. The energy levels that are affeited are for the spin of a
nucleus in the presence of an external magnetic field. Normally all of the nuclei
have spins of the same energy. However, when an external magnetic field is
applied, spins can either align with the field or align against the fie[, so multiple
energy states are possible. In the case of 1H, theie or"u t*o energy levels: o
line
one aligned with the external magnetic field), and G (the one aligned against the
external magnetic field.) The B energy level is defined as higher tlian the cx
energy level. The energetics of the two levels depends on the strength of the
erternal magnetic field and the magnetogyric ratio of a particular nucieus. This
:neans that the energy gap between the two levels also clepends on the strength
of
:he external magnetic field. As the external tnug.r"ii. field increarui th"
::equency of the EM radiation needed to flip the spin in"r"as"s proportionally.

\ny nucleus with an odd number of protons (Z number) or an odd number of


:-ucleons (A number) has a net spin. what is meant by "spin" is that as the
:-ucleus precesses, it generates a weak magnetic field (as etections do).
Just as a
:rarged particle in linear motion generates a radial magnetic field, a charged
:article in rotational motion generates a linear magnetic field. when the atomic
:,ucleus has an odd number of protons (or nucleoni), the spins cannot pair up to
:ancel one another out. The result is that the nucleus has a net spin. In the cises
-t 1H, 13c, and 19F, it happens that there are only two energy ievels associated
,
',1th the spins, so they can be analyzed without complicationl-A nucleus such as
-)-i has spin, but there are more than two energy
states, so its NMR spectrum is
-.o complicated to analyze conveniently.

-:. the absence of any surrounding electrons, all identical nuclei exhibit the same
-:in and therefore require the same energy for excitation in an external magnetic
:::ld. within a molecule, two identical nuclei may be in different eleclronic
. r'ironments. As a result of the difference in their local magnetic fields, caused
:-'' the moving electrons, they do not require exactly the same amount of energy
:: excite the nucleus to a higher-energy spin state. This can also be viewed is
- cal magnetic fields altering the strength of the apptied external magnetic field
- :eded to get excitation (spin-flip) at a set frequency for the EM radiation. NMR
:-achines can be designed to vary the frequency of the radiation or vary the
.::ength of the magnetic external field. The NMR graph we observe typically
:=:ords changes in the magnetic field strength along the x-axis, so we tnint or
l,\lR in terms of varying external magnetic field strength.
l:oton NMR (which uses the 1H nucleus) is the most common form of NMR and
rkes advantage of the magnetic spin associated with the hydrogen nucleus. The
l.:cAT test-writers focus on analyztng the graphs produced uy lumran. There is
:,lrimal 13cxuR on the MCAT. rn"e scaie rot tuNvn is set from 0 to 10 parts
: =r million (pp*) of the total magnetic field of the machine.
Just as an inchls an
..;h because someone made it a standard unit of measurement, NMR is
:easured in ppp of the external magnetic field, because that is the arbitrary
.::.ndard. All lHNMR shift values are relative to a standard compounj,
=:ramethylsilane ((H3c)4si). All twelve of the hydrogens on tetramethylsilane
.:e equivalent, so they absorb at the same value. This value is arbitrarily
';signed to be 0 ppm, and all shift values are referenced against it. Rather than
.- - Lnto other intricacies of NMR, we shall concentrate on how to read the graphs.

-,rpvright O by The Berkeley Review 125 Exclusive MCAT Preparation


Organic Chemistry Structure Elucidation Spectroscopy and Analysis j
Symmetry and NMR Signals M

The best place to begin NMR for the MCAT is with molecular symmetry. Based &
on the symmetry of a molecule, you can determine the number of equivalent tu
hydrogens that it contains. We will consider symmetry within different groups [M

of molecules, starting with the four six-carbon esters shown in Figure 2-30' l!..

ffi
o o MM

il il
H,Ct t- oarr"cHrcH)cH1" HrC' c- oarrlcH?)cH2cH3
&,r
h
a' h' c'd- e a" b' d- e"
m!
Five unique hydrogens labeled Five unique hydrogens labeled
@
a-e in a 3:2:2:2:3 ratio. a-e in a 3 : 1 : 3 : 2 : 3ratio. ffim
mh
o o @!
lt lt dh
H^Ct '-
/-
ocHrcH(cHj)? H^Ct
t- oc1arr.,;. M
a" b- c d-- at ' b''' m
Four unique hydrogens labeled Two unique hydrogens labeled m
a-dina3:2:7:6ratio. aandbinal,:3ratio. W
@
Figure 2-30 dM
W
The first two esters, n-butyl acetate and sec-butyl acetate, each have six unique ffim
carbons, of which five hav-e hydrogens. gach exiribits five signals in its
1HNMR ]b,fr
spectrum and six signals in its 13CNMR spectrum. Isobutyl acetate has five
unique carbons, but only four contain hydrogens. This means that isobutyl qm
acetate exhibits four signals in its I HNMR spectrum and five signals in its
13CNUR spectrum. Tertbutyl acetate has four unique carbons, but only two
contain hydrogens. This means that tertbutyl^acetate exhibits only two signals in
its 1HNMR spectrum and four signals in its IsCNMR spectrum. The presence o{
only two signals in an NMR spectrum makes it easy to identify tertbutyl acetate.

The comparison of symmetry between isomers is highly useful, particularly with


benzene derivatives. Figure 2-31 shows three structural isomers of methyl

q q
anisole, each of which has different symmetry.

OCH" OCH?

H
f
l"' H
b
H
f
1"" CHs
b

H H H
e
.T, H
e
H
c
c c

CHs H H
d d d

Four unique hydrogens Six unique hydrogens Six unique hydrogens


labeled a-d Iabeled a-f labeled a-f
ina3:2:2:3ratio. ina3:1:3:1:1:1ratio. ina3:3:1:1:1:1ratio.
Figure 2-31

Para-methylanisole has six unique carbons, of which only four contain


hydrogens. This means that para-methylanisole has four types of hydrogens and

Copyright O by The Berkeley Review t26 The Berkeley Revieu


Organic Chemistry Structure Elucidation Spectroscopy and Analysis

therefore four signals in its proton NMR. Meta-methylanisole has eight unique
carbons, of which six contain hydrogens. This means that meta-meiirylanlstle
has six types of hydrogens and therefore six signals in its proton NMR.
ortho-
methylanisole also has eight unique carbons, of wtrich six contain hydrogens.
This means that ortho-methylanisole also has six types of hydrogens"and
therefore six signals in its proton NMR. The structures in Figur e-2-Br"are para-,
meta-, and ortho-methylanisole, respectively.

You may have noticed that many problems are often just variations on a single
theme. In Figure 2-32, compounds with comparable i\MR readings are shoirn
side by side to demonstrate similarities in the distribution of lheir unique
hydrogens. Figure 2-32 shows two sets of three isomers that can be distinguished
irom one another using IHNMR by simply looking at the number of "signals.
Butanol and pentanal each have five unique types of hydrogens. The hydrigens
are in exactly the same ratio on both compounds, so thelr lHNMR spectra
"r,f,6it
strong similaritie^s. Each has five signals in its 1HNMR spectrum, ilthough they
iave different 13cxMR spectra arie to a different nrr-L", of unique ciruor,r.
rhe HNMR spectra of the two compounds can be distinguished from one
.1
another by the shift values of the respective signals. Methylpiopyl ether and.2-
lentanone each have four unique types of hydrogens. rhe hydrogens are in
exactly the same ratio on both compounds, so their 1HNMR spectra exhibit
strong similarities. Each has four signals with the same relative area in their
-HNMR spectra, but at different shift values. Diethyl ether and 3-pentanone
each
have mirror symmetry and thus have similar carbons and similar hydrogens due
io this symmetry. There are two unique types of hydrogens in both diettyl ether
:ld 3-pentanone. This means that diethyl ether and 3-pentanone have only two
1HNVn.
'ignals in their

Butanol Pentanal
o
H
./o--CH,-CF:.
. CH;-CHz
e-
il
.r b- d-
Fir-e unique hydrogens labeled
T-t-.f {'!'-cu1cF:
Five unique hydrogens labeled
a-e irr a 7 :2 :2 :2: 3 ratio. a-e in a 1 : 2 : 2: 2 : 3 ratio.

Methylpropyl ether 2-Pentanone


o
o-a*t,-tF'- il
H,C/
dr
a". -C. CH.
b. -;'j
H'rCt - CHr' CH. t '\
a" b' d'
Four unique hydrogens labeled Four unique hydrogens labeled
a-dtna3 2:2:3ratio. a-dina3:2:2:3ratio.

Diethyl ether 3-Pentanone

F'c-cni o-..,; tF'


fl
bb T"- c+{c-ctlr-cYu
b

- rvo unique hydrogens labeled Two unique hydrogens labeled


aandbina3:2ntio. aandbina3:2rctto.
Figure 2-32

a -.rpyright O by The Berkeley Review 127 Dxclusive MCAT Preparation


Organic Chemistry Structure Elucidation Spectroscopy and Analysis

The comparison of butanol to methylpropyl ether and diethyl ether is similar to


the comparison of pentanal to 2-pentano-ne and 3-pentanone. For instance, the
presence of only two signals in the t HNMR spectrum makes it easy to
distinguish 3-pentanone from 2-pentanone and pentanal in the same way it is
easy to distinguish diethyl ether from methylpropyl ether and 1-butanol. You
may see this theme repeated several times, so it is better to know basic trends
rather than specific examples.

In the case of alcohols, such as butanol, the protic hydrogen can be distinguished
from other signals by its broadness. Broadening results from hydrogen-bonding
in solution. Hence, alcohols are easily distinguished from ethers by the presence
of a broad. peak in their 1HNMR spectrum. In the case of NMR, the local
environment of equivalent hydrogens undergoing hydrogen-bonding is not
equal, so they appear at slightly different shift values. The degree of hydrogen- lrer,
-i
illl
bonding varies, so the effect is also varied, causing the signal to be a broadened. f,*::
Hydrogen-bonding causes the broadening of peaks in all types of spectroscopy.
Broad peaks are a dead give-away for protic hydrogens.

These examples were designed to look at symmetry within a molecule. You will
do this over and over throughout the spectroscopy section. The key to predicting
an NMR pattern for a compound is to understand the symmetry of the molecule.
You must be able to identify unique hydrogens and then determine their
respective features. This is where we shall start our analysis. The features we
shall focus on primarily are the integration of the peak, the splitting pattem
(shape) of the peak, and the shift value (measured i. pp*) of the peak. Be sure
that you understand the importance of each of these features and the factors that
can produce changes in them.

lnteg:
-ra n
'^'' t-
--- L

-^-! !:
_.AL L

: ack-r,
_5OU:
either
shou-li
hvdro,

Copyright @ by The Berkeley Review The Berkeley Revieu Copr:


Organic Chemistry Structure Elucidation Spectroscopy and Analysis

1H Nuclear Magnetic Resonance


3-pentanone has two unique types of hvdrogens in a B:2 ratio, its
P-":iy::
lHNMR spectrum has two signals with relative areas o{g,2. Figure 2-33 shows
the signals from the 1Flrultn spectrum of 3-pentanone. Each p"ui i, explained
in
terms of its splitting, integral, and shift value. The unique hyhrogens aie labeled
in the same fashion as they were in Figure 2-32.

r)
ll
tra\ zc\ .zCHz
CH. CH"
a' h'
Being next to a C=O group yields a shift Being next to a CH2 group yields a shift
value between 2.0 and 2.5 ppm. value between 0.9 and 1.5 ppm.
Being,next to a CH3 groupyields a quartet. Being next to a CH2 group yields a triplet.

1 ppm o pp-

Figure 2-33

The zero reference is ignored for analytical purposes, because it is there just to set
lhe scale correctly. The integral is not drawn on the spectrum in this example. In
ntost cases, you will be provided with the relative areas of the peaks, or you will
:e given a summation line to evaluate the relative areas. Either way, you must
:e able to apply the relative areas of the peaks to the quantity of hydrogens that
each peak represents. This is the start of NMR analysis.

Spectrum Analysis
iVe shall start off with how to anaryze the three basic components of the graph:
:'r.tegral (determined by the number of hydrogens making up a signal), splitting
:attern (derived from the coupling between hydrogen neighbors), and shift aalue
determined by the local magnetic field caused by either lone pair electrons in
rrotion or the electronic density associated with electronegative atoms). Each
piece is equally important. At times, one piece of information may be a little
more enlightening than the rest, but on the whole, every bit of data counts.

Integral
The peaks for a signal can be integrated, meaning that the area under the curve
:an be summed up, and set directly proportional to the number of hydrogens
:hat the signal represents. For instance, a CH3 group has a signal with a relitive
area of 3 compared to a CH2 group with a signal of relative area 2, working
rackwards from the integration to the structure, it is possible to deduce the
Sroup from the integration. For instance, a relative area of 5 can be attributed to
either 2 equivalent CH3 groups or 3 equivalent CH2 groups. Further inspection
should reveal which of the two scenarios is responsible for the six equivalent
hydrogens.

11 Copyright O by The Berkeley Review 129 Exclusive MCAT Preparation


Organic Chemistry Structure Elucidation Spectroscopy and Analysis (

Splitting Pattern )r_

The splitting pattern, also referred to as coupling, corresponds to the number of t'
hydrogens on a neighboring atom. Like electrons, nuclear particles have spin rl:

that can be classified as either up or down. The magnetization caused by the '1:

nuclear spin of hydrogen can be felt by the hydrogens on a neighboring atom. :-


Because the spin can be either of two ways, the magnetic field may be additive or :lll,::

subtractive. The random distribution of spins is used to determine the number of ::,L

hydrogen neighbors a group has. For instance, the CH3 group in 3-pentanone iir:
(labeled with an a in Figure 2-33) is next to a C}{2 (labeled with a b in Figure 2- It"
33). The two hydrogens of the CH2 have one of four possible spin combinations: yril

up/up, up/down, down,/up, or down,/down. Every CH3 group next to an rit,r

up / up CH2 group has a slightly higher signal, while every CH3 group next to a liI
down/down CH2 group has a slightly lower signal. Every CH3 group next to an p
up/down or down/up CHZ group has a normal signal, because the opposite 'llri

spins cancel each other. The result is that one out of every four times, the CH3
signal is slightly higher, two out of every four times the signal is unaffected, and
one out of every four times the signal is slightly lower. This is why the CH3
signal in 3-pentanone occurs as a triplet (in a 1 : 2 : 1 ratio).

Likewise, the CH2 group of 3-pentanone is next to a CH3. The three hydrogens
of the CH3 have eight possible spin combinations: up / up / up, down/down/ up, -*

dow n / up / down, up / down/ down, up / up / down, up,/ down / up, down / up / ap, It
or down/down/down. If all three spins are up (up/up/ up), then the net spin is
tl
+3/2. If only two spins are up, then the net spin is +7/2. There are three
combinations where two spins are up and one is down (up/up/down, rt

up / down/ up, down/up / up), so this is three times as frequent as the all spin up li'
combination. The same thing can be done for the one spin up combinations
(down/down/up,down/up/down, upldown/down) and the all spin down fr,
combination. The result is that a quartet is found to be in a 1 : 3 : 3 : 1 ratio.

This is why the CH2 signal in 3-pentanone occurs as a quartet (1 ; 3 : 3 : 1). There :-.
are eight outcomes, but three of them share one value and three of them share j'.
another value, so we see only four different outcomes. Working from a spectrum
i'
to a structure, it is possible to say that a 1 : 3 : 3 : 1 quartet is the result of the
hydrogens on a carbon being next to three equivalent hydrogens, often due to the
presence of a CH3 group as the neighbor. To determine the ratio of the peaks
within an overall signal (like the 1 : 3 : 3 : 1 value for the quartet), you can use
Pascal's triangle for binomial expansion to get the relative area of each peak
within the signal. Table 2-3 shows Pascal's triangle along with a brief
explanation of what the relative numbers are expressing about the shape of the
peak and the abundance of the signal. As the relative amount gets smaller, it is
harder to distinguish a peak from noise in the baseline signal.

Neighbors Signal Shape Pascal's Triangle Ratio of Peaks in Signal


0Hs Sinslet 1 1 peak
1H Doublet 11 2peaks:L:1ratio :r:
2Hs Triplet t27 :2: 1 ratio
3 peaks: 1
3Hs Quartet 1331 4peaks:1:3:3:lratio
4Hs Quintet 74641 5 peaks:1, : 4: 6: 4 : 1 ratio

5Hs Sextet 1 5 1010 5 1 6peaks: 1 : 5 : 10 : 10 : 5 : 1 ratio -:


6Hs Septet 1 6 152075 6 1 Tpeaks: 1 : 6 : 15 : 20 : 15 : 6 : 1 ratio
.- :.-
Table 2-3

ir
Copyright @ by The Berkeley Review The Berkeley Review
Organic Chemistry Structure Elucidation Spectroscopy and Analysis

Shift Value
The shift value is a diagnostic tool for assessing the locai electronic environment.
it is measured in parts per million (ppm) relative to the magnetic field necessary
to detect a standard compound, tetramethylsilane ((H3C)aSi). It is used in
conjunction wiih known shift values to determine what functional groups are
neighboring. In the 3-pentanone example, the CH2 group adjacent to the
carbonyl group feels the magnetic field of the electrons on a neighboring oxygen
and thus requires a stronger external magnetic field to energize the spin levels
than a CH2 group that is next to an alkyl chain. A larger shift value in the
spectrum is thus observed than is typically observed for CH2 groups in
hvdrocarbons. This is referred to as being shifted downfield, which indicates
righer ppm values for the shift. To verify the presence of a C=O (carbonyl
group), consult Table2-4 to find that a value of 2.1 to 2.5 ppm is expected for a
r-rvdrogen on a carbon adjacent (alphn) to the carbonyl. Table 2-4 lists many
.orrunon lUNVR shift values used for analyztng spectra. The bold hydrogen in
each compound in Table 2-4 is the one of interest (to which the shift vaue
;orresponds) and the shift value is measured in units of ppm.

Hydrogen Atom D (ppm) Hydrogen Atom d (ppm)


RCHc 0.8 - 1.0 R2C=CH2 5.0 - 5.8
RCH2R (acyclic) 1.3 - 1.5 RCH=CRz 5.2 - 6.4
RCH2R (cyclic) 1.5 - 1.8 RNH2 1-3
R3CH r.5 - 2.0 RNHCH3 2.0 - 3.2
R2C=C(R')CH3 1.8 - 2.2 ArNH2 3.5 - 5.0
RCOCH3 (ketone) ) 1 _) q RCONHR (amide) 5-9
,\rCH3 2.2 - 2.5 ROH (alcohol) 1 - 5 (broad)

RC=CH 2.5 - 2.6 AIOH (phenol) 4 - 7 (broad)


R.OCH3 (ether) 3.5 - 4.0 ArH (benzene) 7.0 - 7.4
RCH2X (X = Cl, Br,I) 3.0 - 3.8 RCOH (aidehyde) 9.0 - 9.8
RCO2CHa (ester) 3.5 - 4.0 RCO2H (acid) 10 -72

Table 2-4

Erample 2.15
lentanal can be distinguished from 2-pentanone by what lgNIrrtR feature?
\. A 3H triplet at 1.5 ppm
B. A 2H multiplet at 1.8 ppm
C. A 2H triplet at 2.3 ppm
D. A 1H triplet at9.7 ppm

Solution
lentanal has an aidehyde hydrogen, while 2-pentanone does not. The shift value
:-r an aldehyde hydrogen is found between 9.0 and 9.8 ppm. The peak shape is a
:rp1et, because there are two equivalent alkyl hydrogens on carbon 2. The two
- .'-drogen
nerghbors couple with the aldehyde hydrogen to split it into a triplet in
:e lHNMR spectrum. This makes choice D the best choice. 2-pentanone could
:e distinguished by the singlet of relative integration 3 caused by the isolated
:'.ethyl group adjacent to the carbonyl. Because the carbonyl carbon has no
:.','drogens attached, the methyl group of carbon one is not coupled to any other

-opyright O by The Berkeley Review l5r Exclusive MCAT Preparation


Organic Chemistry Structure Elucidation Spectroscopy and Analysis
I
hydrogens, which makes it appear as a singlet in the 1HNMR spectrum. The two
structures are drawn below, along with their proton NMR features. l
m
o o &
lt il A
(-
d
(- f
.-/*\ b c e ,./-\ b c cl r(
H CH2CH2CH2CH3 ^
H:C CH2CHzCH3
Il
Aldehyde hydrogens show signals Because there are no hydrogens on
around 9.7 ppmin the
1HNMR.
the neighboring carbon, the methyl
f,
1HNMR. lfi
group is a singlet in the n
ffin
GT
gM
Example 2.16
\A/hat signals are present in the lIlUVn spectrum of chloroethane? ffim

lffi
A. A downfield doublet and an upfield triplet
B. A downfield triplet and an upfield doublet -
C. A downfield triplet and an upfield quartet ft
D. A downfield quartet and an upfield triplet W
h
Solution
Chloroethane has two unique types of hydrogens. This results in two signals in
lL
its 1HNMR spectrum. The two hydrogens on carbon 1 are split into a quartet bv
n
rf
the three hydrogens on carbon 2. Equally, the three hydrogens on carbon 2 are
split into a triplet by the two hydrogens on carbon 1. The lines split according to
n
the neighboring hydrogens and project down to the spectra. The quartet is
farther downfield than an ordinary alkyl group due to the electron density on the
lfr
chlorine atom. This means that the triplet is upfield, making choice D the best
l[
answer. ffi
tfrr
There are two types of H, so there are two
1HNMR
signals. ag
ffir
CIH
\^ -.Lu n:
nttuTt-t\ l[r
H fi.
n
fl,
n"

are coupied to the three F{t &


on the carbon 2, resulting
iii
,II
I

on carbon 1, resulting in a M
in a quartetdownfield. iiI i
triplet upfield. mM
l]lt ilhs
tttt
tu
ffi
fiF

Actual srectrum
1

Copyright @ by The Berkeley Review 132 The Berkeley Review


Organic Chemistry Structure Elucidation Spectroscopy and Analysis

Erample 2.17
lentanol can best be distinguished from ethyl propyl ether by which of the
:,rllowing features in the lHNMR spectrum?
\. A 3H triplet at 1.2 ppm
B. A 2H triplet at 3.5 ppm
C. A broad 1H peak between 1.0 ppm. and 5.0 ppm
D. The total number of signals in the ether is substantially less
Solution
in alcohol is distinguishable from an ether by its broad peak between 1.0 ppm.
:rd 5.0 ppm, so pick C for best results. The peak is broad due to the hydrogen-
:onding within the alcohol. The broadening of the peak makes it difficult to
=-,-aluate the integration of the alcohol hydrogen peak. Both structures contain a
:;{ iriplet at1.2ppm, so choice A is eliminated. Both structures contain a 2H
:rplet at 3.5 ppm, so choice B is eliminated. Choice D is eliminated, because
--:.ere is a small difference in the number of peaks between the two compor-rnds.

E-rample 2.1.8
',',hich
of the following is a common feature in the 1HNMR spectra of all methyl
retones?
\, A triplet at 1.5 ppm (3H)
B. A quartet at 2.3 ppm (2H)
C. A doublet at 2.3 ppm (3H)
D, A singlet at2.7 ppm (3H)

Soiution
- methyl ketone has an isolated methyl group neighboring the carbonyl carbon
'.;hich has no hydrogens). Having no neighboring hydrogens makes the peak a
.nglet. The protons on a carbon alpha to a carbonyl are found between 2.0 and
I 5 ppm. The signal has a relative integration of 3 hydrogens. The only answer
:hat fits this is choice D, a 3H singlet at 2.1 ppm. The best choice is choice D.

Example 2.19
-- monosubstituted benzene has which of the following in its IUXVIR spectrum?
\. A peak at 1.2 ppm (5H)
B. A peak at 5.3 ppm (5H)
C, A peak at 7.2 ppm (5H)
D. A peak at 8.1 ppm (5H)
Solution
\lonosubstituted benzenes have a single peak around 7.0 ppm. The aromatic
l'drogens appear as one singlet, despite the fact that they are not all equivalent
:r'symmetry. The key to this question is not the integration or the peak shape,
:ut the shift value. Choice D is just a little too high, so it is eliminated along with
:hoices A and B, which are far too low" To do this problem quickly, you should
:e familiar with the common peaks. Choice C is the best answer.

Be aware of certain peaks and features that occur over and over. For instance,
,vhenever you see a triplet and quartet in a 3: 2ratio, you should conclude that
there is an isolated ethyl group (H3CCH2-) in the molecuie somewhere.

Copyright @ by The Berkeley Review r33 Exclusive MCAT Preparation


Organic Chemistry Structure Elucidation Spectroscopy and Analysis

in a 6 : 1 ratio, you should conclude


tr\lhenever you see a doublet and a septet !

that there is an isolated isopropyl group ((H3C)2CH-) somewhere in the I


molecule. Rather than looking at molecules to determine the spectra (going from 1

structure to spectrum), it is important to work problems from the spectrum to the ft


structure. By recognizing the combination of peaks, you will save time in
J
determining the unknown structure. This is very conunon in NMR spectroscopy.
I
We will use symmetric structures at first and then move on to more difficult
(
examples. The MCAT has traditionally asked simple questions about this topic,
t
but it's better to be safe than sorry, so we will present examples that are harder
$
than the questions they have given on previous exams. In the following few
W
questions are sample spectra from which you must determine the corresponding
structures. It helps to solve for the units of unsaturation first. Once these are Tffi

ffi
known, deduce possible functional groups that fit both the heteroatoms in the
formula and the calculated units of unsaturation. For instancet zero units of &
unsaturation and one oxygen can be an aliphatic ether or an aliphatic alcohol. &
Take advantage of the multiple-choice format by eliminating wrong answers as
you come across them. In the case of a compound with zero units of
-
fr
unsaturation and one oxygen, an answer choice of a ketone is eliminated m
immediately. Any structures with rings or n-bonds should be eliminated. This
ability to eliminate wrong answers can be very useful in the multiple-choice il"
format. To gain both insight and experience, try the following spectral problems: l.
c.
D.
Example 2.20
\A/hat is the name of the compound that has the following 1I-nuraR spectrum, and
&
whose formula is C7H14O? W
tu
ffi
:-
fr
mm
ffiF

A. 2,4-dimethyl-3-pentanone
B. 2,2,4,4-tettamethyl-3-pentanone
C. 1,1,3,3-tetramethyl-2-propanone
D. 2,2- dimethyl-3-pentarone

Solution
The septet and doublet in a 1 : 5 ratio are a dead give-away for an isopropd
group. Choice D is eliminated, because it does not have an isopropyl group"
Choice B is eliminated, because it contains too many carbons (nine instead od
seven). Choice C is eliminated, because the structure is misnamed. The bes,t
answer and only remaining choice is A.

Copyright O by The Berkeley Review 134 The Berkeley Revieu


Organic Chemistry Structure Elucidation Spectroscopy and Analysis

Example 2.21
.The
s-hape-of the signal at 2.3 ppm in the luNvtR spectrum in Example 2.20 is
best described as:
A. a quartet.
B. a sextet.
C. a septet.
D. an octet.

Solution
lve counting seven apexes within the signal at2.3 ppm, and seven apexes (peaks)
is referred to as a septet. check to see whether the ratio is 1 : 6 :rs :20: 15 : 6 : 1
to be sure. This seems reasonable, so the best answer is choice C. If you already
know the structure, then you can see that the septet results from six equivalent
hvdrogen neighbors on the methyl groups neighboring the alpha carbon.

Example 2.22
The ratio of the areas under the peaks within a quartet is:
-{.7:2:2:1..
B. 1 :3:3: 1.
C. 2:5:5:2.
D. 1:4:4:'J..
Solution
Bv using Pascal's triangle, you can easily determine the ratio. It is a good idea to
know the ratios of the more common peaks such as a doublet, a triplet, and in
--}'ris case a quartet. A
quartethas a ratio of 1 : 3 : 3 : 1. Choice B is correct.

Example 2.23
'r\'hat is the IUPAC name of the compound represented by the following lHNltR
spectrum, whose molecular formula is C4H3O?

c4H8o

-{.. Butanal
B. Butanone
C. Ethyl ethanoate
D. Methyl propanoate
Solution
Because the formula has only one oxygen, the two esters (choices C and D) are
immediately eliminated. The question is now reduced to determining whether
Jre compound is an aldehyde or ketone. An aldehyde would have a peak in the

I Copyright O by The Berkeley Review 135 Exclusive MCAT Preparation


Organic Chemistry Structure Elucidation Spectroscopy and Analysis
I
proton NMR between 9.0 and 9.8 ppm. There is no peak in that range, so choice
A is eliminated. Only choice B remains. To be certain, butanone E"r
(CH3COCHZCHS) has three types of hydrogens and thus three peaks in its -.
proton NMR spectrum. The peaks are a singlet (3H), a quartet (2H), and a triplet :n
(3H). This is what the spectrum shows, so choice B is correct.
"4t
ffi
C.
Example 2.24
\iVhat is the IUPAC name of the compound represented by the following lHirll,tR
lL-il

spectrum, whose molecular formula is C4H3O2?


50
-.
][jr:

ta
t
H

l,/{1/,1

- :-:

2 ppm 1 ppm

A. Butanal
B. Butanone
C. Ethyl ethanoate
D. Methyl propanoate

Solution i
This question is similar to the previous question, except the ketone and aldehyde
are eliminated, because there are two oxygen atoms in the molecular formula. "0,
Choices C and D show identical peak shapes and integrals in their 1HNMR ffi
spectra" The distinguishing feature is the shift value of each signal. The ethyl C
ethanoate (structure shown on the left below) exhibits a quartet near 4.0 ppm,
making choice C correct. The structure of methyl propanoate is shown on the
right below. Srntr

OO -_E
fri j

,.4.,/tT'---,,
o o
T,t-.-A o/'\,
trr0g:

T" l'T' 1'Tt


*Ltxn:

I.EI
:I:
I -*
3H Singlet 2H Quartet 3H Triplet 3H Triplet 2H Quartet 3H Singlet l_: a.

at - 2.1 ppm at - 3.8 ppm at - 1.0 ppm at - 1.0 ppm at - 2.3 ppm at - 3.5 ppm

Copyright O by The Berkeley Review The Berkeley Review =! JIl


Organic Chemistry Structure Elucidation Spectroscopy and Analysis

Example 2.25
Hydrogens on a carbon adjacent to two equivalent CH2 groups show which type
of signal in a IHNMR spectrum?
A. A1:3:3:lquartet
B. A1:4:4:lquartet
C. A1:3:5:3:lquintet
D. A1:4:6:4:Lquintet
Solution
Having two equivalent CH2 groups adjacent to the site of interest results in a
total of four equivalent hydrogen neighbors. Four equivalent hydrogens split a
signal into a total of five (4 + 1,) peaks. This makes the signal a quintet, which
according to Pascal's triangle (or binomial expansion of any sort) has a ratio of L :
1:6:4: 1. The best answer is choice D.

Example 2.26
\Vhat is the common name of the compound represented by the following
HtWtR spectrum, whose molecular formula is C6H16O2?

-A.. Para-ethoxy phenol (H3CH2COC6HaOH)


B. Ortho-ethoxy phenol (H3CH2COC6HaOH)
C. Para-methoxy anisole (H3COC6HaOCH3)
D. Ortho-methoxy anisole (H3COC6HaOCH3)
Solution
The symmetry in the 1HNVR spectrum is associated with a structure that is also
>r'mmetric. The only way to get two types of hydrogens on a disubstituted
lenzene is to have two equal substituents on benzene para to one another. This
eliminates choices B and D. Based on the formula, this molecule has two
rLethoxy groups para to one another on the benzene. All of the benzene
hvdrogens are equivalent, which explarns why only a singlet is observed. The
best choice is thus answer C. Choice A would exhibit more than two peaks, so it
Ls eliminated.

Copyright @ by The Berkeley Review r37 Exclusive MCAT Preparation


Organic Chemistry Structure Elucidation Spectroscopy and Analysis
I
Recognizing Special Structural Features D
Recognizing special structural features requires knowing some general shift ftm
values (6-values) from memory. You should know that a carboxylic acid @
hydrogen falls in the 6 = 1.0 - 12 ppm range and that the signal is broad. An mm
aldehyde hydrogen falls in the 6 = 9 - 10 ppm range, aromatic hydrogens fal in ffio
the 6 = 7 - 8 ppm range, vinylic hydrogens fall in the 5 = 5 - 6 pp* range, alkoxy
.f'
hydrogens fall in the 6 = 3.5 - 4pp- range, and alphahydrogens fallin the E=2- du
?.5
pp* range. Figure 2-34 shows a molecular structure and its corresponding al
lHNMR spectrum that includes many of these key peaks.
fu@

ffi
,fillfl

o 3H 3H 3H m.

oHa
W
o
b 11H 2H 2H 1H
c d
A A1H

12 ppm 10 ppm 8 ppm 6 PPm 4 ppm 2 ppm


PP* o

Figure 2-34

Be certain that you can match the peaks in the spectrum to the hydrogens in the
structure drawn above it in Figure 2-34. This can be done on the exam using a
chart of values if one is given, but it is not a bad idea to know the values from
memory.

Example2.27
\A/hat is the IUPAC name of an unknown compound with the molecular formula
C3H6O, an IR absorption at 1722 cm-l, and three 1HNVR peak,; one at 9.7 ppm
(1H), one at 2.3 ppm (2H), and one at 1.4 ppm (3H)?
A. Propanoic acid
B. Propanal
C. Propanone
D. Methyloxyrane

Solution
There is an excess of information in this question beyond what is needed to
aswer it. The one and only piece of information you need is the peak at9.7 ppm,
which makes the compound an aldehyde. Pick choice B and move on quickly.

Copyright @ by The Berkeley Review The Berkeley Revier


Organic Chemistry Structure Elucidation Spectroscopy and Analysis

Distinguishing Disubstituted Benzenes


Integrals teil us the number of equivalent hydrogens in a signal and are often
employed to determine the position of substituents on disubstituted benzene
rings. Structures that are highly symmetrical have more equivalent hydrogens
than asymmetrical structures. A 1,4-disubstituted benzene ring (referred to as
para") shows the fewest peaks in the aromatic region of the spectrum of all
disubstituted benzenes, due to its mirror symmetry. Both a 1,2-disubstituted and
a 1,3-disubstituted benzene ring (referred to as "ortho" and "meta" respectively)
have four unique hydrogens in the aromatic region of the spectrum. Figure 2-35
shows two sets of disubstituted benzenes, one set of three with identical groups
and another set of tlrree with two different groups on the benzene.

Case 1: The two substituents on benzene are equal:

;f.
X
Ha X Ha Ha

Hb Ha Ha Ha
U
rlb Hb X
2 different Hs 3 different Hs All Hs are
inal:lratio in a 1 :2:lratio equivalent

Case 2: The two substituents on benzene are not equal:

Y Hb Ha

Hd Hc Y

{ different Hs in 4 different Hs in 2 different Hs


a1:1:1:lratio a1:1:1:lratio inal:lratio
Figure 2-35

:rara substitution is the easiest arrangement to distinguish of the three possible


-.:uctural isomers, because it has a doublet of doublets. The dissimilar heights of
-:s peaks can be attributed to a mathematical phenomenon whereby peaks, as
--:.ev near one another, begin to coalesce. Figure 2-36 shows the aromatic region
: a l HNMR spectrum if u puru substituted benzene ring, where the two
.:bstituents are nonequivalent. Para coupling is a highly recognizable feature.

Enlargement of the aromatic region shows that the


splitting is a doublet of doublets, corresponding to a
para-substituted benzene ring. X
Ha Ha

Hb
7.8ppm 7.6ppm 7.4ppm 7.2ppm 7.0ppm

Figure 2-36

-;.pyright @ by The Berkeley Review Exclusive MCAT Preparation


Organic Chemistry Structure Elucidation Spectroscopy and Analysis o
TE
Example 2.28 rL"AlI

what is the common name of the compound that has the formula C6Hgo, an IR trs':
absorption at7722 cm-], and three notable lHNMR peaks at9.7 ppm (1H, s), 2.3 &,f,1r

ppm (4H. dd), and 2.2ppm (3H, s)? rli J


A. Ortho-methylbenzoic acid &e"r

B. Para-hydroxyacetophenone
*-"
C. Ortho-methylbenzaldehyde J,ffi

D. Para-methylbenzaldehyde Mfl
,ILfu.

Solution *."'
The compound has only one oxygen, so neither a carboxylic acid (methylbenzoic [i *-
acid) nor a hydroxy ketone (hydroxy acetophenone) is possible. Choices A and B :mb{:.

are eliminated. we know that the compound must be an aldehyde from the tnlm

choices that remain, so the lriNvtR peak at 9.7 ppm and the lR absorption at 1me
1722 crn-ldo not help our efforts to identify the compound. The trqrulan signal $fft
at 7.3 ppm is a doublet of doublet (dd), which indicates para-substitution. This
makes choice D the best answer.

Deuterated Solvents for 1HNMR


Because the solvent is in substantially higher concentration than the solute, it is
imperative that the solvent not have any hydrogens. If the solvent has 1H nuclei.
then it would produce the largest signal in the spectrum, eliminating integration
and causing the other peaks to disappear into the baseline. To avoid this
problem, solvents are chosen that have deuterium (2H) instead of the standard
isotope of hydrogen (1u;. one potential problem occurs when protic compounds
are dissolved into deuterated protic solvents, such as D2O. protic hydrogens can
undergo exchange with the protons of the solvent, if the solvent is protic.
Although the dissociation constant (Ku) may be small for compounds such as
alcohols, over enough time all of the hydrogens can be released and then are able
to reform their bonds. rf D2o is present in the solution, then deuterium will
gradually replace protic hydrogens capable of undergoing exchange. This causes
the signal for the protic hydrogen to disappear gradually.

Example 2.29
which of the following compounds does Nor lose a luNIvtR signal after D2o
has been added to a solution containing it?
A. Carboxylic acid
B. Cyclic ether
C. Primary amine
D. Secondary alcohol

Solution
If a compound contains a protic hydrogen, then it loses a peak from its 1HNMR
spectrum when D2o is added to the solution. Primary and secondary amines
have a hydrogen bonded to nitrogen, so thev are protic. This eliminates choice C.
All alcohols have a hydrogen bonded ,o s11:gen, so all alcohols are protic. This
eliminates choice D. A cirboxylic acid hasl dissociable proton, so it readily
exchanges with deuterated water. choice A is eliminated. An ether, whether
cyclic or not, has all of its hycirogens bonded to carbon, so it is aprotic. When ,- f
D2O is added to an ether, no exchange transpires. The best answer is choice B.
i,:,:

Copyright @ by The Berkeley Review The Berkeley Review -:r:


Organic Chemistry Structure Elucidation Spectroscopy and Analysis

13CNMR Spectroscopy
Carbon-13 NMR focuses on the carbons of a compound rather than the
hydrogens. The NMR cannot detect carbon-12, because it has no nuclear spin.
However, carbon-13 has an odd number of nucleons (particles in the nucleus), so
ii has a nuclear spin. For that reason, NMR can be used to detect carbon-13.
Because carbon-13 constitutes only about 1"/' of all carbon atoms, a 13CNMR
requires maly more scans to obtain a spectrum than 1HNMR. This is why the
'raseline
in 13CNMR spectra is noisy (scattered and messy). l3CNltR is used to
rdentify the number of unique carbons in a compound. A typical application of
t'hrs technique is to distinguish the substitution of disubstituted benzenes. Figure
l-37 shows the proton-decoupled (ali singlet spectrum) C-13 NMR spectra for
-,2-dimethylbenzene, 1,3-dimethylbenzene, and 1,4-dimethyibenzene. What is
:reant by "proton-decoupled" is that the compound is irradiated constantly
similar in concept to Dolby noise^reduction) with the coupling energy, so that
iere is no coupling between the IJC and IH atoms. This makes all of the peaks
=inglets, as drawn.

80

# CH"

ppm 140 120 100 BO 60 40 20

Figwe 2-37

llhe shift values for 13CNMR are simple to recall. They come in blocks of 50
rpm. An sp3-hybridized carbon next to a carbon has a shift value between 0 and
50 ppm. An sp3-hybridized carbon next to an electronegative atom has a shift

Copyright @ by The Berkeiey Review t4t Exclusive MCAT Preparation


Organic Chemistry Structure Elucidation Spectroscopy and Analysis j
value between 50 and 100 ppm (usually less than 75 ppm). An sp2-hybidized,
carbon next to a carbon has a shift value between 100 and 150 ppm. A carbonyl tr:
carbon with sp2-hybridization has a shift value between 180 u"a zso ppm. The
shift value increases as the substitution increases, meaning that 3' > 2" > f in
shift value. Quaternary carbons do not show up on the 13cruvR very well, due
to their long relaxation times. \44-rat is meant by a "long relaxation time" is that
the excited energy state of carbon takes longer than a few seconds to dissipate the
energy into solution and relax back to a lower energy level. We will not focus on
relaxation times, but instead will focus only on the application of information
extracted from the spectra.

In the three spectra shown in Figure 2-37, you'lrnote that the number of peaks in
each spectrum corresponds to the number of unique carbons in the compound
the spectrum represents. For the ortho compound, there are four carbons in a 1 :
1:1:1ratio. Thespectrumshows fourpeaks ina 1:1:1:1ratio. Three of the
peaks are from sp2-hyb.idized carbons and one is from an sp3-hybridized
carbon. For the meta-substituted and para-substituted compounds, a similar
relationship between spectrum and unique carbons in the structure is observed.

using spectroscopy data, you should be able to solve any problem following a
systematic procedure. It is best to use l3ctlVR to get-the symmetry oI a
structure and to identify selected functional groups (in a manner similar to IR
spectroscopy). Don't make much more out of i3cNun spectroscopy than this.
Be sure to use your degrees of unsaturation in structure elucidation problems!

Stn
1.

Sper
1. .

Copyright @ by The Berkeley Review t42 The Berkeley Review copl


Organic Chemistry Structure Elucidation Section Summary

Key Points for Structure Elucidation (Section 2)


lsomerism
1. Isomers (compounds with the same type and number of atoms but different
spatial arrangement due to bonding, connectivity, or molecular contortion)
a) Structural isomers (isomers with different connectivitv because of
different bonding)
i. Have different IUPAC names
ii. Can be ciassified as skeletal isomers, positional isomers, or functional
isomers
iii. Their number can be determined by evaluating possible chain
lengths and connectivity
b) stereoisomers (compounds with the same bonding, but different spatial
arrangement)
i. Can be classified as configurational isomers (geometrical and optical)
or conformational isomers
ii. Have same IUPAC root, but a different prefix
iii. Conformational isomers are formed by rotating or contorting a
structure (leading to eclipsed and staggered conformations, with
groups gauche and anti to one another)
iv. Maximum number of possible optical isomers is 2n, where n is the
number of chirai centers in the compound
c) Nlewman projections (front view of moiecule)
d) Cyclic molecules
i. Three- and four-membered rings have ring strain that makes them
highly reactive
ii. Five- and six-membered rings are stable, with six being the more
stable of the two
iii. Cyclohexane (and six-membered rings in general) assume chair
conformation, with groups equatoriai (more stable position) and
axial

Structural Insights
1. Structural symmetry
a) Plane symmetry (mirror plane in molecule splits molecule into equal
halves)
b) Point symmetry (molecule has an inversion point at its center of mass)
c) Units of unsaturation
i. Determined from excess bonding electrons divided by 2
ii. Unirs of unsaturation - 2(#C) + (#N) - (#H)- (#X) + 2
iii. Describes the number of n-bond., ur-ra .i-rg, in a molecule
Spectroscopy
1. IR spectroscopy (used for vibrational excitation)
a) Ranges from 1000 cm-1 to 4000 cm-1 (about 3 kcai/mole to 10 kcal/mole)
b) Correlates bond-stretching and bond-bending to absorbance
c) Used to identify functional groups
d) Key peaks: C=O around 1700 cm-1, O-H around 3400 cm-1 (broad), and
C-H around 3000+ cm-1 (varies with hybridization-- ,p > ,p2 > ,p3j

Copyright @ by The Berkeley Review t43 Exclusive MCAT Preparation


Organic Chemistry Structure Dlucidation Spectroscopy and Analysis

e) spectrophotometer uses salt plates to hold sample, because salt plates


have ionic bonds and therefore do not interfere rn'ith the simpte
molecule's absorbances

2. Ultraviolet-visible (uv-vis) spectroscopy (used for electronic excitation)


a) Ranges from 200 nm to 800 nm, increasing in wavelength as conjugation
increases
b) Typicallv used for analyzing compounds with n-bonds, especially
conjugated systems
c) Peak intensity and wavelength increase as the amount of conjugation
increases

3. NMR spectroscopy (the basics of I HNMR analysis)


a) IUXVIR Integration (Quantitative analysis using relative area under the
curves)
i. Area under the curve for each signal is proportional to the number of
hydrogens responsible for the signal
ii. Connectivity can often be deduced from the integration ratio
b) lUXUR peak shape (coupling and J-values)
i. The number of peaks within a signal equals the number of
neighboring hydrogens plus 1
ii. Hydrogens coupled to one another have the same j-values
iii. The ratio of the area of the peaks within a signai can be determined
using Pascal's triangle
c) lUNVtn shift value (eiectron-rich environments affect shift values by
exerting a magnetic field)
i. Common signals include 9-10 ppm for an aldehyde and around 7
ppm for hydrogens on benzene
ii. "Upfield" refers to shifts at lower ppm values
iii. All shifts are referenced against si(CH3)a, which is assigned a value
of 0 ppm
d) 1HrultR special features (effects of deuterium and structural symmetry)
i. Exchanging of deuterium for protons (peak disappearance)
ii. Para substitution pattern (symmetric benzenes have unique spectra)
iii. Solvent choice (solvent must be invisible)
e) 13CXUR and symmetry
i. The number of signals in 13CNVR spectrum is equal to the number
of unique carbons in the molecule
ii. Alkene carbons have signals between 100 ppm and 150 ppm, while
carbonyl carbons have signals between 180 ppm and 230 ppm
iii. 13C is a rare isotope, so 13CNMR spectra require more scans and
have more noise in their baseline than IUXVn spectra

Copyright @ by The Berkeley Review r44 The Berkeley Review


Stnrcture
I

Elucidation
Passages
14 Passages
l OO Questions

Suggested schedule:
I: After reading this section and attending lecture: Passages I, V, VI, VIII, & XI
Qrade passages immediately after completion and log your mistakes.
II: Following Task I: Passages III, IV XII, & Xlll (27 questions in 55 minutes)
Time yourself accurately, grade your answers, and review mistakes.
III: Review: Passages II, VII, IX, X, XlV, & Questions 98 - lOO
Focus on reviewing the concepts. Do not worry about timing.

REru{ffiNY
l)n.n.v.r.-E.w'
Speci altztng in MCAT Preparation
rsftfficffirrui1ii1iruffiilil#tflff 1 lil
{tndh
mm@{

@m)
r,Mm

I. Chair Conformation ry
(r -7) ttAmrl

mc
II. Rotational Energy Diagrams (B - 14) lffi,Gdli'r

lhm
III. Deuterated Cyclohexane (r5 - 22)
dffi
IV. Infrared Spectroscopy (23 - 2e) ,t5m
,@mo
V. Unknown Alkene Determination (50 - 56) qn*
fr
VI. Structure Elucidation (37 - 43)
VII. NMR and IR Spectroscopy (44 - so)
VIII. NMR Data Table (5r - 57)
IX. Alkene-Coupling Experiment (58 - 64)
X. Unknown Compound Identification (65 - 71)
XI. Carbon-l5 NMR (72 - 78)
XII. Distinguishing Isomers Using lnnmR (7e - 84)
XIII. Structure Elucidation Using lHnMn and IsCNMR (B5 , eO)
XIV. Proton NMR of an Unknown (er - e7)
Questions not Based on a Descriptive Passage (eB - l oo)

Structure Elucidation Scoring Scale


Raw Score MCAT Score
84 - 100 15-15
66 B5 lo-12
47 65 7 -9
34-46 4-6
t-33 t-3
Passage I (Question 1 - 7) t A carbon-deuterium bond is shorter than a carbon-
l-rydrogen bond. Using this idea, how many deuterium
Cyclohexane is not a planar molecule, but in its mosl atoms assume axial orientation in the most stable
stable conformation, fbur of the six carbons that make up the conformation of the following molecule?
ring are coplanar. Studies using I HNMR and X-ray
:rystallography demonstrate that the most stable
:onformation of the molecule has carbon-carbon-carbon bond
-rngles of approximately 107.5' and 1l l' in the ring and that
.nere are two types of hydrogens present, axiaL and equatorial.
The axial hydrogens are bonded directly above and directly
:elow the ring carbons. The equatorial hydrogens lie away
:rom the cyclohexane ring. A. 0
A low-temperature lHNVR stucly was conducted to B. 1

::termine the equilibrium constant for the conversion from c.2


:i axial- 1,2-dimethylcyclohexane to diequatorial- 1,2-dimethyl- D. 3
:..clohexane by way ol a ring-f'lip plocess (Ksq-1,2). This
.llue is directly comparable to the equilibrium constant fbr
::e conversion fiom diaxial-1,4-dimetl-rylcyclohexane to
-.equatorial-1,4-dimethylcyclohexane by way of a ring-f1ip
3 . What is the value of K.O- 1,4 for the conversion of
:rocess (Keq-l,a), given that they are both trcns-substituted trans-1,4-dimethylcyclohexane from its diaxial
:,,clohexanes. The difference between their equilibrium conformation to its diequatorial conformation?
, rnstants is attributed to the gauche and anti orientations A. 0.0029
: -,ssible with the methyl substitucnts at the 1,2 positions. B. 2.16
j:udies have shown that lalger substituents prefer the c. 4.31
.:uatorial orientation of the so-called chair conformation. D. 345
.qure I summarizes the findings of the study.

CHr
I
nttt
I 4, Cis-i,4-dimethylcyclohexane, in its most stable chair

H H,c{{\ conformation, CANNOT have which of these


interactions?
-/--_/ t\eq = ll'J
A.
I
CH3/H gauche
*-H. B. CH3/H anti
C. CH3/CH3 gauche
9H..
D. H/H anti

s,cG4,
'
K"o+ +.Zl
\--l-{/'cH\
5 . Which of the following accurately describes the value of
K"O for the conversion from one chair conformation [o
, .l: the other chair conformation for the compound cis-t,2-
Figure 1 Ring-flipping of trans-dimethylcyclohexane dimethylcyclohexane?

The values fbr the equilibrium constants can be applied


A. Less than or equal to 0
Jetermine the relative steric hindrance of one substituent
B. Greater than 0 and less than I
, rpared to anolher. A bulkier group exhibits greatet' steric C. Equal to 1

" rdrance, so the equilibrium lies more towards the more D. Greater than 1

.rle of the two possible chair conl'irrnations. Hence, a


.-3ater equilibrium constant irnplies that there is a greater
---ee of steric hindrance in the less stable confblmation.
6. The value of K"O for the conversion of cls-1,3-
I . Which of the following is the most stable orientation of dirnethylcyclohexane from its diaxial conformation to
a substituent on a cyclohexane molecule? its diequatorial conformation is:

A, Axial orientation ol a chair conlormation A. less than 0.22.


B. Equatorial orientation of a cl.rair conlotmation B. between 0.22 and 1.

C. Bridge orientation of a boat conlormation C. between I and 4.3 i.


D. Oar orientation of a boat conlormation D. greater than 4.31.

-:pyright @ by The Berkeley Review@ 147 GO ON TO THE NEXT PAGE


7. Which three-dimensional conformation corresponds to Passage ll (Questions 8 - 14)
the 3-hydroxy-cls-decalin, shown below?
Alkanes are hydrocarbons that contain all sigma bonds.

qlr"'
H
Sigma bonds have linear electron density (electron density
that is localized between the two nuclei of the bonding
atoms). This allows for free rotation about a sigma bond.
Rotation about sigma bonds is continually occurring at
temperatures above absolute zero, although the rate of the

&F
rotation varies. However, the rotation does not necessarily
complete a full 360' cycle about the sigma bond.
OH Some conformations encountered during rotation are of
high energy (due to steric repulsion) and others are of low
energy (due to minimal steric interactions). The most stable
conformation occurs when the largest groups are as far apart
as possible. When two groups are as far apart as possible,
the conformation is referred to as staggered, and the bulkiest

ffi*f=," D. substituents are said to be anti to one another. The least


stable conformation occurs when the largest groups interfere
with one another. This is known asfully ecLipsed. Drawn in
Figure I is an energy diagram for the counterclockwise
rotation about the C2-C3 bond fbr R-2-methyl-1-butanol.

bo
b
+.1

g l8o 360
Degrees displaced from initial state

Figure 1 Energy during rotation about the C2-C3 bond

The three apexes occurring at 60', 180' and 300' on the


graph are not of equal energy. In 2-methyl-1-butanol, carbon
2 is a stereocenter. Because of this asymmetry, none of the
eclipsed or staggered conformations are equal in energy. Al1
visual projections show asymmetric steric interactions.
Although the molecule is constantly rotating about its bonds,
it assumes its most stable conformation most of the time.

8 . Which of the following structures represents the


molecule at the 240" point on the graph in Figure 1?

B.
H.C CH.
Hf{,,cH:oH
C. D.
H:C.
sr,'HH
J\
H
\CH2OH

CHr
TF{::""
Copyright @ by The Berkeley Reviewo GO ON TO THE NEXT PAGE
9. The 60'point on the graph in Figure 1 represents the 1 3. The reason there is no rotation diagram for trans-2-
structure when it is: butene is that:
A. eclipsed and the methyl substituent on carbon-2 A. gauche is not favorable for alkenes.
interferes with carbon-4. B. anti is not favorable for alkenes.
B. eclipsed and carbon-l interferes with carbon-4. C. steric hindrance does not affect alkenes.
C. staggered and the methyl substituent on carbon-2 D. rotation about a n-bond requires energy in excess of
interferes with carbon-4. room temperature.
D . staggered and carbon-l interferes with carbon-4.

10. Which of the following structures represents the 14. Which of the following compounds contains a single
molecule at the 330" point on the graph in Figure 1? bond about which complete rotation is not possible?
A. B. A. Dipropyl ether
9Hr B. 2-butanone
C. 2,3-butanediol
D. Methylcyclopentane
H
CHzOH HOH2C

CH2OH

H
CHc H:C

I l. Which of the following statements BEST explains why


the MOST stable conformation of 2-amino-1-ethanol is
gauche?
A. Hydrogen bonds are strongest when the two
substituents have gauche orientation.
B. Hydrogen bonds are weakest when the two
substituents have gauche orientation.
C. H is bulkier than NH2, due to the inductive effect.
D. H is bulkier than NH2, due to resonance.

[ 2. The strongest hydrogen bond occurs between which of


the following?
A. A lone pair on nitrogen bonded to an H on nitrogen
B. A lone pair on oxygen bonded to an H on nitrogen
C. A lone pair on oxygen bonded to an H on oxygen
D. A lone pair on nitrogen bonded to an H on oxygen

-opyright @ by The Berkeley Review@ 149 GO ON TO THE NEXT PAGE


Passage lll (Questions 15 - 22) 1 6. Addition of D2 gas and palladium metal instead of H2
gas and palladium metal to deuterobenzene would have
A researcher wishes to determine the relative stability of shown what ratio of equatorial hydrogens to axial
axial orientation versus equatorial orientation for deuterium hydrogens in its most stable chair conformation?
and hydrogen on cyclohexane. To do so, she treats benzene
(COHO) with D2S04/D2O at 100"C for rhirry minutes to A.6:5
synthesize monodeuterobenzene (COHSD), which is then 8.5:2
treated with H2 gas and palladium metal under 90 psi of C.3:2
pressure to yield monodeuterocyclohexane (C6H11D). For D. 2:3
monodeuterocyclohexane (C6H11D), there are two possible
chair conformations, one with the deuterium having axial
orientation and the other with the deuterium having equatorial
orientation. At room temperature, the interconversion
between the two chair conformations is too rapid to study and 17. The addition of H2 gas and platinum metal ro
all eleven hydrogens appear as a singlet 1.38 ppm in the chlorobenzene (C6H5CI) leads to a product whose most
IHUUR. At low temperatures, the interconversion through stable conformation is:
ring-flip from one chair-conformation to the other is slowed A. boat with chlorine anti.
greatly, so that axial and equatorial hydr-ogens generate
different signals in the IHNMR. As a resulr, the ring-flip
B. boat with chlorine gauche.

process can be monitored using IHNMR spectroscopy. C. chair with chlorine axial.

A IHNMR was recordecl at -89'C in deuterochloroform


D. chair with chlorine equatorial.

solvent, A hydrogen with axiai orientation shows a lUNIrrtR


shift of 6 = l.5l ppm, while a hydrogen with equatorial
orienrarion shows a IHNMR shift of 5 = i.25 ppm.
Integration shows that the relative area of lHNIr,lR signals is 18. The D-C-H bond angle about the deuterated carbon is
1.12: l in favor of the 6 = 1.25 ppm signal. Because a closest to which of the following values?
larger amount of equatorial hydrogen is obser-ved than axial
hydrogen, the deuterium has axial orientation in the more A. 90'
favorable chair confbrmation. B. 109.5'
The researcher proposes that a diff'erence in bond length c. 120'
between the C-H bond and the C-D bond, rather than a D. 180"
difference in atomic size between hydrogen and deuterium,
accounts for the equatorial preference of hydrogen over
deuterium. The difference in bond length is attributecl to the
gleater relative mass of deuterium compared to carbon versus
the lesser relative mass of hydrogen compared to carbon. 19. The most stable form of cis- 1,3,5-trimethylcyclohexane
has the chair conformation with:
Because the center of mass remains constant when a bond is
stretched, the greater difference in mass between hydrogen and
carbon than deuterium and carbon makes the carbon-hydrogen
bond stretch more asymmetrically than a carbon-deuterium
bond. A carbon-hydrogen bond stretches more than a carbon-
deuterium bond, and thus occupies a greater amount of space. Hsc ctl 3
Despite the differences in bond length, the bond angles in
cyclohexane remain similar, between 107.5' and I 1 l'. A. three methyl groups in the equatorial position and
no methyl groups in the axial position.
15, The researcher reached the ultimate conclusion that a B. two methyl groups in the equatorial position and
bond between carbon and deuterium is shorter than a one methyl group in the axial position.
bond between carbon and hydrogen, based on the tact C . one methyl group in the equatorial position and
that: two methyl groups in the axial position.
A. the deuterium favors the equatorial orientation. D . no methyl groups in the equatorial position and
B. the deuterium f'avols the axial orientation. three methyl groups in the axial position.
C. the interconversion between the two possible chair
conformations of the deuterocyclohexane molecule
through ring flip is rapid a[ room temperature.
D. the lgNVtR shift at 6 = 1.25 ppm is fbrrher uplield
than the IHNVIR shil'r ar 6 = l.5l ppm.

Copyright O by The Berkeley Review@ GO ON TO THE NEXT PAGE


2 0. How many units of unsaturation are there in C6H5D? Passage lV (Questions 23 - 29)
A. 3 In most research laboratories, fourier transform infrared
B. 4 (FTIR) spectrophotographers are used to obtain infrared
c. 5 spectra. The FTIR spectrophotographer works by passing an
D. 6 electromagnetic pulse of multiple frequencies through a
sample and then collecting and analyzing outgoing radiation.
The difference between the output signal and a reference
signal is digitized by computer and broken down into a set of
component sine waves (this is the Fourier transform process).
The signals are processed and recorded to yield the same
11. The reason that the diaxial orientation for cis-3- spectra as those obtained using outdated variable frequency IR
hydroxycyclohexanol (a cis-1,3-diol) is preferred over spectrophotorneters.
the diequatorial orientation is that the hydroxyl groups:
One advantage of the FTIR machine is that the wave'
A. are smaller than hydrogens, so they exhibit no number for each signal is given precisely. A disadvantage is
pref'erence for the less hindered equatorial that it is not possible to focus on one peak by using a
orientation. monochromatic light pulse. Focusing on one peak with a
B. are larger than hydrogens, so they exhibit a monochromatic beam can be done in rate studies, although
preference for the less hindered axial orientation. the IR has a rapid shutter speed, faster than any reaction
C. are larger than hydrogens, so they exhibit a (including diffusion controlled reactions).
pref'erence for the more hindered equatorial The IR information is most useful if certain peaks are
orientation. understood. For instance, an O-H bond in a compound can be
D. can form an intramolecular hydrogen bond from a recognized by the broad peak it displays around 3300 cm-1,
1,3-diaxial orientation, while they cannot form an although the exact value varies with the degree of hydrogen-
intramolecular hydrogen bond from the 1,3- bonding. A carbonyl bond is found around of 1700 cm-1. If
diequatorial orientation. a carbon has spj-hybridization, the bonds it forms to
hydrogen are found just below 3000 cm-l. All of this
information combines into a nice packet of data used to
deduce the structure of a compound. Figure 1 shows the IR
spectrum for 2-heptanone:
I L Which structure represents the MOST stable form of
cis- 1,4-ethylmethylcyclohexane?

A. cH: B.
H,CN
' >'--L CH:CH-1

CH2CHj

4000 3600 3200 2800 2400 2000 1800 1600 1400 1200 1000
C D.
HCh CH2CH3 Figure I Infrared Spectra for 2-heptanone

The information extrapolated from the IR spectra can be


CH2CH3 CH: coupled with NMR (nuclear magnetic resonance) data to form
a powerful combination. For instance, aromatic hydrogens
are found in the 7 to 8 ppm range in an 1HNMR spectrum.

23, An absorbance between 1700 cm-l and 1740 cm-1


would NOT be present in:
A. ethyl propanoate.
B. butanal.
C. 2-pentanone.
D. diethyl ether.

The Berkeley Reviewo l5l GO ON TO THE NEXT PAGE


24. The IR spectrum for a straight chain monosaccharide 2 8. Which of the following isomers of C4H6O would NOT
has all of the following absorbance values EXCEPT: have an IR signal at 1715 cm-1?
A. 3300 cm-l. A. 2-methylpropanal
eill@
B. 2980 cm-l. B. Butanal
C. 2300 cm-1. C. Butanone ']rMt,"[
m!&m
D. 1715 cm-1. D. Tetrahydrofuran :6r1ttlriu

{@fu
ifi'mrrrrlrm

qnmliftr

tfupr
2 5. Which of the following isomers of C3H6O2 exhibits a
WMffi
ilil&,m
broad IR signal around 2850 cm-l? 29. Hydrolysis of an ester could be supported by which of
Ml t
A.oB. o
these IR spectroscopic data?
,6&Lm

"oA.H2cH3 H3co
A cHs
A.
B.
C.
D.
The appearance of a signal around 1700 cm-l
The disappearance of a signal around 1700 cm-l
The appearance of a signal around 3300 cm- 1

The disappearance of a signal around 3300 cm-l


u

c.o D'
oo,4..
H,cnrcoAn

2 6. Which of the following pairs of compounds could be


distinguished by their splitting patterns in the proton
NMR region between 7 and 8 ppm?

A. Methylpropanoate from ethylethanoate


B. 3-methyl-2-hexanone from 2-methyl-3-hexanone
C. l,4-methylphenol from l,4-ethylphenol
D. 1,4-methylphenol from 1,3-methylphenol

27 . To distinguish a tertiary alcohol from a primary alcohol


(the tertiary alcohol exhibits more steric hindrance to
hydrogen-bonding than the primary alcohol does), it
would be best to focus on which of the following IR
features?

A. The width of the peaks near 3300 cm-l


B. The length of the peaks near 3300 cm-l
C. The width of the peaks near 1700 qm-l
D. The length of the peaks near 1700 cm-1

Copyright @ by The Berkeley Review@ 152 GO ON TO THE NEXT PAGE


Passage V (Question 30 - 36) 3 1. Which of the following I UNUR shifts would
observed for Compound A?
An unknown alkyne with a molecular mass of 122.2
-: mole is treated with H2lPd and BaSO4 to convert it into a A. 12.0 ppm
..s-alkene. The cis-alkene is isolated in high purity. The cis B. 9.7 ppm
..rene is then treated with high-pressure ozone (O3) gas and C. 7.5 ppm
nc metal to convert both of the alkene sp2-hybridized D. 5.5 ppm
:.rbons into carbonyl carbons. Because the cis-alkene was
, -ned by hydlogenation of an alkyne, it is disubstituted with

=.:h alkene carbon holding one hydrogen. This means that


-: products of ozonolysis are both aldehydes. Two unique 32. Which of the following IR absorbances would
- rducts are isolated from the product mixture. The two observed for Compound B?
--known products are designated as Compound A (C3H6O)
A. 1125 cm-1
--J Compound B (C6H1gO). An lHNIrrtR spectrum is B . 2200 cm-l
,.ained for Compound B and is shown in Figure I below.
. -: lHNMR is cariecl out using CDCI3 as the solvent.
C. 1620 cm-l
D. 3550 cm-l

3 3. What is the molecular formula for the original alkyne?


A. C9H14
B. C9H16
C. C9H1s
D. C9H26

34. Which of the fbllowing IR absorbance values is


Figure I 1HNMR spectrum of Compound B indicative of an alkene?

For Compound A, spectral data were obtained from an IR


A. 3550 cm-l

:::r,rum using pure Compound A in liquid form between B. 2200 cm-1


" . -:lates. Table 1 lists the key IR absorbances collected for C. 1"725 cm-l
-rfound A D. 1620 cm-1

Shilt(cm-11 Inrensir.y
2962 strong
2912 medium 35. Which of the following structures corresponds
2106 mediurn
Compound A?
1126 strong
1212 strong A.oB. o
all other peaks are irrelevant

Table 1 IR absorbances of Compound A

The structures of Compounds A and B can be deduced


t,"A.r, HicH2c
A,,
qreat accuracy from the spectral data in the passage.
-r engaging in structure elucidation, some information is
: useful than others. As a general rule, NMR data are C.

O
-:d last, as they have the most information.

-\ compound with one degree of unsaturation and two CHr


r\ygens CANNOT be:

A. a cyclic ketone.
B. a cyclic ether.
C. acarboxylicacid.
D. an ester.
: . right @ by The Berkeley Review@ 153 GO ON TO THE NEXT PAGE
3 6. Which of the lollowing structures corresponds to Passage Vl (Questions 37 - 43)

'{x:,
Compound B?
An unlabeled bottle containing an unknown compound is
A. found in a lab storage locker. The compound is an odorless
liquid that does not evaporate rapidly when the bottle is left
uncapped. A lab technician labeis the bottle Compound T.
Compound T exhibits three signals in its proton magnetic
resonance spectrum. The three signals are listed in Table 1.
C .o shift Integration Shape J-Value

oA 6 = 2.5 ppm (broad)

6 = 3.1 ppm

6 = 4.3 ppm
1H
1H
2H
triplet
singlet
doublet
3Hz
NA
3Hz

Table 1 1HNUR signals of Compound T

Shorthand for describing nuclear magnetic resonance


spectra describes the chemical shift value (6), the number of
hydrogen atoms (relative area under each signal), and the
coupling along with the respective coupling constant, J.
Signal shapes can sometimes be described by single letters.
such as s = singlet, d = doublet, m = multiplet, q = quarter-
and t = triplet.
For the unknown compound, Compound T, three bands
at 6 50 (0, 73.8 (d), and 83.0 (s) appear in the carbon-li
magnetic resonance spectrum. The splitting in the l3CNMR
is due to the hydrogens directly bonded to the carbon. The
important absorbances in the infrared spectrum of Compounc
T are found at 3350 cm-l (broad),2988 cm-l, 2116 cm-1 , anC
1033 cm-1. The spectral data can be applied to determine the
symmetry of a compound as well as the functional groups ot
,#!
the compound. The IR peak at2116 cm-l is indicative of a "

triple bond, indicating the presence of either an alkyne or a


nitrile in Compound T. There is no spectral evidence tc
suggest that a nitrogen atom is present in the compound, sl n

the most logical assumption is that there is a carbon-carbori (


triple bond present in the structure.
I
From the spectral data, the lab technician concludes tha:
the compound contains no carbonyl functionality. However"
the compound contains a functionality that is involved ir
hydrogen-bonding, explaining the relatively slow evaporatior-
of Compound T. Using chemical tests and a polarimeter, the
lab technician determines that there is no mirror symmetry i:
the molecule. This implies that the number of signals in the
l3CNtvtR spectrum is also the number of carbons ii
Compound T.

37. The shift at 1033 cm-l in the IR is caused by the C-C


bond of:
A. an ether.
B. an ester.
C. a carboxylic acid.
D. an alcohol,

Copyright @ by The Berkeley Review@ GO ON TO THE NEXT PAGE


38. The absorbance at2116 cm-l in the IR implies that the 43. the l3CNIr,tR peak at 50 ppm can
be attributed to the:
compound is an:
A. CofaC=O.
A. alkane.
B. CofaC-O.
B. alkene.
C. H next to C=O.
C. alkyne. D. H next to C-O.
D. alcohol.

39. The l3CNIrrtR doublet ati3.B ppm can be attributed to


which type of carbon?
A. The terminal C of a terminal alkyne
B. The internal C of a terminal alkyne
C. Cofanaldehyde
D. Cofanketone

tt). The number of carbons in an asymmetric molecule that


shows seven peaks in the 13CNMR is:
A. 3.
B. 4.
c. 7.
D. 14.

4 1. Hydrogens that are coupled to one another have the


same:
A. shift value.
B. peak integration.
C. area under theirpeaks.
D. same J-value.

Compound T is which of the following compounds?


A. B.
H- C: C- Ct-te H- C= C- Chtz
'\ ,/.H
\
OH c
il
o
c. D.
HO- C= o\ H-C=C-cH^
"- - cH, \o,

-:'pyright @ by The Berkeley Review@ GO ON TO THE NEXT PAGE


Passage Vll (Questions 44 - 50) 4 6. According to IUPAC nomenclature convention, what is Fa
the name of Compound III?
From the leaves of a tree that grows wild along the coast
A. Methyl ethanoate
ofEcuador, three esters were extracted using ether. The three
esters were separated by fractional distillation and purified by
B. Methyl propanoate
column chromatography. Once isolated and purified, a C. Ethyl methanoate
IUNVIR spectrum was obtained for each compound. The D. Propyl methanoate
three spectra are shown in Figure 1 below. All three spectra
were obtained using the same NMR spectrometer in CDCI3
solvent from the same bottle. A dead give-away for an ester
is the shift for the hydrogens attached to the carbon bound to
the oxygen of the alkoxy group. These hydrogens are found 4 7. Which of the following IR absorbances would you NOT
between 3.5 ppm and 4.0 ppm. Compound I has a molecular expect for an ester? I.
formula of CgHgO2, Compound II has a molecular formula A. 3500 cm-1
of CgH1602, and Compound III has a molecular formula of B. 2980 cm-1
CaHs02.
C. 1685 cm-1
Compound I D . 1300 cm-1

lrr
i'" 4 8. Which of the following structures represents Compound
6 pom 4 ppm 2 ppm
tr?
Compound II A. B.
O o
lm

ppm 4 DDm
ll2H
illi il oa cH2cH3 ocH 2cH 3
ill I

6 2 PPm

Compound III
C. O D. :1
llil{:

I
il,'"
I
lr"
ilr
I
d:,. Oclr*" ,T!*

'r t]
rll
llr

3 pDm 2 ppm 1 ppm rl-

gj,

Figure I 1HNMR spectra of Compounds I, II, and III *.!n-:'


49. For the spectrum of Compound III, the three 1HNMR I i,]"
4 4. Which of the following structures is Compound I?

d
signals are which of the following, respectively? .i[[,u:-
.o B. o A. Triplet, doublet, triplet

C.
ctl 3

D.
v
..A,,/ocH. B. Triplet, quartet, triplet
C. Singlet, doublet, triplet
D. Singlet, quartet, triplet
O

o*. o"oYo' 5 0. Which of the following would NOT work as an NNfR


solvent for an ester?
A. CDC13 (chloroform)
4 5. It important that each sample use the same solvent in B. C6D6 (benzene)
order to: C. D3CD2COD (ethanol)
A . ensure the same reactivity in the magnetic field. D. D2CO (formaldehyde)
B. increase the pH of the compounds in solution.
C. view any common impurities between samples.
D . allow fbr extraction of protic compounds.
Copyright @ by The Berkeley Review@ r56 GO ON TO THE NEXT PAGE
Passage Vlll (Question 5t - 57) 53. Pentanal is BEST distinguished from 2-pentanone by a
peak:
Nuclear magnetic resonance spectl'oscopy is a powerful
:ool for distinguishing isomers. By cornparing shift values A. above 1700 cm-l in the IR.
measured in ppm), coupling consrants (J-values), and B. berween 2.0 to 2.3 ppm in rhe IHNMR.
rntegration ofpeaks (the area under the curve), it is possible C. near 3.5 ppm in the IHNMR.
io deduce a structure with great accuracy. The coupling D. near 9.7 ppm in the IHNMR.
lonstants help us deduce which hydlogens are neighbors of
,lne another. For instance, a carbon con taini ng two
:quivalent hydrogens influences the signal for hydrogens on
::re neighboring carbon to foln.r a triplet. The integration is 5 4. To confirm the presence of an alcohol, it would be best
,re area under the curve that is directly proportional to the to add:
-.umber of hydrogens within that signal. Table 1 is a general
A. D2O with NaOD and observe whether the broad
,sting of proton NMR shift values.
peak grows.

Hydrogen Shift (ppm) Hydrogen Shift (ppm)


B. D2O with NaOD and observe whether the broad
peak disappears.
-RCH3 0.8 RCH2R, 0.9
C . H2O with HCI and observe whether the broad peak
-COCH3 2.1 -COCH2R 2.3 grows.
-OCH3 3.5 -OCH2R D. H2O with HCI and observe whether the broad peak
-CH=CH2 5.5 -CH=CHr 5.3 disappears.
\H 7-8 -CH2CH=CH2 2.3
CH=O 9.7 -CO2H to-12

Table 1 IHXVR shift values fbr common hyclrogen types


5 5. A methyl ketone always has which of the following
proton NMR absorbances?
When IHNMR informarion is coupled with IR shifr A. A 3H singlet between 2.0 to 2.3 ppm
.lles, it is possible to narrow the structure down quickly to B. A 3H triplet between 2.0 to 2.3 ppm
: possibility. Important IR absorbances are 3500 crl-1 iO-
': . 1700 cm-l 1C=O;, anct 1600 cm-l 1C=C;. There are
C. A 3H singlet between 3.5 to 3.8 ppm
.:::r absorbances, but from these three values, it is possible D. A 3H triplet between 3.5 to 3.8 ppm
,, estimate many other values. As the strengLh of a bond
r:reases, it exhibits a higher absorbance value in the IR.
l:; heavier the atoms in the bond, the higher the absorbance
: IR. This intbrmation makes it possible to predict shift
-:e
5 6. What is the common name for the compound with the
fbrmula CaHgO, an IR absorbance at Il21 cm-l , and
-- res lor other bonds. For instance, a C-C bond must be
:is than 1600 cm-I in absorbance in the IR, three NMR peaks at 9.7 ppm (lH),2.2 ppm (lH), and
because a
- i.,-:on-carbon single bond is weaker than a carbon-carbon 1.0 ppm (6H)?
i " -cle bond. However, the irnportance o1 NMR A. Tetrahydrofuran
:E.Iroscopy is greater than that of infrared spectroscopy. B. 2-methylpropanal
C. Butanal
i l. ]-Methyl-3-pentanoneconrains:
D. Butanone
-\. 3 non-equivalenthyclrogens in a9 : 2 : I ratio.
B. 4 non-equivalenthydrogens in a4 : 3 : 3 : 2 ratio.
C, 4 non-equivalenthydrogens in a6 : 3 : 2 : 1 ratio"
D. 5 non-equivtrlent hydrogens in zi 3 : 3 : 3 : 2 : I 5 7. A compound with one oxygen in its formula, one
degree of unsaturation, and no IR absorbance between
ratio"
1600 cm-l and 1750 cm-I must be:
A. a ketone.
B. an aldehyde.
il" What in the proton NMR is a dead give-away for an C. an alkene.
isolated isopropyl group?
D. cyclic.
{. A 6H doublet and a lH septet
B. A 6H singlet and a lH sextet
C. A 6H septet and a lH doublet
D. A 6H sextet and a lH singler

- ::', nght O by The Berkeley Revie',v@ 157 GO ON TO THE NEXT PAGE


Passage lX (Question 58 - 64) 60. Which of the following compounds has the same P@
number of signals in their carbon-13 NMR as there are
A 1HNMR was run for the cis and trans isomers of the carbons in the compound?
molecule shown in Figure 1 below. The spectra obtained for
A. An ortho substituted benzene with two identical
the two geometrical isomers are also shown in Figure 1. The
substituents
cis compound can be distinguished from the trans compound
using the coupling constants for the vinylic hydrogens. The
B. A meta substituted benzene with two identical
substituents
trans species has a larger coupling constant due to better
transfer of magnetization in the trans orientation. The shift C .A meta substituted benzene with two different
values on the spectra are listed in parts per million (ppm) of substituents
the frequency of the spectrometer. The hydrogen on the D . A para substituted benzene with two different
carbon adjacent to benzene can be tbund downfield from the substituents
other vinylic hydrogen due to the presence of oxygen.
"Downfield" implies higher shiti values.

n,.-G cH- cH- ocHj 61. Where do the vinylic hydrogens appear in IHNMR?
A. Above 7.0 ppm
Spectrum I 7.25 ppn (4 H) B. Between 4.5 and 7.0 ppm
]H 5.37 pptn (l H)
4.88 ppm (1 H) C . Between 2.0 and 4.5 ppm
3.76
2.25
ppm (3 H)
ppm (3 H)
D. At less than 2.0 ppm

rllr
ilfl ]l]i llLl
PPmz65432lo 6 2. Which of the following absorbances wouid be found rn
the IR spectrum for the compound in Figure I ?
Spectrum II 3 3H
7.25 ppm (4 H)
H
5.37 ppn (l H) A. 3500 cm-l
4.88 ppm (l H)
4H
3.76 ppm (3 H) B. 2220 cm-l
2.25 ppnt (3 H)
C. 1120 cm'l
rllr IHIH D. 1640 cm-l
iltiltil ilil
PPmz6s432lo
Figure 1 1HNMR spectra of the cis and trans isotners
63. Had the compound had meta substitution rather than
para substitution, the ratio of the benzene hydrogens
The spectra as drawn are different enough to distinguish
would be which of the following?
the trans and cis compounds fiom one another. The shilt
values does not help to distinguish the cis lrom the trans A. 1 :1
compound, the coupling does. B. 1 :3
C" 1:2:l
5 8. Spectrum I is associated with the: D. 1:1:1:1 ill:'tn:1

A. trans compound, because the J value is smaller.


FitE'ru
B. cis compound, because the J value is smaller.
C. trans compound, because the J value is larger.
D. cis compound, because the J value is larger.
64. Had the compound had an ethyl group on benzene rather
*"'"
iu. -

than a methvi group, which of the following would be I Jr lrl

observed in the proton NMR? intmr i:


)illi;IlL,I
5 9. The methyl group on benzene appears in the IHNMR A . Doubiet (2H) and triplet (3H) ll,il T1:.
.i

at: B . Doublet (3H) and triplet (2H) [:ifL l[ 1.i"

A 7.25 ppm. C . Triplet i2H) and quartet (3H) L


" l:iiL'tt:ti

B. -5.37 ppm. D . Triplet (3H) and quartet (2H) iud t'

t,i l,nr;'
C. 3.76 ppni. ,il]fiI'rl',':r:
D. 2.25 ppm. pr|trnTI- '

Copyright O by The Berkeley Review@ r58 GO ON TO THE NEXT PAGE


Passage X (Question 65 - 71)
6 5. The shift at 1738 cm-l in rhe IR can be attributed to the
C=O of:
An unknown compound, labeled Compound B, has only
:u'o singlets in its proton magnetic resonance spectrum.
One
A. an aldehyde.
shows a shift of 6 1.42 ppm and the other shows a shift
of 6 B. a carboxylic acid.
-.96. ppm, with relative intensities (from the integration of
1HNMR
C. an ester.
:tre spectrum) of 3 : l. The decoupled carbon-l3 D. a ketone.
:ruclear magnetic resonance spectrum for Compound
B shows
:lur signals (with one of greater intensity ihan the other
'lree) with shifts at 6 22.3 ppm,
2g.l ppm,79.9 ppm, and
- r0.2 ppm. The signal at ljo.2 ppm is atributed
to a
:rrbonyl carbon. For carbons of equal hybridization, the
:rgher the shift value in the carbon-l3 NMR, the greater the 66. The molecular ratio of hydrogens in the structure is
:-ectronegativity of the atoms bonded directly to that carbon. which of the following?
:igure I shows both the 1HNMR ana l3CNUR spectra of A. 3:1
l mpound B.
B. 6:2
C. 9:3
D. 12: 4

6 7. The l3CNUR peak at 79.9 ppmcan be attributed to:


A. an H bonded to C adjacent to C=O.
B. an H bonded to C adjacent to C_O.
C. the C in rhe C=O bond.
D. the C in the C-O bond.

6 8. Which of the following compounds would NOT show a


l3cNnaR peak above 100 pfm?

A. Methyl benzoate
B. 3-methyl-2-hexanone
C. 3-methyl-2-penranol
D. 3-methylpentanal

Figure t lHNltn and I3CI.Ur,IR spectra of Compound B


69. The lgNVtR peak at 1.96 ppm can be artribured to a
The carbon-13 NMR information leads to the conclusion hydrogen bonded to a carbon:
fibi.: there are four nonequivalent carbons tn the compound.
Tre important bands in the infrared spectrum of Compound B A. adjacent to a C=O bond.
iurir,: found at 1738 cm-l
, 1256 cm-l, and ll73 cml1. The B. adjacent to a C-O bond.
innr:d at 1738 cm-1 is attributed to the stretching of a
carbonyl C. ofaC-Obond.
*n'r:d. Elemental analysis of Compound B shows that it D . of a C=O bond.
rr:alns two oxygen atoms. The spectral data, in
r.:unction with mass percent values from elemental
umn-vsis, can be combined to determine the structure of
inpound B. Overlapping spectral data, such as an IR
,niur*orbance at 1738 cm-l and a l3CNUR signal at 170.2
can be used to verify aspects of the structure.

::vright @ by The Berkeley Review@ GO ON TO THE NEXT PAGE


70. in IHNMR, a singlet is explained as the evidence of Passage Xl (Questions 72 - 78)
hydrogens of the signal being coupled to:
Carbon-13 NMR can be used to determine the number ol
A. equivalent hydrogens on all adjacent carbons. r,rnique carbons in a compound. Carbon-13 NMR is similar
B. non-equivalent hydrogens on all adjacent carbons. to hydrogen-1 NMR in that it generates separate peaks for
C. only one hydrogen on an adjacent carbon. each unique isotope, in this c"s. 13C. Many I3CNMR
D. no hydrogens, because there are no hydrogens on spectra are recorded without coupling (known as decoupled
any of the adjacent carbons. spectra), so all peaks appear as singlets. Table I lists the
approximate shift value for selected types of carbons. It is a
brief guide to determining the types of carbon represented bi
each I3CNMR peak. The range for the ppm of each signal is
approxirnate and on occasion, a peak may fall outside of the
range.
7 1. Compound B is which of the fbllowing?
Carbon ppm Carbon ppm
A.oR' .C-OH 50-70
R2C=O 205 - 220
Hrc RHC=O 185 - 200 -C-CI 40-45
CH: R2C=CH2 t20 - 140 -C-NH2 35-45
HIC CH: H:C CH: R2C=CH2 r 10 - 120 -C-Br 25-35
RC=CH 12 85 F{3C-C=O 20-35

"''K)rn .XY
C. D. RC=CH 65 -10 R2CH2 l0-30
HrC- - O- -CH-r Table 1 l3cxltR shilt values

HsC CH: O H:C CHr O should be noted that the peaks fbr carbons with r:
It
hydrogen directiy attached are less intense than other carbo:
peaks. Carbonyl carbons and quaternary carbons therefb:.
generate shorter peaks than those for other carbons in t1::
spectrum. For peaks that represent rnore than one carbon, th:
intensity increases, but not in a way that is easily integrate;
Integration is generally not carried out on 13CNMR spectra.
A researcher used I3CNMR to distinguish two structur--
isomcrs formed when toluene (methylbenzene) was acylate:
using acetyl chloride by comparing the I3CNMR spectra :t
the two compounds. The molecular tbrmula fbr both isome:i
is CgH160. Spectra fbr both compounds were recorded usi:'s
the same NMR instrument for the same period of time in lt:l':
same concentration. The only clifTerence between the t''n:
samples invoived the isomers themselves. The spectral d:-l
tbr the two isomers are listed below:

I:
Ison-ier 2-1,34, 110, ll1, 123, 130,206

lsonierll: 31, 35, 108, 115, 119,123,126,131,?t-';

The dift'erence in the number of 13CNUR signals lisl:r


for each isomer is cause<l by the lack of symmetry betse:r
the two isomers. Isomer I has nine carbons but shows ol"'
sevcn peaks in the l3CNMR, so it must have some type :$i
symrnetry that equates carbons within the structure. B'-;tr
isomers ale benzene rings with a n-rethyl substituent anC ar
acetyl group attached to carbons on the benzene ring.

Copyright @ by The Berkeley Review@ GO ON TO THE NEXT PAGE


72. Which of the following reactions CANNOT be 76. How many signals would be seen in the carbon-l3
monitored by a change in the shift values in the NMR of para-methoxy benzaldehyde?
l3CNvtR spectra?
A. Four
A . Oxidation of a secondary alcohol to a ketone B. Five
B . Nitration of ethyl benzene to para-nitroethylbenzene
C. Six
C . Deprotonation of a carboxylic acid
D. Eight
D . Reduction of an alkene to an alkane with hydrogen
gas and nickel catalyst

77 . What is true about the units of unsaturation for the two


isomers?
- 3. Which of the following structures represents the
structure of Compound I?
A . Isomer I, because it is more symmetric than Isomer
II,has more units of unsaturation than Isomer II.
A. B. ov.+t B. Isomer I, because it has less unique carbons than

d
Isomer II, has fewer units of unsaturation than
Isomer II.
C. Isomer I and Isomer II each have three units of
unsaturation.
Gl3 D. Isomer I and Isomer II each have five units of
C. D. unsaturation.

I
il cH2cH
78. For the compound 2,2-djmethylbutane, which carbon
shows the peak of lowest intensity?
A. Carbon
B. Carbon 2
1

I C. Carbon 3
r D. Carbon 4
s Which of the following functionalities are NOT present
t! in either of the two isomers?
A. Aldehyde
D

f,
B. Alkene
C. Ketone
D. Methyl

How can it be explained that the least intense peak in


the spectrum for Compound II is found at209 ppm?
A . Carbons with sp2-hybridization show low intensity
l3CNMR peaks.
B. Carbons with spj-hybridization show low intensity
l3cNuR peaks.
C. Chiral carbons show low intensity l3CNVtR
peaks.
D. Carbons with no hydrogen atoms attached show
low intensity l3CNUR peaks.

)pyright @ by The Berkeley Review@ l6l GO ON TO THE NEXT PAGE


Passage Xll (Questions 79 - 84) 7 9. Which of the following is the common name for the
compound represented by Spectrum I?
Coupling in proton NMR is used to determine the
relative positioning of hydrogens within a compound. A. Ethyl acetate
Hydrogens are considered to be coupled when they are on B. Acetone
neighboring carbons. Their respective magnetic fields C. Methyl acerare
influence the signals for one another. The effect is mutual, D. Isopropyl formate
so the coupling interaction is equal for all coupled hydrogens.
This can be seen in terms of identical J-values. A J-value is
also known as the coupling constant and is the distance 80. A triplet in the proton NMR is associated with t-be
between adjacent peaks within a proton NMR signal. hydrogens:

the lftNMR spectrum for two isomers of C4H3O2 are A. of a CH2 group.
collected under identical conditions using the same NMR B. on a carbon next to a CH2 group.
machine. Figure 1 shows the spectrum for Isomer I, while C. of a CH3 group.
Figure 2 shows the spectrum for Isomer II. D. of a carbon next to a CH3 group.

Spectrum I 13 H
8 I . What type of compound is represented by Spectrum H-t
A. An ethyl ester
B. A methyl ester
C. An ethyl ketone
D. A methyl ketone

82. A lgNirAR signal in the range between 6 and 8


indicates the presence of which of the following?
A. a hydroxyl proton.
4ppm 3ppm 2ppm lppm 0 B. an aldehyde hydrogen.
C. abenzenehydrogen.
Figure 1 1HNMR spectrum of Isomer I D. a carboxylic acid proton.

83. Which of the following features in the proton


spectrum CANNOT be used to distinguish an ester [h
of a carboxylic acid when the two compounds have ,ct- .

same molecular formula? il.


A. A sharp peak between 2.0 and 2.5 ppm 0-,
B. A sharp peak between 3.5 and 4,0 ppm D-.
C. A sharp peak between 10.0 and 12.0 ppm
D. The observation that no peak integrates to a re
ratio of 1.

3pp- 2ppm lppm 0


84. How many unique proton NMR signals are expected
frdmmm
4-heptanone?
Figure 2 IHNMR spectrum of Isomer II A. Two A-m
B. Three f-p
Both of the isomers are esters. The exact connectivity of ,lU- s
the esters can be deduced from the shift values of the singlet
C. Four
and quartet. IHNVIR signals around 3.5 to 4.0 ppm are due D. Five
to alkyl groups bonded to an oxygen while alpha hydrogens
typically show shift values between 2.0 and 2,5 ppm. The
integration information verifies the substitution of each
carbon in the ester.

Copyright @ by The Berkeley Review@ 162 GO ON TO THE NEXT PA


Fassage Xlll (Question 85 - 90) 8 7. What can be concluded from the data obtained using
UV-visible spectroscopy?
A chemist sets out to determine the structural identity for
fih;ee structural isomers, using data from NMR spectroscopy,
A. Compound I is a conjugated diene.
Ltu.L:iviolet-visible spectroscopy, and mass spectroscopy to B. Compounds II and III are carbonyl compounds.
:tify each cornpound. The following spectral data are C. Compound I is a ketone.
:l:rved for three separate compounds, all with the forrnula D. There is no n-bond in Compounds I and II.
: Ht oo.

:-"pound I:
r-:CNMR' 211 ppm (1), 31 ppm (1), 22 ppm (i), 17
ppm (1), 1l ppm (1)
:HNMR: 2.32 ppm (triplet, 2H),2.08 ppm (singlet,
3H), 1 .12 ppm (multiplet, 2H), 0.96 ppm 8 8. Which of the fbllowing is NOT a valid conclusion from
(triplet,3H) the spectral data for Compound III?
--\'-Visible: 268 nm and 189 nm A. The absence of a broad peak in the 1HNMR means
,".iound II: the compound cannot be an alcohol.
rCNMR: 12 ppm (1),24 ppm (2), 13 ppm (2) B. The absence of a peak above 175 ppm in the
HNMR: 3.71ppm (multiplet, iH), 1.88 ppm (broad,
l3CNltR means the compound cannot be a
1H), 1.33ppm (multiplet,4H), 1.14 ppm carbonyl.
(triplet,4H) C. The absence of an absorbance above 180 nm
,f--Visible: No intense peaks above 180 nm confirms that the structure must contain a ring for
:, its unit of unsaturation.
:cLutd III:
'CNMR: 68 ppm (2),21ppm (2), l0 ppm (1) D. The large number of peaks in the 13CNMR means
the compound is an ether with little symmetry.
FI-vMR: 3.58 ppm (triplet, 4H), 1.28 ppm
(multiplet, 4H), 0.92 ppm (multipl et, 2H)
-,-"'-Visible:
No intense peaks above 180 nm

1.ll NMR spectra are obtained using deuterated


.::form as solvent. The mass spectrometer show an
l-'e peak at 86 amu for all three isomers, contirming their 8 9. What is the IUPAC name for Compound I?
':ular fbrmula. Elemental analysis shows that carbon A. Pentanal
:'" drogen are not the only atoms present.
B. 2-Pentanone
ilompound II is what type of compound? C. 3-Pentanone

A, An alcohol D. 3-Methylbutanone
B. An ether
C . A ketone
D. An alkene

90. The integration of a signal in a proton NMR is useful


for determining the:

^arbon-13 NMR is A. local magnetic field experienced by a hydrogen"


useful for determining all of the
'.llowing EXCEPT the: B. neighboring hydrogen atoms.
C. presence of an atom other than hydrogen or carbon.
A. number of unique carbons in a structure.
D. relative quantities of unique hydrogens in the
B. presence of a carbonyl group. compound.
C. substitution of a benzene ring.
D. geometry about a double bond.

right @ by The Berkeley Review@ 163 GO ON TO THE NEXT PAGE


Passage XIV (Questions 91 - 97) 91. Which of the following is the IUPAC name for rhe
compound represented by Spectrum A?
Proton NMR is a valuable tool used to deduce the
structure of unknown organic compounds. It helps one A. Phenylmethane
distinguish between structural features of two similar B. Toluene
compounds. The three useful components of the spectral data C. Methylbenzene
fbr extracting structural information are the shift value (6, D. Orthoxylene
measured in ppm), the coupling and coupling constants (peak
shape), and the integral (the area under the curve of a signal).
From the shift value, infbrmation about the electroncgativity
of adjacent atoms may be obtained. Coupling is used to
determine the number of hydrogens on any atoms bonded to 92. In Spectrum B, what is the ratio of the areas under
the atom bound to the hydrogens producing the signal. The three peaks?
integral is directly proportional to the number of hydrogens A. 3 :2:3
in the compound, so it can be used to find the ratios of B. 1:1 4
hydrogens in the cornpound. Figure I and Figure 2 show the
l gNVtR spectra for two simple organic structures, C. 2:3 5

Compound A and Compound B, along with the molecular D. 1:2 3

tbrmula of the compounds they represent.

Spectrurl A
CzHrc
9 3. What peaks are expected for 2-bromopropane?
A. A 6H sextet and a 1H singlet
5H B. A lH sextet and a 6H singlet
3H C. A 6H seprer and a lH doublet
D. A 1H septet and a 6H doublet

8 ppm 6 ppm 4 ppm 2 ppm 0 pprn


94 . Which of the following compounds would NOT har;
quartet in its proton NMR spectrum?
Figure I IHNMR spectrum of Compound A
A. o B. o
Spectrum B
C3H602 (H3C)2HC
{ cH2cH3 H3CH2C(H3C)2C
A
I o D.
C. o

j (H3C)2HCH2C
A CHe H3CH2CH2C
A

95. Which of the following compounds would NOT ha"'e


iL) ppm 5 pprn o ppm doublet in its proton NMR spectrum?
A. 2-methyl-1-pentanol
Figure 2 lHNMR spectrum of Compound B B. 3-methyl-1-pentanol
C. 4-methyl-1-pentanol
Structures can often be deduced usins onlv some of the
lUlltR D. A11 isomers of methyl-1-pentanol have double-;
information, such as the coupliig information and
their proton NMR spectrum.
the integration. Often, the coupling infbrmation is the most
important of all the data. The coupling constants can give
intormation about the connectivity of the structure, as well as
irints about the three-dimensional orientation of atoms within
:ne n.iolecuie.

f.-,pvright O by The Berkeley Review@ GO ON TO THE NEXT P.{


'r 6. The iodoform test involves the addition of hydroxide
anion and iodine to a carbonyl compound. If a carbon Questions 98 through 100 are NOT based on a
descriptive passage.
contains three alpha hydrogens, then the iodine will
react with the carbonyl compound to yield a yellow
precipitate. A compound with a positive iodofonn test 9 8. The following two molecules are best described as:
would likely have which of the following signals in its

a
proton NMR?
A. Singlet --
B. Doublet
C. Triplet
A. structural isomers.
D. Septet
B. geomelrical isomers.
C. optical isomers.
D. the same molecule with altered spatial orientation.

The broadness ofthe signal around 10 ppm in Spectrum


B is explained as a signal caused by hydrogens:
A. on a carbon involved in resonance. 9 9. The following two molecules are best described as:

B. coupled to more than eight hydrogens.


C. on a carbon involved in hydrogen bonding.
D. involved in hydrogen bonding.

A. structural isomers.
B. geometrical isomers.
C. optical isomers.
D. the same molecule with altered spatial orientation.

10 0. How many structural isomers of C5H12 are possible?


A. 3
B. 4
c. 5
D. 6

l.B 2.C 3.D 4.C 5.C 6.D


1.8 8.C 9.A 10.A 11.A 12.D
13. D 14. D 15. B 16. C 1',7. D 18. B
19. A 20. B 2r. D 22. D 23. D 24. C
25. A 26. D 21. A 28. D 29. C 30. A
3r. B 32. A 33. A 34. D 35. B 36. C
31. D 38. C 39. A 40. C 41. D 42. A
43. B 44. B 45. C 46. B 41. A 48. B
49. D 50. C 51. C s2. A 53. D 54. B
55. A 56. B 57. D 58. C 59. D 60. C
6i. B 62. D 63. D 64. D 65. C 66. C
61. D 68. C 69. A 10. D 11. D 72. C
13. C 74. A 15. D 16. C 77. D 18, B
19. A 80. B 8i. B 82. C 83. A 84. B
85. A 86. D 87. C 88. D 89. B 90. D
91. c 92. D 93. D 94. C 95. D 96. A
91. D 98. B 99. C 100. A

- ::.t O by The Berkeley Reviewo ENOUGH CHEMISTRY... FOR NOW!


Structure Elucidation Passage Answers

L. Choice B is correct. The most stable form of the cyclohexane ring is the chair conformation. The most s
position on the chair form is referred to as equatorial. Combine these two facts and the result is choice B.

a Choice C is correct. Because the three deuterium atoms are cis with respect to one another, they cannot all
axial nor all be equatorial. The most stable orientation (most stable chair confirmation) has as many deute
atoms with axial orientation as possible. However, because the deuterium atoms are all mutually cis to
another, the structure must have at least one deuterium with equatorial orientation. The best choice (
consequently your choice) is C, which is drawn below.
D D

2 axial deuteriums and 1 axialdeuterium


1 equatorial deuterium 11 2 equatorial deuteri

J. Choice D is correct. As the reaction is written, the value of Kgt-1,4 must be greater than 1, because the prod
more stable than the reactant. The reaction is favorable in the forward direction as written. This elimi
choice A, a value less tiran 1.
The question now focuses on whether the conformational change with Ksq-1,4 is more favorable than
conformational change with K"O-1,2, because the value of Kqq-1,2 is 4.31. In the case of the 1,2-disubstih
compound, there are both diequdtorial versus diaxial interactions as well as gauche versus anti interactions
consider. The diequatorial orientation is better than the diaxial orientation, because with diaxial there
eclipsed interactions with the axial hydrogens. Howevet, the anti orientation of the methyl groups is
than the gauche orientation.
Overall, diequatorial preference over diaxial is a more important factor than a preference for gauche over
so the value of Keq-l,Z is greater than 1. The 1,4-disubstituted compound has no gauche-versus-anti ini
between the methyi groups to consider, because the carbons are far apart. Thus, the conformational pre
purely an effect of the diequatorial orientation being preferred over the diaxial orientation. This makes
value of Keq-1,4 greater than the value of Keq-1,2, making choice D the best answer. Drawn below is one
worth (apprbximately equal to 1000 words worth) of explanation.
Methyl groups are apart from Methyl groups are close to one Methyl grouPs collide with Methyl groups do not collide
one another when diaxial. another when diequatorial. axial Hs when diaxial. one another when diequatorial
CH. 9Hs
I,CH1
rr3L

I - = !,"guubtl
oiulgl Diaxial < Diequatorial
cF{3 -H.C,
Anti > Gauche
cFI3
No anti vs. gauche factor

1,2-diaxial orientation results in anti orientation, while 1,4-diaxial orientation results in steric hindr
1,2-dieqatorial orientation results in gauche orientation. from diaxial interactions with hydrogens, w
The equilibrium- still favors product (the right), but not l,4-diequatorial has no eclipsing steric hindr
as much as the L,4-equilibrium does. Equilibrium favors the products more than with 1

4" Choice C is correct. In order for substituents to be gauche or anti to one another, they must be bonded to ca
that are connected to one another. In the case of 1-4-dimethylcyclohexane, the methyl groups are not
adjacent carbons, so the two methyl groups cannot be gauche or anti to one another. This makes choice C
There are hydrogens on every carbon, so H can be gauche to methyl. For the H to be anti to a methyl, the
group must assume axial orientation. The oniy possible chair conformation of cis-1,4-dimethylcyclohexane
one methyl group axial and the other in an equatorial orientation. This makes choices A, B, and D valid.
Copyright @ by The Berkeley Review@ 166 Section II Detailed Exp
Choice C is correct. Both of the chair conformations possible for cis-1,2-dimethylcyclohexane are equivalent in
energy. In both of the chair conformations, one methyl substituent assumes axial orientation and the other
methyl substituent assumes equatorial orientation. The equilibrium constant for the ring-flip process is equal to
1, because the energy levei of the product is equal to the energy level of the reactant. This makes choice C
correct. Do the correct thing and pick C. As a point of interest, the value of KuO can never be less than or equal to
0 (products and reactants always have some positive quantity), so choice A is an absurd answer.

HsC

HsC

Both structures have one methyi equatorial and the other methyl axial

Choice D is correct. With cis-1,3-dimethylcyclohexane, the cis-1,3-diaxial interactions (steric repulsion)


between the two methyl groups makes the diaxial orientation less stable than the trans-1,2-diaxial orientation.
This decrease in stability in the conformation drawn on the reactant side results in a greater value for Ksq as
written. This means that K"O-1,3 ,K"q_I,2, which is choice D.
Methyl groups are apart from Methyl grouPs are close to one Methyl groups collide with Methyl groups do not collide with
one another when diaxial. another when diequatorial. one another when diaxial. one another when diequatorial.
CH.
IJ
.r, a, fn,
In, S^
cF{"
1,2-diaxial orientation results in anti orientation, while 1,3-diaxial orientation results in increased. steric
1.2-dieqatorial orientation results in gauche orientation. hindrance. This has an effect on the equilibrium by
I?re equilibrium still favors product (the right), but not as shifting it heavily to the right. 1,3-diequatorial
much as the 1,3-diaxial-to-diequatorial equilibrium does. has no eclipsing sieric hindrance.

Choice B is correct. The key to this problem is drawing the two hydrogen atoms on the bridging carbons cis to one
another. When the hydrogens are cis to one another on adjacent carbons, one hydrogen assumes axial orientation,
while the other assumes equatorial orientation. As a consequence, the carbon-carbon bonds to the left ring must
also be axiai for one and equatorial for the other. The structure is drawn below with the hydrogens noted.
Choice B is the best answer.

OH
Note that the two hydrogen atoms and
the hydroxyl group are all cis and up.
Equatorial
Axial

Equatorial--+11

Choice C is correct. Because 240' is at the nadir (low point) of the graph in Figure 1, it correlates with the
staggered structure. This eliminates choices A and B. The compound has R stereochemistry at carbon two which
makes the correct answer choice C and eliminates choice D, which has S stereochemistry.

Choice A is correct. Because 60" is at a local apex (high point) of the graph in Figure 1, it correlates with an
eclipsed structure. This eliminates choices C and D, which have the compound in its staggered confirmation.
The 60' point is not the highest point on the energy diagram, so it does not involve the largest groups (carbon 1
and carbon 4) interfering with one another. This eliminates choice B. The best answer is choice A, with the
two methyl groups eclipsing one another.

-:-:vright O by The Berkeley Review@ t67 Section II Detailed Explanations


10. Choice A is correct. The 330" point on the graph is near (30" away from) the most stable conformation (nh:
has anti orientation of the CH2OH group and the CH3 group of carbon 4). This eliminates choices C and
Because of R stereochemistry, the correct choice is A. Stereochemistry is difficult to see in the Newrr
projection and can be seen more easily in other projections. Drawn below is a way to convert the Neu,c:
projection back to the stick-wedge drawing and a subsequent evaluation of stereochemistry.
CHs

H:9

H
cH2oH H CH.OH
H\\"t"'

11. Choice A is correct. Conditioning may cause yoll to respond automatically that the best orientation is the ane
with the fewest repuisive interactions. This is often true, but it does not tell the entire story in this case. The,
most stable orientation can also be the result of the strongest attractive interactions. Hydrogen-bonding betn'
the hydroxyl and amine groups occurs only from gauche orientation, where the two gtorpi are close enough
bond. Hydrogen-bonding cannot occur between substituents with anti orientation. This *uk"r choice A correct.

1' Choice D is correct. Hydrogen bonds have some acid-base character to them, so the most favorable pro::
transfer reaction is a good indicator of the strongest hydrogen bond. Because the amine is more basic than i
hydroxyl group, the nitrogen is the lone pair donor. Likewise, the hydroxyl is more acidic than the amine
the hydroxyl is the hydrogen donor. This makes choice D correct.

13. Choice D is correct. Because of the planar nature of electron density in a n-bond, rotation about a double bc:
requires that the n-bond be broken. This is not observed under thermal conditions. To convert a cis n-bond int;
trans n-bond, UV light is needed. Pick D to tally big points. The drawing below shows that a 90' rotation abc
the C-C breaks the r-bond. It requires substantial energy to break a rc-bond.
ru-bond

R1
90" rotation R,
----------------
R3
R2

1.4. Choice D is correct. The only single bond about which rotation is not possible is a single bond between two ar
in a cyclic compound. The only cyciic compound of the answer choices is methylcyclopentane, choice D. The
answer is therefore choice D. One item of notable interest is that both n-bonds and rings lower the entropl
compound by lowering its degrees of freedom (i.e., its ability to rotate freely).

1.5. Choice B is correct. The most stable form of cyclohexane is the chair conformation (as opposed to the i:
conformation), with the smallest substituents (determined by bond length) in the axial position. The inteq
from the proton NMR shows that the ratio of peaks for 1H is 1.12 : 1 in favor of the 6 = 7-.25 ppm shift, the
due to the equatorial hydrogen. This indicates that more hydrogens are located in the equatorial position=i
1.25 ppn) than the axial orientation (6 = 1.51 ppm). Consequently, the preferred conformation has deuteriu-cr
the axial position. For this reason, choose B. Drawn below are the two chair structures and their equilibriurc" I
0
6 = t.st ppm ----->H
L{ D m

6 = 1.25 ppm
--->H {
,0

t[
rU

rfi
- H
6axiaiH:5equatorialH (IHNMRintegral 7.2:7) 5 axialH:6equatorialH llUruVRintegral 1: T
,fl
Actual ratio is I.1.2:1, rvhich shows H prefers the equatorial position, so the right structure is more stable.
Copyright @ by The Berkeley Review@ l6a Section II Detailed Explanat
Choice C is correct. Using D2 gas rather than H2 gas would have produced C6H5D7. The addition of the D2 is
in syn addition for all 6 deuteriums that are added, producing two possible conformational isomers, one having
three hydrogens equatorial and two hydrogens axial (choice C) and the other having two hydrogens equatorial
and three hydrogens axial (choice D). Choice C is more stable, because hydrogens prefer the equatorial
orientation. Choose C if you want to be a star. The structure is drawn below:

.-

- D
3 axialH:2 equatoriaiH
3axialH:2equatoriaiH 2axiaIH:3 equatorialH
H prefers the equatorial position, confirming that the structure on the right is
more stable. The K"O for this ring flip is > 1. The ratio is 2 axial: 3 equatorial.

This question could have also been answered without knowing the exact chemistry. The passage infers that
hvdrogen prefers the equatorial position over deuterium. This means that in the most stable orientation, there
are more hydrogens with equatorial orientation than axiai orientation. This eliminates choice D. Because
there are only five hydrogens present (you start with five on deuterobenzene), the sum of the two numbers must
be five. This eliminates choices A and B and makes choice C the best answer. Learn to answer questions as
quickly as you can, whether you use organic chemistry knowledge or common sense.

Choice D is correct. The addition of H2 gas to chlorobenzene results in the hydrogenation of the benzene ring
and the formation of CH11Cl (chlorocyclohexane). For chlorocyclohexane, there are two possible stable
conformational isomers, one having a ring structure in a chair conformation with the chlorine equatorial
(choice D) and one iraving a ring structure in a chair conformation with the chlorine axial (choice C). Chlorine
is larger than hydrogen, so it prefers to be in the equatorial orientation. Choice D is more stable so choose it.

Clroice B is correct. The hybridi zatron of carbon remains sp3, whether it is bonded to deuterium or to hydrogen'
The angle therefore should be around 109.5". It is stated in the passage that the bond angles are between 107.5"
and 111", which are both nearest to 109.5" of all the choices. Pick B for best results on this question.

Choice A is correct. The cis form of 1,3,5-trimethylcyclohexane allows for all three methyl groups to assume
identical orientation in terms of axial or equatorial. This means either choice A or choice D is the best answer.
Equatorial orientation is more stable than axial orientation, so choice A is the best answer. The conformation is
shown below:
H

H
Hsc

Choice B is correct. Like benzene, C6H5D has three n-bonds and one ring. The best answer is choice B. If you
recall the formula for units of unsaturation, (2(#C) - (#H) + 2)/2, you can calcuiate the units of unsaturation,
knowing that D behaves the same as H. This would also yield a total of 4.

Choice D is correct. A hydroxyl group, OH, is larger than a hydrogen, so choice A should be eliminated. The
axial orientation is -or" iritld"redin terms of sterics, so choice B is invalid. Choice C is a good explanation for
why the two hydroxyl groups would be found in a diequatorial orientation, but the question is looking for an
expianation foi why diaxiai, rather than diequatorial, is the preferred conformation. This eliminates choice
C. When the two hydroxyl groups are both in axial orientation, they can exhibit 1,3-diaxial interactions.
This is normally considered to be steric repulsion; but in the case of two hydroxyl grouPs, there exists the
ability to form hydrogen bonds. This makes choice D the best answer.

::',right O by The Berkeley Revierv@ r69 Section II Detailed ExPlanations


,, Choice D is correct. Only in choice C and choice D do the rings have their substituents with cis orientation.
Choices A and B are eliminated, because the substituents are trans to one another. The more stable conformer
has the larger substituent (the ethyl group) in the equatorial position. This describes choice D.

23. Choice D is correct. An IR absorbance between 1700 cm-1 and 7740 cm-1 is the result of a carbonyl (C=O) group, as
stated in the passage. Of the choices, only the_ether doesn'tcontain a carbonyl group, so it is the ether thai
does not have an IR absorbance between 1700 cm-1 and.7740 cm-1. Pick D for best t"rrttr.

24. Choice C is correct. A straight-chain monosaccharide has O-H groups, a C=O functionality and C-H bonds.
These groups have IR absorbances for bond stretching of 3300 cm-r,l7I5 cm-1, and 2980 cm-1 respectively. This
makes choice C the best choice. You are required to know common values for the IR absorbances. No common-
value peak is found around 2300 cm-1, so thit should lead you to answer choice C.

25. Choice A is correct. A broad signal near 2850 cm-1 indicates the hydroxyl group of a carboxyiic acid (O-H witin
hydrogen-bonding). A hydroxyl group exhibits hydrogen-bonding, so the signal is broad. Because the O-H
bond of a carboxylic acid is weak, its absorbance is lower than that of standard hydroxyl groups. Of the ansirer I

choices, only choice A has a carboxylic acid functionality, let alone a hydroxyl gio,rp. plit a for happiness. :

26. Choice D is correct. The region between 7.0 and 8.0 ppm in the proton NMR can be attributed to hydrogens on I
aromatic ring. This immediately eliminates choices A and B, which have no aromatic rings associated rri ,ii

them. The difference between ethylphenol and methylphenol can be demonstrated by eithei the total numb t
of hydrogens (as shown in the integration) or by the coupling of the alkyl portion of the compounds. The
grouP exhibits a 2H quarter and a 3H triplet, while the methyl group exhibits a 3H singlet. The extra
group associated with the ethylphenol compared to methylphenol is found in the 2.0 to 2.5 ppm region, not (
aromatic region. This eliminates choice C. To decide between the two compounds in choice D, one can look iu
the coupling of the hydrogens on the benzene ring, found in the range between 7.0 ppm and 8.0 ppm. fr
splitting for the hydrogens on a benzene ring with para-substitution is symmetric, while the splitting for
hydrogens on a benzene ring with meta-substitution is asymmetric. This is the distinguishing iactor betu (
the para-substituted and meta-substituted phenols in choice D. The best choice is answer D. u
,n Choice A is correct. Because the broadness of hydroxyl peaks is associated with hydrogen-bonding, '@
r
rffi
decreased hydrogen-bonding of a tertiary alcohol is reflected as a narrower absorbance in the IR spectru-n:.
compared to a primary alcohol. The alcohol peak is found weli above 3000 cm-1, so the best answer ii choice

28. Choice D is correct. An IR absorbance at 1775 cm-1 is rndicative of a carbonyl group (the stretching of a C
bond)' Aldehydes and ketones contain a carbonyl group, so choices A, B, and C are eliminated. Only the
(tetrahydrofuran) does not have a carbonyl functionality. Pick D and bask in the glow of correctness.

29. Choice C is correct. The hydrolysis of an ester results in the formation of a carboxylic acid and an a
Both the ester and the carboxylic acid have carbonyl groups, so each has an IR absorbance around 1700 c
This means that both before and after hydrolysis, there is a signal around 1700 cm-1, eliminating choices A
B. An ester has no hydroxyl group, so initially there is no signai around 3300 cm-1. Following hydrolysis,
an alcohol and carboxylic acid are formed, so a signal for the hydroxyl group appears. In particular"
hydroxyl group of an alcohol shows an absorbance around 3300 cm-1. This means that during the course cd-
hydrolysis of an ester, an IR signal appears around 3300 cm-1, making the best answer choice C.

30. Choice A is correct. Because a ring takes one degree of unsaturation and a carbonyl takes one d
unsaturation, a cyclic ketone has two degrees of unsaturation. The compound has only one
unsaturation, so it cannot be a cyclic ketone. The correct answer is choice A. A cyclic ether has one ring and
bonds, so it has one degree of unsaturation. This eliminates choice B. A carboxylic acid has no rings and
bond, so it has one degree of unsaturation. This eliminates choice C. An ester has no rings and one n-bond.
has one degree of unsaturation. This eliminates choice D.

Copyright @ by The Berkeley Review@ t70 Section II Detailed Expl


Choice B is correct. Ozonolvsis cieaves the double bond of an aikene, converting it into two carbonyl compounds
(either an aldehyde or a ketone). In this reaction, the products formed are aldehydes, as mentioned in the
passage. Aldehvdes can be identified by their peak around 9.7 ppm in the 1HNMR. Choose B for best results.

Choice A is correct. Compound B, like Compound A, is an aldehyde. Like Compound A, it shows an IR


absorbar-ice near 7725 cm-1 for the stretching mode of the C=O bond. This makes choice A correct.

Choice A is correct. Compound A has one degree of r-rnsaturation (due to the carbonyl), and Compound B has two
degrees of unsaturation (one due to the carbonyl and the other due to a ring). The second degree of unsaturation
has to be from a ring, because there can be no double bond in the product from ozonolysis (had there been a n-
bond, the ozone would have reacted with it.) This means that the original alkyne must have had a ring and a
triple bond, which in turn means that the original alkyne had three degrees of unsaturation. The alkyne had
nine carbons (the sum of the carbons from Compounds A and B). A nine-carbon compound with no units of
unsaturation has the formula CSHZO. For every unit of unsaturation, two hydrogens are removed from the
formula, so a hydrocarbon with three units of unsaturation must have the formula C9H1.4, choice A. You can
verify tlris with the molecuiar mass, which is roughly 122 g/mole, as stated in the passage.

Choice D is correct. An alkene is distinguishable by an IR absorbance between7620 cm-1 and 1660 cm-1 for the
stretching of the C=C bond. You should know that i carbonyl C=O bond absorbs at around 1700 cm-1, and being
that a C=C bond is slightly weaker than a C=O bond (they are both double bonds, but a carbonyl bond is
slightly shorter), it takes siightly iess energy to stretch the C=C bond than the C=O bond. This means that a
C=C bond has an absorbance slightly less ihar-r 1700 cm-1. Only choice D is less than 1700.*-1, so choice D is
tl-re best answer. The test requires that you have some iR peaks in your memory; but for ones you don't recali,
estimate them by comparison to the values you know. Choose D in this question for the feeling of correctitude.

Choice ll is correct. The products from the ozonolysis of the alkene are both aldehydes, so Compound A must be
an aldehvde, which makes choice B the best answer. All of the choices have the correct units of unsaturation,
so the bcst answer rnust be determir-red from the presence of the aldehyde proton.

Choice C is correct. As stated in the passage, Compound B must be an aldehyde, which eliminates choice A.
The con'rpound cannot contair a double bond (the product from ozonolysis cannot have a double bond between
carbons), so choice B is eliminated. The lHXVR integration shows a peak ratio of 7:1:4:4, which indicates
that there are no methyl groups in the compound. A methyl group would have shown a relative ratio in the
integrai of 3. This eliminates choice D. Choice D can also be eliminated, because it contains too many carbons.
This narrows it down to choice C, which does in fact show four nonequivalent hydrogens in a ratio of 1:1:4:4. It
is vital that you solve this question by a muitiple-choice elimination process rather than structural deduction,
because in a multiple-choice format, eliminatir-rg invalid structures is faster than elucidating the correct one.

Choice D is correct. The question here is not what type of bond is causing the shift, because all four answer
choices are C-O bonds. The question is: "What type of molecule is Compound T?" Because of the broad IR
absorbance at 3350 cm-1, tire compound has H-bonding, so it must be an alcohol. The correct answer is therefore
choice D. Choices B and C should be elimilated, because the passage states there is no carbonyl. Choice A can
be eliminated, because ethers exhibit no hydrogen-bonding.

Choice C is correct. An absorbance at 2\76 cm-1 implies that the compound is an alkyne. This is trivial
knowledge for the most part, but the passage provided the information, in case you don't have carbon-to-carbon
triple-bond IR data committed to memory. You can deduce the IR absorbance vaiue for an alkyne knowing that
a C-C boncl has an IR absorbance between 1100 cm-1 and 1300 cm-1, and that a C=C bond has an IR absorbance
between 1620 cm-1 and 1680 cm-1. As the bor-id strength increases (or bond length decreases), the IR shift value
incre;rses. Choices C is the best answer. The answer is given at the end of the tl-rird paragraph in the passage.

Choice A is correct. A doublet in the 13CNMR is due to a carrbon with one hydrogen attached to it. Because a
ketone carbon and an internarl alkyne carbon do not have hydrogens attached to them, they would be singlets
rather than doublets. This eli.mi.nates both choice B ancl choice D. There is no peak in the IR spectrum,
inciicating that there is a C=O present (no peak around 1700 cm-1), so choice C is eliminated. The only choice
left is choice A, the best choice.
,:vright O by The Berkelev Reviervo t7l Section II Detailed Explanations
40. Choice C is correct. If a compound has no symmetry, then thereis a 13CNMR peak for every carbon in -*-*
compor-rnd. The number of carbons in a compound exhibiting seven 13CWR peaks is therefore r"rr"rr. pick C :.r
best results.

47. Choice D is correct. When hydrogens are coupled to one another, they have the same coupling constants :
values)' This makes choice D the best answer. Two different hydrogen groups have different shift values, .,:
choice A is eliminated. Choices B and C are the same answer, so they shouldboth be eliminated. This lear'=
only choice D as the best answer.

42. Choice A is correct. From tl-re NMR data, we know that Compound T has three carbon atoms and four hydrog.:
atoms, so the structure must contain only three carbons. Choices B, C, and D al1 have four carbonr, ,o thuy .-,=
eliminated without involving the spectroscopic data. The only choice that fits the formula is choice A. Or^.::
this question is resolved, it can help to solve some of the other questions. This happens often on the MCAT. m
hllL.
43. Choice B is correct. The 13CNMR peak at 50 ppm is due to a carbon, not a hydrogen. This eliminates choices
-
and D. A carbonyl shows a 13CNMR peak around 200 ppm, so choice A is eliminafed. Only choice B remains,
you sl-rould pick it. The absence of an IR peak around 1700 cm-1 should reaffirm that a carbonyl is not present '.,:
Compound T. This means that choice B is the best answer.
-

Choice B is correct. The total number of hydrogens in the compound, according to the NMR is eight, of whr::
five are on the benzene ring. This eliminates choice C. The pisug" states thal the compound is an ester,:.
choice A, a ketone, is eliminated. The best answer is choice B, because the methyl shift is around 4.0 ppm. Th,-
is tough to know without an NMR chart. In the passage, it is mentioned that the alkoxy group hai a shift :
the 1HNMR between 3.5 and 4.0 ppm. if you can'f recali a fact, search for it in the passage.

45. Choice C is correct. The three separate IUXMR spectra are to be compared to determine the compounds :
solution. It is important that nothing varies between samples, so the same solvent should be used i1 each ca.s=
Any impurity peak wouid be common to all of the spectra and thus could be eliminated. This makes choice C
the best answer. A common solvent has no effect on the pH or the behavior of a protic species in the lF{Nlt:
Although lHUltR invokes an externai magnetic field, the solvent has no bearing on the iielcl.

46. Choice B is correct' This question is typical of spectra-to-structure-to-name questions. There are four carbons :
Compor,rnd III, so choices A and C are elj.minated, bec;ruse they have only three carbons, according to t:-
nomcnclature. Translating the narrles of cl-roices B and D yields the following two structures:

o o

H3CH2C
A ocH3 H:C
A OCH2CH3
Methyl propanoate Ethyl ethanoate
Both A anci C would have identical peaks of a 2H quartet, 3H triplet, and a 3H singlet. The key feature is ti.
3H singlet at approximately 3.5 ppm. This indicates that the methyl group is attached to the ester oxygen :i
in choice B. You should pick B to score.

47. Choice A is correct. The four answer choices are the result of the following bonds: O-H (3500 sp3C--
(2980 cm-l), C=O (1685 cm-1;, and C-O (1300 cm-1). An ester contains an sp3-f-g bond, a C=O bond, "^-t),
and a C-i
bond, but it does not contain an O-H bond. This means that there will be no O-H stretcl-r (thus no IR peak at i:
near 3500 cm-1; associatecl with the IR spectra of an ester. Cl-roose A and be a stellar achiever at choosr-r.:
correct answers^

48. Choice B is correct. Compound II has two oxygerls in its formula, so choices A ancl C are out immediately. T:'.
question here is reciuced to placing the a1ky1 group either on the ox\1gen (as in choice B) or on the carbon., -
carbon (as in choice D)" Because the peak for the two hydrogens is around 4.0 ppm, the ethyl group must'r=
attached to the oxygen, making choice B the best answer. You should note that when there is a jH quartet ar.:
a 3H tripiet in the spectra, there is an isolatec-l ethyl group ir-i the n-rolecule.

Copyright @ by The Berkeley Reviewo 172 Section II Detailed Dxplanations


+Y. Choice D is correct. Taken sequentially, the signals are a singlet (one apex), a quartet (four apexes), and a
triplet (three apexes). This makes choice D the correct choice. The shape of a peak, you should recall, is due
to the presence of hydrogens on the neighboring carbon.

Choice C is correct. An NMR solvent should be inert and show no peaks in the spectra. All of the compounds are
deuterated, (contains 2H rather than 1H) so they show no peaks in the 1HNMR spectrum. The most reactive
solvent listed is the alcohoi. The problem with an alcohol is that when it is heated or given enough time, it
can undergo a transesterification reaction and change the ester. This makes choice C the best choice. An ester
should be soluble in all four of the solvents. It is a general rule that protic solvents are bad choices for the
NMR, because they exchange for hydrogens and are typically more reactive than aprotic solvents.

Choice C is correct. 2-methyl-3-pentanone has four nonequivalent hydrogens, according to the symmetry of the
molecule. The structure of 2-methyl-3-pentanone is drawn below:

H*C
"l
6 a hydrogens
bl c
1 b hydrogen

'll
A../tt\ ,/t'=-.ff, 2 c hydrogens
3 d hydrogens

o
This resuits in a ratio of 6:3:2:l for the four nonequivalent hydrogens, which makes choice C the best answer.

Choice A is correct. An isolated isopropyl group has two equivalent methyls groups (six hydrogens total)
adjacent to a CH group (one hydrogen). The six hydrogens of the methyl groups are expressed as a doublet
(being adjacent to one hydrogen), and the one hydrogen of CH are expressed as a septet (being adjacent to six
equivalent hydrogens). This results in a proton NMR with a septet (1H) and a doublet (6H). This is choice A,
so choice A is the best of all possible choices,

Choice D is correct. Pentanal is an aldehyde, while 2-pentanone is a ketone. An aldehyde is best


distinguished from other compounds by a peak near 9.7 ppm for the aldehyde proton. Choices A and B should
be eliminated, because both pentanal and l-pentanone have a peak in the IR just above 1700 cm-1 for the C=O
bond and a peak in the proton NMR between 2.0 and 2.3 ppm for the alpha hydrogens. Neither comPound has a
proton NMR peak between 3.5 ppm and 4.0 ppm, because that shift value is attributed to hydrogens on a carbon
bonded to oxygen. Only choice D, a peak near 9.7 ppm, is for a peak that is unique to the aldehyde, so the
presence of the peak confirms that the compound is pentanal, while the absence of the peak supports that the
compound is 2-pentanone.

Choice B is correct. An alcohol hydrogen is rnildly acidic. This means that when a base is added to an alcohol,
ihe protic H on oxygen can be removed. If the alkoxide formed is placed in deuterium-labeled water, then the
alkoxide can remove deuterium from water to form a deuterium-labeled alcohol. Deuteriums do not appear in
the 1HNMR, so the peak for the H on oxygen disappears, making choice B the best answer.

R_d:\
r\ "\/ e
+oo1 D-\ +Ro
o

H----/ H

D-dl"\/
u e
-- o
+on1 +DO
D---/ D

-
Choice A is correct. A methyl ketone has an isolated CH3 group adjacent to a carbonyl. Because the methyl
group has no hydrogen neighbors, it has no coupling, and thus is a singlet between 2.0 and 2.5 ppm. Pick A.

-::r-right @ by The Berkeley Review@ 173 Section II Detaited Explanations


56. Choice B is correct. A 1HNMR peak at 9.7 pprr- indicates that the compound is an aldehyde. This eliminat.s
choice A (an ether) and choice D (a ketone). The ratio of 1,:7:6 lor theirea of the signali indicates that the:: ry
are two equivalent methyl groups (that accounts for six equivalent hydrogens) and two unique hydrogens on +;,: tr5.
compound. Only choice B has two equivalent methyl groups, so choice B is the best answer. Theie are fc::
unique types of hydrogens on butanal, so butanal would show four signals in its 1HNMR, not just three signals.

57. Choice D is correct. Having no IR absorbance between 1600 cm-1 and 1750 cm-1 indicates that the compound h.:-.
neither a C=C bond nor a C=O bond in its structure, both of which have IR signals between 1600 cm-1 and 1li,
cm-1' This eliminates all of the choices except choice D. The compound ,rr.rit b" cyclic to account for the o:*
degree of unsaturation.

Choice C is correct. In Spectrum I, the distance between the peaks in the aikene region (coupling constant :;
greater than it is in Spectrum II. A larger coupling between vinyiic hydrogens is attributed to the tra:=
compound. This is choice C.

59. Choice D is correct. The methyl group has three hydrogens, so the signai for the methyl group on benze:.
cannot be the peak at either 7.25 ppm and 5.37 ppm, because neither of those peaks contains three hydroge:--
This eliminates choices A and B. The alkoxy methyl group on the oxygen is found farther downfield than f *
methyl on benzene. Thus from the values on the spectra, the methyl on benzene (3H singlet) comes at 2.25 pp::
which is choice D.

60. Choice C is correct. In order for the disubstituted benzene compound to have the same number of peaks in -:*
carbon NMR as it has carbons, the compound must not have symmetry. Symmetry results in equivallnt carbc:-.
resulting in fewer 13CXUR signals. the only asymmetric iompound u-orlg tire answer choices is the n.-:.l
substituted benzene with two different substituents. Choice C is correct. The choices are drawn below:

A. B.X C.X D{
I I

cc /Cb,' c,/cu.-
i-' il
au
1" 'iiT"'f.
?Cr -
C.\
^
t Cr--. c.1 lzC.
CcX udY
$'
6 Carbons 6 Carbons 6 Carbons 6 Carbons
3
l3cltttR peaks 4
13CNMR
peaks e
13CNMR
peaks 13CNMR
4 peaks

61. Choice B is correct. The 1H peaks (the vinylic hydrogens) in the spectrum fall between 4.88 and 5.37 py=".
implying that the hydrogens on the alkene are found in this range. Hydrogens on an alkene are referred tc m
vinylic hydrogens. This means that vinylic hydrogens are found in the range of 4.5 ppm to 7.0 ppm. This ma..:s
choice B the best choice.

Choice D is correct. The compound contains a C=C double bond, which has an IR absorbance between 1620 c::'')
and 1660 cm-1. The best choice is 1640 cm-l, choice D. An IR absorbance of 3500 cm-1 is due to a O-H bond, ar. Ji
absorbance of 2220 cm-1 is d"ue to a C=C bond, and an IR absorbance of 7720 cn-7 is due to a C=O bond.

63. Choice D is correct. With meta-substitution, all of the benzvlic hydrogens would be different, because --:*
benzene compound would be asymmetric. There would thus be four nonequivalent hydrogens in a 1:1:1:1 ra-r:,
Picking D is a beautiful thing on this question.

64. Choice D is correct. An ethyl group on benzene is composed of a CH3 group next to two equivaient hydrogi--r,.,
(making it a 3H triplet) and a CH2 group next to three equivalent hvdrogens (making it a 2H quartet). I:
describes choice D.

Copyright @ by The Berkeley Review@ t74 Section II Detailed Explanatio


Choice C is correct. Because the compound contains two oxygen atoms, choice A (an aldehyde) and choice D (a
ketone) are both eliminated. There is no peak in the IR spectrum that is indicative of an O-H bond, so the
compound is not a carboxylic acid. This eliminates choice B. The only choice left is choice C.

,ffi. Choice C is correct. The molecular ratio of hydrogens cannot be 3:1 in that region of the proton NMR spectrum,
given the shift values. In order for there to be only one hydrogen on a carbon, there must be other groups
ittached to that carbon. If the other group was caibot'r-based, ihere would be more signals in the IHNVIR
spectrum than two and more signals in the 13CNMR spectrum than four. If the groups contained no hydrogen,
then they would affect the shift value, making it farther downfield than 2.0 ppm. They are alkyl hydrogens,
so the ratio is most likely 9:3, caused by the presence of equivalent methyl groups. Any alkyl group besides
methyl would show coupling and not singlets. The structure must therefore have a tertiary butyl group (nine
equivalent hydrogens forming a singlet) and an isolated methyl group (three equivalent hydrogens forming a
singlet). Choose C for best results.

Choice D is correct. A 13CNMR peak is for carbon, not hydrogen, which eliminates choices A and B. The
passage states that the carbon of the C=O bond is found at170.2 ppm., which eliminates choice C. The best
answer is choice D. Carbons bonded to an electronegative atom that arc spJ-hybridized typically show a
signal between 60 and 90 ppm.

tr!" Choice C is correct. As stated in the passage, the peak at 170.2 ppm is attributed to a carbonyl carbon.
Carbonyl carbons are found between 770 and220 ppm in the 13CNMR. Choice A is an ester, choice B is a ketone,
and choice D is an aldehyde, of which all three contain a carbonyl group. This means that choices A, B, and D
can be eliminated, because they all exhibit a signal in the l3CNltR that is greater than 100 ppm. An alcohol
has a peak around 70 ppm to 80 ppm for the alcohol carbon, but not greater than 100 ppm. The best answer is
choice C.

Choice A is correct. The shift value for hydrogens on an alpha carbon (the carbon adjacent to a carbonyl) is
found to be between 2.0 ppm. and 2.5 ppm. The alpha hydrogens are described in the answer choices as
hydrogens on a carbon adjacent to a carbonyl (C=O) bond. This is choice A; your choice for a question like this.

Choice D is correct. A singlet in a 1HNMR spectrum occurs when there is an isolated hydrogen (or group of
equivalent hydrogens), with no hydrogens on the neighboring atoms. This eliminates choices A, B, and C, and
makes choice D the best answer.

Choice D is correct. Compounds A and C are eliminated, because they do not contain two oxygen atoms, as is
stated in the passage. Choice B is eliminated, because the CH3 of the methoxy group would show a shift value
between 3.5 ppm and 4.0 ppm. The peak for the lone methyl group is found near 2.0 ppm, which indicates that
the methyl group is adjacent to the carbonyl. The correct structure is choice D.

:a Choice C is correct. The oxidation of a secondary alcohol to a ketone results in a carbon that goes from a
i3CIrIptR signal of roughly 75 ppm to a signal of roughly 200 ppm. This conversion can be monitored.easily by
13CNMR, so choice A is valid. Nitration of ethylbenzene results in a carbon that changes from a TTCNMR
signal of roughly 115 ppm to a signal of roughly 140 ppm,.lecause the nitro group changes the immediate
chemical environment. This conversion can be monitored by 13CNVR, so choice B is valid. Deprotonation of a
carboxylic acid. does not change the immediate environment (adjacent atoms) of any carb.oSr so no drastic change
in change in any l3CNltR sifral is observed. This conversion cannot be monitored Uy isg*tR, so choice C is
invalid, and thus is the best choice. Reduction of an alkene to an alkane results in a carbon that goes from a
13CNMR signal of roughly 100 ppm to a signal of roughly 20 ppm. This conversion can be monitored by
l3CNVln, so choice D is valid.

:i Choice C is correct. According to the reaction in the passage, the compound is an aromatic ketone. The peak at
206 pprn, according to the data in Table 1, confirms that Isomer I is a ketone. Choice D (the aldehyde) is thus
eliminated.. Becau-se there are only seven peaks for the 13CNMR, the compound must have symmetry, so
choices A and B are eliminated. The best answer (one that contains only seven unique carbons) is choice C.

J:n-right O by The Berkeley Review@ 175 Section II Detailed Explanations


74. Choice A is correct. An aldehyde, according to Table 1, is found between 185 and 200 ppm, and no peak is fou_i.:
in this range for either structural isomer. Choice A may be the corect choice. It stated in the pu5ug" that c,:
occasion, the shift may not fall exactly in the range given. An alkene, according to Table 1, is found beiween 1- -
and 140 ppm, and there are peaks found in this range for both structural isomers. Choice B can thus i.
eliminated. A ketone, according to Table 1, is found betr.r'een 205 and 220 ppm, and peaks are found in this rans=
for both compounds. Choice C is thus eliminated. A methyl group, according to Table 1, is found between 10 a-r;
35 ppm, and there are peaks found in this range for both structural isomers. Choice D is thus eliminated. Tf.-
best answer is choice A.

/J. Choice D is correct. According to the passage, the least intense peak is caused by a carbon with no hydroge:,.
attached. This makes choice D the best answer. The carbons with no hydrogens attached in a disubstitute:
benzene derivative with a carbonyl are two of the benzene carbons (the beniene carbons with a substituer:
attached) and the ketone carbon. It has sp2-hybtrdization, not sp3-hybridization, and it is achiral. Th-"
eliminates choices B and C. While it has sp2-hybridization, that is not the cause of its low intensity. Chor.:
D is a better answer than choice A. This is a question that rewarded the test taker who sifted through t:-
information in the passage. This will happen on occasion, because even the science sections are reading u*u*..

76. Choice C is correct. Para-methoxybenzaldehyde has a total of eight carbons in its structure, but the molecu-.
contains a mirror plane that reflects two pairs of equivalent carbons. Using symmetry, this means that the:=
are only six unique carbons, so the best answer is six signals, choice C. The structure is shown below:

o\^-t
!u
I
CD
.c/-\ c.
ilt
oC-.. Cd
eC -Z
lr
ocH3

Choice D is correct. Because the two compounds are isomers, they have the same molecular formula. Havrr:
the same molecular formula results in having the same units of unsaturation. This eliminates choices A and E
Each compound has a benzene ring and a carbonyl group. The benzene ring has three n-bonds and the one rir-r:
There are four ur-rits of unsaturation due to the benzene ring alone, so choice C is eliminated. When the carbor.-. -
E
n-bond is accounted for, there are five units of unsaturation in the molecule, so the best answer is choice D. &F

78. Choice B is correct. It is stated in the second paragraph of the passage that quaternary and carbonyl carbor..
generate peaks of low intensity. There is no carbonyl in the compound, but carbon 2 of 2,2-dimethylbutane ha-.
four other carbons attached, making it a quaternary carbon. The best answer is choice B, carbon 2.

79. Choice A is correct. Spectrum I contains a quartet (2H), a triplet (3H) (this combination is a dead give-an-a:
for an isolated ethyl group), and a singlet (3H) (a dead give-away for an isolated methyl group)- The t',=
question here is whether the ethyl group or methyl group is attached directly to the oxygen of the ester grou;
The degrees of unsaturation (1) and number of oxygens (2) tell you the compound must be either an ester or :
carboxylic acid. The lack of a broad peak between 10 - 12 ppm eliminates the possibility of the compound bef :
a carboxylic acid, so the compound must be an ester. Because the quartet is so far ciownfield (at a higher sh,:-
value), the ethyl group is attached to the oxygen. This makes the best choice an ethyl group on the meth'. -
ester, whose common name is ethyl acetate. Choose A for best results.

80. Choice B is correct. A triplet is the result of coupiing to the neigl-rboring hydrogens (there are two hydrogens c.
the adjacent carbon in the case of a triplet). The integral (quantitr. of hvdrogens for the signal) has no effect c:
the shape of the signal, meaning that the peak shape does not tell vou any information about the hydrogens c-
the signal, only about the neighboring hydrogens. This eliminates choices A and C. A triplet is therefore th=
result of neighboring a CH2 group. Choose B for a grade A, genuine, altogether correct, best answer.

Copyright @ by The Berkeley Review@ 176 Section II Detailed Explanations


Choice B is correct. Spectrum II contains a quartet (2H), a triplet (3H) (this combination is a dead give-away
for an isolated ethyl group), and a singiet (3F{) (a dead give-away for an isolated methyl group). Because the
3H singiet is found around 4.0 ppm, the methyl group is attached to the oxygen. This makes the best choice a
methyfgroup on an ethyl ester. Choose B for the happiness of another correct answer. The drawing below lists
how the name and structure are determined for the compound.
Singlet in the 3.5 to 4.0 PPm range

/il
H;C-.
o cHzcHj
methyl
--J propanoate
Nomenclature rules state that the alkyl group on oxygen is named first, followed by the ester chain' This
makes this compound methyl propanoate.

Choice C is correct. This is one of those trivial facts that you should know. A peak in the neighborho od ol 7
ppm is indicative of aromatic hydrogens, which are found in benzene compounds. Pick C, to score bigl

Choice A is correct. A carboxylic acid has one proton that forms a broad peak between 10 ppm and 12 ppm in the
1nruVtR. The hydrogen in question is the acidic proton of the carboxylic acid. Because there is only one proton,
the peak between 10 ppm and 12 ppm has an integration value of one hydrogen, so choices C and D are
eliminated. An ester nui at'r alkyl group attached to the noncarbonyl oxygen of the ester. Protons on the first
carbon from the oxygen have a peak between 3.5 ppm and 4.0 ppm. This eliminates choice B, leaving only
choice A as the possible answer. The peak between 2.0 ppm and 2.5 ppm is the result of alpha hydrogens,
which ot" preser-ri in both an ester and a carboxylic acid. 'lhis means that a peak between 2.0 ppm and 2.5 ppm
cannot be used to distinguish an ester from a carboxylic acid. The correct answer is choice A'

Choice B is correct. 4-Heptanone is a seven-carbon ketone with the carbonyl directly in the middle- The
structure is symmetric, so there are many equivalent carbons and hydrogens. There are four unique carbons, of
which tl-iree contain hydrogens. This resulti in three signals in the 1HNMR for 4-heptanone. The best answer
is choice B.

Choice A is correct. For Compound II, the absence of a 13CNMR signal between 180 ppm and 230 ppm supports
the idea that it has no C=O group. This eliminates choice C. No peak above 5 pp* in the
rHNMR spectrum
confirms that there is no double bond. This eiiminates choice D. This means that the unit of unsaturation in the
1HNMR spectrum suPPorts,the idea
compound must be the result of a ring. The presence of a broad peak in the
IrCNMR
that the compound is an alcohol, eliminating choice B and making choice A the best al1swer. The
spectrum shows that there is great symmetrv in the structure. The choices are either cyclopentanol or 2,3-
dlmethylcyclopropanol. The integral of the proton NMR says that there are mostly CH2 groups present,
which iurro., ryclopentanol over 2,3-dimethylcyclopropanol. Cyclopentanol is drawn below:
Broad------>1tO H<- Multiplet
(1H) \ / (1H) 13cNMR,

H"C
,zt\ CH"
There is a mirror planecutting throughthe
molecule, so there are three unique carbons,
resulting in three different signals.
MuitiDiet
42c- v (4FU

\,2
Tripiet (4H)

Copyright @ by The Berkeley Review@ Section II Detailed ExPlanations


86. Choice D is correct. Each unique carbon within a molecule exhibits a unique signal in the 13CNMR spectrum, so
choice A is valid. This eliminates choice A. A carbonyl carbon has a slgnaiaround 180 ppm in a 13CNVR
spectrum, which b d+lf:=llom other signals in the 13CNMR spectru-. ih" presence of a carbonyl group can
be identified using l3CNl'tR spectroscopy, so choice B is valid and thus elirninated. The substitution of a
benzene ring affects the symmetry of the molecule. For instance, if the substitution is para, then there is a
mirror plane in the molecule. This results in fewer unique carbons, which is seen with 13CNMR spectroscopy.
This makes choice C a valid statement, and therefore it is eliminated. The geometry about a doulle bond, cis
versus trans, does not express itself in l3CNltR spectroscopy. Typicaily, geJmetry is determined by looking at
constants of the vinyiic hydrogens in 1HNMR spectroscopy. bhoice D cannot be determined using
E
ll:::_"-p_t*g
IrCNMR spectroscopy, so choice D is the best answer. 91"

87. Choice C is correct. According to the data in the passage, only Compound I has a UV absorbance above 175 nm.
This means that only Compound I has a r-bond. There is only one unit of unsaturation, so Compound I can have,
at most, one n-bond. This means that Compound I cannot be a conjugated diene, which eliminites choice A. If it
were a conjugated diene, there would be a UV absorbance above 200 nm for the n to rc* transition. Compounds II
and III cannot be carbonyl compounds, according to their UV data. If there was a carbonyl gro.rp on the
molecule, there would be a n-to-n* transition around 190 nm and an n-to-n* transition around jOO n*. this
eliminates choice B. Because Compound I has two UV absorbances and only one n-bond, it must be a carbonyl
species of some sort, While it is not possible to decide between an aldehyde and ketone based on this
information, choice C is a solid answer. Because Compounds II and III show no UV absorbance above 1,75 nm,
there is no n-bond present. However, choice D refers to Compounds I and II, not II and III, so choice D is
eliminated. Choice C is the best answer.

88. Choice D is correct. For Compound III, the absence of a broad peak in its 1HNMR spectrum confirms that there
is no alcohol in the compound. This makes choice A a valid statement, and thui the incorrect choice. The
absence of a peak around 180+ ppm in its 13CNMR spectrum confirms there is no C=O present. This makes choice
B a valid statement, which eliminates it. The absence of an absorbance above 180 nm in the UV-visible
spectrum implies that there is no n-bond ir-r the compound, confirming that the structure must be cyclic to account
for the one unit of unsaturation. This makes choice C a valid statement, which eliminates it. The I3CNIrAR
shows very few signals (only three), which implies that there is great symmetry in the structure. It must be a
symmetric cyclic ether. This makes choice D an invalid statement, making it the best answer.

89. Choice B is correct. Key features from each spectrum must be extracted. From the molecular formula, we know
there is one unit of unsaturation and one oxygen. This means that the compound must contain either a ring, a
C=C bond, or a C=O bond. All of the answer choices fit these criteria, so we must use the spectroscopic dita.
the l3CNVtR data show a peak at 211 ppm (1), and that no two carbons are alike. This means that the
compound is a carbonyl, which does nothing to eliminate any choices. If the compound were 3-pentanone, it
would show only three signals in the 13CNVR due to its symmetry, which eliminates choice C. The fact that
no two carbons are alike also eliminates choice D, which has two equivalent methyl groups. the 1HN\4R data
shows a 3H singlet at 2.08 ppm, and no peak between 9 and 10 ppm. This means that the iompound is a methyl
ketone and not an aldehyde, which elj.minates choice A and choice C. The ratio of the hydrogen signals in the
lHNltn (3 :2 :2: 3) supports choice B. The structure is shown below:
o
nm and 268 nm
ll :v-vis:
18e

tV',. CNMR: 21 1 PPm


n"r/'T-.r'/
1HNMR,
t' I t'
rrJptet v,,rlipturrrlpt"t
t
silgr"t
(3H) (2H) (2H) (3H)
13CNMR,
All five carbons are different, so there are five different signals.
There can be no branching, because the proton NMR shows that the integral values are 3,2,2,3. This implies
that the structure is linear.

Copyright @ by The Berkeley Review@ t7a Section II Detailed Dxplanations


90. Choice D is correct. Integration is used to determine the reiative quantity of hydrogens within a signal by
looking at the area of the signal. Integration does not change with magnetic environment, so choice A is
eliminated. The neighboring hydrogen atoms affect the splitting, not the integral, so choice B is eliminated.
Integration does nothing to determine the presence of atoms other than hydrogen, so choice C is eliminated.
The best answer is choice D.

91. Choice C is correct. Five hydrogens constituting a singlet with a shift value between 6.0 and 8.0 ppm indicates
that the compound is a monosubstituted benzene. The three remaining hydrogens make up a methyl group. This
now becomes a nomenclature question, rather than a spectroscopy question. The correct name for a methyl group
attached to benzene is methylbenzene. The common name for methylbenzene is toluene. Choose C for optimal
results.

92. Choice D is correct. The formula contains six hydrogens in all, so the sum of the ratio values must equal 6. The
first peak is shortest, the middle peak is the second tallest, and the last peak is the tailest. This means that
the values must be ascending. The only combination of ascending values adding to 6 is 1:2:3. Choice D is the
best answer.

93. Choice D is correct. The compound 2-bromopropane has two unique types of hydrogens, so it has two peaks in its
IUNUn spectrum. The two terminal methyl groups are equivalent, so they are seen as one signal with an
integration of 6. The middle carbon (carbon 2) has one hydrogen, so it has a signal with an integration of 1. The
peak shape is determined by adding 1 to the number of hydrogens on the adjacent carbons. The six equivalent
hydrogens have one hydrogen neighbor, so there is a doubiet of integration 6. The one hydrogen has six
hydrogen neighbors, so there is a septet of integration 1. Choice D is the best ansr.ver.

9'1. Choice C is correct. A quartet is the result of the observed hydrogens being coupled to three equivalent
hydrogens. This is often the result of hydrogens that are adjacent to a methyl group on one side and no other
protons on the other. The quartet hydrogens are in bold face, and the neighboring three hydrogens are boxed in
the drawing below. Choice C is the oniy structure that shows no quartet in its proton NMR spectrum.
A. o B. o

(H3C)2HC
A cHr@ fnrburclHrcyrc
A CH:

C. o D. o

(H3C)2HCH2C
A cHg H3CH2CH2 C
A cHr@
95. Choice D is correct. A doublet is the result of hydrogens on a carbon that neighbors a carbon with only one
hydrogen attached (most easily recognized as a tertiary carbon). In each of the first three answer choices, the
methyl group attached to the interior of the carbon chain is bonded to a carbon with only one hydrogen (a
tertiary carbon), which results in every compound having a doublet with an integration of three hydrogens.
This leaves choice D as the best answer.

q5. Choice A is correct. A positive iodoform test, as stated in the question, is caused by a compound with three
aipha hydrogens on one carbon. This means that the iodoform test is positive for a methyl ketone, which
would have a CH3 group adjacent to a carbonyl (there are no hydrogens on a carbonyl). With no hydrogens on
the neighboring carbon (carbonyl), there is no coupling and thus a the peak is a singlet. Pick A for the pleasure
of correctness. The iodoform test works by removing an alpha hydrogen to form an anion. The anion
subsequently attacks iodine, adding an iodide to the alpha carbon. This is repeated two more times, until there
are three iodides bonded to the alpha carbon. The CI3 group is a great leaving group, and it forms a yellow,
oily compound when protonated.

-opyright @ by The Berkeley Reviewo Section II Detailed Explanations


97. Choice D is correct- The hydrogen responsible for the broadness is the carboxylic acid proton. It is bondecl to an
oxygen, making choice A an incorrect answer. The couphng to eight or more other hvdrogens would result in a
multiplet made of many sharp peaks, not a broadened signal, which eliminates ciroice B. Choice C can be
eliminated, because hydrogens on carbon do not form hydrogen bonds. the hydrogen forming the hydrogen bond
in this molecule is bonded to oxygen. This makes choice D the best answer.

Choice B is correct. Because the compound on the left has cis orientation about ttrre internal double bond, while
the compound on the right has trans orientation abor-rt the internal doubie bond, the two compouncls must be
geometrical isomers of one another. Pick choice B, and you won't be sorry. In case you were considering choice
C, the two compounds have the same absolute configuration, so they cannot be optical isomers.

99. Choice C is correct. Both double bonds are trans, so choice B is immediately eliminated. Both have the same
connectivity, so that eliminates choice A. The last thing to check is the chiral centers, and each has just one
chiral center. The compound on the left has R chirality, while the compound on the right has S chiralitr.
This makes the two structures optical isomers, choice C. If you flip the structure on the right to align with the
structure on the left, it can be seen that the chirai center has changed" A change in a chiral center results in an
optical isomer.

,ars".s
100. Choice A is correct. The best way to do this probiem is the systematic counting of carbon backbones, starting
with the iongest carbon chain possibie (five carbons). The tally for each possible carbon backbone is drar,r,r
beiow. There are only three possible structures: pentane, 2-methylbutane, and 2,2-dimethylpropane. Tlr-c
structures that are in fact structural isomers must have different IUPAC names. Pick A to be a correct anst'e:
picker person.
C
I

C-C
I

I I
C C

Copyright O by The Berkeley Review@ Section II Detailed Dxplanations


Section III
Stereochemistry
by Todd Bennett

Molecule fits template!


Rcenlelill>Y>a>l

Molecule does not fit template!


Scenterilr>v>a>.J

TIEBITEIEY
l) g. p- y.1.-6. u/'
Speciahztng in MCAT Preparation
Stereochemistry Section Goals
oB Be able tq identify stereocenters and chiral com
A stereogenic center (often referred to as a chiral center) is most commonly made up of a central
atom (gsually carbon) wlth four unique substitutents aitached. The stere6centei ii'identified as
R (lf,t1l : or S (Latin,. sinister) to define its orientatio" i.rpuce. g";;ti;;ilI t"qiiir"
:,it-h.*
that you identify lec,tusl
the number of chiral centers and often label them accordini to convention.'The
cite.d example of a,non-t1.pical situation involves allene, which has sf-hybridized carbons
lSguentlY
that can be chlral. An example of allene enantiomers is drawn below:

H H

Ntrt,,,-
& -i,n_^_^

ct/
CI cl

a? Be able to and stereoisomers.


Youmustunderstandthedifferencesbetweenenantiomersu''da
9"*""*.'.fl_1":"_.::Tf?.yf:ji9T
casesinvol,vi"g lh"toanh epimers- rnlJi't i-'o,' special
op.tica] inactivity and structure. iL"
such as anomers giu; irvo
M<ist importantly, when
:"g?.t:
be able to identify their relationship if they are'stereoisomers" of onestrucLures,
another.

Be familiar with common exam of chiral molecules.


It should be as second nature to yog that sugars occur naturally in the D form, *fli.f-r defined bv
having the penultimate carbon wittr R sterJochemistry. The iypical exception ir ;;;;ith " ;t;A
Vpes where one of the sugars in the antigenic
"in
determin-ant is L-fticose. lt shbuld be as second nahure
l.o,y,o:1la1a1ino.acids
oicur naturally the L form, which is defined by il;ing the;lpd;";il;
ylti-::1"^r:o-:,1:iil]V Jexqep,t for cysteine). A typical exception is seen wi[h transiriptidase enzymes
wnere the acttve amtno acrd is D-alanine.

Be able to use rotation data to i


@? an unknorun
JustIiketheboilrngpointandthemeltingpoint,theopticalrotationi'upnyffi
uyrrldr rurdr.rurr ls d pftyslcal pl.opeftv tnat can
rrrL rrrcrrrrrS PUrrrr, .rrE
l"'""'; _"': :"'::.,6
Deused to Identlty a molecule. The Ihe opti"cal,rotation
optical rotation is a measurement df of t'he rotalion ol pti""J
o1,.r nlan,,d
::,T:9P,i1""l,ilt: i?],"jq" the rotation
p,."hlt::1,!,qht bi a solution of the cipticatly active .o*po.rtld. A;.-;;;;;ppli;"ti;"'il;h;
ldentrtlcatron an unknown susar.
sugar.

Be able to disti ish ic substitution mechanisms.


There are two mechanisms for nucleophilic-substitution that you must know. fft" fitst ir tt-r" Slryf llili:li
and Sg2. They are
ancl the second is the S1x2. aredefined
deTined bv the number of leactants
ieactants in the rate cletermininc,
determining step
sfen ilit i:
of the mechanism. You.must 6e able io predict the reaction from the l.ltiri.."ailio"s, t"co"g"ir'"
the reaction from the intermediate or trahsition state, and identify the reaction it;;ih6 p;;f,;;r. )il[*
The differences include solvent, strength.of leaving group, sterii hindrance, ability to slabilize a lh*:
carbocation intermedia te, and stereoch"emistry. :!* -

iX!:tlt

Be able to recognize typical nucleophiles and leaving groups.


You must recognize what makes a go-od leaving grgup, and what effect this has on the reaction. The :tLlt
leaving group is dependent
strength of a lehvi d onthe solvent and can be qredicted from the acidity of the
conjugate acid of tlieleaving gtg*p:
p Equally, the strength of anuileopl-rile can be predictdd from r*
the basicitv
rne ot the nucleophilel
Daslclty of nucleoohilel Agpin, dolvaiion and st&ic hindrance
tllgli

a nucleophile. it"y;;;l";ih;;;"dil;l &r&


-[gfi!
$Ima:

|]t&i0![

ilUMll'r

tumrrn
Organic Chemistry Stereochemistry Introduction

stereochemistry involves the asymmetry of a molecule. we can consider the


asymmetry as a whole or the asymmetry about specific atoms in the molecule,
most often carbon. If there is asymmetry within a molecular structure, the
compound's reactivity, physical properties, and stability are all impacted. The
study of stereochemistry has direct implications in the biological applications of
molecules.
In this particular section we shall address the concept of configurational
isomerism and the many different classifications of configurational isomers. As
with ail isomers, configurational isomers have the same atoms within the
molecule, but they differ in some manner so that the molecules are not
superimposable on one another. No matter how a compound is rotated or
contorted, it is not superimposable on its configurational isomer. As a
consequence of their different configurations, one configurational isomer may
have the correct arrangement of atoms to offer minimal steric hindrance in a
chemical reaction with another asymmetric molecule, while another
configurational isomer proves to be too stericaily hindered on one side to
undergo reaction. This is frequently seen with enzymatic chemistry, where
enzymes have several stereogenic centers and are highly specific about which
configurational isomer can bind and undergo a reaction.
Configurational isomers can be categorized as either optical isomers or
geometrical isomers. optical isomers rotate plane-polarized light while
geometrical isomers are structures with limited rotation. In addition to that
categofization, configurational isomers can also be categorized as either
enantiomers or diastereomers. Enantiomers are nonsuperimposable mirror
images while diastereomers are nonsuperimposabie structures that are not
mirror images. The two categorizations are not mutually exclusive; meaning a
pair of configurational isomers could be enantiomeric optical isomers,
diastereomeric optical isomers, enantiomeric geometrical isomers, or
diastereomeric geometrical isomers.
In this section we shall also address nucleophilic substitution. We will consider
the two mechanisms for nucleophilic substitution: the S511-mechanism and the
Sp2-mechanism. In a nucleophilic substitution reaction that proceeds by an SN1-
mechanism, the leaving group leaves to form a carbocation intermediate before
the nucleophile attacks. An S511-reaction has a unimolecuiar rate-determining
step. In a nucleophilic substitution reaction that proceeds by an Sp2-mechanism,
the nucleophile attacks the electrophile from the opposite side of the leaving
group and forces the ieaving group off of the electrophile. An Sy2-reaction has
oniy one step, a bimolecular step. We will compare and contrast the conditions
and features of an Sp1-reaction with that of an S52-reaction to establish a set of
criteria you can use when deciding which mechanism (Sp1 or Sp2) is applicable
for a given nucleophilic substitution reaction.
We will consider the impact of stereochemistry on reactant interactions,
transition state formation, and product distribution. We will present the basic
tenant that if the reactants are optically active, then the product mixture is likely
optically active, and at the very least enantiomerically rich in one configurational
isomer (possible a geometrical isomer, which is optically inactive.) We will also
consider enantiomeric distribution in a product mixture and discuss ways to
increase the optical purity.

Copyright @ by The Berkeley Review r85 Exclusive MCAT Preparation


Organic Chemistry Stereochemistry Configurational Isorners

Confi gurational trsorners


Stereochemistry
Stereochemistry centers around the formation, orientation, and reactivity of
molecules with stereogenic centers, referred to as stereoisomers. The molecules
we shall consider in this section are configurational isomers.

Configurational Isomers
Configurationai isomers have identical bonds, but they have a different spatial
arrangement of their atoms, no matter how the structures are contorted.
Common examples, with which you are familiar, are optical isomers. Opticat
isomers, due to their asymmetry, rotate piane-polarized light. This is used as a
diagnostic test to identify a specific configurational isomer. We shall first look at
asymmetry and chirality, as configurational isomers are based on chirality.

Asymmetry
A molecule with asymmetry has a site about which there is uneven distribution
of the bonded atoms. Analyzing symmetry is critical, because at least one of two
stereoisomers must be asymmetric in some manner if the two structures are not
superimposabie. To understand stereoisomers, it helps to be familiar with mirror
plane symmetry and chirality (molecular asymmetry). Figure 3-1 shows the
asymmetry of carvone and the symmetry of 2,2-dichloropropane.

C1

-H5
Hsc CHs
No plane of symmetry, so Plane of symmetry, so
the compound is chiral. the compound is achiral.

Figure 3-1

Chirality
Chiral is the term assigned to a molecuie with no plane of symmetry, therefore a
chiral molecule has an asymmetric structure. Simply put, chirality is the "left
and right handedness" of a molecule. From our perspective (keeping at the leve1
of this test), a chiral molecule has at least one stereogenic center present. A
stereogenic center is an atom within the chiral compound^that has asymmetrr-
abor.rt it. For our needs, chiral (asymmetric) carbons are sf -hybridized carbons
with four unique substituents attached. Within 2-chloropentane there is one
stereocenter (chirai carbon), as emphasized in Figure 3-2 below. Figure 3-2
shows the two configurational isomers (enantiomers) of 2-chloropentane in such
a manner that the two structures are mirror images of one another. The plane
mirror reflects a configurational isomer as its image.

-;---- -'CHTCH2CH3
plane I '
mlrror I

'tLl

-_____;;__.v,1:
I-ug" cl :tl
sp3-hyb.idirotitor-t vith four different groups attached. &n
-;
Figure 3-2

Copyright O by The Berkeley Review The Berkeley Revieu


Organic Chemistry Stereochemistry Configurational Isomers

What makes this important is that an atom with four unique substituents
attached has two possible ways that the substituents may be connected (which
ate mirror images of one another). The two structures, mirror images that have
identical bonds, are stereoisomers that may exhibit different chemical properties
despite identical physical properties as one another. The biological ramifications
of chirality are important. For instance, humans digest only D-sugars (D refers to
one of the two possible stereogenic orientations associated with the penultimate
carbon within a sugar backbone), because enzymes bind and react only with D-
sugars. If this seems unclear, the following everyday analogies may help:
a) A soew with right-handed threads does not fit into a left-handed nut.
b) Your right hand does not fit rnto a left glove.
:) A key with a groove on only one side and its mirror image do not open the
same iock.
Exampie 3.1 shows some examples of pairs of butane moiecules that have four
'l1ique substituents on carbon two. The goal of the question is to develop skills
:or quickly recognizing when two structures represent conformational isomers
ciscussed in section II) versus when they represent configurational isomers.

Example 3.1
,ihich of the following structurai pairs represents the same molecule and not a
:air of configurational isomers?
\.

&
F{3CH2C F{rCHrC&I
H
D
D.
OH

&
irL
cH2cH3 ""-&rx,.,,,
!Br
i

-,./YcHrcH3& cH2cH3
CHg

D. cH.cH" CH"Cl
l'J t-
:ul2clY/H & H3cH2clY/H

! olution
- :u must decide whether the compournds are either enantiomers or the identical
,:r-Lpound with different spatial orientation. Rotate the molecules in your mind
' - see if the atoms overlay. If you do this successfully, then you will see that they

=:e identical only in choice C. However, if this is hard for you to visualize, try a
.=: of models. A shortcut you may recognize is that when two of the substituents
':e interchanged, the chirality of that stereocenter changes. In choices A, B, and
l' two substituents have interchanged, making them enantiomeric pairs.

-:pyright O by The Berkeley Review r85 Exclusive MCAT Preparation


Organic Chemistry Stereochemistry Confi gurational Isomers

We will start with a traditional approach to stereochemistry probiems and then


slowly work our way into shortcuts and visualization tricks. The first pair of
molecules in Example 3.1 are mirror images of one another, because if you rotate
either structure by 180" about its carbon 1 - carbon 2 bond, the structures are not
superimposable. According to our shortcut, when tlvo of the substituents
interchange their locati.ons (H and I in this case), because there is only one chiral
center, the tr,r'o molecules are mirror images of one another. Mirror images that
are non-superimposable are defined as enantiomers. The substituent exchange
shortcut shoulcl make it easier to recognize enantiomers.

The molecules in choice C in Example 3.1 are identical. If the left structure is
rotated counterclockwise by 120" about the carbon 2 - carbon 3 bond (as shown in
Figure 3-3), the identical structure and orientation are formed. Note that when a
structure is rotated by 720" about a bond, the three other substituents interchange
their locations on the molecule. The conclusion from this is that when three
substituents are different from one structure to another, those two structures
represent different conformational isomers (orientations) of the same compound.

C1
I

zGH:- BrlYcH2cH3

Figure 3-3

Rotating molecuies in your mind becomes easier with practice, although if the
ski11 is never fully developed, you can stil1 answer stereochemistry questions br
following a few simple rules. If two groups are interchanged and the rest of the
molecr,rle remains in place, then the two structures are configurational isomers. If
there is a mirror plane between two molecules and no mirror plane within the
molecule, then the two structures are configurational isomers.

In addition to the symmetry of a compound with respect to another molecule. lx

there is also internal symmetry to consider. For internal symmetry, you can look :
;
for either a plane within the molecule that reflects equal halves, or an inversiorr.
point. An inversion point is a point at the molecule's center of mass through 1i

which a line passing through that spot will intersect the same atom at the same l
lr
distance, no matter which direction you proceed (positive or negative direction
on the 1ine. Within asymmetric molecules, each stereogenic center is assigned a s
letter, R or S, to describe its stereochemistry.
llr ;

Determining Absolute Configuration "i


The identification and naming of a chiral center is based on nornenclature
convention. There is a set of guidelines, the Cahn-Ingoid-Prelog rules, to follol '11
I

for determining R and S for a stereocenter (chiral carbon). The Cahn-Ingold- J


Preiog rules to determine the stereochemical orientation (R or S) are as follows: :tr"r

1) First, you must prioritize (from heaviest to lighiest) the substituents that are $."
attached to the carbon of the stereocenter according to the atomic mass of the l:t

atom directly bonded to the chiral carbon. (Get it from the periodic table) lill lr ,

2) Second, you must orient the molecule in such a way that the substituent wil;. iut!1,

priority number four points behild the plane of the molecule'


3) Third, you must draw a circular arc from substituent 1 through 2 and on to l zu.

If the arc is clockwise, it is referred to as R. If the arc is counterclockwise, it r.


;i.*
referred to as S.

Copyright O by The Berkeley Revierv ra6 The Berkeley Revieu 'l,r


Organic Chemistry Stereochemistry Configurational Isomers

R is from the word rechts, which is right in Latin, while S is from the word sinister
which is left in Latin. If you point your thumb in the direction of substituent
number four on a compound with R-stereochemistry, the fingers of your right
hand will curl from one to two and on to three. Thus, R-chirality can be thought
of as right handedness. The same holds true for your left hand with an s-center.

Figure 3-4 shows a generic R molecuie and a generic s molecuie as oriented


according to convention. The steps of rotation presented, take the structure from
a standard view to a view with the fourth priority substituent eclipsed, to the
Newman projection from which the stereochemical identity is derived.
1
Reposition by
rotating by 90" about Redraw to
C.6irot-C1 bond see chirality
_______>

Clockwise: R

3
Reposition by Redraw to
rotating by 90' about see chlrality
C.51.o1-C1 bond
---------->

Counterclockwise: S

Figure 3-4
Knowing the terminology is key; it is recognition, not recail, that is emphasized
on a multiple-choice exam. Here are some nndified definitions of common terms.
Chiral Center (stereochemical center):
A carbon with four unique substituents attached. Any carbon with four unique
substituents has two different orientations that it can assume (R and S). What is
neant by "unique substituents" is not four different atoms, but four unique
$oups including the atoms attached to the four atoms bonded directly to carbon.
For example, carbon two of 2-chloropentane (see Figure 3-2) is chiral, because it
ras a chlorine (priority 1), a propyl group (priority 2), a methyl group (priority
it, and a hydrogen (priority 4)bonded to it. These four substituents are different
:rom one another, therefore they are four unique substituents.
R-center:
-\ carbon center that when you look down the bond from the chiral carbon to the
:ourth priority substituent (usually a C-H bond) in a way that you can't see the
iourth priority substituent, the remaining substituents form a clockwise arc
,r-hen moving from priority one to priority two and on to priority three according
io the priority rules. This can be thought of as a right-handed molecule when
llacing your thurnb in the direction you're looking and curling your fingers to
natch your right hand to the structure.
S-center:
-1,carbon center that when you look down the bond from the chiral carbon to the
:ourth priority substituent (usually a C-H bond) in a way that you can't see the
:ourth priority substituent, the remaining substituents form a counterclockwise
:rc when moving from priority one to priority two and on to priority three
:ccording to the priority rules. This can be thought of as a left-handed molecule
'";hen placing your thumb in the direction you're looking and curling
your
:rngers to match your ieft hand to the structure.

Copyright @ by The Berkeley Review ta7 Exclusive MCAT Preparation


Organic Chemistry Stereochemistry Configurational Isomers

Prioritizing Substituents to Determine R and S


To prioritize, first you must look at the four atoms directly bonded to the
asymmetric carbon. You then rank those atoms according to their atomic mass
with the heaviest atom taking ihe highest priority. if two atoms are equal (as is
often the case n,ith carbon) you must continue down the molecule following the
bonds outward from the chiral center. Figure 3-5 shown below presents
examples with the priorities labeled on the molecules.
23
!n'
3

f',
Ir Tt' T"'
n4$'r n, H,cH,tA';;, "'t-t. "tY:.d
sI
1
"r'*'it ::,"=
I>Br>C>H O>C>D>H Br>C=C>H C-C-C-C
CH2CH3t CHs C=O > C=C > C{ > C-H

Figure 3-5

Shortcut to Determine R and S


As with so many other topics in organic chemistry, such as nomenclature, R and
S questior-rs become easy and redundant with time. Once they become easy,
there are useful quick tricks to help you to identify chiral centers as being either
R or S. For instance, when the fourth-priority substituent is sticking out from the
rnolecule, the molecule must be rotated. To save time, it is easiest to first solve
for the arc using the structure as it is, and then take the opposite chirality for the
center. In the interest of saving time, this works well for use on tl're MCAT.
Substituent number four can either be behind the plane, in front of the plane or
in the plane. In each case, there is a technique to apply to arrive at the chiral
center easily. Many techniques shall be presented, so choose your favorite.
Figure 3-6 shows how to get the chirality easily when the structure is drawn irr
the conventional manner.

'(')t4
,l,.i(f-},.t-,, H?CH2 H"

t, Ho H
H in back of plane .'. H in front of plane .'.
Take whai you observe as is. Reverse what you observe.
Clockwise Arc = R Clockwise Arc reverses to S

iY)
H3CH

1
HO CHs
H in plane close to back group .'.

Take what you observe as is. Reverse what you observe.


Ctlunterclockwise Arc = S Clockwise Arc reverses to S

Figure 3-6

Copyright @ by The Berkeley Review The Berkeley Review L{._


Organic Chemistry Stereochemistry Configurational Isomers

Example 3.2
The following molecule has what type of chiral
- orientation?
H"C
,\
\
cH2cH3
Ho$tyc
H
A.R
B.S
C. The molecule has no chiral center.
D. The compound is meso.
Solution
The compound has one chiral center, so it cannot be meso (to be meso requires an
even number of chiral centers). The compound is chiral, because carbon two has
four different substituents attached it. The molecuie is therefore either R or S.
The priorities are oH > CH2CH3 > CH3 > H. Correct arignment of the
substituents shows that the compound has an R chiral center. A
counterclockwise arc connects priorities 1,,2 and 3. Because the H (priority 4) is
in front, the arc should be reversed. Pick A for best resuits.
Priority +S HsC
Priority #2
CH2CH3 Priority #4 in front
Priority#1 HO\\) Counterclockwise = R
H
Priority #4

Example 3.3
r'\4rat is the chirality
of the triol below according to the Cahn-Ingold-preiog rules?
o CH"OH

OH OH
{. 2R,35,45,6R
B. 2R,3R,4R
C. 2R,3R,43
D. 2R,35,45
Solution
larbon six is not a chiral center, because there are two methyl groups present
lhis eliminates choice A. The chiral centers are 2R, 35, 45 (choice D), as shown.
o o CH"OH o cH2oH
V< 4S

oH oH CHs
Clockwise Arc and #4 Counterclockwise Arc and Clockwise Arc and #4
in back .'. take as R #4 in back .'. take as S in front ,'. reverse to S

-,:pvright @ by The Berkelev Review ra9 Exclusive IvICAT Preparation


Organic Chemistry Stereochemistry Confi gurational Isomers

Example 3.4
Which of the foilowing compounds have R orientation?

C1
HgN* HOH2C
I
H
nrcnrc4/;cn, H CH2OH HOH
2

Compound I Compound II Compound III


A. Compound T only
B. Compound II only
C. Compound III only
D. Compounds I and III only

Solution
Hydrogen points out in each of the compounds. Whichever arc is seen from this
view must be reversed to get the arc that would be seen from the correct view.
The priorities in Compound I are: Ci > CH2CHg > CHe > H. Compound I has an
R chiral center. The priorities in Compound iI are: NH3+ > COZ- > CH2OH > H.
Compound iI has an S chiral center. The priorities in Compound III are: OH >
CHO > CH2OH > H. Compound III has an R chiral center. Choice D is best.
Compound I Compound II Compound III
(2-chlorobutane) (Serine) (D-Glyceraldehyde)
tcl o

3
HOH2
2
o-
H3CH2 H3

Counterclockwise Arc and Clockwise Arc and #4 Counterclockwise Arc and


#4 in fiont .'. reverse to R in fiont .'. reverse to S #4 in front .'. reverse to R

Example 3.5
What is the stereochemical orientation of the foilowing molecule?

"\-
H .CH,
Eil
r,.J)y'-'a.,
A. 2R,3R
B. 2R,35
C. 23,3R
D. 2S,3S

Solution
For the first chirai center (carbon 2), the fourth priority (hydrogen) is in the plane :i
close to the group in back (a reversing position). An arc from priority one to two
and on to priority three is counterclockrvise. However, because H is in a reverse -s
position, the chirality is R. The second chiral center (carbon 3) has H in front of
the plane, so it is in a reversing position too. An arc from priority one to two and lt*
on to priority three is clockwise. Horr-ever, because H is in a reverse position, the i,:
chirality is S. The best answer is thus 2R, 35 which makes choice B correct. Il

lr:

Copyright O by The Berkeley Review r90 The Berkeley Review


Organic Chemistry Stereochemistry Configurational Isomers

Figure 3-7 shows a summary of the tricks presented in Figure 3-6 and applied in
Examples 3.2 through 3.5.
If Priority #4 is in the plane far from If Priority #4 is in the plane near
group in back, then take arc as is group in back, then reverse arc.

If Priority #4 is in front, If Priority #4 is in back,


reverse the arc to opposite. then take arc as is.

Figure 3-7

An alternative short cut in determining the chirality of a compound involves


interchanging substituents to generate an easy structure to solve, and then
assigning the opposite chirality to the original molecule. This method is based on
the idea that it is easiest to solve for chirality when the fourth priority substituent
is in back and that when two substituents are interchar-rg"a, tn" chirality is
rnverted. Figure 3-8 shows the application of this method.

HOH2C< cHs HoH2r< H.J

'@ , lHtz
'HO H*
Priority #4 not in back ... Priority #4 now in back of plane .'.
interchange #4 and group in back New structure has a ciockwise arc, so it is R
The original compound must be S

CH"OH H2OH

Priority #4 not in back ... Priority #4 now in back of plane .'.


lnterchange #4 and group in back New structure has a clockwise arc, so it is R
The original compound must be S

Hgcl Hscj 1oH

, 2
HOH2C HOH2c
Priority #4 not in back .'. Priority #4 now in back of plane .'.
lnterchange #4 and group in back New structure has a clockwise arc, so it is R
The original compound must be S

Figure 3-8

These are two-dimensional tricks that may be done on paper. There are other
ticks that involve contorting your hand to model the molecule. No one method
is more accurate than another is, so once you find the one you prefer, hone it in
and use it. We will cover the three methods in class. One involves pointing in
the direction your eye should be looking, and forming an arc that goes from
oriority 1 to 2 to 3. Another involves using your fingers to represent the bonds in
the molecule. And as mentioned earlier, one method involves matching your
thumb and curling fingers to a molecule.

Copyright @ by The Berkeley Review l9r Exclusive MCAT Preparation


Organic Chemistry Stereochemistry Confi gurational Isomers

Optical Rotation
Optical rotation is a physical measurement of the rotation of plane polarized light
by a solution with a chiral molecule. A soiution containing a pure compound of
known concentration (dissolved into solvent) in a standardized cuvette rotates
piane-polarized iight the same amount each time. Consequently, the specific
rotation (optical rotation under specific condiiions) is a physical measurement
(like melting point and boiiing point) that may be used as a diagnostic test for the
identity of a compound. This is common with sugars which have multiple chiral
centers. The direction of the rotation is signified by either (+) or (-) orientation
foliowed by the degrees of rotation. The (+) denotation describes clockwise
rotation of light while the (-) denotation describes counterciockwise rotation of
light by the molecule. If the R-enantiomer of a compound lotates the light in a
positive direction by X", then the S-enantiomer of the cornpound rotates light by
X' in the negative direction. The measurement is taken with a polarimeter which
is made from a sample tube sandwiched between two polarizing plates' The
plates are rotated in a manner to allow for the greatest amount of iight to pass
through. If the plates are rotated 90' away from the orientation with highest
intensity, then the new orientation of the plates does not allow for any light to
pass through. A sample polarimeter is shown in Figure 3-9 below.

Solution of Chiral Compound

light enters light rotates rotated light exits

z porarlzet
^nd

Figure 3-9

Chiral molecules may be assigned a "+" or "-" preceding their name to indicate
the direction that the compound r,r'il1 rotate plane polarized light. For instance.
(+)-2-butanol rotates tight in a clockwise direction while its mirror image, (-)-2-
butanol, rotates light in a counterclockr.r'ise direction. There are compounds with
R-stereochemistry that rotate plane polarized light in a clockwise fashion anC
other compounds with R-stereochemistry that rotate plane polarized light in a
counterclockwise direction. Consequently, R stereochemistry does not
necessarily correspond to either (+) or (-) rotation of plane polarized light. Ir
biochemistry, the designation of D and L is based on threose, where (+) was
originally assigned to D-threose and (-) rvas assigned to L-threose. Flowever.
because of the vast multitude of sr-rgars, there is no correlation between D and L
designation and (+) or (-) rotation oi pl3ns polarized light.

Copyrighi O by The Berkeley Review t92 The Berkeley Revieu


Organic Chemistry Stereochemistry Configurational Isomers

Having chiral centers does not always result in the rotation of plane polarized
light. Meso compounds have opposing chiral centers that cancel one another out,
resulting in no net rotation of plane polarized light. Meso compounds are
therefore opticnlly innctiue, meaning they have a specific rotation of zero.

Polarimeters measure the optical rotation of a soiution. The specific rotation of a


compound, [a]p, is calculated from the optical rotation using Equation 3.1.

observed rotation in degrees


[o]5 = (3.1)
(length of sample in dm)(concentration of sample it-r ItuTt)
mL
As seen in Equation 3.1, the standard cell length of a cuvette is 10 centimeters
(one decimeter), and the standard concentration is 1 gram solute per milliliter
solution. The superscript T refers to temperature as measured in Celsius and the
subscript D refers to monochromatic light from a sodium lamp, known as the D-
band. Specific rotation is the optical rotation observed under specific conditions.
If the solution is too concentrated, then the rotation of tight is greater than it
should be. If the solution is less concentrated than standard conditions, then the
rotation of light is less than it should be. For this reason, the observed optical
rotation is converted to specific rotation to determine purity.

Types of Configurational Isomers


Until now, we have focused on the chirality of molecules and their stereogenic
centers. A moiecule is a chiral molecule when it is asymmetric. However, often
times, a molecule can be asymmetric in more than one way. There are terms that
describe the relationship between two stereoisomers. The relationship requires
determining whether the structures are mirror images and whether they are
superimposable. Configurational isomers can be classified as either enantiomers
or diastereomers. Enantiomers and diastereomers have the same bonds, but a
different spatial orientation of their atoms. Enantiomers are configurational
isomers that are nonsuperimposable mirror images (reflections that you can't
overlay). Diastereomers are configurational isomers that are nonsuperimposable
and that are not mirror images. Thus, to classify a pair of configurational isomers
as either enantiomers or diastereomers requires evaluating whether the
structures are mirror images and whether they are superimposable.

There is a wonderful short cut for determining whether two compounds are
enantiomers or diastereomers. Consider a molecule that has two chiral centers, R
and R. To be mirror images, ail of the chirai centers must differ, because each
chiral center must switch when it is reflected (just as a left hand in the mirror
turns into a right hand). If you were to place an R, R molecule into the mirror it
rvould reflect an S, S molecule, so the S, S molecule is the enantiomer of the R, R
molecule. This means that to be enantiomers, all of the chiral centers must
differ between the two configurational isomers. If no chiral centers differ, then
the two structures are identical (the same molecuie). If one of two stereocenters
differs, then the two compounds are neither mirror images of one another nor the
same molecule. This makes the two compounds diastereomers. If only a few,
but not all chiral centers differ, then the compounds are diastereomers. Listed
below are the modified definitions of enantiomers and diastereomers.

Enantiomer; Enantiomers are configurational isomers in which all of the chiral


centers in each molecule are different from one another.

Diastereomer: Diastereomers are configurational isomers in which at least one,


but not all of the chiral centers in each molecule is different from one another.

Copyright @ by The Berkeley Review r93 Exclusive MCAT Preparation


Organic Chemistry Stereochemistry Configurational Isomers

These modified definitions should prove to be easier to use than the traditional
definitions. Recall how these definitions were derived and they will be easy to
remember and apply. Figure 3-10 shows two pairs of enantiomers and Figure 3-
11" shows two pairs of diastereomers.

HgC. HsC cH2cH3


\J /cF{2cH3

8,,'f Vott
Enantiomeric pair because both'of the chiral centers are different

I
rl

(
& u
H'v M

M
ffi
{m
Enantiomeric pair because both bf the chiral centers are different
s
@
Figure 3-10
lt

cH2cH3 n
m
&

Diastereomers, because only one of the chiral centers (the left one) is different
A,
n
0
ocH2cH?
OCH"CH"J
-.--
^
Ht rt

Diastereomers, because only one of the chiral centers (the right one) is different

Figure 3-1L

After practice (and thus on your exam), you should be able to just scan structures
to look for interchanged substituents (chiral centers that are different.) First, look
for any chiral centers (sf-carbons with four unique substituents). From that
point, compare the comparabie chiral centers in the two structures. If ttr
structure is oriented in a similar fashion, but two substituents are in differsfr
positions, then the chiral center is different between the two compounds. If ttrc
structure is oriented in a similar fashion, but three substituents are in differeril
positions, then the chiral center is the same between the two compound.*
Finally, it is a matter of deciding i-f a1l, some, or none of the chiral centers on tlu
two molecules are different and then determining their relationship.

Copyright @ by The Berkeley Review t9,4 The Berkeley Revien


Organic Chemistry Stereochemistry Configurational Isomers

Example 3.5
Ihe following pair of compounds is best described as which of the following?
cl. H1C
\-J
c(cH3)3 C(CH.):
" '
\___./
H.,c$\'/ Yrort
HH "vCi
\r"
OH
A. Diastereomers
B. Enantiomers
C. Identical achiral compounds
D. Identical chirai compounds
Solution
\Vhen the orientation of the molecule remains constant and three substituents
:hange their location, this implies that the compound has been rotated about that
chiral center. The left chiral center is just rotated between the two compounds,
thus it has the same chirality. When two substituents interchange their location,
this implies that the chiral center changed. The right chiral center has changed,
because the H and aldehyde group have interconverted. This means that only
one out of the two chiral centers has changed its orientation between the two
structures. The two compounds are therefore diastereomers of one another,
naking choice A the best answer.

Example 3.7
F{ow can the relationship of the following pair of molecules be described?

CHs
-{. Diastereomers
B. Enantiomers
C. Identical achiral compounds
D. Identical chiral compounds
Solution
ln this example, the mirror plane between the two molecules can be seen easily as
lhey are drawn. So without rotating or counting chiral centers, the two
:ompounds can be identified as enantiomers of one another. Enantiomers are
nonsuperimposable mirror images. Choice B is the best answer.

Example 3.8
The following pair of isomers is best described as which of the following?

HO HHO.H
\-,)cH, &
Hrcrsf \ f",.",
H OCH3 ocH3
A. A pair of anomers
B. A pair of constitutional isomers
C. A pair of diastereomers
D. A pair of enantiomers

Copyright @ by The Berkeley Review r95 Exclusive MCAT Preparation


Organic Chemistry Stereochemistry Confi gurational Isomers

Solution
The two structures are not aligned in an equivalent fashion, so one of the trt'o
structures must be rotated into a structure equivalent to the other structure.

A H\

J"; #'
OCH?
Itwo 60' rotations) "t.&--J"
/ poc",
H CHe

H"C H
ro'&_/- -z &
/ \ao.tt,
H CHs H ocH3
After rotating the structure, it is easier to see that the left chiral center has tn'c
substituents thut huu" interchanged between the two structures, thus it ha-' (I

changed chirality. The same is true for the right chiral center after rotation, thus I
it toJhas changed chirality. This means that both chiral centers have changed, sc
the compoundi are e.rut iio*"ts. There is no need to determine the chiralitv c:
u

u
the stereocenters (R or S) within molecules to determine whether they are r(
enantiomers or diastereomers. Deciding whether two molecules are enantiomers u
or diastereomers is as easy as asking whether aII of the stereocenters or just sonr
of the stereocenters have changed their orientation between the two compounds'
(All centers differ = enantiomers; Some centers differ = diastereomers) L
T

Example 3.9
The following molecules are best described as:

Hsc CHs HeC CHs

Br H & Br H

H CI C1 H

CHc
A. diastereomers.
B. enantiomers.
C. identical achiral comPounds.
D. identical chiral comPounds.
Solution
The structures are drawn as Fischer projections, which represent the top vierr
the all eclipsed form of the molecule. In a Fischer projection, the side gloups
coming olrt ut you in the three dimensional perspective,. The two isomers h
two c(iral ."r-tiuru. Only the chiral center with the chlorine is different r'r'
comparing the two compounds. The chiral carbon with bromine has
changed, t".urrr" the subitituents have not moved' Only one out of the
chiraf centers di{fers, so the compounds are diastereomers. This makes choice
the best answer.

Copyright @ by The BerkeieY Review r96 The BerkeleY


Organic Chemistry Stereochemistry Configurational Isomers

Example 3.10
Ihe physical properties of the follolving stereoisomers relate in what way?
cH?oH

H.c{ou CH:

Br-+H & CH:


I

CHs
\. Same boiling point; different melting points
B. Same density; different boiling points
C. Different boiling points; different melting points
D. Different densities; same boiling point
Solution
The chiral center on carbon 2 is different between the two structures, because the
CH3 and OH are interchanged. The chiral center on carbon 3 is not different
:etween the two structures, because the H, CH3 and the Br have all
,nterconverted. When three substituents interconvert, the chiral center is not
:hanged. This means that one out of two chirai centers differ, so the two
:ompounds are diastereomers. Diastereomers have different physical properties
rcluding melting point, boiling point, and density. Pick C for best results.

Example 3.11
lhe following pair of isomers is best described by which of the following terms?
H:cr'''1)''\$oH
r,.r,"'Q ",tt aH
\. Anomers
B. Diastereomers
rl. Enantiomers
D. identical meso compounds

Solution
-ne two compounds are mirror images of one another, so they are enantiomers,
:.::oice C. To see the mirror plane relationship, one of the compounds must be
::tated 180'. The drawing below shows the right structure berng rotated.

nr.r,,O..rr\oH
"r@
t""O"rr\oH "or'O".,,tCHg
I
nirror ima

- -.pyright @ by The Berkelev Review Exclusive MCAT Preparation


Organic Chemistry Stereochemistry Confi gurational Isomers

Meso
Meso compounds are individual structures which contain a mirror plane slicing
through the middle of the compound and an eaen number of chiral centers
symmetrically displaced about the mirror plane. The net optical rotation of a
meso compound is 0". It is zero, because the opposing chiral centers on each haU
of the molecule cancel one another out, Ieaving no net rotation of plane polarized
light. Meso compounds are referred to as optically inactiae. Remember the
phrase, "Me so inactive", a high-energy rap lyric that describes your physicai
state while studying for the MCAT. Figure 3-12 shows a meso compound (it has
been rotated into a side view to see the mirror plane more easily).

top view side view

Figure 3-12

A meso compound may be identified by an inversion center in the middle of the


molecule. Figure 3-13 shows two conformational isomers of a meso compound.
one in its most stable conformation (where it has an inversion point), and the
other in its least stable conformation (where it has a mirror plane of symmetry.
A meso compound has the same number of R-stereocenters as S-stereocenters.
Inversion point

CHa
I!
:
Figure 3-13

Example 3.12
The reflection of a meso compound can be classified as which of the following?
A. Identical to the original compound
B. An enantiomer of the original compound
C. A diastereomer of the original compound
D. An ameso compound from the fifth dimension where evil lurks and the sock m

,[
that disappear from laundry loads in our world gather.
t
Solution
5
A meso compound when viewed in a mirror reflects the identical compounc
This makes choice A the best choice, although choice D is a close second. A:

"u"l H
example of a meso compound and its reflection is drawn below. 'n

HOO H "aI

.i:
:
\,/ ln
mirror plane tr
--t

Copyright @ by The Berkeley Review 198 The Berkeley Revieu .


Organic Chemistry Stereochemistry Configurational Isomers

Stereoisomerism
As mentioned before, stereoisomers are compounds that have identical bonds
but their atoms differ in spatial orientation. \Mhen a molecule contains more than
one chiral center, the maximum number of stereoisomers increases exponentially
with each new chiral center according to the equation 2n, where n is the number
of chiral carbons in the molecule. There are less than 2n stereoisomers, if one of
the possible structures is meso. If there are an odd number of chiral centers, the
structure cannot be meso, so there is exactly 2n possible stereoisomers. For
example, consider the compound 3,4,5-trimethyloctane, which has three chiral
centers and thus eight possible stereoisomers. 3R, 4R, SR-trimethyloctane is just
one of the eight possible stereoisomers. Table 3-1 shows the possible
stereoisomers for compounds with a variable number of chiral centers.

Chiral Centers Maximum Stereoisomers


Stereoisomers
1 2 RorS
2 4 R& RS, SR, or SS

3 8 RRR, SRR, RS& RRS, SSR, SRS, RSS, OT SSS

Table 3-1
lVe are most concerned with stereoisomers when a molecule contains more than
rne chiral center. If there is only one chiral center, there cannot be diastereomers.
Stereoisomerism is important in biological sciences, because only a very few
riological compounds have just one chiral center (some amino acids being
amongst of these few) with some proteins having in excess of 200 chiral centers.

E-xample 3.13
I{ow many stereoisomers are possible for the following structure?
OH OH

o CHs CH:
-\. 4
ts.8
c. 72
D. 16

Solution
lhe molecule is drawn in a way to make you mistakenly see four chiral centers if
- ou don't pay close attention, but the number of chiral centers is only three.

larbon five is not a chiral center, because there are two methyl substituents
::'iached to it (one that is drawn as a methyl substituent and the other that is
j:awn as carbon six of the longest chain). By having two methyl groups
.::ached, it does not have four different substituents attached, thus it is not a
::riral carbon. This means that only carbons two, three, and four are chiral. The
:-aximum number of stereoisomers is derived using the equation 2n, which is
::rs case is 23. Because there is an odd number of chiral centers, the compound
::mot be meso, so there are eight stereoisomers. The correct answer is choice B.

-:,:vright @ by The Berkeley Review r99 Dxclusive MCAT Preparation


Organic Chemistry Stereochemistry Configurational Isomers

Example 3.14
The relationship of the following pair of compounds is best described as:

CHs OH
A. Anomers
B. Diastereomers
C. Enantiomers
D. Structural isomers

Solution
The hydroxyl group is attached to different carbons in the two structures. In the
left structure the hydroxyl group is on carbon 4 while in the right structure, the
hydroxyl group is on carbon 2. This makes the two compounds structural
isomers, which makes choice D the best answer. you may recall that if the two
compounds have different IUPAC names, then they are structural isomers. By
virtue of the hydroxyl being in a different position, the two compounds in
question have different IUPAC names.

Example 3.15
Which of the following compounds islare optically inactive?
L 2R,3S-dibromopentane
II. 2S,3R-dichlorobutane
III. 1R,2R-diiodocyclopentane
A. Compound I only
B. Compound II only
C. Compound III only
D. Compounds I and III only
Solution
To be optically inactive, the compound must either be achiral or meso. All of the
compounds iisted have stereocenters, so achiral is not a possibility. The questio:. :
is whether or not each structure is meso. To be meso, the compound must b=
symmetric and have an even number of chiral centers equally displaced abou:
T]f
the internal mirror plane (i.e., R on one side and s on the other). Compound I s
eliminated, because it is not symmetric about the a plane (the mirror plane wou.l; 1l!18

have to slice through carbon three to break the five carbon species into halves
To be symmetric, carbons two and four would have to have the sam" mrt

substituents, which they do not. Compound I is chiral, so it is optically acti,,'= ,&-


and therefore eliminated. Compound III is eliminated, because it does not havr il"
opposite chiral centers (they are both R). To be meso, carbons one and trt: C.
would have to have opposite absolute configuration, which they do nc: l!i't

Compound III is chiral, so it is optically active and therefore eliminated. Tl:


only choice left is Compound II, which is symmetric, because it has a mirrir
plane that slices through the bond between carbon two and carbon three. Th:
best answer is choice B.

Copyright @ by The Berkeley Review The Berkeley Revie*


Organic Chemistry Stereochemistry Stereochemistry in Reactions

Stereoisomer Formation
The utility of stereochemistry lies in the selectivity of chiral reactants for one
another. \Mhen reactions involve chiral reactants, they are often selective for one
stereoisomer over another. This is a staple of enzymatic selectivity. Howevet,
when a reaction involves reactants without any chirality, the formation of
stereoisomers is random and follows basic probability. Most reactions in organic
chemistry produce stereoisomers. The type of stereoisomers formed depends on
the chirality of the starting reagents. \44ren a symmetric nucleophile can attack a
planar species from either side, there usually are two enantiomers formed in
equal proportion. When a symmetric nucleophile is hindered from attacking a
planar species from one side more than the other (due to a chiral center in the
electrophile that creates greater steric hindrance on one side than the other), there
are two diastereomers formed in unequal proportions. Stereoisomers can result
from electrophilic addition reactions with alkenes as well as substitution
reactions. Figure 3-14 shows an example of an electrophilic addition reaction
that forms two enantiomers.
.. O O..
o
ll o
t,to\ t t\ St",
l"o
j*", &
,rrc'c\nrcr,, .rC-. ,C-.
HsC CH2CH3 HsC CH2CH3
Planar carbonyl I

l nocu"
HO. OCH,
""\ c-.ft &
\l'._-, I "

z ,rC-.
HsC cH2cH3 HsC cH2cH3
Enantiomers

Figure 3-14

Example 3.16
The addition of alkyl magnesium bromide (RMgBr) to a carbonyl in ether adds a
:iew alkyl substituent to the carbonyl carbon, resulting in conversion of the
:arbonyl into an alcohol. The addition of H3CMgBr to R-2-methylcyclohexanone
n diethyl ether yields which products?
-A.. One meso compound
B. Two diastereomers
C. Two enantiomers
D. Two epimers

,opyright @ by The Berkeley Review Exclusive MCAT Preparation


Organic Chemistry Stereochemistry Stereochemistry in Reactions

Solution
In this reaction, the methyl group can add to either the top or bottom of the
planar carbonyl group. This results in a new chiral center that can be either R or
S. However, there is already a chiral center present in the reactant that is not
involved in the reaction, which retains its original chirality. The chiral center
present in the reactant does not change during the course of a reaction, so the
products cannot be enantiomers. This eiiminates choice C. The compound is not
a sugar, so choice D iseliminated. It is not meso, so choice A is eliminated. One
of the two chiral centers differs between the two stereoisomer products, so thev
are diastereomers. Choice B is correct. Due to steric hindrance from the ethll
group on carbon two, the product mixture of the two diastereomers is not 50/50.
o HO

0 .*rCHe
+ HrCMgBr -----> .'rrCHs .,tfCHe

When two enantiomers are formed, they are formed in equal quantity, and the
product mixture is said to be rncemic. When the two enantiomers are equalh-
present, there is no net rotation of plane poiarized light. When tu'o
diastereomers are formed, they are formed unequally, so the product mixture has
a major and a minor product. When the two diastereomers are present ir.
unequal amounts, there can be a net rotation of plane polarized light.
Enantiomers can be formed in an unequal ratio if a chiral catalyst is present. This
:
leads to the concept of enantiomeric excess, used to analyze product distributions E
from reactions with a chiral catalyst (most often an enzyme).
:i:
.4_
Example 3.17
ffi
\Atrich of the following reactions produces no optically active compounds? ,I
A. 2-Butanone treated with NaBH4 in ether followed by acidic workup. D
B. (Z)-Z-butene with KMnO4 in base (sgr addition of two hydroxyl groups).
C. Reduction of HN=C(CH3)CH2CH3 using LiAiH4 in thf solvent. i:
D. S-2-Bromobutane treated with NaCN in ether solvent. :
::i
Solution
For a compound to be optically inactive, it must either be meso or achiral. In
choice A, NaBH4 adds a hydrogen to the carbonyl carbon from either side,
resulting in a racemic mixture of alcohols. Choice A is eliminated. In choice B,
KMnO4 adds a pair of hydroxyl groups to ihe alkene carbons to from a syn
vicinal diol. Because the alkene is svmmetric to begin with, symmetric addition
results in a symmetric product. The product is a meso diol, so choice B is the
best answer. In choice C, LiAIH4 adds a hydrogen to the imine carbon from
either side, resulting in a racemic mixture of ami.nes following workup. Choice C
is eliminated. In choice D, a good nucleophile attacks an alkyl halide, resulting in
inversion of stereochemistry. One chiral species is formed, so choice D is out.

Choice B\ /
\-*/ + KMnOr
Symmetric about plane
Symmetric -+
additioi-r HO OH HO -t' OH
n
Symmetric product: meso vicinal diol

Copyright @ by The Berkeley Review The Berkeley Review


Organic Chemistry Stereochemistry Stereochemistry in Reactions

The only way to get a meso compound from an addition to an alkene, is to have a
symmetric addition to a symmetric alkene or an asymmetric addition to an
asymmetric alkene. The best answer is choice B. Choices A and C involve
racemization which is common when a nucleophile attacks an sp2-hybridized
carbon. Choice D is the result of an SN2-reaction, which inverts the chirality.
Because the compound starts optically active, inversion generates an optically
active product.

Enantiomeric Excess
Enantiomers rotate light in opposite directions of one another, but with equal
magnitude. When both enantiomers are present in equal quantities in soiution (a
50-50 mixture), the solution exhibits no net rotation of plane-polarized light.
Based on this idea, when a mixture is not in a 50-50 ratio, then the net rotation of
light by the solution is not zero. The farther the value deviates from zero, the
greater the difference in concentration of the two enantiomers. From the
observed rotation of the solution, the percentage of the enantiomer in excess can
be derived. Equation 3.2 shows how to determine the enantiomeric excess from
the observed specific rotation. The enantiomeric excess is the difference in
percentage between the more abundant enantiomer and the less abundant
enantiomer.

measured specific rotation


'oee (enantiomeric excess) = x 700% (3.2)
specific rotation of the pure enantiomer

Example 3.18
i\4rat enantiomeric distribution would account for a specific rotation of +13.6' if
',Jre pure enantiomers have specific rotations of +27.2" and -27.2' respectively?

-\. (+)-enantiomer =25ok and (-)-enantiomer =75%


B. (+)-enantiomer = 33oh and (-)-enantiomer = 67.h
C. (+)-enantiomer = 67oh and (-)-enantiomer = 75%
D. (+)-enantiomer =75o/o and (-)-enantiomer =ZS%

Solution
Because the net rotation is positive, the (+)-enantiomer must be in higher
concentration than the (-)-enantiomer. This eliminates choices A and B. To
determine the exact quantity, Equation 3.2 canbe applied.

"/oee =+13'6 x 100% =!x 100% = 50'/oinfavorof the(+)-enantiomer


27.2 2
The exact ratio is found using the following reiationship:
'i'(+)-enantiomer + %(-)-enantiomer = 100% and (+)-enantiomer - (-)-enantiomer = Bjok
(+)-enantiomer = 75o/o and (-)-enantiomer = 25"/o, choice D

Copyright @ by The Berkeley Review 203 Exclusive MCAT Preparation


Organic Chemistry Stereochemistry Nucleophilic Substitution

Nubledphiliru1i;$fib$$.flfi ffi,bn
Nucleophilic Substitution
One application of stereochemistry is in nucleophilic substitution reactions.
Nucleophilic substitution involves the attack of an electropositive carbon by a
nucleophile (Lewis base) to dislodge an atom or functional group (referred to as
the leaving group). This is a recurring reaction in organic chemistry and it
involves the substitution of one functional group for another. Nucleophilic
substitution can proceed by more than one reaction pathway. It can proceed bv
the two-step 51111 mechanism, or it can proceed by the one-step 5512 mechanism.
Nucleophilic substitution reactions are based on the fundamental chemistrv j
concept that negative charge seeks positive charge. The electron pair of the fi

nucleophile hunts for an electron deficient carbon to attach to. It will be s


important to understand the steps of the reaction for both mechanisms, as lli
reactants proceed to products. Figure 3-15 shows an example of a nucleophiJic ri[

substitution reaction. rl
s
M
..-..o
.1. + H.CO M

,M

d
Figure 3-15

We will discuss the mechanism of this reaction shortly, but for now, there are
some fundamental definitions with which to be familiar. Listed below are the
most important definitions. Each definition is followed by some genere-
comments about the relative reactivity of the species and how to discern i5
reactive strength.

Nucleophile
The species donating an electron pair in a nucleophilic substitution reactic'n
(Lewis base). As its name implies, it loaes (philes) a positiae charge (nucieo'
Nucleophiles must have an available pair of electrons to share. Nucleophile
strength is closely approximated by its base strength, although steric factors
(nucleophile size) affect nucleophilicity. Small nucleophiles are generally better ffii
nucleophiles. This is to say that steric hindrance plays a larger role in s
nucleophilic substitution reactions than proton transfer reactions. The strength
of the nucleophile does not perfectly correlate with base strength, but it is close
enough to say that it parallels. A short list of nucleophilic strength in water m
solvent is as follows: .Lq

SH-> CN- > I-> OR- > OH-> Br-> NHg > C6H5O- > CH3CO2- > Cf > F- > ROH > HzO d
It should be noted that if the base is too strong, an elimination reaction can occur I
(as is the case with OR- and OH-). The solvent also has an effect in that (
nucleophiles that can hydrogen bond are hindered in protic solvents, because f,
they are solvated. The solvation by water binds the electron pair of the
nucleophiie and reduces its nucleophilicity. This phenomenon explains why SH-,
CN- and I- are stronger nucleophiles in water than OH- despite being weaker
bases than OH-. In aprotic solvents and the gas phase, nucleophilicity more
closely parallels basicity. The big difference in nucleophilicity is that size of the
anion is not as important as it is with basicity.

Copyright O by The Berkeley Review The Berkeley Review


Organic Chemistry Stereochemistry Nucleophilic Substitution

Electrophile
The species accepting an electron pair in a nucleophilic substitution reaction
(Lewis acid). The electrophile holds the reactive .urborl and the leaving group.
The weaker the bond between the leavrng group and the carbon, the better the
electrophile. Electrophile strength can be ipproximated by the stability of the
leaving group once it is off of the electrophile. Electrophiiic carbons typically
nave a partially positive charge.

Leaaing group
The functionai group that dissociates from the eiectrophile in a nucleophilic
substitution reaction. The more stabie the leaving group, the weaker it ii as a
rase. This means that the strength of a leaving gio"p can be predicted by the
.trength (pKu) of its conjugate acid. The theory is that the more stable the
-eaving group, the less basic the leaving group, and thus the more acidic the
:onjugate acid of the leaving group. The strengih of a leaving group increases as
:he pKu of its conjugate acid decreases. This is most true in wafer, but can also
be
seen in organic solvents. group strength increases the strength
as of the
:ond between carbon andf-eaving
the leaving gro.tp J""r"ur"r. This is why iJdine is a
retter leaving group than fluorine.

?.scemic mixtrn.e
'+ product mixture that has an even distribution of enantiomers, 507u of each
=nantiomer, in the product mixture. A racemic mixture is the observed product
-'""hen the mechanism
involves an intermediate where the reactive site is'an sp2-
-^,., bridized carbon (like a carbonyl or carbocation) and
the molecule is symmeiric
-:ras r-lo other chiral centers). There is no such thing as a racemic mixture
of
..iastereomers, because diastereomers have at least two chiral centers associated
''ith them, and a chirai center present in the reaction hinders attack of one side of
-: e electrophiie relative
to the other, causing the distribution to not be fifty-fifty.

--.1 nucleophilic reactions involve a nucleophile, an electrophiie, and a leaving


;:oup. some, but not all, nucieophilic reactions generate a racemic mixture.
a racemic mixture is generated or not depends on the reaction pathway.
",hether
lhe fundamental question in a nucleophilic subititution reaction is, ;does the
- ;cleophiie come in first, or does the leaving group leave first?,, This is the basic
-..Jference between the sry1 and the slrJ2 mechanisms. The nucleophile attacks
::st in an S]N2 reaction mechanism, while in the leaving $oup leaves first in an
:';1 reaction mechanism. we will look at these two scenarios in more detail.

Erample 3.19
-, iavorable nucleophilic substitution reaction has all of the following EXCEpT:
d. a good leaving group.
l. a reactant with a weak bond to the leaving group.
:. a strong Lewis base as the nucleophile.
f . a weak Lewis base as the nucleophile.

- - nvright @ by The Berkeley Review 20s Exclusive MCAT Preparation


Organic Chemistry Stereochemistry Nucleophilic Substitution

Solution
A favorable nucleophilic substitution reaction is one that forms a stronger bond
than the one broken. A good nucleophile is one that forms a stronger bJnd with
carbon than the bond between carbon and the leaving group. A good leaving
group is one that forms a weak bond with carbon, thus minimal eneigy is needeJ
to break the bond between it and carbon. when the leaving group ii strong, ihe
reaction is said to be favorable, so choice A is a valid statemet-,i. choic" e is
eliminated. A weak bond to the leaving group makes it a good leaving group, so
choice B is also a valid statement. This eliminates choiie B. The n.t"t"optl1t" I

should be a strong Lewis base, so choice C is a valid statement and choice Dls an
invalid statement. This eliminates choice C and makes choice D the best answer.
You might note that to determine the favorability of a nucleophilic substitution ,i

"".rt"", t"" """,


, leophile and leaving group. L

sru2
In an sjr]2 reaction, the nucleophile attacks prior to the ieaving group leaving. lr- e
u:1"1.9, the nucleophile comes in from the backside and pushes ine teaving gio.rp e
off of the electrophile. An important factor to consider is the transition state tha: rm

forms during the reaction. Transition states cannot be viewed directly (thei:
-
lifetimes are too short), but evidence in the product (inversion of configuration ai
a chirai carbon) infers they exist. Backside attack by a nucleophile causes thr-.
inversion at the chiral carbon. Certain features in the reactants (nucleophile an;
electrophile) and the product (if it is chiral) indicate that the reaction proceedec
by an slr]2 mechanism. These are idiosyncrasies of the reaction, and they dictatr
the pathway the reaction chooses. Each property favors one of the trr l
mechanisms. They can be used to distinguish an slr]2 reaction from an sl-
reaction. Figure 3-16 shows a generic mechanism for an sl.I2 reaction.
II
Trigonal bipyramidal
Transition state &
I ,&riil
R+ s,
I
I
Nuc---f---'L
n
Nuc: L'G'--+ .G. ------> Nuc L.G.- G
ln"
l'==
H HH
Figure 3-16

The reaction takes place in one step, so the rate of an sry2 reaction depend.s .-mr
both the concentration of the nucleophile and the concentration of t:e
electrophile. The nucleophile initiates the reaction by attacking the electrophie
and forcing the bond between the carbon and the leaving group to stretch a:rd
weaken. At the same time that the nucleophile approaches the electroph:-rc
carbon, the electron density of the nucleophile repels the substituents on tw
electrophilic carbon and thus they form the trigonal bipyramidal transition sta:e-
As the leaving group begins to leave, the substituents on the electrophilic carb,cnmr
begin to fold in the direction of the less hindered side of the mblecule (l
hindered because the leaving group has left). The hybridization finishes as #-
Table 3-2 lists some key features associated with an sp2 reaction, according
observation order. What is meant by observation order is that the first featr::
(features of the reactants) are observed before the reaction begins, the ser
features (features of the transition state) are observed during the reaction,
the last features (features of the products) are observed after the reaction end-s-

Copyright @ by The Berkeley Review 206 The Berkeley


Organic Chemistry Stereochemistry Nucleophilic Substitution

Reactant Features Course of Reaction Features Product Features

rhe preference for an 51112 An 51112 mechanism forms a five' A single enantiomeric product
::,.echanism is 1' > 2" > 3" in ligand transition state during is formed
:erms of electrophiles the middle of the reaction (No racemic mixture)
The S-ligand transition state is 5512 reactions exhibit second
-{n S11i2mechanism is favored
i'.-ith a good nucleophile the highest energy state and it order kinetics
exists for just a split second (rate = klNucllElectl)
ln 5512 mechanism is favored Steric forces destabilize the Sl12 reactions are one-step
-l poiar, aprotic solvents such transition state by forcing bond reactions, so they have fast
- ethers and ketones angles to values less than 109,5' rates of formation

Table 3-2

lie reaction in Figure 3-17 proceeds by an Sy2 mechanism, because the


:-ectrophile is primary and it has a good nucleophile. With a primary
=-ectrophile, the reaction must proceed by an 5512 mechanism. The ether solvent
polar and aprotic, which further favors the Sp2 reaction pathway.
-.

,\
H"C CH"

NC:@ + r --+
Et2O NC-J-
+I@
Good H'''y Y,o"
D
Stable
\ucleophile D Anion
1" Electrophile Inversion Product

Figure 3-17

E"xample 3.20
-ll of the following are associated with an 51112 reaction EXCEPT:
A. backside attack of the electrophile by the nucleophile.
ts. inversion of stereochemistry.
C. nucleophile concentration affecting the reaction rate.
D. rearrangement of alkyl groups from reactant to product.
5olution
ohis question focuses on the fundamentals of an S1.tr2 reaction. For an St12
:eaction to proceed, the nucleophile must attack the electrophile from the
--rpposite side as the leaving group in a collinear fashion reiative to the bond to
:he leaving group. This is referred to as backside attack, so choice A is valid, and
--nus eliminated. Backside attack results in inversion of stereochemistry if the
tlectrophilic carbon is a chiral carbon. This makes choice B valid, which
eliminates it. Because there is just one step in an 51112 reaction, the rate depends
rn all of the reactants, including the nucleophile. This makes choice C valid, and
thus eliminates choice C. Rearrangement can occur when there is a carbocation
present, because carbocations lack a bond. Carbocations are associated with 51111
reactions, not 51112 reactions, so choice D is invalid and thus the correct answer to
:his question.

Copyright @ by The Berkeley Review Exclusive MCAT Preparation


Organic Chemistry Stereochemistry Nucleophilic Substitution
j
sp1:
Lr an s|J1 reaction, the leaving group leaves before the nucleophile attacks. Lr
essence/ the nucleophile waits until the leaving group has left, allowing it more
room to attack. The slr]1 reaction rate does not depend on nucleophile
concentration. once the leaving group has dissociated, a planar caiionic
intermediate forms. Evidence for the intermediate comes from kinetics data as
well as stereochemicai evidence provided by the products. The carbocation
intermediate has a long enough lifetime to be detected using spectroscopy. Both
rearrangement (hydride shifts and alkyl shifts) and a mixture of stereoisomers
(formed from the spl-intermediate) are observed with sNI1 reactions. The
nucleophiie is free to attack from either side of the carbocation intermediate, if
the carbocation is symmetric. As a result, a racemic mixture of enantiomers is
formed as the product mixture. Figure 3-18 shows a generic mechanism for an
5111 reaction.

R
R Nuc attacks
,."(
720'A 109.5"
,onf"$G" I fromrhe,9r__ Nuc

R"
-> Vf ffi;=-.
R,, R, from the right 109
'[r
R>.. -R; L.c.

Trigonal planar
Carbocation Intermediate R"

Figure 3-18
The reaction takes place in two steps, where the first step is the slowest. In the
first step, the bond between the carbon and the leaving group breaks. As the
leaving group begins to leave, the substituents on the carbon foia in the direction
of the less hindered side of the molecule, allowing the-bond-angles to increase
from 109.5' to 120" as the carbon re-hybridizes from ry3 to sp2. ihis results in a
slight increase in stability, accounting for the intermediate being at a lower Ll
energy level than the first transition state in the energy diagram. In addition to .:
re-hybridization, the planar cation is solvated, which also increases the stability -,':
of the intermediate. The nucleophile can attack the carbocation intermediate and
.q,.
displace the solvent from either side of the carbocation intermediate. This
D
displacement of solvent and the re-hybridization from sp2 back to sp3 causes a
C
decrease in stability from the intermediate to the second transition state. Finally,
D
as the new bond is formed, the energy level decreases until it reaches the level of
the products. Bond formation is an exothermic process. The hybridization of the
central carbon finishes atsp3. Table 3-3 lists features associited with the sx11
reaction. Like in Table 3-2, the features are iisted according to observation order.

Reactant Features Course of Reaction Features Product Features


The preference for an 5511 Steric hindrance pushes the A racemic mixture forms
mechanism is 3" > 2' > 7" leaving group off of the when the electrophile
in terms of electrophiles electrophile has chirality
An S11 1 mechanism is The intermediate is a planar, SN 1 reactions exhibit
favored in a protic solvent three-ligand carbocation with first order kinetics
such as alcohol sp2-hybrrdtzatton (rate = k[Elect])
An Sry1 reaction is seen An intermediate is observed in Sry1 reactions are slow
with a poor nucleophile addition to transition states two-step reactions
Table 3-3

-:pvright O by The Berkeley Review The Berkeley Review


Organic Chemistry Stereochemistry Nucleophilic Substitution

The Sxyl reaction can be complicated by rearrangement, because of the


carbocation intermediate formed. If a secondary carbocation (R2CH+) is formed,
it can rearrange to form a tertiary carbocation (RgC*), if a tertiary carbocation is
possible. For alkyl carbocations, the relative stability is 3" > 2' > 1", the same
preference that is observed with free radicals. The features in an S5i1 reaction are
opposite of those in the Sp2 reaction. These features of a reaction can be used to
predict whether a reaction will proceed by an 51111 or 51112 reaction mechanism.
The reaction in Figure 3-19 proceeds by an Sp1 mechanism, because the
electrophile is tertiary and it has a good ieaving group.
H3CH2CHTC
'\
o
NH3
H?CH2CH2C H3c\f
\ H3CH2C
H3N: + I+& *Io
Average tr.o'YF gH2cH2cH. stable
Nucleophile H3CH2C | ' ' Anion
3" Electrophile
ol
,,*_\,,.,r,
cH2cH3
Racemic Mixture of Products

Figure 3-19

The reaction is favorable, because the leaving group is a good leaving group and
the nucleophile forms a stronger bond with carbon than the leaving group.

Example 3.21
Ihe addition of ammonia to R-3-iodo-3-methylhexane at low temperature would
vield:
A. one product with R configuration exclusively (retention of stereochemistry).
B. one product with S configuration exclusively (inversion of stereochemistry).
C. two products in an enantiomer mixture.
D. two products in a diastereomeric mixture.

Solution
First, we must determine whether the reaction proceeds by an S1r11 or 51..12
mechanism. The electrophile (R-3-iodo-3-methylhexane) is tertiary, so the
reaction proceeds by an S1'tr1 mechanism. The chiral center is lost with the
formation of the carbocation intermediate, because the intermediate is pianar
with symmetric sides. This results in a racemic product mixture of two
enantiomers. Choice C is a swell answer for this question.

Copyright @ by The Berkeley Review 209 Exclusive MCAT Preparation


Organic Chemistry Stereochemistry Nucleophilic Sutistitution

In Figure 3-19, the electrophile is tertiary. In cases where the reactive carbon is
secondary, an slr]1 reaction can be complicated by rearrangement. This is shown
in Figure 3-20.

H.C
"\ H,.C
"l H"C
"l
shift
Hv \'
,,L--r

-([t
hydride

"'h
"-c H

-
(H3C)2HC
- c* H
/\ /\
HsC CHs HsC CHe
2'carbocation 3" carbocation

HlCH,C
" 'l H3CH2C
I HAN: \
\-
li,- urcsf.NH'
HsC CHa
- HsC
3" carbocation achiral product

Figure 3-20

Rearrangement is rapid, because it is an intramolecular process. L:r the example


in Figure 3-20, the secondary carbocation rearranges to form a more stable
tertiary carbocation before the ammonia nucleophile attacks the carbocation
intermediate. This results in a tertiary product. The halide leaving group is not
basic enough to deprotonate the ammonium cation formed from the substitution
reaction, so the product remains as a cation.
If the electrophile has a chiral center at a site other than the electrophilic carbon,
an Str11 reaction will form both a major and minor product. The major product
results from the transition state with least steric hindrance.

CHICH? CH,,CH" CH,CH"


YYYY CH.CH,

-.> ry.s
OCI e.'C*
Attack from the backside is
O'"?:il; Ort:il;
Major Minor
more favorable than frontside Product Product
attack due to steric hindrance.

Figure 3-21

In the example in Figure 3-27, the ethyl group in front of the plane interferes with
the attack by the nucleophile, which results in an uneven distribution of
diastereomers as the product mixture. The major product is formed when
ammonia attacks the less hindered face of the carbocation (backside attack in this
example). The minor product is formed when ammonia attacks the more
hindered face of the carbocation (front side attack in this example).

Copyright O by The Berkeley Review 2to The Berkeley Review


Organic Chemistry Stereochemistry Nucleophilic Substitution

Distinguishing an Sp2 reaction from an SpL reaction


The first thing to look for when determining the mechanism by which a
nucleophilic substitution reaction will proceed, is the substitution of the
electrophile. Tertiary and allylic (adjacent to a n-bond) electrophiles wiil proceed
by an Sx11 mechanism while methyl and primary electrophiles will proceed by an
5512 mechanism. This is the first factor to view. If the electrophile is secondary,
then the reaction can proceed by either mechanism. After considering the
substitution of the electrophile, the next feature to consider is the nucleophilic
strength. The stronger the nucleophile, the more likely the reaction will proceed
by an 5512 mechanism. The better the leaving grosp, the more likely the reaction
will proceed by an 51111 mechanism. Lastly, you should consider the solvent. If
the solvent is protic (capable of forming hydrogen bonds), the reaction wili have
a tendency to proceed by an 5111 mechanism. If the solvent is aprotic (not
capabie of forming hydrogen bonds), the reaction will have a tendency to
proceed by an Sry2 mechanism. These factors can be applied when looking at the
reactants.

If rate data are given, then the mechanism can be inferred without ambiguity.
The rate law associated with an 5111 mechanism is shown in Equation 3.3, while
the rate iaw associated with an Sp2 mechanism is shown in Equation 3.4.

S1.tr1 Rate = k [Electrophile] (3.3)

5512 Rate = k [Nucleophile][Electrophile] (3.4)

If the rate of a reactionchanges as the nucleophile concentration is varied, the


reaction is proceeding by an Sp2 mechanism. Conversely, if the rate of a reaction
does not'change as the nucleophile concentration is varied, the reaction is
proceeding by an 5511 mechanism. Because the solvent can affect the strength of
a nucleophile, solvent and nucleophile are often considered together. The rates
of both reactions vary with a change in electrophile concentration.

The energy diagrams for the two mechanisms also differ. There is no
intermediate associated with an 51112 reaction, only a transition state. There is ar"t
intermediate and two transition states associated with an 5511 reaction. Figure 3-
22 shows the energy diagrams for the one-step S52 reaction (on the left) and the
two-step Sp1 reaction.
sru1 Intermediate
Transition state
+
t

Reactant
bo
H
G)

F]
Product

Reaction co-ordinate -------> Reaction co-ordinate -+


Figure 3-22

Copyright @ by The Berkeley Review 2tl Exclusive MCAT Preparation


Organic Chemistry Stereochemistry Nucleophilic Substitution

The energy level increases at the start of each energy diagram, because a bond is
being broken. In the case of the S1r11 reaction, the intermediate is of lower energy
than the transition state, because the carbocation can rehybridize to the leil
crowded sp2-center rather than an sp3-center and the intermediate can be
solvated in a protic solvent. The increase in energy from the intermediate to the
second transition state is associated with rehybridization to the more crowded
sp3-center and the intermediate losing solvition to allow the nucleophile to
attack. It is important to be able to recognize these diagrams and apply the
information they contain to conceptuai questions.

Example 3.22
The addition of sodium methoxide to s-2-bromohexane at low temperature
would yield:
A. one product with R configuration exclusively (retention of stereochemistry).
B. one product with S configuration exclusively (inversion of stereochemistry).
C. two products in an enantiomeric mixture.
D. two products in a diastereomeric mixture.

Solution
First, we must determine whether the reaction proceeds by an sIrI1 or sry2
mechanism. The electrophile (S-2-bromohexane) is secondary so the reaction can
proceed by either an sl.J1 or Sry2 mechanism. The nucleophile is a strong
nucleophiie, so we can assume the reaction will proceed via an Sp2 mechanisml
This results in inversion of the chiral center and the final product having R
stereochemistry, so choice A is the best answer. The low temperature is
important, so that there is little to no E2 product formed. An E2 reaition results
r" r^" trt-",t"" t

Example 3.23
The following reaction shows what relationship between nucleophile
concentration and reaction rate?
H?CCH2S-+ H,CCHTBT -----+ H3CCH2SCHTCH, + Br-
A. The reaction rate increases in a linear fashion with increasing nucleophile
concentration.
B. The reaction rate increases in an exponential fashion with increasing
nucleophile concentration.
C. The reaction rate does not change with increasing nucleophile concentration.
D. The reaction rate decreases in a linear fashion with increasing nucleophile
concentration.

Solution
The reaction has a primary electrophile and a good nucleophile, which favors an
s512 mechanism. The rate equation associated with a reaction proceeding by an
s}..J2 mechanism is rate = k lElectrophile][Nucleophile]. The equation shows that
the reaction rate is directly proportional to the nucleophile concentration. The
rate increases in a linear fashion wiih increasing nucleophile concentration, as
stated in choice A. The best answer is choice A. Choice B should be eliminated,
because the rate of a nucleophilic substitution reaction does not depend on the
concentration of any species in an exponential fashion. Choice D should also be
eliminated, because the rate will not decrease with additional nucleophile. It will
either increase in a linear fashion or not change.

Copyright @ by The Berkeiey Review 212 The Berkeley Review


Organic Chemistry Stereochemistry Nucleophilic Substitution

Example 3.24
A transition state with no intermediate is associated with which of the following
reactions?

A. H3CCHZO- + H3CCH2BT ----t> H3CCH2OCH2CH3 + Br-


B. H3CCH2OH + (H3C)3CBr ------> H3CCH2OC(CHa)3 + HBr
C. (H3C)3CSH + (H3C)2CHOHz* + @3C)3CS+HCH(CH3)2 + H2o
D. NH3 + (H3CH2C)3CBr --.> (H3CCH2)3NH3+ + Br-

Solution
As shown in Figure 3-27, no intermediate is associated with an sx12 mechanism,
so we must find the reaction most likely to proceed by an sN2 mechanism. The
reaction most likely to proceed by an Sl.]2 mechanism should have a good
nucleophile and ideally a primary electrophile. A low temperature is important
here so that there will be little to no E2 product formed. Choice C is the ieaction
of a secondary electrophile with a poor nucleophile, so it will likely proceed by
an S51 reaction mechanism. This eliminates choice C. Choices B and D involve
tertiary electrophiles, therefore they definitely will proceed by an SlrJ1 reaction
mechanism. This eliminates both choice B and choice D. Choice A has a primary
electrophile and a good nucleophile, which makes it the most likely to proceed
by an sg2 mechanism, and therefore makes it the best answer. The ethoxide
anion is also a strong base, so elimination is possible in choice A. Despite the
competition with the E2 reaction, choice A is still the best answer.

Reaction Kinetics
The rate'of a reaction depends on several factors. The rate depends on the
available energy for the molecules to collide, orient, and break the necessary
bonds. The rate depends on the likelihood of the molecules colliding. For an
sirl2 reaction, the rate depends on the availability of nucleophile, while it does
not depend on the nucleophile concentration in an Sry1 reaction. Consider the
5512 reaction shown in Figure 3-23 with an ethoxide concentration, [CH3CH2O-],
of 0.01 M a 2-bromopropane concentration, [CH3CHBTCH3], of 0.01 M and a k1*
of 2.53 x 10-2 L.mol-1.s1 at 2gBK.

HqCH?CO:

Figure 3-23

The concentrations are low, so the reaction is very slow. Plugging the values into
Equation 3.4 yields a rate of 2.53 x 70-6 M per second. The reaction rate may be
increased by increasing the reactant concentrations, increasing the temperature,
or by adding a catalyst. A catalyst stabilizes the transjtion state complex and
lowers Eu.1. Transition states are short-lived complexes. In the course of the
reaction, reactants collide with the correction orientation (from backside attack)
to form the transition state complex, when eventually splits io generate the
products. Figure 3-24 represents the species of the S5tr2 reaction in Figure 3-23 at
different stages in chronological order over the duration of the reaction.

Copyright O by The Berkeley Review 213 Exclusive MCAT Preparation


Organic Chemistry Stereochemistry Nucleophilic Substitution

..4- | - ..5
CH"

H,cH"co ----+----Bri

Reactants draw close to start H CH3


Transition State Transition State starts to split
bond formation and form the
as the bond breaks to form the
Early Complex (C"u.1r). Late Complex (C1u1s).
H.C
..6- '\ ..6- ..5- ..6-
H3cH2co. --- Br! H3CH2CO.- - - /::' Bri
;f \"
HsC CHs
Ceurly Cl"t.

H.CH"CO
Reaction co-ordinate
J L ..

Yg"
-/'n'

Figure 3-24

Physical Properties of Stereoisomers


Enantiomers have identical physical properties (boiling point, melting point, and
density to name a few), while diastereomers have slightly different physical
properties. Because they have slightly different physical properties,
diastereomers are easier to separate than enantiomers. Enantiomeric mixtures
are difficult to purify, because a racemic mixture often has stronger
intermolecular forces than the pure enantiomer. Table 3-4 lists the physical
properties of the two enantiomers and the racemic mixture of 2-butanol.

Form Chirality CTD Boiling Point Density Index of Refraction


(+) S + 13.5" 99.4'C 0.808 1.398
(-) R - -t-J.5 99.4'C 0.808 1.398
(r) R/S 0' t01.2"c 0.840 1..442

Table 3-4

lrom the data in TabIe 3-4, it can be seen that a racemic mixture allows the
molecules to get closer together. This can be thought of when considering your
hands, where a left and right hand fit together nicely. It is common that a
racemic solid mixture has a higher melting point and greater density than either
enantiomer.

Copyright O by The Berkeley Review The Berkeley Review


Organic Chemistry Stereochemistry Nucleophilic Substitution

Separating Stereoisomers
One of the most chailenging tasks a synthetic organic chemist faces is the
separation of stereoisomers. If a reaction generates a new chiral center in the
product, then it will be complicated by stereoisomerism. To generate a pure
stereoisomer as a product, chirality must be invoked at some point. From
biochemical examples, we know that enzymes (chiral polypeptides) orient
molecules in a specific fashion, allowing just one stereoisomer to form, The
chirality of the enzyme helps to select for the desired product. In organic
chemistry, there are compounds known as chiral nuxiliaries, which introduce
chiraiity to, or exaggerate existing chirality within, a reactant molecule. Chiral
auxiliaries serve in a similar fashion to an enzyme. When aiming for one specific
stereoisomer, it is often easiest to select for it in the reaction. If not, a mixture of
stereoisomers is formed and chirally selective separation techniques must be
applied.

Chirally selective separation techniques come in two types. The first involves
employing an enzyme (or chirally selective molecule) to react specifically with
one stereoisomer within the mixture. By reacting and therefore introducing a
new functional group to only one stereoisomer, the two enantiomers now have
different physical properties and can easily be separated. Once separated, the
same enzyme can be employeci to return the compound back to its original form.
An example of such an enzyme ts porcine renal acylase, which selectively acylates
the N-terminal of L-amino acids. By acylating the L-amino acid, it is no longer a
zwitterion at neutral pH, while the D-amino acid is a zwitterion. Because one
carries a net charge, it is easily separated from the other.

The second chiraily selective separation technique involves invoking chirality in


an existing separation technique. For instance, a column chromatography gel can
be made from a pure stereoisomer. If the column is made with an R-alcohol for
instance, then when a racemic mixture of alcohols is added, the S-enantiomer has
a greater affinity for the column and thus has a greater elution time. This is the
basic principle behind affinity chromatography in biochemistry, where an
antibody is bound to the column so that it can selectiveiy bind an antigen. Chiral
columns in organic chemistry are not as specific as enzyme columns and they
hinder solutes, but do not actualiy bind them. In theory, chiraiity could be
invoked in any organic chemistry separation technique, including distillation,
but only it is chiral columns that are commonly used.

Copyright @ by The Berkeley Review 215 Exclusive MCAT Preparation


Organic Chemistry Stereochemistry Summary

Summary
This section involved a few basic concepts. From this particular section, you
should be able to identify R and S chirality, determine the relationship between
stereoisomers, understand the nuances of nucleophilic substitution, distinguish
between the slri1 and sIN2 reactions, and apply stereochemistry to organic lab
techniques and biochemistry.

Determining R and S: When a structure is drawn in dash-and-wedge style, if


priority number four is in back or two substituents away from the substituent in
back, then take the arc as is and assign the corresponding chirality (Clockwise =
R; Counterclockwise = S). If priority number four is one substituent away from
the substituent in back, then determine the arc and assign the opposite chirality
of the arc.
Enantiomer: Enantiomers are stereoisomers which are nonsuperimposable
mirror irnages (reflections that you can't overlay). They have the same bonds,
but they have a different orientation of atoms in space, as do all stereoisomers.
Enantiomers can be thought of as stereoisomers in which all of the chiral centers
have different orientation between the two molecules. Enantiomers have
identical physical properties.
Diastereomer: Diastereomers are stereoisomers which are nonsuperimposable
and that are not mirror images. They too have the same bonds and a different
orientation of atoms in space. To be diastereomers, the compounds must contain
a minimum of two chiral centers. Diastereomers are stereoisomers in which at
least one but not all of the chiral centers have different orientation between the
two molecules. Diastereomers have close, but not identical, physical properties,
sy1 Reactions: Preferred when an electrophile is tertiary, when the solvent is
polar and protic, and when the electrophlle has u good leaving group, Sx1
reactions form a planar carbocation intermediate that can undergo
rearrangelnent to form a more stable carbocation. Sru1 reactions result in a
racemic mixture when the reactive carbon is the only chiral center in the reactant.
The rate of an SX11 reaction depends on only the electrophile and not the
nucleophile (rate = k[Electrophile]).
5672 Reactio;,zs: Preferred when an electrophile is primary, when the solvent is
polar and aprotic, and when the nucleophile is good. 5112 reactions form a
transition state complex as the nucleophile forces the leaving group off from the
electrophile. SN2 reactions result in inversion when the reactive carbon is a
chiral center. The rate of an 5112 reaction depends on both the electrophile anC
nucleophile (rate = k[Nucleophiie][Electrophile]).
Stereoisomer Mixtures: A fifty-fifty mixture of enantiomers is said to be a racemr:
mixture. The diastereomers in a product mixture formed from a chemica,
reaction are referred to as major and minor, because they do not occur in a fiftv-
fifty ratio. Separating one diastereomer from another is easier than separatirrg
one enantiomer from another. To separate enantiomers, chirality must b.
incorporated into the separation technique.

Copyright O by The Berkeiey Review 216 The Berkeley Revien


Organic Chemistry Stereochemistry Nucleophilic Substitution

Key Points for Stereochemistry (Section 3)


Chirality and Asymmetry
1. Chiral Molecules
a) Have asymmetry within their structure due to atoms that are unevenly
substituted
i. Stereogenic carbons are sp3-hybridized carbons with four unique
substituents
ii. Chiral moiecules are predominant in many organic reactions
b) Stereogenic carbons are assigned an absolute configuration of either R or
S to describe their chirality
i. Priorities are assigned according to atomic mass of the atoms
attached to the stereogenic center. If two atoms are identical, then
you proceed along its connectivity until there is a difference
ii. when priority #4 is in back, a clockwise arc connects priorities #1,
#2, and #3 in R-stereogenic centers. When priority #4 is in back, a
counterclockwise arc connects priorities #I, #2, and #3 in S-
stereogenic centers.
iii. To determine whether a center is R or S, you can place your thumb in
the direction of substituent #4 and curl your fingers from priority #1,
through priority #2, and on to priority #3. Only one of your hands
can do this. If it is a right hand that does this, the stereogenic carbon
has R-chirality. If it is a left hand that does this, the stereogenic
carbon has S-chirality.
c) short cuts for determining R and s involve the positioning of priority #4
i. If priority #4 is in back, then the arc determines the chirality
(clockwise for R and counterclockwise for S). If priority #4 is in
front, then the arc must be reversed to determine the chirality (a
clockwise arc is reversed to represent S and a counterclockwise arc is
reversed to represent R). If priority #4 is drawn in the plane close to
the group going back, then the arc is reversed to determine the
chirality. If priority #4 is drawn in the plane far away from the
group going back, then the arc as is determines the chirality.
ii. \Ahenever two groups are switched, the chirality reverses

Configurational Isomers
1. Same connectivity, but different spatial arrangement of atoms
a) Can be categorized as either optical isomers or geometrical isomers
i. Optical isomers rotate plane-polarized light
ii. Geometrical isomers differ about a feature in the molecule about
which rotation is not possible (n-bond or ring)
iii. Optical isomers are identified by a standard rotation value
b) Can be categorized as enantiomers or diastereomers
i. Enantiomers are nonsuperimposable stereoisomers that are mirror
images
ii. Diastereomers are nonsuperimposable stereoisomers that are not
mirror images
iii. Enantiomeric optical isomers are better thought of as stereoisomers
where all of the chiral centers differ
iv. Diastereomeric optical isomers are better thought of as stereoisomers
where some, but not ali, of the chiral centers differ

Copyright O by The Berkeley Review 217 Exclusive MCAT Preparation


Organic Chemistry Stereochemistry Section Summary

Stereochemistry in Reactions
7. Stereoisomers are formed when a nucleophile attacks an asymmetric
moiecule in multiple ways
a) Racemic mixtures form when there is no preexisting chirality
b) Diastereomers are formed in a major/minor distribution when one of the
reactants has a chiral center at a non-reactive site
i. Mixtures are resolved by using chiral reagents or lab techniques that
invoke chirality.
ii. Enzymatic reactions use a chirai catalyst to cause the reaction to
drastically favor the formation of one stereoisomer over all other
possible stereoisomer products
iii. When two enantiomers are present in unequal amount, there is said
to be an enantiomeric excess. Enantiomeric excess is used to describe
the success in a stereoselective synthesis
c) Enantiomers have identical physical properties as one another while
diastereomers have different physical properties.

Nucleophilic Substitution
1. Proceeds by either an Sp1-mechanism or Sp2-mechanism
a) In an Sg1-mechanism, the electrophile is highly substituted, the solvent
is protic, and a carbocation intermediate is formed because the leaving
group leaves in ihe first step. There is potentially rearrangement and the
product mixture is often racemic. The reaction rate only depends on the
electrophile.
b) In an S52-mechanism, the electrophile is minimally substituted, the
solvent is polar and aprotic, and a transition state is formed because the
nucleophile attacks to force the leaving group off in the only step. There
is inversion of chirality so the product mixture is often the opposite
chirality of the reactant and the reaction is fast. The reaction rate
depends on both the nucleophile and the electrophile.

51

Copyright @ by The Berkeley Review 2ta The Berkeley Review


Stereochemistry
Passages
15 Passages
I OO Questions

Suggested schedule:
I: After reading this section and attending lecture: passages I, II, vll & x
Grade passages immediately after completion and log your mistakes.
II: Following Task I: Passages IV V IX, & XIII (28 questions in 56 minutes)
Time yourself accurately, grade your answers, and review mistaKes.
III: Keview: Passages III, VI, VIII, XI, XII, & Questions 92 - IOO
Focus on reviewing the concepts. Do not worry about timing.

B
SpeciaLtztng in MCAT Preparation
#fi il rtffi-iffit :lffiffings.

I. Isoleucine and Threonine (r -7)


II. Unknown Stereochemically Active Compound (B - 14)

III. Classification of Isomers (15 - 21)

IV. Stereoisomers and Optical Activity (22 - 28)

V. Enantiomers from Diels-Alder Reaction (2e - 35)

VI. Proposed S51 SYnthesis (36 - 42)

VII. Nucleophilic Substitution (43 - 4e)


(5O - s6)
vlil. Nucleophilic substitution of chlorocyclohexane
IX. Leaving GrouP Strength (s7 - 63)

X. Reaction Rates of Nucleophilic Substitution (64 - 70)

XL Elimination versus Substitution Experiment (7r -77) lI:r


lll.r:

(78 - 84) .lr-:

XII. Enantiomeric Excess rlltut

XIII. Stereochemistry in Synthesis (Bs - el) illl:,"

rLllJ

(e2 - lOO)
Questions not Based on a Descriptive Passage

Structure, Bonding, and Reactivity Scoring Scale

84 - 100
Passage I (Questions '1 - 7) 4 . The amino acid glycine has an H as its side chain. What
would you predict for the optical rotation for naturally
Isoleucine is oneof eight essen.tirrl amino acids. The occuning glycine?
::n essential is applied to amino acids that humans cannot
::rduce, and therefore must take in through diet. Isoleucine A. (.+)32'
:, .sts as a zwitterion in aqueous solution. The natural form B. (-) 32'
isoleucine has the same chirality as other naturally c. 0'
-:urring amino acids at carbon number two. Isoleucine is D . More information is needed. (Never choose this!)
- lnd naturally in the L form. Naturally occurring amino
.,-ds have an S chiral center on carbon two (except cysteine).
-..rleucine has a second chiral center in addition to the one at
-'rbon two. Seventeen of the amino acids we code for have
: ,.rctly one chiral center, with glycine (which has no chiral
5 . If L-isoleucine were found to have an optical rotation of
-62' from plane-polarized light studies, what would you
-
'rter) and threonine (which has two chiral centers) being the predict for the optical rotation of its enantiomer?
::er exceptions. The second chiral center of L-isoleucine
-:s fixed chirality, so a diastereomer of isoleucine varying at A. -62'
. : side chain is not a biological substitute for L-isoleucine. B. +62'
. .sure 1 shows the zwitterion form of L-isoleucine. c. -31'
D. +31'

CH 2CH r
6 . Which of the following is the side chain of threonine?

o-
A. -CH(CH3)2
L-Isoleucine B. -CH(oH)CH3
Figure 1 Zwitterion form of L-Isoleucine C. -CH2CH2OH
D. -CH2OH
The only other amino acid coded fbr by human beings
.:at has a chiral side chain attached is threonine. The
.ireonine side chain contains an alcohol functionality. Like
.oleucine, the side chain chiral center must be specific for
.ie amino acid to be biologically incorporated. Threonine is 7 . Only twenty-five percent of synthesized isoleucine can be
..so an essential amino acid. The diaslereomer of threonine used biologically. This is best explained by which of
.:at varies at the side chain is known as "allo-threonine." the following explanations?
A. Only 25Vo exists as a zwitterion in the body.
1 . What is the stereoconfiguration for isoleucine? B. 15Vo of synthetic isoleucine does not have the
A. 2R. 3R correct side chain.
B. 2R, 35 C. In synthesizing isoleucine, the two chiral centers
C. 25,3R result in four stereoisomers being formed. Only one
D. 25, 35 of the fbur is biologically usable.
D. In synthesizing isoleucine, the two chiral centers
result in eight stereoisomers being formed. Only
2. If the side chain chiral center were changed, the new two of the eight are biologically usable.
structure would be which of the fbllowing?
A. An enantiomer of isoleucine.
B. A diastereomer of isoleucine.
C. An epimer of isoleucine.
D. ldentical lo isolcucine.

3. Most naturally occurring amino acids have which


stereochemical orientation ?

A. R
B. S
C. E
D. Z

Copyright e hy The Berkeley RcviewG GO ON TO THE NEXT PAGE


Passage ll (Questions 8 - 14) 10. What is the stereochemistry of Compound P?

In two controversial laboratory studies, Compound P,


A. 35, 45
shown in Figure I below, has been determined to reduce
B. 3R, 45
constipation in the Southern European Red-Eared Jumping
C. 35, 4R
Lizard during mating season. The effects are less significant D. 3R, 4R
during courting periods and does nol occur at all during the
first three days following the New Moon. The two studies
compared this particular stereoisomer, along with other 1 1. The following structure relates in what way to
stereoisomers and structural isomers of Compound P, to Compound P, the Southern European Red-Eared
determine the effects of the drugs. The dosages used were
Jumping Lizard' s constipation medication?
held constant between compounds. The disagreement
between researchers came in determining the binding activity
ofeach stereoisomer and its subsequent reactivity.
HOH ;"

cH 2cH 3
H H HO Ct-12CHs -!:
*rt
A. It is an enantiomer of Compound P
OH B . It is a diastereomer of Compound P

Figure 1 Compound P
C. It is identical to Compound P
D . It is a meso structural isomer of Compound P
The exact mechanism for the constipation reducing jr-
behavior is not known, but it is speculated to work in ::i*.
conjunction with sex hormones to induce muscle relaxation.
In other studies, the compound has been applied to the 12. What is the stereochemical orientation of the followins
c,-

compound?
abdomen of the Saskatchewan Green-Nosed Squatting Frog to "1::r'

test for similar effects. To date, no solid conclusions have - i'


been formed as to the effect of Compound P on constipation
lr: :.;

in other organisms. Being such an important chemical to the


bowel process of reptiles and amphibians everywhere, a great
-"&
deal ofresearch is destined to be under way. Researchers have
continued to develop other structural isomers that will
hopefully show similar effects in creatures such as the very
A. 15, 25 :Ft
rare Yellow-Tongued Sabertooth Crotch Cricket.
B. lR, 25
C. 15,2R
8 . If treated with PBr3, the OH groups can be converted D. lR, 2R *'it:r:

into Br groups through a reaction which proceeds by an


5512 mechanism. The dibromo product formed from lEri ";

Compound P would show which of the following chiral lf-:r:


assignments?
13. Enzymatic active sites are all of the following -i:r':
EXCEPT:
A. 35, 45
A. chiral speciiic.
B. 3R, 45
B. size specific. L:,
C. 35, 4R
D. 3R., 4R
C. functional group specific.
D. isotope specific.

9 . Assuming Compound P is made by treating an alkene


with KMnO4 in basic water, what is the geometrical
14. The enantiomer of Compound P has:
orientation of the alkene precursor to Compound P?
(KMnOa adds two hydroxyl groups with syn orientation) A. the same boiling point as Compound P

A. E. B. a higher melting point than Compound P.


C. A lower density than Compound P.
B. Z.
D. The same specific rotation as Compound P.
C. Either, the reaction has no stereoselectivity.
D . Neither, the reaction has no stereoselectivity.

Copyright @ by The Berkeley Review@ GO ON TO THE NEXT PAGE I.lrr"


Passage lll (Questions 15 - 21) 16. Which of the following explanations does NOT account
for an observed optical rotation greater than that of a
Isomers are compounds with identical formulas but a positive literature value for the pure species?
different arrangement of atoms in space. There are two basic
types of isomers: structuraL isomers (also known A . Both enantiomers are present in an unequal amount.
as
constitutional isomers) and stereoisonters (which can further B. The solution is too concentrated.
be categorized as either configurational isomers or C . The cell for the polarimeter is too long.
conformational isonters). Structural isomers are defined as D . The wavelength of light being used is greater than
raving different connectivity of atoms, also referred to as that of the standard sodium light used.
different bonds. Ethanol and dimethyl ether are an example of
structural isomers.
Stereoisomers are isomers which have the exact same
'londs,
but diff'er by the position of their atoms (substituents) 17. How many degrees of unsaturation are present in a
'.rithin space. Stereoisomers can be classified as either compound with formula C11H21NO2?
-tptical isomers or geometrical isomers. Optical isomers 4.4
rave different spatial arrangement due to asymmetry about an
.tom with in the molecule. A good example of optical
B. 3
.somers are the R and S enantiomers of a given molecule.
c.2
3eometrical isomers have a different spatial arrangement of D. I
:ioms with fespect to a molecular plane. Cis and trans
:utene are a good example of geometrical isomers.
A common, and often challenging, task in an organic
:remistry laboratory is to separate and distinguish the 18. How many structural isomers are possible for the
)omers formed fiom a chemical reaction. Of all isomeric formula C6Hla?
:ixtures, it is easiest to separate and distinguish structural A. rl
somers, which have varying physical and chemical B. 1
::operties. Geometrical isomers also have different physical
::operties, and can be separated readily. They can often be
c. 6

::stinguished from one another by their melting or boiling


D. 5

: rints. Of isomers, optical isomers are the most difficult to


,:parate. To separate optical isomers, a pure chiral material
-"n be used in a chemical reaction, which is then followed by
::ecipitation or extraction. Once separated, the specific 19. The chiral centers for the following molecule are:
::tical rotation value may be used to identify the enantiomer (note: OH is located on carbon 2)
'-.3t was isolated.

Other traditional laboratory techniques can be modified to


,:ploy chirality to help separate optical isomers from one
..other. As a general rule, the distinguishing features
: -'lween isomers can be used to separate them in a laboratory
::ocedure, although some are considerably easier than others.
-:r identifying a purified isomer, spectroscopy and physical A. 2R, 3R
r: lperties are most often employed. B. 2R, 35
C. 25, 35
- 5. The pure R enantiomer of some compound has a D. 25, 3R
specific rotation of +32.2'. A sample you make in lab
has a specific rotation of +16. l'. This is best explained
by which of the following mixtures?
A. 100Vo R enantiomer present.
B. 7 5Vo R and 25Vo S enantiomers present.
20. Which of the following CANNOT form optical
isomers?
C. 50Vo R and 50% S enantiomers present.
D. 25Va R and l5Vo S enantiomers present. .
A A four carbon gem diol
B. A four carbon vicinal diol
C. A fourcarbon secondary amine
D. A four carbon secondarv alcohol

- ,pyright @ by The Berkeley Review@ 223 GO ON TO THE NEXT PAGE


2 L. How many stereoisomers are possible for the following Passage lV (Questions 22 - 28)
structure?
Stereocenters are important features in chiral organic
OH
compounds. Stereocenters are responsible for physical
properties, chemical reactivity, and biological function. An
example of the correlation between physical properties and
chirality is shown with the stereoisomers of tartaric acid.
Drawn in Figure 1 is the generic structure of tartaric acid. and
a data table of the physical properties of the stereoisomers:

HO
4.32
B. 64 (oH)
c. 128 (oH)
D.256
o
Figure 1 Tartaric Acid

Table 1 shows the physical properties of the thre;


stereoisomers of tartaric acid and the physical properties c:
the enantiomeric mixture.

Form m.p. 0D Density H2O sol. @ 20'C

(+) 168-170 + 12" 1.7598 139 g/l00ml


(-) 168-170 12' 1.7598 139 g/100ml
meso t46-148 0" r.5996 125 g/l}0mL
(t) 205-207 0' 1.7880 2l g/l}0mL
Table 1

The differences in physical properties can be attributed ::


lattice formation in the solid phase. The compound can pa;r.
most easily when it is symmetric. This can be seen in t:* {
density and the melting point of each stereoisome: ts
Examples of biological activity are numerous. A comni:,'r
f
example involves the digestion of D-sugars. Our bod,v car
D
metabolize D-glucose yet it cannot metabolize L-glucose i::e
enantiomer of D-glucose). A recent example involves ne
compound L-Dopa. L-Dopa is used as an anti-Parkinson dr-r4
while D-Dopa has no effect and can in fact be toxic in larg
enough doses. L-Dopa is drawn in Figure 2 below.
HO NHz

Figure 2 L-Dopa

22 . Which of the following physical properties would hi'*u


-{"
the same value for morphine and a diastereomer ui{
E"
morphine?
A. Melting point.
I
D"
B. Density.
C . Molecular mass.
D. Specific rotation.
Copyright @ by The Berkeley Review@ GO ON TO THE NEXT P.{
How can a compound with an optical rotation of 2 6. What can be concluded about the packing of molecules
+233.0'be discerned from a compound with an optical in the crystal lattice of the stereoisomers of tartaric acid?
rotation of -121 .0"2
A. The meso compound packs most tightly while the
A . The intensity of the light is greater with the (+) enantiomer and (-) enantiomer pack the same.
posilive optical rolation.
B. The meso compound packs least tightly while the
B. The sample with +233.0" oprical rotarion when (+) enantiomer and (-) enantiomer pack the same"
diluted to half of its original concentration would
C . The meso compound packs most tightly of all of
show an optical rotation of +116.5".
the stereoisomers. The (+) enantiomir packs more
C. The larger the absolute value of the optical tightly than the (-) enantiomer.
rotation, the greater the density of the compound.
D. The meso compound packs most tightly of all of
D. It is not possible to distinguish the two compounds the stereoisomers. The (+) enantiomer packs less
fiom one another. tightly than the (-) enantiomer.

Given that the specific rotation of D-Glucose is +52.6.,


what can be said about the specific rotation of D_
mannose (the C-2 epimer of glucose)? Note: An 27 . How many stereocenters are present in the molecule
epimer is a diastereomer that varies at onlv one camphor which shows an optical rotation of +44.3"?
stereocenter.

^vo cHs

u-f I
oH

so-FH I

CH:
H-1- I
oH
A. 0
B. I
u
-1- I
oH
c.2
CH2OH D. 3

D_Glucose
A. It cannotbe +52.6', -52.6', or 0'.
B. It must be either +52.6', -52.6', or 0'.
C. It is 0'.
D. It is -52.6 ". 28. Which of the following statements best explains why
an R/S enantiomeric mixture has a higher melting point
than pure samples of either the R enantiomer or the S
enantiomer?
How many stereoisomers are possible for penicillin V? A . It requires more energy to break the hydrogen bonds
within a pure compound than within a mixture of
H
compounds.
H'coc6Hsl/ I

t\a.r, B. Enantiomers readily form covalent bonds with one


another.

o -CHr
C . The covalent bonds are weaker when the material is
one pure stereoisomer then when it is a mixture of
,h.y co2H
D.
two or more stereoisomers.
The R-stereoisomer packs more tightly with its
enantiomer than it does with itself.
A. X

8,2
c" 4
D. 8

, :vright @ by The Berkeley Review@ GO ON TO THE NEXT PAGE


Passage V (Questions 29 - 35) 31. The lemon flavored isomer has what stereochemisr,
associated with it?
The Diels-Alder reaction of isoprene upon itself at 100'C
yields two enantiomers of limonene in a fifty-fifty ratio. The
A. One chiral center with R stereochemistry
diene of one isoprene molecule reacts with the less hindered B. One chiral center with S stereochemistry
double bond of another isoprene molecule to form the C. Two chiral centers with R, R stereochemistry
product. Because isoprene is a planar molecule, there is an D. Two chiral centers with S, S stereochemistry
equal chance fbr the reaction to occur on the top face as the
bottom face of isoprene. The Diels-Alder reaction that
synthesizes limonene is shown in Figure 1 below.

32. Which of the following physical properties is MOSI


likely different for the two enantiomers?
A. Boiling point
B. Density
Figure 1 Diels-Alder Condensation Reaction of Isoprene C. Alcohol solubiiity
D. Optical Rotation
The two enantiomers that are formed have similar
physical properties, but have different applications in the
flavoring agent business. One enantiomer has a lemon scent
while the other has an orange scent. The two enantiomers of
limonene are shown in Figure 2 below.
3 3. When hydroborane (BH3) reacts with limonene, ir a.rj
boron to the less hindered carbon of both alkenes ::n
hydrogen to the more hindered carbon of both alken::;,

^"-&""&
What is the major product when hydroborane reacts $:17
R-limonene?

lemon odor orange odor

ca
Figure 2 Enantiomers of Limonene

The percentage of each enantiomer in the product mixture

p,
can be altered only with the addition of another chiral reagent
involved in the transition state. To isolate either the lemon
flavoring or the lirne flavoring, the chosen enantiomer must
be separated from the product mixture by one of several
possible laborator"y techniques that involve chirality.

29. "-u\.4 n-u-=A


Which of the following techniques does NOT work ro
isolate one enantiorner from the plesence of an
enantiomeric mixture?
A. Adding the mixture to a chiral gel in a column
chromatography.
B. Distillation of the product mixture. 34 . What can be concluded about the olfactory receptors
C. Crystallization of the mixrure with the addition of A. They are symmetric (achiral) because thel ;:anr

an enantiomerically pure compound. distinguish between enantiomers.


D . Filtering through an enzymatic membrane. B. They are asymmetric (chiral) because ther :m
d istinguish between enantiomers.

30. Which of the following will lead to a product mixture


C . They are symmetric (achiral) because they c.::nm.
distinguish between enantiomers.
composed of more than fifty percent of one of the
enantiomers (the product mixture not being racemic)'/
D. They are asymmetric (chiral) because they c-rmum
distinguish between enantiomers.
A. Carrying the reaction out with a chiral solvent.
B. Carrying the reaction out with achiral catalyst.
C. Carrying the reaction out at a lower temperature.
D. Carrying the reaction out at a higher concentration.

Copyright @ by The Berkeley Review@ 226 GO ON TO THE NEXT P{


3 5. If the following molecule were the reactant rather than Passage Vl (Questions 36 - 42)
isoprene, how would the results differ?
A chemist intended to study the effect of periphery
chirality on a nucleophilic substitution reaction. To 50 mL
of methanol at 45'C, the chemist added 0.10 moles of
(l S,2R)-2-methylbromocyclopentane in the hopes of carrying
out Reaction 1, which is shown below.

A. The product mixture would no longer be racemic.


HrC Br
IJ CHroH'
HrC
Li
OCHr

&
"'u''
B.
C.
D.
The product mixture would still be racemic.
The products would be achiral.
The products would have four stereocenters each.
O O major minor
product product

Reaction I
The reaction was carried out for one hour at 45"C in an
aqueous solution buffered at a pH of 5.0. Excess methanol
was removed from the product mixture by fractional
distillation under reduced atmospheric pressure. The
atmospheric pressure was reduced to lower the boiling point
of methanol, in the hopes that additional reactivity would be
minimized at a lower temperature. The optical rotation for
the crude product mixture was found to be 0', which is
contrary to what the chemist had expected. The chemist had
expected that the crude product mixture would exhibit optical
activity, although the exact value would be different than that
of the reactant.
The chemist reevaluated the proposed reaction, Reaction
1, and decided that the temperature and the pH should be
changed. Under the reaction conditions used, the proposed
reaction proceeds by a mechanism that is susceptible to
rearrangement. The chemist also failed to consider other
reactions that compete with nucleophilic substitution at
elevated temperatures, such as elimination. Under different
conditions, the chemist found that optical activity could be
retained.

36. The chemist attempted to carry out what type of


reaction?
A. Syl
B. S52
c. Er
D. Ez

3 7. Which of the fbllowing reaction mechanisms wouid


explain a 0' optical rotation?
A . An Spl reaction with inversion of configuration
B. An Sy2 reaction with rearrangement
C. An Sy2 reaction with inversion of configuration
D. An E1 reaction with rearrangement

:r'right @ by The Berkeley Review@ 221 GO ON TO THE NEXT PAGE


3 8. The two products from the proposed reaction are related 42. The following distribution of products can best L,'e
Pas
in what manner? explained by which ofthe explanations?
A. They are enantiomers of one another. _:_
B. They are diastereomers ofone another. ",u,s.", ::[:
C. They are identical to one another. :rtu:

D. HrC CHr
They are different orientations of the same meso
compound. t-
/ \
su'
cHroH maJor
:::::
":

.-> -

"'ry,
":ira
-u
J:i
\,) 'fui:
.F--

3 9. To increase the amount of substitution reaction "tr ."

observed, the chemist would likely change the minor


MI;:
temperature and pH in what way?
A. Increase the temperature and decrease the pH A. The intermediate undergoes a hydride shift. \,
B. Decrease the temperature and increase the pH B. The methyl group on the carbon adjacent to tr:
carbocation influences the attack of methanol.
C. Increase both the temperature and the pH
D. Decrease both the temperature and the pH
C . The methyl group on the carbocation influences r,:
attack of methanol.
D. The intermediate undergoes a methyl shift.

4 0. The small amount of substitution product isolated was \r


found to have both the OCH3 group and the CH3 group
both on the same carbon. This can best be explained in
what way?
A. First an elimination reaction took place followed
by a Markovnikov addition reaction.
B. First a Markovnikov addition reaction took place
lbllowed by an elimination reaction.
C. A hydride shift occuned.
D. A methyl shift occured.
llllus .,:

unit;
urrTI
ill"ri-ullJ

41. PBr3, when added to an alcohol, converts an OH group


into a Br substituent with inversion of configuration at
the carbon. The mechanism is an Sy2 substitution of
the Br for the OH. What alcohol can be used in this
reaction to produce the starting reagent in the proposed
synthesis in Reaction 1?
A. (1R,2S)-2-Methylcyclopentanol
B. (1R,2R)-2-Methylcyclopentanol
C. (1S,2R)-2-Methylcyclopentanol
D. (1S,2S)-2-Methylcyclopentanol

Copyright @ by The Berkeley Review@ 228 GO ON TO THE NEXT P{


f,assage Vll (Questions 43 - 49) 45. Reaction of (2R,3S) 2-bromo-3-methylpentane with
ammonia yields which of the following products?
Nucleophilic substitution is the process by which
-nctional groups on an alkane can be exchanged. It is
A . (2S,3S) 2-amino-3-methylpentane
,:mmonly viewed as a reaction which can proceed by one of B. (2R,3S) 2-amino-3-methylpentane
'.o paihways. The first of the pathways is named S51 I for C. (2S,3R) 2-amino-3-methylpentane
. rate dependence on one reactant. The second pathway is D . A racemic mixture of A and C.
-.-erred to as Sp2 1br its rate dependence on two reactants.
.:.e data in the tables were gathered for reactions in which the
-
-:leophile was varied in two different experiments involving
,'r different electrophiles (one primary and the other 46. A reaction in which the specific rotation of the starting
.::iary). The reactions were monitored using UV material is + 32" and the product (which still contains a
:3rtroscopy wl-rere the magnitude of the rate of disappearance chiral center) is 0" is which of the following?
: the peak corresponding to the electrophile varies directly A. Sp1
,h the reaction rate.
B. SNr2

\ucleophile Reaction rate w/ CH3CH2CH2CI C. S5E1


D. SpE2
H3CNH2 4.7 x 10-2 M/sec
NH: 4.2 x l0-2 M/sec
H3CSH 2.1 x 10-2 M/sec
47 . Which of the following electrophiles is the best choice
H3COH 8.2 x 10-3 M/sec to react with NaOCH3 to yield the following ether?

Table 1 Reactions with n-Propylchloride H

\ ucleophile
H3CNH2
NH:
Reaction rate w/ (Hf C)f CCl
3.3

3.6
x l0-4 M/sec
x 10-4 M/sec
H:C-o {r'"""'
A. (R)-2-chlorobutane
H3CSH 3.3 x 10-4 M/sec
B. (S)-2-chlorobutane
H3COH 3.5 x 10-4 M/sec C. (R)-2-aminobutane

Table 2 Reactions with t-Butylchloride


D. (S)-2-aminobutane

The reaction rates listed in Table I and Table 2 represent


: initial rate ol'cach reaction. All other variables that can
:ct the reaction rate besides the nucleophile, such as 48. Which of the following graphs BEST represents the Cl-
concentration as a function of time for the reaction of
-Derature and concentration, were held constant between
.;S
ammonia with 1-chloropropane?
.

l From the data presented in the passage, which of the


fbllowing is thc best nucleophile?
A. H3CNH2
B. NH3
C. H3CSH
D. H3COH
Time Time+
-->
1 . A nucleophilic substitution reaction proceeds MOST
rapidly with the leaving group on what type of carbon?
A. 1'
B. 2"
c. 3'
D. The reaction rate is independent of the degree of
substitution. Time Time ---+
---------_->

rvright @ by The Berkeley Revier.v@ GO ON TO THE NEXT PAGE


49. Monitoring the following reaction by optical roration Passage Vlll (Questions 50 - 56)
would yield which of the fbllowing graphs?
Br
It is possible to exchange one functional group or. I
HjCH2C
substituted alkane for another by performing a nucleoplu.,:
+ HqCSH -+ substitution reaction. There are two versions of nucleophi,r:
tz-
-/ substitution, known as S,y1 and S,rr'2. The number in ea:r
HjCH2CH2C CH: reaction describes the rate dependence. The rate of an S.;
reaction depends only on the electrophile concentration ;-;
not on the nucleophile concentration. The rate of an S.,l
reaction depends on both the concentration of nucleophile cr;
the concentration of electrophile. The difference between ;*
two mechanisms boils down to the sequence of the steps. -:
Sp1 reactions, a leaving group first leaves followed :'
nucleophilic attack of the carbocation intermediate. In S.,-
reactions, the nucleophile attacks the electrophilic carb':i
forcing the leaving group off of the molecule. I: ,
mechanistic study, a secondary alkyl chloride w4s rsa1ei r,il
Time -------> Time two different nucleophiles to get the same ether product. ; j(
rate data were collected for each.

NaOEt
t - t ---------------- OEt
HOEr.0'

Reaction 1

_-_=> L-l--->HOEI
Time Time pH=5,0"

Reaction 2

Tables I and2 show the initial rate data for three


trials lbr each of the two recctions.

Reaction 1

Rate (M/s) ICH3CH2ONa] IC6H11Cl]


1.32 x 10-2 0.05 M 0.0-5 M
2.63 x 10-2 0.10 M 0.05 M
5.25 x l0-2 0.10 M 0.10 M

Table I Initial Rates for Reaction 1

Reaction 2

Rate (M/s) ICHTCHzOHI lCoHrrCll


1.93 x l0-3 0.20 M 0.05 M

1.95 x l0-3 0.40 M 0,0.5 M


3.83 x 10-3 0.40 M 0.10 M

Table 2 Initial Rates for Reaction 2

Based on the data presented, the nature of the mechan,n:


(whether it follows Sy1 or Sp2 kinetics) can be determl:.:r
The key to the analysis is to observe the change in rate a! 'rrw
nucleophile concentration changes. As a rule, Sy2 reaci,,rn,
are fil\tcr than S5 l reactions.

Copyright @ by The Berkeley Review@ 230 GO ON TO THE NEXT PAG.M


5 0. Reaction 1 and Reaction 2 are best described as what 5 5. Which of the following reactions is most likely to
type of reactions? proceed by an Sy2 mechanism?
A. Reaction I is an Syl; Reaction 2 is an Sp2 A. H3CO- + (H3C)3CBr -+
B. Reaction I is an Sp2; Reaction 2 is an Spl B. H3COH + (H3C)3CBr -+
C. Reaction I is an El; Reaction 2 is an Sp2 C. H3CO- + (H3C)2CHCHBTCH3 -+
D. Reaction I is an Spl; Reaction 2 is anE2 D. H3COH + (H3C)2CHCHBTCH3 -+

5 1. All of the following are associated with Reaction 2


EXCEPT: 5 6. Which of the following energy diagrams corresponds to
A. inversion of the chiral center. an exothermic Sp2 reaction?

B. a carbocation intermediate. A. B.
C. the rate depending on the electrophile.
D. a greater rate when a protic solvent is used than
when an aprotic solvent is used.

5 2. A product mixture from a nucleophilic substitution Rxn Coordinate Rxn Coordinate


reaction on an enantiomerically pure compound that
yields a product distribution of 87Vo R and 13% S can
best be explained by which of the following?
A. The reaction goes purely by an 5112 mechanism.
B. The reaction goes mostly by an Sp2 mechanism
with some Sp1 mechanism transpiring.
C. The reaction goes mostly by an Syl mechanism
with some 51,12 mechanism transpiring.
Rxn Coordinate
D. The reaction goes purely by an SNI1 mechanism.

5 3. If bromine were used as the leaving group from the


cyclohexane in lieu of chlorine, what effect would you
expect on the rate? (Note that a C-Cl bond is stronger
than a C-Br bond)
A. Both the S51l and Sy2 rates would increase.
B. The Spl rate would increase, while the Sp2 rate
would decrease.
C. The Sy1 rate would decrease, while the Sp2 rate
would increase.
D . Both the Spl and Sp2 rates would decrease.

5 -1. Which of the following is the BEST nucleophile?


A. tteCO-
B. H3COH
C. Cl-
D. HCI

Sopyright @ by The Berkeley Review@ GO ON TO THE NEXT PAGE


Passage lX (Questions 5Z - 63) 58. Which of the following compounds is the MOST
reactive when treated with cyanide nucleophile?
- -ucleophilic substitution reactions, the reactivity of
a: ..::,rophile dictates the reaction rate. The reactivity ofthe A. CH3CH2F
..=::-rphile is correlated to the str.ength of the leaving group. B. CH3CH2OC6H5
,..: :lectrophile with the better leaving group is the more C. CH3CH2SCH3
::::tive electrophile, and thus reacts faster and undergoes the D. CH3CH2BT
::re favorabie nucleophilic substitution reaction. A good
.::r ing group is stable once it has left the electrophile, so a
s:able leaving group does not readily donate its electron pair. 59. Which of the following is the MOST srable leavin.
Thts implies that a good leaving group is a weak base. The group?
ueaker the compound is as a base, the stronger its conjugate
acid is. As acid strength increases, the pKu of the icicl
A. HCN
decreases. This ultimately means that the lower the pKu of B. CN.
the conjugate acid o1'the leaving group, the more reactive the C. H3CCH2S-
electrophile. D. H3CCH2SH
Based on the pKs values, it is possible to predict the
relative reactiviry of various electrophiles. The favorability
60. The best explanation of why NaSCH3 is a bette:
of a nucleophilic substitution reaction can be approximated
nucleophile than NaSCH(CH3)2 is which of ri_:
by comparing the pK2 value of the conjugate icid of the
fbllowing?
nucleophile with the pKu value of the conjugate acid of the
leaving group. Equation 1 can be ernployed to approximate a A. Inductive effect of methyl is weaker than isopropr .

reactivity constilnt, C, for the reaction: B. Resonance affects only three carbon fragments
C. Hybridization of carbon varies with substitution
C = 10[pKr(H-Nucleophile) - pKo(H-Leaving group)]
D. Steric hindrance is less with the methyl gr<_rup
Equation 1

A reaction can be classified anywhere front very fbvorable 61. The reaction of sodium cyanide with 2-iodobutane is:
to very unfavorable. The C value can be used as follows to
A. very favorable. li:
approximate the tavorability of a reaction:
B. slightly favorable.
If C > 103, then the reaction is very favor.able
C. slightly unfavorable. Lll a

If 103 > C > l, then the reaction is slightly favorable D. very unfavorable.
If I > C > l0-3, then the r-eaction is slightly unfavorable
t\,,
If 10-3 > C, then the reaction is very unfavorable
62. The reaction of methylthiol (CH3SH) wirh R-2-buta::, .
Tablelists pKn values 1br the conjugate acids of sorne
1
common leaving groups.
is which of the following? -
A. Very favorable.
Acid pKa Acid PKa B. Slightly favorable. a_
HI -r0.5 HCN 9.1 C. Slightly unfavorable.
HBr -8..5 C6H5OH 10.0 D. Very unfavorable.
HCI -1.0 H3CCH2SH 10.5
HF 3.3 Hzo 15.1 63. How can it be explained that fluoride, F-, is a bei:.r
nucleophile than iodide, I-, in eiher solvent but a uc:l;l
Table I nucleophile in alcohol solvent?
The pK6 data given in Table I can be userJ to pr.edict the A . In a protic solvent such as alcohol, F- is hinde::,rr
favorability of a nucleophilic substitution reaction. The by hydrogen bonding, and cannot migrate as we.. r iir"i:
accuracy is within experimental error fbr substitution stuclies.
B. In an aprotic solvent such as ether, F- is hinde,:r
Equation I does not hold well in protic solvents due to by hydrogen bonding, and cannot migrate as wel. LLtt]tri :

variations in nucleophile strength from hydrogen bonding.


C . In a protic solvent such as alcohol, F- exhibits : I
hydrogen bonding, so it cannot migrate as we1l.
57. Which of rhe fbllowing compounds is the BEST r.4
electrophile'l
D. In an aprotic solvent such as ether, F- exhibirs - I
hydrogen bonding, so it cannot migrate as we11.
A. (CH3)3CI
B. (CH3)jCBr
C. (CH3)3CCI
D. (CH3)3CF

Copyright @ by The Berkeley Review@


GO ON TO THE NEXT PACM
Passage X (Questions 64 - 70) 65 . What difficulty arises if Reaction 1 is carried out using
a secondary propyl electrophile instead of the methyl
The strength of a leaving group can be deternined by electrophiie?
:ramining the reaction rate of either the S51 or Sp2 reaction. A. The electrophile exhibits more steric hindrance.
The rates fbr both reaction mechanisms show a linear B. The electrophile exhibits less steric hindrance.
jependence on the concentration of the electrophile. It can be
C. There is the chance of an elimination side reaction.
.nferred from kinetic data that an electrophile with a better D. The chance for an elimination side reaction is
.eaving group undergoes nucleophilic substitution at a faster reduced.
.ate than an electrophile with a worse leaving group. A
:esearcher designed a study that varies the leaving group on an
:lectrophile while keeping all other factors constant. Factors 66. Generally, nucleophiiicity and basicity run parallel to
.hat influence the rate include temperature, nucleophile one another. What can be said about the correlation
,trength, solvent, and concentration. In a valid study, all of between the leaving grouprs basicity and the strength of
.hese factors should remain constant between trials. the leaving group?

A difficulty that can arise with this experiment is A . The less basic the leaving group, the better it is as
a leaving group.
:ompetition between substitution reactions and elimination
:eactions. To alleviate the problem of the competing B . The less basic the leaving group, the worse it is as
a leaving group.
:limination reaction, a one-carbon electrophile is chosen.
i\-hen the electrophile has only one carbon, the substitution C . The more basic the ieaving group, the better it is as
:eaction must proceed via an S52 mechanism. Reaction I a leaving group.

.hown below is a generic reaction representing each of the D. There is no observable correlation between basicity
and leaving group strength.
:irteen triai runs.
Nuc + CH3-X -+ Nuc-CH3 + X
Reaction 1
o/. Which of the following does NOT directly aff'ect the
strength of the nucleophile?
There were fbur nucleophiles and fbur leaving groups A . The nature of the solvent.
-sed in the experiment to account for sixteen combinations. B . The basicity of the nucleophile.
lable I is a matrix showing the four nucleophiles ancl lbur C . The steric bulkiness of the nucleophile.
:aving groups used in the generic reaction. The vaiue listed
r each box in the table is the log of the reaction rate.
D. The quality of the leaving group on the
electrophile.

\x
\
-+
OSO3CH3 I CI OCHr
)JucJ
68. Which of the following could NOT be determined from
H:N -3.82 -3.31 -4.11 No Rx a similar experiment carried out with an Sy1 reaction
CN. 1.39 1.tl L92 No Rx instead of the Sy2 reaction'?

H3CS- -2.16 1.92 )Aa No Rx A. The strength of the leaving group.


H3COH -5.21 -4.80 -5.10 No Rx B. The strength of the nucleophile.
C. The ellect of temperature.
Table 1 D. The effect of varying solvent.

The pKu values fbr the conjugate acids of the leaving


::oups can be used to estimate reactivity. The relative pKus 69. The best explanation tor the lack of any observed
':e pKulg.6gCHr) t PKa(HCl) > PKa(HOSOTCH:) > PKa(HI)' reaction when NH3 was added to H3COCH3 is that:

The rate for the reaction is measured in rnolar per second, A. NH3 is a poor nucleophile.
:erefore the less negative the log value of the initial reaction B. NH3 is a poor leaving group.
:.rte, the faster the reaction. Because the rate of an S52 C. -OCH3 is a poor nucleophile.
-:action depends on both the nucleophile and the electrophile,
D. OCH3 is a poor leaving group.
.ris experiment can be used to determine the strength of both
- -rcleophiles and leaving groups.

70. A nucleophile can also be classifled as which of the


: J . The relative strength of the nucleophiles is best following?
described by which of the fbllowing relationships?
A. ALewisacid.
A. CN- > CH3S- > NHr > HOCH3 B. A Lewis base.
B. CN- > CH3S- > HOCH3 > NH: C. An oxidizing agent.
C. CH3S- > CN- > NH3 > HOCH3 D. A reducing agent.
D. CH3S- > CN- > HOCH3 > NH3

Jopyright @ by The Berkeley Review@ 233 GO ON TO THE NEXT PAGE


Passage Xl (Questions 71 - 77) 7 2. When a secondary alkyl bromide is treated with stronl
base at 60'C, what type of reaction occurs?
It is theorized that under identical conditions, while
rarying only the temperature of a solution, it is possible to A. Sp1
convert a reaction that yields purely substitution product (a B. S1q2

substituted alkane) into a reaction that yields purely c. Er


elimination product (an alkene). Studies have shown that D. Ez
favorable conditions for an elimination reaction involve
higher temperatures (elimination is endothermic and often
has a greater activation energy than the competing
substitution reaction). Elimination is carried out in the
presence of eithel a strong bulky base (82) or a strong acid
(Et) in solution. This implies that weak bases at low 7 3. The chirality of the reactant can BEST be described a.
temperature react as nucleophiles rather than as bases. which of the following?
Reaction i, drawn below, was designed to verify this theory. A. 2R, 3R
Br B. 2R, 35
D C. 25, 3R
H$\l
+ Nuc ---> Product D. 25, 35
H:C CH:

Reaction I
The idea was ro monitor the reaction by the optical
rotation of plane polarized light. Only the SyZ reaction 7 4. In Trial II, the -28" optical rotation for the prodr_:l
shows retention of some optical activity although the exact mixture can BEST be explained by which of ::,:
value of the specific rotation (tolp) is not predictable. Table following explanations'/
I shows the final specific rotations for each of the six trials A . The reaction goes purely by an 5612 mechanism.
of Reaction 1, where either the nucleophile or temperature
were varied. The initial specific rotation for the deuterated
B. The reaction goes purely by an E2 mechanism.
alkyl bromide is +24'. C . The reaction is an Sp 1 reaction with sor:r*
competing elimination reaction side produ:::t
Trial Temperature Nucleophile [cr]p products present.
Trial I l0'c NH: -JO
D. The reaction is an Sy2 reaction with so:rr
Trial II 60"c competing elimination reaction side proc::1.
NH: -28"
present.
Trial III l0'c NaOH -38"
Trial IV 60'c NaOH 0"
Trial V r0"c NaNH2 0'
Trial VI 60'c NaNH2 0'
Table 1 7 5. Which of the following statements is true regarding :u
interchanging of stereoisomers for the reactant?
The two cornpeting reactions when a good nucleophile
is present in solution, in the absence of an acid, are thi E2 A . When the enantiomer of the reactant is used. ::u
and 5512 reactions. The internal alkene is the predominant same geometrical isomers are formed, so either:nc
reactant or its enantiomer may be used.
product when elimination is observed. It is fbund that
deuterium is less acidic than a proton due to the cleuterium B. When the enantiomer of the reactant is us:'L
isotope effect rooted in the shorter bond length associated different geometrical isomers are formed, sc ir
with the deuterium-carbon bond. An E1 r.eaction can enantiomer cannot be substituted for the reactan:
compete if the leaving group is a good leaving group and it C . When a diastereomer of the reactant is used. ::r'c
is situated on a tel'tiary carbon. same geometrical isomers are formed, so either :lrw
reactant or its diastereomer may be used.
71. Based on the data listed in Tabie l. Trial I is D . When a diastereomer of the reactant is usac
predominantly what type ol reaction?
different geometrical isomers are formed. s: ur

A. Spl diastereomer cannot be substituted for the reacta:r-


B. 5512
c. Er
D. Ez

Copyright @ by The Berkeley Review@ GO ON TO THE NEXT P.{GM


An elimination product for this reaction would have Passage Xll (Questions 78 - 84)
what optical rotation?
Not ali stereocenters are chemically reactive. When a
A. +24'
reaction is carried out on a molecule with an unreactive
B. 0"
stereocenter present, there exists the possibility that
c. -24'
diastereomerswill be formed in unequal quantities, due to the
D. -42' asymmetry in the molecule. This influence is referred to as
stereochemical control. Reaction l, drawn below,
demonstrates this principle.
CH2CHj CH2CHj

..t\OH
1. BHr(etrO)
To support the theory that an E,2 reaction mechanrsm rs ---->
2'fuoz ,'r/CH3
taking place, it would be best to use chiral centers on cHs oH (aq)
which carbons of a deuterated 2-bromobutane?
A. Carbon2only +48'
[o]o = +48'
[cr]o [cr]l = +18"
B. Carbon 3 only Reaction 1
C. Both calbon 2 and carbon 3
D. Carbons l, 2 and 3 The two products, Compound A and Compound B, are
nonsuperimposable and are not mirror images. In
hydroboration, the hydroxyl group prefers to add to the less
hindered carbon of the n-bond, so the reaction is referred to as
anti-Markovnikov. The hydroborane prefers to add to the less
hindered fbce of the molecule, which means that the two
products are present in unequal amounts. Their percentages
can be fbund using Equation 1 below. The percenlages
determined using this equation can be referenced against the
quantitative values obtained using GC analysis.

Oobs=xacxa+(l -xu)cx6
Equation 1

uo6, is the observed optical rotation for the mixture and


xu is the mole fraction of component a in the mixture.

The same phenomena can be observed any time a


nucleophile is attacking an ,p2-hybridized carbon of an
asymmetric molecule. This means that unequal amounts of
diastereomers may be observed with Sy 1 reactions,
electrophilic addition reactions, and carbonyl addition
reactions. Reaction 2, shown below, is an Sp1 reaction
involving the forrnation of two diastereomers.
o @
NHr
.$\ ...'St'
:NH:
___l-

[s]o = +62' lulo = +30"


Reaction 2

78. For Reaction 2, what is the observed specific rotation


fbr the product mixture?
A . Greater than 62"
B. Between 46' and 62"
C. Between 30" and 46'
D. Less than 30'

. :yright @ by The Berkeley Review@ GO ON TO THE NEXT PAGE


7 9. What would be the specific rotation for Reaction 1 if 84. How many stereogenic centers (chiral carbons) at: rPa
Compound A is 807o of the diastereomeric mixture? present in the alkene reactant in Reaction 1?
A. 48' A. 0
B. 42" B. 1

c. 33" c.2
D. 18' D. 3

8 0. The two products in Reaction 2 are best described as:


A. enantiomers.
B. epimers.
C. diastereomers.
D. identical. Tlll_t

L\.

8 1. The products of Reaction 1 can be distinguished from


one another by all of the following methods EXCEPT:
A. specific rotation.
B. melting point.
C. retention time on a GC.
D. IR spectroscopy.

82. Which of the following structures best represents the


most stable conformer of product A of Reaction 1?
A' B'Hn
ay
M.o/>cHt ""#
,GH.CH3

HrcH2i
C.
OH CH2CHj 9Hr

cH2cH3

8 3. What are the orientations of the three chiral centers in


the reactant in Reaction 2, starting with the chirai center
on which the iodine is attached and moving clockwise
around the cyclopentane?
A. S,R,S
B. R,R,S
C. S,S,S
D. R,S,S

Copyright @ by The Berkeley Review@ GO ON TO THE NEXT P{GmI


Passage Xlll (Questions 85 - 91) 85 . All of the following reagents, when added to Compound
3a, result in a product with more asymmetric carbons
A chemist sets out to perform a multistep synthesis. than compound 3a EXCEPT:
lhe first step, Reaction 1, is a standard Diels Alder reaction. A. Br2(l)/CCla(l)
oo B. Hz(e)/Pd(s)
C. cold KMnO4(aq) at pH = l0

"..\. q"ri,. D. NH3(l)

o o 8 6. Which of the following is Compound 3b?


Compound 1 Compound 2 Compound 3
A. o B. o
Reaction I
..rr( ..r(
Compound 3 represents a mixture of enantiomers. The o
o
:irture undergoes a chirally specific laboratory technique to ,,,,,(
.oiate Compound 3a, shown below, from Compound 3b. HgC HeC
o
C. D.

HsC
o
Compound 3a

In the second step, Reaction 2, Compound 3a is treated


rth meta-chloro peroxybenzoic acid, mcpba, in ether to form
,'.
87 . What laboratory technique would be MOST effective in
!ompound 4. obtaining a pure enantiomer from a racemic mixture?
o A. Adding the mixture to a chromatography column
filled with a gel with both enantiomers bound to it.

o B. Adding the mixture to a chromatography column


filled with a gel with just one of the enantiomers
HgC bound to it.

o C. Distilling the mixture using a vertical column


Compound 4 filled with beads that contained both enantiomers
bound to their surface.
Compound 4 then undergoes Reaction 3 to form D. Using a chirally pure carrier gas in a gas
Compound 5. chromatography experiment.
o o
8 8. What is the major product, most abundant stereoisomer,

-.:e("HicNH2H3;iq"
o o
formed in Reaction 2?
A. B. o
Compound 4 Compound 5

Reaction 3 o
In Reaction 4, Compound 5 is hydrolyzed using water at HrC"
90"C to form Compound 6.

H3CHN
OH
HO OH
Hec

Compound 6

Copyright @ by The Berkeley Review@ 237 GO ON TO THE NEXT PAGE


89. All of the steps in the overall synthesis shown in the
passage generate an optically active product mixture Questions 92 through
EXCEPT: descriptive passage.
A. Reaction I
B. Reaction 2 92 . The following molecule has which of the lollou i:g
stereocherhical orientations
C. Reaction 3 ?

D. Reaction 4
ocH3

90. What is a likely side product of Reaction 3, if excess


-oH
amine is used?
A.
A. 2R, 3R
B. 2R, 35
C. 25,3R
D. 25, 35

B. 93. The following molecule has which of the


stereochemical orientations ?

CHr
H3CN NCFI3
HsC

C.

H?CHN ocH3

HO A. 2R, 3R
HsC B. 2R, 35
C. 25, 3R
D. 25, 35
D.

H"CHN 94.
NHCFL
The following pair of molecules can best be described x
which of the following?
HO NHCr-t
HsC

CHr
t?a
HHO CH2CH3

and
91. The final product mixture following Reaction 4 can best
be described as: HOH
A. an enantiomeric mixture with up = Q'. \ts cH2cH3
B. a diastereomeric mixture with ap = Q".
C. a diastereomeric mixture with up * 0".
D. a meso mixture with up * 0". CH:
A . Diastereomers
B . Enantiomers
C. Epimers
D. Anomers
Copyright @ by The Berkeley Review@ GO ON TO THE NEXT PAGI f,r:
9 5. How many stereoisomers are possibie for the molecule 100. Which of the following compounds does NOT show
1,2,3-trifl uoropentane? any optical rotation of planar light?
4.2 A. 2R, 3R dibromobutane
B. 4 B. 2R, 35 dibromobutane
c. 6 C. 2R, 3R dibromohexane
D. 8 D. 2R, 35 dibromohexane

,6. Addition of KMnO4(aq) at pH = l0 generates a vicinal


diol with syn stereoselectivity. What does the addition
of KMnO4(aq) at pH = l0 to E-2-butene would yield
which of the following?
A. Two enantiomers
B. Two diastereomers
C. Two meso compounds (not identical)
D. One meso compound

9 7. Which of the following compounds is optically active?


A. 2R,3s-dibromobutane
B. 2R,4s-dibromopentane
C. 2R,4s-dibromohexane
D. cis-1,3-dichlorocyclohexane

9 8. Which of the following compounds CANNOT be


optically active?
A. 2-chlorocyclopentanol
B. 2-chlorocyclohexanol
C. 3-chlorocyclohexanol
D. 4-chlorocyclohexanol

1.D 2.8 3.B 4.C 5.8 6.8


9 9. Only twenty-five percent of synthesized isoleucine can 1.C 8.C 9.B l0.B 11.B 12.4
be used biologically. This is best explained by which 13. D 14. A 15. B 16. A 17. C 18. D
of the following explanations? 19. B 20. A 21. B 22. C 23. B 24. A
25. D 26. B 21. C 28. D 29. B 30. B
A. Only 25Vo exists as a zwitterion in the body. 31. A 32. D 33. A 34. B 35. A 36. A
B. 15Vo of synthetic isoleucine does not have the 31. D 38. B 39. B 40. C 41. B 42. B
correct side chain. 43. A 44. A 45. A 46. A 41. A 48. A
C. In synthesizing isoleucine, the two chiral centers 49. A 50. B 51. A 52. B 53. A 54. A
55. C 56. B 57. A 58. D 59. D 60. D
resuit in four stereoisomers being formed. Only
61. A 62. D 63. A 64. A 65. C 66. A
one of the four is biologically coffect. 61. D 68. B 69. D 70. B 1r. B 72. D
D. In synthesizing isoleucine, the two chiral centers 73. B 14. D 75. A 76. B 77. C 78. B
result in eight stereoisomers being formed. Only 19. B 80. C 81. D 82. A 83. C 84. B
one of the eight are biologically correct. 85. D 86. A 87. B 88. B 89. A 90. C
91. C 92. A 93. D 94. A 95. B 96. A
91. C 98. D 99. C 100. B

Copyright @ by The Berkeley Review@ LIFE BEYOND CHEM STARTS NOW


Stereochemistry & Nucleophilic Subst'n Passage Answers
Choice D is correct. Using the Cahn-Ingold-Prelog rules for substituent priority and drawing
the appropria:.
arcs, the chiraiity for the two stereogenic carbons of isoleucine is determined as iollows:

H:9 H Priority #4 in back


(an as isposition)
*HrN Countercioskwise Arc
Priority #4 in back CH2CFI3 ChiralitY isS
"'
(an as lsposition)
Countercloskwise Arc
Coz-
.'. Chirality isS
Isoleucine
Both of the chiral carbons have S chirality, which makes the compound 2S,3S. It wouid be swell of you:-
choose D. The chiral center on carbon two of an amino acid must be S according to the rules discussed in :: .
Passage, so choices A and B could have been eliminated eariy. Regardless, the chiral center on the side cha-:
of the amino acid must be solved for. Determining R and S ii actuiily rather simple when you get the hang .:
it. The key is to find a method that works for you and hone it in by repeated .rr". '
2. Choice B is correct. If the side chain (carbon three of isoleucine) were to change the orientation of its chr.-
center while carbon 2 retained the orientation of its chiral center, then only
ot" o] the two chiral centers lyoL-:
differ between the two compounds. The two stereoisomers (isoleucine and the other compound) would ::
classified as diastereomers. The correct choice is B.

Choice B is correct. This question can be solved from straight memorization. Naturally occurring amino ac:-.,
are "L" as in life and natural. It is also stated in the passage that naturally occurring amino acids har'- :
stereoconfiguration and that S stereochemistry is associated with L-amino acids. The besi answer is choice B

4. Choice C is correct. With H as the side chain, carbon two of glycine (the alpha carbon) has two hydrog..
-,
attached, thus there is no chiral center present on glycine. Neither of the two carbons in glycine have::-:
different substituents attached. The absence of a chiral center in glycine results in an opticai rotation o: ,
Choose C and be happy.

5. Choice B is correct. Enantiomers are nonsuperimposable mirror images, thus ail of the chiral centers ,:.
different between the two structures. If all of the chiral centers are reversed, then the specific rotation shc*-:
be completeiy reversed, which would lead to a value of +62" rather than -62". It would be terrific if you \{e:- ::
choose B. Enantiomers always have the same absolute value for the specific rotation, only the sign (direc-* :
of rotation) differs.

6. Choice B is correct. As stated in the passage, the side chain of threonine is an alcohol (eliminating cholct ..
and it is chiral (eliminating choices A, C, and D), all choices except answer B are eliminated. Only chor:t S
contains a carbon that is asymmetric (chiral center).

7' Choice C is correct. Isoleucine contains two chiral centers, one for the alpha carbon and one in the side ch,*-
Plugging into the stereoisomer equation 2n where n is the number of chiral centers, there are four pos'-- -r
stereoisomers for the isoleucine structure. Because isoleucine contains two chiral centers that must have s1.;- -,-
orientation, only one of the four stereoisomers wili have the correct chirality to be biologically usable. Ti.,-" ,.
stated in the passage in two fragments. The best choice is C.

8. Choice C is correct. PBr3 converts the OH group of an alcohol ir-rto a Br group through an S1.;2 reaction. Bec: --,o
the reaction is by way of an S52 mechanism, the chiral centers inverts. If you recall, chiral centers inr e :: -Ln

Sp2 reactions, but not S1l11 reactions. This means the product shows stereochemistry of 35, 4R. Choose C fc: --:':
sensation of correctness and satisfaction.

Copyright @ by The Berkeley Review@ 240 STEREOCHEMISTRY EXPLANATIC,i\]$


9. Choice B is correct. If you rotate the original structure so that the hydroxyl groups are syn, then the two alkyl
groups of the molecule also have syn orientation. The original alkene must therefore have the two alkyl
groups cis to one another, resulting in Z geometrical alignment. Pick B for best results.
HO

cH2cH3

h B{r R"t"t
"
H
+ cH2cH3

OH cH2cH3 H
Hydxroxyl groups are syn So, utt have been cis
"rlu*ust
10. Choice B is correct. Using the rules of priorities, the following is determined:
Prioritv #4 in back
Clockwise = R
74

cH2cH3

H Priority #4 in front
Clockwise = S

This makes the compound 3R,45. Choose B for optimal correctness and the satisfaction that goes with it.

11. Choice B is correct. On the third carbon, the OH and H groups have interchanged, so that chiral center has
changed. On the fourth carbon, the ethyl group, hydrogen and hydroxyl group have interchanged, so that
chiral center has not changed. When only one out of two (some, not all) chiral centers change, it is not a mirror
image, nor identical (superimposable), making the two compounds diastereomers. Pick answer B.
s
r o HOH

cH2cH3

e
il ?.,
M HH H HO CH2CH3
mt Compound P Mystery Compound

12. Choice A is correct. Using the rules of priorities, the following is determined.
H3C
H in front, so reverse
chirality from R toS
m!
H in back, so chirality
h is S as shown
ile
:hr This rnakes the compound 15, 25. Choose A.

[3. Choice D is correct. The active site of an enzyme carries out a highly specific function (reaction), so they must
be highly selective in terms of reactivity. As implied by the passage, active sites are highly specific in terms
of chirality. This eliminates choice A. Although it is not stated in the passage, you should know that the
& active site has specific dimensions, so it is size specific. This eliminates choice B. Active sites are highly
for specific for the functional groups involved in a chemical reaction, so choice C is eliminated. Because isotopes
hc show the same chemical reactivity, enzymes are unable to distinguish isotopes. This means that enzyme
active sites are nof isotope specific, making choice D the best answer.
lopyright @ by The Berkeley Review@ 241 STEREOCHEMISTRY EXPI,ANATIONS
't4. Choice A is correct. Enantiomers have the exact same physical properties, such as boiling point,
melting point,
and density. This makes choice A correct and eliminates choices B and C. Enantiomers have the
same
magnitude for specific rotation, but with the opposite sign (in the opposite direction). This makes
choice D an
incorrect answer, and leave choice A as the best choice ini sea of many choices (weil maybe not
many, but four.)

15. Choice B is correct. A specific rotation for an enantiomeric mixture that is positive means that there is an
excess of the enantiomer with the positive rotation (in this case the R enantitmer). If the two enantiomers
were present in a fifty-fifty ratio, then the specific rotation would be 0". This eliminates answer choice C.
Choice D is eliminated because an excess of th" S stereoisomer would result in an overall negative opticai
rotation, making choice D invalid. If the mixture were in fact 100% of the R enantiomer, thei the specific
rotation would be +32.2" ' This is not the case either, so choice A is eliminated. By the process of elimination.
choice B must be the correct answer. The problem could have been solved mathematically as follows:

%R (+32.2) + 7"5 (-32.2) = +L6.1.


%R (32.2) - %S (32.2) = +16.1
32.2 (%R - "/oS) = 16.1
%R - %S = 0.5 = 50/o; and %R + %S = 700'/"
Thus2(%R-%S)=%R+%S
2%F.-2"k5=%R+%S
%R = 3 %S, so o/,R = 75oh and %S = 25"/o

16. Choice A is correct. Because one enantiomer aiways has the opposite optical rotation of the other enantiome:
(one is negative and one is positive with the same magnitude), li Uotn enantiomers are present in
solution, the
absolute value of the rotation must decrease from the absolute value of the pure enantiomer. The observec
optical rotation is an average of the enantiomers in solution. Therefore, choice A cannot account for an obser'ec
specific rotation greater than that of the pure species, because the mixture of the two enantiomers would cause
the value to be less than the literature value (given that the literature value is positive). Choices B and C
both lead to an increase in the observed rotation. The standard rotation is based orrmonochromatic light from;
E
sodium iamp. If monochromatic light from another source is used, it will interact differentlyiwith the
compound, resulting in a different optical rotation. Whether the rotation is greater or lower is uniertain, bu:
because it is possible that the rotation is greater, choice D is eliminated.

17. Choice C is correct. The formula for units of unsaturation is Units of unsat = 2(#C)+1(#N)+2-1(#H) For a
2
2(77)+7(7)+2-7(27)
compound with a formula of C11H21NO2, the calculation is: Unitsof unsat - - 4 -'
22
Pick C. Remember that oxygen is not included in the formula for degrees of unsaturation. Iflrou didn't rememte:
the formula, then count the bonding electrons for the compound (44 for the eleven carbons, 21 for the hydrogerr.
3 for the nitrogen, and 4 for the two oxygens). There are a total of 72 bonding electrons, so the compound has lr
bonds total. There are 35 atoms in the molecule, therefore only 34 bonds (minimum) are necessary to connect r;,e
atoms and form the molecule. There are 36 bonds when only 34 are needed, thus there are two extra bonds. Th-
long-winded path still leads to answer choice C.

18. Choice D is correct. The only way to do this problem is to count isomers systematically. Start with the longe=:
chain (6) and list all the possible isomers. Then look at chain lengths of one less carbon (5) and list all of tho-
possible isomers. This continues until the possible backbones are depleted. For this question, the carbi:
backbone is shown for all of the possible isomers. There are five isomers total, so pick D.

longest chain = 4 carbons


rongest chaln
^h.i- _
rnnnact chain
longest = 5< carbons
^-, A
= 6 carbons

c-c-c-c-c-c c-c-c-c-c c-c-c-c_c


\l
c_c_c_c c_c_c_c
llrtt
ccccc
-
Copyright @ by The Berkeley Review@ 242 STEREOCHEMISTRY EXPLANATIO\5
19. Choice B is correct. It is stated that the OH substituent is on carbon 2. The OH is attached to an R chiral center
while the Br is attached to an S chiral center. Pick B. The rationale is shown below:
I
Br
If the Br and H were switched, the
compound would be R. Because they
are not switched, it must be an S.
Li\ Looks like an S center from
the arc, but because H is in
front, it reverses to R.
,,F,,,Y

20. Choice A is correct. A geminal diol has two hydroxyl groups on the same carbon (think of 'geminal' as being
equivalent to gemini, meaning that the two OH groups are twins.) Because the two hydroxyl groups are on the
same carbon, that carbon cannot be asymmetric. As a result, the molecule is achiral, so it cannot form optical
isomers. This makes choice A the best answer. A vicinal diol has two hydroxyl groups on neighboring carbons
(think of 'vicinal' as meaning vicinity or vicino, the Spanish word for neighbor). Because this ensures that the
second carbon has a hydroxyl group, the second carbon must be asymmetric. As a result, the molecule is chiral,
so it can form optical isomers. This eliminates choice B. Four carbon chains with a secondary functional group,
whether it is an amine or alcohol, have a carbon with four different substituents (H, CH3, CH2CH3, and the
group), so they are chiral. Being chiral, four carbon chains with a secondary functional group can form optical
isomers. This eliminates choices C and D. Choice A is the best answer.

21. Choice B is correct. In polycyclic systems, if there is no plane of symmetr1z, then all tertiary and quaternary
carbons are stereogenic centers. In addition, any secondary carbons with a functional group are also stereogenic
centers. The compound has two quaternary carbons, three tertiary carbons, and one secondary hydroxyl group.
This means that there are six chiral centers. Given that there are two possible orientations at each chiral
center and the compound is not meso, the total possible number of unique stereoisome rs ts 26, which is 64, Pick B,
and feel the warmth of correctness.

22. Choice C is correct. As indicated in Table 1 in the passage, diastereomers show different physical properties.
The melting points of diastereomers are different, because the molecules pack into their respective lattice
structures differently. The density is different between isomers, because the two diastereomers have different
conformations that also pack into their respective lattice structures differently. The optical rotation of
diastereorners must be different given the fact that they are identified by their differences in optical rotation.
This eliminates choices A, B, and D. Because diastereomers are isomers, and they have the same molecular
formula, thus they have the same molecular mass. The correct answer is therefore choice C.

23. Choice B is correct. When using a polarimeter, an observed optical rotation of +233.0" and -1.27.0'would yieid
the same reading (given that a full circle is 360"). To discern one optical rotation from the other, the sample
should be diluted to reduce the rotation. If the actual optical rotation is in fact +233.0', then the lower
concentration would show a rotation less than +233.0' (less clockwise). If the actual optical rotation is in fact
-727.0", then the lower concentration wouid show a rotation less than -727.0" (less counterclockwise). If the
solution concentration were cut in half for instance, the rotation would be either +116.5" or -63.5". The change in
rotation can therefore determine the original rotation value. The only answer that indicates changing the
concentration is choice B.

24. Choice A is correct. If D-glucose has an optical rotation of +52.6' , then the enantiomer of D-glucose (L-glucose)
must have an optical rotation of -52.6'. Mannose, the C-2 epimer of glucose (the diastereomer of glucose that
only differs at carbon two) is neither of these two structures (L or D glucose), thus it does not show an optical
rotation of either + 52.6' or -52.6". Mannose is chiral and not meso, so it cannot have an optical rotation of 0".
The best answer is choice A.

25" Choice D is correct. The number of stereoisomers (assuming that there is no meso structure), can be determined
by raising 2 to the power of the number of chiral carbons (stereocenters). There are three chiral carbons
(stereocenters) associated wiih penicillin V, thus there will be eight (23) stereoisomers for the structure of
penicillin V. The 2n formula represents the maximum number of stereoisomers. For every meso structure, you
must subtract one frorn the total. Choice D is correct.

Copyright O by The Berkeley Review@ 243 STEREOCHEMISTRY EXPLANATIONS


26. Choice B is correct. The greater the density of a compound, the more tightly packed the compound
is in its
crystal lattice. This means that this question is a read-the-chart-to-fitr"a-tnu-d"r-rsity question.
The mesc
compound is less dense than the other compounds according to the data in the table, thus it
must pack least
tightly of all of the choices. The best answer is therefore choice B.

27. Choice C is correct. Stereocenters can be identified quickly as sp3 carbons with four different
substituents
attached. In camphor, there are two carbons that fit this deicriptibn. The correct choice is answer
C. Dral'n
below is camphor with the two chiral carbons (stereocenters) labeled:
Top Side

CH:

28. Choice D is correct. If the hydrogen bonds in a pure compound were stronger than the hydrogen bonds in a
mixture of stereoisomers, then the intermolecular forces would be greate"r in the p,rr"- .orripound. As a
consequence, the pure compound would have the higher melting point, which is the exact opposite of the
premise. Choice A is an untrue statement, and thus it is eliminated. Physical properties, such as'melting point
result from intermolecuiar forces, not covalent bonds. Enantiomers are no
with one another than diastereomers. Choice B is an untrue statement that -ore lit ely to form covaleni bonds
does not explain the observei
melting points. Choice B is eliminated. Covalent bonds are not affected when a .o*porr1d melts, so choice C
should be eliminated. The best explanation is choice D, because when the moleculei pack more tightly, ther
exert stronger forces on each other. Because the enantiomeric mixture (R with S) has a higher
-"itlng po*,
than the pure enantiomer (R with itself), the forces are in fact stronger in the enantiomeric mixture than the
pure compound. This makes choice D the best answer.
]!

29. Choice B is correct. To separate enantiomers from one another, the medium must be chirally pure (be chiral
with only one enantiomer present). The best method is the use of chiral gei in colum. .hro^uiography. The
two enantiomers exhibit different migration rates down the column, because the two adhere to thJcoiumn to a
different extent. Choice A is a valid methocl. Distiilation r,vill nof separate the two enantiomers, so choice B
is the correct answer choice. The mixture can be selectively crystallized with a pure R or pure S compound. ThL
is often carried out with tartaric acid. Enzymes are chiral, so chiral .o-porrr.i, pass thiough an enzyme filter
at different rates. This makes choice C and D vaiid.

30. Choice B is correct. To prevent the product mixture from being racemic, chirality must be present in the
transition state. A change in temperature does not affect the chirality of the products. Choice Cis eliminated.
The concentration does_not affect ihe alignment of the molecules in the transition state, only the frequency witi.
which the reactants collide to form the transition state. This eliminates choice D. The presence of a chira-
center in the solvent cloes not affect the chirality of the transition state unless the soivent is involved in the
transition state. Choice A cannot be eliminated yet, but it is not a likely choice. The only change that vr'i--
definitely affect the distribution of enantiomers is the addltion of a chiral catalyst which affects the
transition state. This is the whole idea behind the activity and specificity of enzymes (chiral catalysts) i-r.
biological reactions. The best answer is therefore choice B.

31" Choice A is correct' Both enantiomers have oniy one stereocenter, therefore choices C and D are eliminatec
The lemon fiavored extract is the structure on the left, which has its lone stereogenic center in R configgration
This eliminates choice B and makes choice A the correct answer.

Copyright @ by The Berkeley Review@ .)At


STEREOCHEMISTRY EXPLANATIONS
Choice D is correct. The one physical property that definitely changes with the chiral center is the optical
rotation. The specific rotation of a product mixture measures its enantiomeric purity (percentage of each
enantiomer). The boiling point, density, and solubility in a given solvent does not vary between enantiomers.
These physical properties can vary between diastereomers, but enantiomers are identical in their packing and
intermolecular forces, unless the alcohol solvent is chiral and optically pure. The best answer is choice D.

Choice A is correct. The regio-chemistry is correct in all of the answer choices (the boron has attached to the
less hindered carbon of both alkenes). Hydroboration, you may recall, proceeds with anti-Markovnikov
regioselectivity. The reaction calls for the R enantiomer, which has the alkenyl group sticking out of the
plane (the structure on the left in Figure 2, labeled "lemon odor", has R stereochemistry). This eliminates
choices C and D. Because the boron and the hydrogen add syn to one another, the methyl substituent on the ring
must be trans to the bridging boron. This eliminates choice B and makes choice A the best answer.
nrC4,

First addition Second addition


of BH3(Et2O) of BH"(EI2O)
H-B

R stereocenter -> Trans addition to methyl Choice A

Choice B is correct. The two enantiomers have a different flavor (and thus different smell), so they must bind
the olfactory receptors differently. This eliminates choices C and D. Because they recognize the difference
between the two enantiomers, they too must be asymmetric (and thus chiral). The correct answer is choice B.

Choice A is correct. The isoprene molecule has no chiral centers, so the product is a racemic mixture. The
molecule listed in the question as an alternative reactant has a chiral center present that will influence the
orientation in the transition state. The product would contain three chiral centers (one new chiral center
formed and one each present in the two reactant molecules). The product would be present in a mixture of
diastereomers. Diastereomers cannot be present in a racemic mixture, therefore the product mixture would not
be racemic for the new reaction. The best answer is choice A.

Choice A is correct. The proposed product is a mixture of diastereomers formed from a nucleophilic substitution
reaction. Because two diastereomers are formed, the proposed reaction must have been predicted to proceed by
an Sp1 mechanism. The best answer is choice A.

Choice D is correct. Any nucleophilic substitution reaction that proceeds with either inversion of a chiral
center or retention of a chiral center remains chiral, and thus is optically active. This eliminates choices A, B,
and C. The product from elimination is an alkene, which has no chirality, and thus no optical activity. The
correct answer is elimination, choice D. The rearrangement clause, although true, has little bearing.

i8. Choice B is correct. There are two chiral carbons present on each compound. The tertiary carbon retains its
chirality between stereoisomers, but the methoxy carbon has different chirality in the two stereoisomers. This
means that one out of two of the chiral centers differs, making choice B the best answer.

39. Choice B is correct. To increase the amount of substitution product that forms, the amount of elimination
product that forms must be reduced. The elimination reaction is by way of an E1 mechanism, because no strong
base is present. To reduce the amount of E1 product, the amount of acid should be reduced, and the reaction
should be carried out at a lower temperature. This means that the temperature should decrease (elimination is
favored at higher temperatures) and the pH should increase. The best answer is choice B.

40. Choice C is correct. There is a secondary electrophile, protic solvent, and a poor nucleophile present, so the
substitution reaction takes place by an SX11 mechanism. The presence of the methyl and methoxy groups on the
same carbon can be explained by rearrangement. There is a secondary carbocation formed when the leaving
group leaves. When the hydride shifts, a tertiary carbocation forms. This is a more favorable intermediate, so
the reaction proceeds via a hydride shift before the nucleophile attacks. The best answer is choice C.
Copyright @ by The Berkeley Review@ 245 STEREOCHEMISTRY EXPLANATIONS
41, Choice B is correct. Given that the conversion of the hydroxyl group into a bromine goes with inversion o;
stereochemistry, the alcohol must have opposite chirality at carbon 1 as the bromoalkane in Reaction 1. The
passage states that the reactant is (1S,2R)-2-methylbromocyclopentane, so the alcohol precursor must har.e
chirality of 1R and 2R. The best answer is B. The reaction and chirality is shown below.

#
CFL

d
H in back on C-2,
so take as is: R Br
.)'rrOU .,rrr\OH ______*
PBre

42.
H in front on C-1,
so reverse it: R

Choice B is correct. The preference for the major product can be attributed to steric hindrance in the transition
I
illl'
state (the transition state is asymmetric). The carbocation is sp2-hybridized,, so it is planar. The only steri:
hindrance comes from the adjacent methyl group (which is above the ring). The adjacent methyl, by beir.e
above the ring, influences the nucleophile to attack from below the ring. The best answer is choice B.

43. Choice A is correct. The first reaction in Table 1 proceeds by way of an 5512 mechanism, because the
electrophile is a primarv alkyl halide. The second reaction in Table 1 proceeds by way of an 51'11 mechanisrr.
because the electrophile is a tertiary alkyl halide. Only the rate of an 51112 reaction depends on the
nucleophile, therefore to determine the best nucleophiie, the data from the reaction with the primary aik-, -
halide (first set of data in Table 1) should be used. The best nucleophile is the compound that has the fastes:
reaction rate, which according to Table 1, is methylamine (H3CNH2). Choose A if you're a table believer.

44' Choice A is correct. From the data in Table 1, the faster reactions are observed with 1-chioropropane an othe:
electrophiles, so it is a safe and valid assumption that the reaction with the primary aikyl halide proceed.
most rapidly. Choose A.

45. Choice A is correct. The reaction can proceed by either an S511 or Sl112 mechanism with a secondary alk,,'-
halide as the eiectrophile. If the reaction were to proceed by an 51112 mechanism, then the product wouid be
the 25, 35 stereoisomer. Only the stereocenter from which the bromine substituent left underwent a change r,..
chirality (inversion), thus only that stereocenter will show a change in its orientation. If the reaction were tc
proceed by an S1r11 mechanism, then the intermediate would undergo rearrangement, and thus the ammoni"
wouid attack the third carbon leaving an achiral product. Given the answer choices, choices B and C car.:
form, and there is no achirai choice, therefore the reaction proceeded by an SNI2 mechanism. This means tha:
inversion of the second carbon will transpire to yield 25,35. Choose A.

46. Choice A is correct. To have optical activity and lose it during the course of the reaction eliminates choice B
because the 51'12 reaction proceeds with inversion (thus an optically active product is formed). By definitior
the product mixture as described in the question is racemic. A racemic product mixture is associated with th.
51111 reaction. Pick A.

47, Choice A is correct. The ether product has S stereochemistry as drawn and was formed by a substitutic:.
reaction using NaOCH3 as the nucleophile. Sodium methoxide (NaOCH3) is a strong base, and thus it is also a
good nucleophile. Because the nucieophile is good, the reaction must have proceeded by an 51112 mechanisn'.
Because the final product has S stereochemistry, the starting material (electrophile) must have had F.
stereochemistry to form the S product from inversion. This eliminates choices B and D. For the substitutior.
reaction to proceed, the electrophile must have had a good leaving group. Ammonia is not a leaving group
therefore the chlorine leaving group is the better choice. This makes the correct answer choice A.

48. Choice A is correct. The reaction proceeds at the fastest rate during the first segment of the reaction becaus:
initially the concentration of L-chloropropane (a reactant in the rate determining step) is greatest and bot:r
reactants (nucleophile and electrophile) are depleted over time. Chloride anion is the leaving group, thus rls
concentration will increase over time. A11 of the graphs show increasing concentration. Over time, the
concentration of 1-chloropropane gradually decreases, thus the reaction rate decreases graduallyi this resul-
in a slower production of Cl- anion. Graph A best depicts this gradual decrease in reaction rate. Choose A anc
be a wunder sfudent.

Copyright @ by The Berkeley Review@ 246 STEREOCHEMISTRY EXPLANATIONS


49. Choice A is correct. Because the electrophile is a tertiary alkyt halide, the reaction is an Sry1 reaction, which
proceeds with racemization. The optical rotation of the product mixture following an 5511 reaction is 0". This
eliminates choice B. Graph C (the schizophrenic graph) shows the correct final optical rotation, but no
reaction will proceed with the erratic change in rotation. Choice D shows that the reaction proceeds at a
constant rate until the reaction is complete. This would be seen with a zero order reaction, not a first order
reaction' The SIrI1 reaction is first otder, so answer choice D is eliminated. Graph A best depicts the gradual
loss of chirality. The optical rotation will never switch to the opposite sign unless there is inversion *hi"h it
not possible with an sI.I1, reaction. Choose A and make your support group proud.

50. Choice B is correct. The ether product shown in both Reaction 1, and 2 is the result of a substitution reaction, not
an elimination reaction, so choices C and D are eliminated. The data in Table 1 correlates to Reaction 1.
Because the rate of the reaction varies directly with both the concentration of the nucleophile and the
concentration of the electrophile, Reaction 1 must be an 5512 reaction. The data in Table 2 correlates to Reaction
2' Because the rate of the reaction varies directly with the concentration of the electrophile, but does not vary
with the concentration of the nucleophile, the reaction must be an 51111 reaction. The rate of an 51111 reaction
only depends on the concentration of the electrophile, and does not vary with the concentration of the
nucleophile. This means that you must choose B to live up to your potential.

Choice A is correct. Because the rate of Reaction 2 varies directly with the concentration of the electrophile,
but does not vary with the concentration of the nucleophile, Reaction 2 must be an St'tr1 reaction. an S51t
reaction undergoes racemization, not inversion, so choice A cannot apply to Reaction 2. In addition, Reaction 2
has no chirality, so choice A is invalid. The best answer is choice A. A carbocation intermediate corresponds
with an Sp1 reaction, so choice B is valid and thus eliminated. All nucleophilic substitution .eactions,
whether it is an 5111 or S1rJ2 rnechanism, have a rate that depends on the electrophile. Choice C is valid, and
thus eliminated. A protic solvent helps to stabilize the carbocation intermediate and the leaving group, so a
protic solvent increases the rate of an 5511 reaction. This makes choice D valid, and thus eiiminateJ it. Choice
A is in fact the top dog of choices.

52. Choice B is correct. If the reaction were to proceed purely by an 5512 mechanism, the product would be 100% R,
because the reactant is enantiomerically pure and the SNI2 reaction results in complete inversion. If the reaction
were to proceed purely by an S1r11 mechanism, the product mixture would be 50% R and 5Ao/" S, because the
reaction goes through a planar carbocation intermediate resulting in a racemic mixture. The mixture is 87% R
and 13% S, which is closer to the products of an 51'12 mechanism than the products of an 51111 mechanism. It is
not a pure reaction so the best answer is choice B.

53. Choice A is correct. Because the carbon-bromine bond is weaker than the carbon-chlorine bond, it is more easily
broken. This makes the bromine a better leaving group than chlorine. An alkyl bromide is therefore a more
reactive electrophile than an alkyl chloride. With a better electrophile, the reaction is faster for both the
Sp1 and the Sir12 mechanisms, because they both depend on the electrophile. This makes choice A correct.

i4. Choice A is correct. The strongest nucleophile is most willing to donate its lone pair to carbon. The answer
choices include two conjugate pairs. The conjugate base is the better nucleophile of the pair, so choices B and D
are eliminated. HCI is a strong acid while methanol is a weak acid, so methoxide is a stronger base than
chloride. This means that methoxide is more willing to donate electrons than chloride, and therefore
methoxide is the better nucleophile. Pick A and see your score improve.

f5. Choice C is correct. An SNJ2 reaction favors a primary electrophile over second aty ot tertiary electrophile, and
a good nucleophile is required. Choices A and B can both be eliminated, because the electrophiles are tertiary
(and tertiary electrophiles proceed via the Sry1 mechanism). Choice C is better than choice D, because
methoxide is a better nucleophile than methanol. Pick C for optimal results.

56, Choice B is correct. An 5512 reaction proceeds by way of a one-step mechanism, which eliminates choices A and
C. An exothermic reaction has the products at a lower energy level than the reactants, which eliminates
choice D and leaves choice B as the correct answer. The apex of the graph represents the transition state, and
the absence of a valley on the graph implies that there is no intermediate for the reaction. Choice A is an
exothermic 5111 reaction, choice C is an endothermic Sp1 reaction, and choice D is an endothermic Sp2 reaction.
Copyright @ by The Berkeley Review@ 241 STEREOCHEMISTRY EXPLANATIONS
57. Choice A is correct. The best electrophile is the compound with the best leaving group. The best leaving group
is the leaving group with the strongest conjugate acid. In this case, iodide is the best leaving group, beciuse HI
(hydroiodic acid) is the strongest conjugate acid of the choices listed. This makes choice A the best answer.

58. Choice D is correct. This question requires that you identify the best electrophile. Again, the best electrophile
is the compound with the best leaving group. The best leaving group has the strongest conjugate acid, which rn
this question is the bromide leaving group. The ranking of the conjugate acids for the leaving groups are: HBr >
HF > HOC6H5 > HSCH3. It is in your best interest to choose D.

59. Choice D is correct. A leaving group, once it has left an electrophile, must have at least one lone pair (as a
result of the heterolytic bond cleaving). This stipulation eliminates choice A, because the carbon of the cyanic
acid has no lone pair, and the nitrogen does not interact with a carbon to be a leaving group. The most stable
leaving #oup is the weakest base. Of the three choices left, CH3CH2SH has the strongest conjugate acic
(CH3CH2SH2+ is a stronger acid than CH3CH2SH and HCN), thus CH3CH2SH is a weaker base than
CH3CH2S- and CN-. CH3CH2SH is the weakest base of the choices remaining, therefore it is the best leaving
group. Choose D to score a point in this contest of point collecting.

60. Choice D is correct. The difference between the two molecules is the alkyl group. According to the question, the
smaller molecule is the better nucleophile. The inductive effect would predict that the electron donating
methyl groups would make the larger alkyl group more electron donating and thus more nucleophilic. This is
the opposite of what is observed, so choice A is eliminated. Resonance is not a factor, because there is no r-
system. Choice B is eliminated. The hybridization is sp3 in both cases, so the difference in nucleophilicit',-
cannot be attributed to hybridization. This eliminates choice C. Steric hindrance predicts that the smaller
nucleophile has less interference in the transition state, thus iL is a better nucleophile. In this case, steric
hindrance plays a larger role than the inductive effect in the reactivity of the nucleophile. The correct ansn'er
is choice D.

61. Choice A is correct. Sodium cyanide is a good nucleophile and iodide is a great leaving group, therefore thi.
reaction shouid be very favorable. The difference between their pKu values ts 79.6, which implies that the
Ksq for this reaction is near 1919'6 = 4v1919. This defines a reaction that goes completely to product, whic-i.
supports the evaluation that the reaction is very favorable. Do the correct thing, choose A.

62. Choice D is correct. Hydroxyl groups are terrible leaving groups. Thiols are average to poor nucleophiles. The
reaction between a poor nucleophile and an electrophile with a poor leaving group should be very unfavorab:e
by intuitio.n. The difference between their pKu values is -5.2, which implies that the K"O for this reaction'.
near 10-5'2 = 6 x 10-6. This defines a reaction that stays predominanlly as reactant, rirhich supports the
evaluation that the reaction is very unfavorable. Choose D,

63. Choice A is correct. In protic solvents, there is hydrogen bonding, so choice C is eliminated. Equally, in a:,
aprotic solvent, there is no hydrogen bonding, so choice B is eliminated. Hydrogen bonding affects fluoride an:
not iodide, so the fact that fluoride is a worse nucleophile than iodide in alcohol implies that hydroger,
bonding reduces the nucieophilicity of fluoride. This can be attributed to hindrance to migration caused t'i
hydrogen bonding. The best answer is choice A. Choice D can be eliminated, because if it were true, then the
opposite relative nucleophilicity would be observed for fluoride and iodide.

64. Choice A is correct. The strength of a nucleophile can be measured by its reaction rate in a second-orde:
nucleophilic substitution reaction (an SX12 reaction). The nucleophiles are listed in the first column of Table 1
Any other column can be used to determine the relative strength of the nucleophiles, because all other factors i
the reaction are constant. The less negative the value in ihe table, the faster the reaction, and therefore th.r
better the nucleophile. The CN- has the lowest value in all three columns, so the best nucleophile is CN-. Thri
eliminates choices C and D. The question now is to determine whether the ammonia (NH3) or methanc-
(HOCH3) is the better nucleophile. The stronger nucleophile is ammonia, because in each column, the 1e..
negative value is associated with ammonia. The best answer is choice A, CN- > CH3S- > NHg > HOCH3.

Copyright @ by The Berkeley Review@ STEREOCHEMISTRY EXPLANATIO\,(


55. Choice C is correct. As mentioned in the passage, the methyl electrophile is chosen to avoid the complication
of the competing elimination reaction. It is not possible to form a dbuble bond with only one carbon in the
reactant (at least two carbons are required for the formation of a double bond). This makes choice C the best
answer. Steric hindrance increases when using the isopropyl electrophile in lieu of the methyl electrophile,
but that is not necessarily a difficulty. The effect should be uniform u"ross the reaction chart, so choices A and
B can be eliminated.

66. Choice A is correct. The best leaving group is the functional group that takes electrons from carbon and retains
them to the greatest extent. Retaining electrons can also be viewed as not sharing electrons. By not sharing
electrons, an ion or molecule can be viewed as being a weak base. The strength of i leaving group is generally
correlated (in a linear fashion) to the strength of the conjugate acid of the leaving group. aJ un icid becomes
stronger, the conjugate base becomes weaker. This implies that it is valid to compare the strength of the
ieaving grouP in a linear fashion with weakening base strength. This makes choice A, "The less basic the
leaving group, the better it is as a leaving group,' the best answer. Choices B and C are essentially the same
answer, therefore they should both be eliminated.

67- Choice D is correct. The strength of a nucleophile can vary with many reaction features. Depending on its
nature, a solvent hinders a nucleophile to a varying degree. For instance, if a solvent is capable of fbrming
hydrogen bonds, then it will hinder the attack of nucleophiles that are capable of forming hydrogen bonds-.
This can be seen in the differing nucleophilic strength of halides as they are observed in apiotic and protic
solvents. The fluoride is the strongest nucleophile of the halides in aprotic solvents whilelt is the weakest
nucleophile of the halides in protic solvents. This eliminates choice A. The nucleophilicity of a compound can
be correlated to its basicity in terms of a Lewis base. Generally, for a nucleopiile that is more basic than
another, it is the better nucleophile of the two, with steric hindrance responsible for most deviations from that
pattern. This eliminates choice B. The strength of a nucleophile reduces with increasing bulk. This implies
that nucleophilicity can vary with steric hindrance, which eliminates choice C. The only answer choice left is
choice D. The leaving group is independent of the nucleophile in nucleophilic substitution reactions. This
makes choice D the best answer.

68. Choice B is correct. Because the rate of an Sp1 reaction depends only on the leaving group breaking free from
the electrophile in the rate-determining step, the nucleophile is irrelevant .to the reaction rate f-or an Sy1
reaction. The strength of a nucleophile cannot be determined by a rate study in which the nucleophile does not
influence the rate. The rate can vary with changes in the leaving group strength (which can be viewed as
changes in the electrophile), the temperature (temperature always affects the rate of a reaction), and solvent.
It is only the strength of the nucleophile that cannot be determined from the reaction rate data of an S1q1
reaction. Choose B for yet another chance to flash a happy "I just got another one right" smile.

59. Choice D is correct. Choice B can be eliminated immediately, because NH3 is the nucleophile and not the
leaving group. Choice C can be eliminated immediately, because -OCH3 is the leaving group (if it were to
react) and not the nucleophile. The data in Table 1 shows that no reaction was observed each time that the
electrophile was dimethyl ether (CH3OCH3). O.t the other hand, the data in Table tr shows that ammonia
(NHg) is a reactive nucleophile with the other three electrophiles used in the experiment (CH3OSO3CH3,
CH3I, and CH3CI). This implies that the lack of reactivity can be attributed to the electrophile rather than
the nucleophile. The leaving group in the cases where dimethyl ether is the electrophile is a methoxide anion
(-OCH3). Pick D to prosper and score... well score at least.

70. Choice B is correct. By definition, a nucleophile is a lone pair donor, which by yet another definition is a
Lewis base. This makes this question a freebie and the correct answer choice B.

71. Choice B is correct. From the low temperature of the reaction and the retention of optical activity in the
product, it can be inferred that the reaction proceeds by way of an 51112 mechanism. Elimination and S1g1
reactions produce products that lose their optical activity. Choices C and D can be eliminated because the
alkene products would show no optical rotation because they lose both stereocenters in the formation of the
alkene. The products from a reaction proceeding by an Sry1 mechanism in this case would be a mixture of
diastereomers (not enantiomers), which would lead to an optical rotation close to zero. Enantiomers are
obtained if the reactant is symmetric. The best (although not perfect) answer is choice B.

Copyright @ by The Berkeley Review@ 249 STEREOCHEMISTRY EXPLANATIONS


72. Choice D is correct. At high temperature, the predominant reaction is elimination. This eliminates choices C
and D. Because a strong base (NaOH) was used, the mechanism must have been an E2 rather than an E1
mechanism. The best answer is choice D. This can be verified by looking at Trial IV in Table 1, which shou'
identical reaction conditions as the reaction in the question. The loss of optical rotation in the product impl,v
that the reaction was an elimination reaction. A substitution reaction would yield some sort of optical
activity. You have to know that base infers that the mechanism is E2.

/5. Choice B is correct. There are two chiral centers in the reactant, located at carbons two and three. This can be
inferred from the answer selections. The number four priority is hydrogen on both chiral centers and
conveniently it is in the back in both cases. The first chiral center (carbon two) has priorities bromine > deutero
ethyl > methyl which form a clockwise arc. The first chiral center is thus R. The second chiral center (carbon
three) has priorities ethylbromide > methyl > deuterium which form a counterclockwise arc. The seconi
chiral center is thus S. The solution is drawn below:

I
u'\.
n"}V
2
H
carbon 2 = R carbon 3 = S

The correct answer is 2R,35, which makes choice B correct.

74. Choice D is correct. If the reaction proceeded purely by an SNI2 mechanism, then the optical rotation would b,e
the same as was observed for trial I(-36"), a purely 5512 reaction. This eliminates choice A. An eliminatior.
reaction would yield an optically inactive product so the optical rotation observed for an elimination reactio:.
would be zero. This eliminates choice B. The optical activity observed implies that an S52 reaction must har-e
been occurring to some degree. The reduction in optical activity must be attributed to the Presence of sone
impurities (from some side reaction). The best answer is choice D.

75. Choice A is correct. Because the enantiomer is a mirror image of the reactant, it forms a transition state wher, -:
is eliminating that is a mirror image of the reactant's transition state. This means that the products are ali:
mirror images, but without stereogenic centers, they can be rotated to match as identical compounds. Th;
symmetry presents itself in the product as an identical geometric isomer. Because a diastereomer varies at on-i
one chiral center, it is not a mirror image of the reactant when it is eliminating. This asymmetry presents itse''
in the product as d"ifferent geometric isomers. This may not make sense in words so the drawing below shou s
the products.
BrH Br H D
CH"
H$FT
HsC CHs
rotation to correct
alignment H$tjl
HsC
' Elimnation
"
rtrr"g
with
b"* - HEC CH:
Reactant Cis Methyls

Br H CH.

"T,>+"
HsC CHs
rotation to correct
---------=--+
alignment
H\ty
H:C
Elimnation
a strong base
with -
HaC
>< D
Diastereomer Trans methyls

HeC

-..,i';*"
H CHs
rotation to correct
aiignment - H D
Enantiomer Cis Methyls

The enantiomer forms the same products while a diastereomer forms geometric isomers, making the best ansl,:s"
choice A.
Copyright O by The Berkeley Review@ 250 STEREOCHEMISTRY EXPLANATION$
76. Choice B is correct. The elimination product would be optically inactive, because it will lose
both stereocenters
(chiral carbons) upon forming the alkene. This means that the specific rotation of the
alkene product is zero
making choice B the best answer.

77. Choice C is correct. An E2 mechanism involves the removal of a proton alpha to the leaving
group which then
shifts the electrons into a n-bond forcing the leaving group off. The mechanism is concerted, iiplying
that both
events occur simultaneously. This results in very predictable geometry with respect to the
p.oirct a-lkene. To
support the operation of the E2 mechanism, both carbons that become involved rn the n-bond should
be chirally
labeled to trace the reaction. If the mechanism is specific, then the final product is a specific
geometrical
isomer. This is why the reactant is deutero labeled at iarbon three. Therefore-, the best answer is choice
C.

78' Choice B is correct. Because the electrophile is a tertiary alkyl halide, Reaction 2 proceeds by way of an S.u1
mechanism. The two products have specific rotations of +62' and +30' respectiveiy as writtln. A fifty
mixture of the two products would yield an observed specific rotation of +46'" (the average of the values ior
/ fifty
the
two diastereomers). Because of steric hindrance from the bonds to the six membereJring, the nucleophile
(It{Hs) prefers to attack from the backside of the molecule. This makes the major product Lompound
A, the
first product in Reaction 1. The product has a specific rotation of +62', thus'the specific rotution of the
-ujgt
mixture is closer to +62" than +30". The specific rotation is between 46' and 62', which
answer. -ik", choice B the best

79' Choice B is correct. The specific rotation for the mixture is found by taking a weighted average
of the specific
rotations for the components in the mixture, which is essentially what Equition 1 does. Becarise there
is more
of the component with an ct = +48" than the component with an o = +18', the averaged value should be
closer to
+48' than +18'. However, because it is not purely one component, the specific rotation for the mixture
must be
less than +48" (within the range of +33" to +48'). The best answer is therefore +42', choice
B. The exact
can be determined mathematically as follows: 'alue
oobs = 80% (+a8") + 20'/" (+18') = 38.4 + 3.6 = 42'
80' Choice C is correct. The two products formed in Reaction 2 both have identical bonds to one another
and three
chiral centers each. In comparing the two sttuctures, only one of the three chiral centers differs, making
the
two structures diastereomers. If some, but not all, of the chiral centers differ between two stereoiso^"rr,1h"y
are not superimposable nor are they mirror images. This, by definition, makes them diastereomers.
The best
answer is choice term epimers describes diastereomers that differ at one chiral center, but it applies
9 Jh"
specifically to the backbones of sugars.

81' Choice D is correct. The two products for Reaction 1 are diastereomers of one another so they have different
specific rotations' This means that they can be distinguished by their different specific rotation values. This
eliminates choice A. Because they have different geometry (asymmetry), thef pack differently into their
respective solid lattices and thus they exhibit different melting points. Tiis eliminates choice B. Because of
their varying asymmetry, they bind a gas chromatography differently (due to a difference in steric
hindrance) and thus they show different retention times on the "ot.,*n
gas chromatogiapher (GC). It can be inferred
from the last sentence in paragraph two of the passage that the two products hurr" dlff"r"nt retention times on
the gas chromatographer (GC), since the concentration values can be d.etermined (and thus verified in this
example) using the GC. This eliminates choice C. Infrared spectroscopy measures the type of bonds in the
molecules, therefore it is difficult, if not impossible, to diitinguistr ihe two diaster"o*ers by infrared
spectroscopy' Diastereomers have identical bonds as one another. The best answer is choice D even if vou have
no idea what infrared spectroscopy does.

82" Choice A is correct. The first product has the methyl and ethyl groups both up and the hydroxyl group down.
To retain the chiral integrity shown in Figure 1, the chair confoimatior,, have ethyi and mjhyl Lp with
the hydroxyl down. This eliminates choices B and C, which have the -.rrt exact opposite geometry (ethyl and
methyi are down and the hydroxyi group is up). The most stable conformer hai the least steric repulsion,
which idealiy occurs if the three substituents are in equatorial orientation rather than having the three
substituents in axial orientation. Because the substituents are on adjacent carbons, they alternate upTdown/up,
which allows all the substituents to be equatorial. This makes the best answer choice A.

Copyright @ by The Berkeley Review@ 251 STEREOCHEMISTRY EXPLANATIONS


83. Choice C is correct. The three chiral centers are determined as follows:
41

Substituent number four is in front, Substituent number four is in back, Substituent number four is in back,
so the arc must be reversed from so the arc is correct as drawn. The so the arc is correct as drawn. The
clockwise to counterclockwise. counterclockwise arc makes the counterclockwise arc makes the
This makes the chiral centerS. chiral center S. chiral center S.

The correct answer is S, S, S, which makes choice C the correct answer.

84. Choice B is correct. In the alkene reactant in Reaction 1, only one carbon has four different substituent=
attached, therefore only one carbon is chiral. The best answer is choice B. The chirality is specified in th=
reactant for the ring carbon with the ethyl substituent attached.

85. Choice D is correct. This question is asking to determine which reactions generate new chiral centers. Wher
treating an alkene with bromine, two bromides are added, one to each of the double bond carbons. The result -
that the two double bond carbons become new chiral centers. This eliminates choice A. When treating a:
alkene with hydrogen gas in the presence of a catalytic metal such as palladium, two hydrogen atoms ar-
added, one to each of the double bond carbons. The result is that the double bond carbon with the methyl grou:
becomes a new chiral center. This eliminates choice B. When treating an alkene with permanganate in bas-:
water, two hydroxyl groups are added, one to each of the double bond carbons. The result is that the two doub,i=
bond carbons become new chiral centers. This eliminates choice C. When Compound 3a is treated with a goc:
nucleophile like ammonia, the acid anhydride is cleaved. The result is that one carbonyl group becomes -
amide and the other becomes a carboxylic acid. Neither of these groups are chiral, so no new chiral centers a::
formed. The best answer is choice D.

86. Choice A is correct. It is stated in the passage that Compound 3 represents a mixture of enantiomers, -: -
Compound 3b is the enantiomer of Compound 3a. Enantiomers are mirror images of one another, therefore ail --:
their chiral centers differ. There are two chiral centers in Compound 3, so Compound 3b must have the opposr::
chirality of Compound 3a at these two chiral centers. This is true in choice A, so choice A is the best answer.

87. Choice B is correct. To separate a compound from its enantiomer, the technique must select for one of the t',"":
enantiomers. This requires that there is chirality (asymmetry) in the technique. If the mixture tra\ = j
through a column with one enantiomer attached, the other enantiomer is likely to adhere to the column as -:
travels. This means that one enantiomer r,r'ill migrate quickly while the other travels slowly. This ma-.-
choice B a strong answer. If the column has both enantiomers bound, then both of the free enantiomer: :
solution will be hindered by the column, slowing equally. This does not result in separation. Choice A '
eliminated. The same logic can be used to eliminate choice C . If the distilling column has beads with b - -:
enantiomers, then both of the free enantiomers in solution will exhibit the same affinity for the beads, a::
will not separate as well. Choice C is eliminated. The compounds do not interact with the carrier gas in ;*
chromatography to any notable extent. This means that choice D will do nothing to help separate --- {
enantiomers. Choice D is eliminated.

88. Choice B is correct. Reaction 2 converts Compound 3a into Compound 4, using a peroxyacid. The oxirane ::L:
form from either side of the n-bond. Flowever, because of the steric hindrance above the ring with the aLk.- *
(caused by the six-membered ring), it is preferential to form the epoxide on the back side. Both bonds to oxr-;*nl
in the epoxide must be on the same side, so choices C and D are eliminated. Because of steric hindrance, ci^ - -:*
B is a better structure than choice A. The best answer is choice B.

Copyright O by Thc Berkeley Review@ 252 STEREOCHEMISTRY EXPLANATIO\$


89. Choice A is correct. It is stated in the passage that Reaction 1 generates a mixture of enantiomers. When
enantiomers are formed, they are formed in a racemic mixture. The result is that the optical activity is zero,
because the two isomers cancel one another out. This means that Reaction 1 generates an optically inactive
product mixture. Once the enantiomer mixture is separated, isolating Compound 3a, the subsequent reactions all
start with an optically active starting material, resulting in optical activity in the end. Reactions 2 and 3 form
diastereomers and Reaction 4 cleaves the anhydride to retain the same chirality as the starting material.
Because the materiai is optically pure at the start, the product of reaction 4 is also optically purel The best
answer is choice A.

90. Choice C is correct. Methyl antine, like all amines, is a good nucleophile. Tl-re first equivalent reacts with the
most reactive electrophile, which in this case is the epoxide ring. However, an anhydride is also highly
reactive. In the event excess amine is added, it can easily add to the carbonyi, cleaving the anhydride and
generating a carboxylic acid and amide. Choices A and B can be eliminated immediateiy, becar-rse the three-
membered ring is highly unstable and will not reform once broken. The OH group is a bad ieaving group, so it is
not likely that an amine will substitute for the hydroxyl group, once the airhydride is cleaved, so choice D is
elimrnated. The result is that choice C is the best answer.

91. Choice C is correct. In Reaction 2, a mixture of stereoisomers is formed that is never resolved. As a result, the
reactant in Reaction 4, Compottnd 5, represents a mixture of stereoisomers. Two of the chiral centers are the
same in all of the stereoisomers, so they cannot be enantiomers (where all of the chirai centers differ). This
eliminates choice A. The product is not meso, it does not have an internal mirror plane, so choice D is
eliminated. Because the mixture starts with optical activity, and Reaction 4 does nothing to affect the
chirality, the product mixture in Reaction 4, Compound 6, must be optically active. This makes choice C the
best answer.

92. Choice A is correct. If you are well versed ir-r r-rsing the thumb technique, then you can place your thumb in the
direction of the C-H bond and curl your fingers from priorities 1-2-3 using a right hand for both stereocenters.
This makes both centers R. For some, it may be easier to rotate the molecule to the side view rather than using
the Newman projection to deterrnine the chirality. From the side perspective, it is much easier to see the three
dimensional orientation of the molecule. As shown in the determination below, the molecule has 2Ii., 3R
stereochemistry. Choose A; be a chem star.

CFI?

ocFi3 H in back; R stereocenter

OH
OH
H in back; R stereocenter

93- Choice D is correct. With Fischer projections, you must remember that when an H is drawn on the side, it
represents an H coming out at you ln a three dimensional perspective. Hence, whatever arc you determine from
the Fischer projection must sutsequently be reversed to get the chirality of the stereocenter. in this example,
both chiral centers generate ciockwise circles in a two-dimensional perspective. But after reversing ihe
clockwise circles to counterclockwise becanse of the hydrogen atoms are in fiont, both centers have S chirility.
Choose D and be a scholar.
CH"
| ' Clockwise arc with H in front;S stereocenter
1/l
rF+T-
\y OH

-Fs.,,.
\ Clockwise arc r,r'ith H in front; S stereocenter
ocF{3

Copyright O by The Berkeley Review@ 253 STBREOCHEMISTRY EXPLANATIONS


94. Choice A is correct. On the second carbon, the OH and H groups have interchanged, so that chiral center differs
between the two stereoisomers. On the third carbon, the OH and H groups have not interchanged, so tha:
chiral center is identical in the two stereoisomers. On the fourth carbon, the ethyl and methyl groups hale
interchanged, so that chiral center differs between the two stereoisomers. When two out of three (i.e., some
but not all) of the chiral centers differ, the two stereoisomers are neither superimposable, nor are they mirro:
images of one another, which defines diastereomers. Pick A and be jovial.

95. Choice B is correct. In the molecule, carbons two and three are chiral centers. However, carbon one is not r
chiral center, because it contains two equivalent hydrogen atoms. The maximum number of stereoisomers i.
equal to 2n where n is the number of chiral centers in the molecule, which in this case is 2. This yields a total c:
fot.l (22) stereoisomers. Be stellar and choose B.

96. Choice A is correct. Syn addition of equivalent to a trans double bond in an alkene results in the formation c:
two enantiomers (specifically the R,R and S,S enantiomers) as shown below. Choose A for best results. A mes:
compound can be obtained from syn addition to a cis alkene, where the aikyl groups are equal on the alkene.
H.C Fi HO OH H
\_J
CFI"
\
/.,-, \
/ KMnOalaq)
__-+ . H:cr,l /-.,,,"H
c'
H CFL
pH=10 HrcY \"'*, ,,,Tt
" Syt-,addition H HO
CH: OH
Trans reactant (asymmetric) (symmetric) As).rnmetric products

97. Choice C is correct. If a compound has stereocenters but is not optically active, this implies that the compou:.:
must be meso. To be meso, a compound must have a mirror plane in the molecule about which the chiral centef.
are evenly displaced. This mirror plane slices the molecule into two equivalent halves. 2R,3S-dibromobuta:.=
and 2R,4S-dibromopentane each have equivalent mirror halves (thus mirror symmetry) so they are both me>:
compounds. Cis-1,3-dichlorocyclohexane has equivalent halves as wel1, thus it is a meso compound. Or-.
2R,4S-dibromohexane does not have two equivaient halves. The correct answer is thus choice C.

98. Choice D is correct. 4-chlorocyclohexanol contains no stereocenters (chiral carbons), therefore it cannot rota.:=
plane polarized light. The result is that 4-chlorocyclohexanol is not optically active. Choose wisely ar.:
choose D. The other three compounds have two chiral carbons and therefore are all opticaily active. It:'
possible for the compound (4-chlorocyclohexanol) to exhibit isomerism in the form of geometrical isomers r.-'
and trans), but geometrical isomers do not rotate plane polarized light.

99. Choice C is correct. Isoleucine contains two chiral centers, one for the alpha carbon and one in the side cha-
Plugging into the stereoisomer equation 2n, wl-rere n represents the number of chiral centers, there are tc..:
possible stereoisomers for the isoleucine structure. Because isoleucine contains two chiral centers that mu-.
have specific orientation, only one of the four stereoisomers has the correct chirality to be biologically use:...,
Choose C to be a successful point collector.

100. Choice B is correct. For a compound with chiral centers to be optically inactive, it must be meso. and tt-.
contain an internal mirror plane of symmetry. The molecule must have R, S chiraiity to be meso. T-,-:
eliminates choices A ar-rd C. The mirror plane must be through the C2-C3 bond according to the chiral centers -:
the ansll'er choices. This mirror plane is possible only with butane, so the correct answer must be choice B.

Copr ri-sht O by flie Berkeley Review@ STEREOCHEMISTRY EXPLANATIO\i


Section HV Alkanes
a)

b)
Structures and Physical Properties
i. Aliphatic and Cyclic Alkanes
Alkane Reactivity

Hydrocarbon i. Free Radical Llalogenation


ii. Mechanism
iii. Energetics
Reactions
by Todd Bennett
iv. SelecLivity
llydrocarbon R.eactions
a) Elimination Reactions
i. Ez Mechanism
ii" E1 Mechanism
iii. Carbocation Stability
iv" Electrophilic Addition
v. Regioselectivity
vi. Steneoselectivity
Geraniol vii. Stereoisomer Formation
b) 1,Z-Additi<ln versus 1,4-Addition
Me c) Pericyclic Reactions
i. Diels-AlderReaction
ii. Cope Rearrangement
iii. Claisen Rearrangement
Limonene
Terpenes
a) Classification (Carbon Count)
b) l'ormation and Synthesis
c) Connectivity
d) Spectroscopic Properties

Speciahzing in MCAT Preparation


Ilydroctrrbons and Keactions
Section Goals
Recoanize the nomenclature associated with the alkanes and dienes.
The Greek prefixes and suffixes associated with the hydrocarbons must be common knowledge.
Know the terminology so that when names are presented in passages, you can draw the structure
or recognize the structure in an answer choice. Be able to recognize the positioning of the n-bonds
in conjugated n-systems.

aB Know the general mechanism for free radical addition to an alkene.


Free radical halogenation of an alkane follows a simple three-step process. A free radical halogen
is generated by the initiation reaction, requiring heat or light. The free radical halogen starts the
propagation reactions which undergo a continuous chain reaction until it is terminated. Termination
i"uitidtlr involve two free radicals coming together to form a bond. The overall reaction is a sum
of the propagation steps.

Know the mechanisms associated with elimination reactions.


@? You must know the E1 and pZ mechanism in detail. Know which reactant, solvent, and catalyst
combination will result in which mechanism. As a rule, E1 occurs in acid and E2 occurs in strong
base. Rearrangement can offer complications in an E1 reaction. E2 reactions require the acidic proton
and leaving group to be anti to one another.

Know the general mechanism for electrophilic addition to an diene.


There are several reactions which involve the addition of an electrophile to one of the n-bonds in a
diene. You must recognize the diffrence between the 1,2-addition and 1,4-additions of haloacids
and water across a conilgated diene. Recognize the stereochemical results associated with electrophilic
addition reactions of allienes.

Know the general mechanism for electrocyclic reactions of polyenes.


"? The most common electrocyclic in organic chemistry is the Diels-Alder reaction. You must recognize
it and be able to predict r6latir.e reictivity of diflerent reactions based on electronic factors"and
steric hindrance.'You must also recognize that the six-electron transition state in the Diels-Alder
reaction is also found in sigmatropiclearrangements like the Cope and Claisen rearrangements.
You should have a basic understanding of n-systems.

Recoqnize the isoorene subunits in natural terpenes.


Terpenes are natural oils derived from a five-carbon species. Isoprene is 2-methyl-1,3-butadiene,
altliough the biologically active molecule is a derivative. Terpenes can be extracted and/or distilled
from natural products.'You should be able to identify the isoprene units in biologically common
terpenes and terpenoids.
Organic Chemistry tlydrocarbons and Reactions Introduction

The crux of organic.chemistry centers on reactivity. In organic chemistry


there
are many classes of reactions. Th"y are organized accoiding to mechanistic
features. Common
.reaction types incrude acid-base reactions (section 1),
nucleophilic substitution (section 3), free radical reactions (typically halogenation
of an alkane or polymerization of an alkene), elimination, electrophilic Jdditlon,
electrophilic aromatic substitution, electrocyclic reactions, oxidation-reduction
reactions, and carbonyl substitution reactions (section 5). There are other types
of reactions, name reactions, which we shall address by functiond grtlp.
However, in this section, we shall address the reactions of alkaner ur,a tt-r"
reactions of n-bonds, as classified by their mechanisms.
Free radical reactions follow three basic mechanistic steps: initiation,
propagation, and termination. Whether we consider free radical halogenation of
an alkane or free radical polymerization of an alkene, the first step of ih" pro.u5
is initiation. Initiation entails the homolytic cleavage of a weak'sigma bond in
only a small number of the molecules present in tne reaction mixture. In
halogenation reactions, the halogen-to-hilogen bond is cleaved to initiate the
reaction. In free radical polymerization, the oxygen-to-oxygen bond of a
peroxide initiator is cleaved to generate a low concentratiot of fr"e radical
compounds. The product of a free radical halogenation reaction is an alkyl
halide. An alkyl halide can und"ergo nucleophilic substitution reactions (as we
saw in the stereochemistry section) and elimination reactions. Elimination
reactions result in the formation of a n-bond, which opens the door to a plethora
of new reactions. starting with an alkane, ,oori u.ry compound can be
synthesized.
Although the MCAT test writers do not require that you memorize the reactions
of alkenes, as they have required in the past, you are expected to know the
general classes of reactions that involve n-bonds. when the n-bond of an alkene
ac-ts- as a nucleophile by attacking an electrophile, this starts an
electrophilic
addition reaction. The ru-bond is not limited to being part of an alkene, u, lt .u1
also be part of a diene, triene, or any other conjugaied system of multiple rc-
bonds. V\4ren there is more than one n-bond in the system, we must consider the
possibility of the addition reaction occurring at different reactive sites. The
nucleophiiic n-bond can also be found on benzene, although the aromaticity of
the n-system drastically reduces the nucleophilicity of a r-bond. However, if the
electrophile is strong enough, the n-bond oi benrene can attack it and eventually
e>rqhange the electrophile for a hydrogen, retaining the aromaticity of the system.
This is known as an electrophilic aromatic substitulion. The last reaction we shall
consider for n-bonds is the Diels Alder reaction, an electrocyclic reaction
involving the addition of an alkene to a conjugated diene to form a cyclohexene
moiety.
From this section on, we shall focus on the reactivity of organic compound.s. To
best prepare for the MCAT, your goal shourd not be to memorize every reaction,
but instead, learn a few simple, common mechanisms and have a conceptual
picture of how they work. If you can summarize the contents of this secti,on in
terms of the nucleophilicity of a n-bond, no matter what compound contains the
rc-bond, then you have a solid grasp of the topic at the MCAr level. Reaction
details will be provided in the passage, so from this point on, know the general
reaction and work on your information extraction skills by reading graphs,
tables, and data charts,

Copyright O by The Berkeley Review 257 Dxclusive MCAT Preparation


Organic Chemistry llydrocarbons and Reactions Alkanes

Alkanes
Alkane Structure
Alkanes contain only carbon and hydrogen atoms, and all of the carbons have
spr-hybridization. AII of the hydrogens in aikanes use s-orbitals to form bonds.
Alkanes are held together exclusively by o-bonds. In any alkane, there are only
two types of bonds present: C-C bonds (which are osD3-s'3) and C-H bonds
(which are oro3-r). Both types of bonds present in alkanis aie shown in Figure 4-

cffic
1 as molecular orbitals.

orp3-tp3
@. os-sP3

Figure 4-1
Alkanes can be classified as either aliphntic (straight chain form) or cyclic
(containing a ring in its structure). Aliphatic alkanes hive a molecular formula of
CnH2r, 12 while cycioalkanes with one ring have a morecular formula of C,.,H2r.,.
For each additional ring in a cyclic alkane, the molecule has two less hydrogen
atoms. Table 4-1 lists some common linear alkanes and cyclic alkanes up to-ten
carbons, It should be noted that at least three carbons are needed to form a ring.

C^H2', * 2 for linear alkanes (no rings)


Methane CHa Hexane CoHra
Ethane CzH.e Heptane CzHrc
Propane csHs Octane CgHrs
Butane C+Hro Nonane CsHzo
Pentane CsHrz Decane CrcHzz

C^H2., for cycloalkanes (with one ring)


Cyclopropane CsHo Cycioheptane CrF{u
Cyclobutane C+Hs Cyclooctane CsHro
Cyclopentane CsHro Cyclononane CsHra
l
Cvclohexane CoHrz Cyclodecane CroHzo

Table 4-1
I

Copyright @ by The Berkeley Review 258 The Berkeley Revie*


Organic Chemistry tlydrocarbons and Reactions Alkanes

Alkane Properties
The physical properties of concern associated with an alkane are its solubility
features, its density, its melting point, and its boiling point. Alkanes are
hydrophobic, nonpolar molecules. They can also be defined as lipophilic, which
implies that they are highly soluble in oils such as the lipid membrane of a cell.
As with all compounds, their physical properties vary with mass and branching.
As the molecular mass increases, both the boiling point and melting points
increase. As the branching increases, the boiling point decreases. Table 4-2
shows the physical properties of several aliphatic alkanes. From the data in
Table 4-2, the effects of mass and branching on the physicai properties can be
observed.

Name
Boiling Melting Density Mass
Isomer
Point Point (g/mL) (g/mole)
CH+ Methane -762"C -183"C 76.043
H3CCHe Ethane -89'C -183'C 30.070
H3CCH2CH3 Propane -42"C -787'C 44.097
H3C(CH2)2CH3 Butane 0"c -138'C 58.124
H3C(CH2)3CH3 Pentane 36'C -130'C 0.557 72.151
H3C(CH2)aCH3 n-Hexane 69"C -95.3'C 0.659 86.778
(HrC)zCH(CHD2CH3 2-Methylpentane 60'c -754"C 0.654 86.1.78

H3CCH2CH(CH3)CH2CH3 3-Methylpentane 63'C 118"C 0.676 86.778


(H3C)3CCH2CH3 2,2-Dimethylbutane 50"c -98"C 0.649 86.178
(H3C)2CHCH(CH3)2 2,3-Dimethylbutane 58'C -729"C 0.668 86.778
H3C(CH2)5CH3 n-Heptane 98'C -90.5"C 0.684 100.205
H3C(CH2)6CH3 n-Octane 126"C -57"C 0.703 1.1.4.232

H3C(CH2)7CH3 n-Nonane 151"C -54'C 0.778 128.259


H3C(CH2)sCH3 n-Decane 174'C -30"c 0.7362 142.286

Table 4-2

Example 4.1
What is the molecular weight of 2,2-dimethyl-4-propyl-5-cyclopentylnonane?
A, 228.21,grams,/mole
B. 266.51, grams/mole
C. 268.57 grams/mole
D. 280.54 grams/mole
Solution
First, we must determine the number of carbon atoms and hydrogen atoms.
Dimethyl accounts for two carbons, propyl accounts for another three, pentyl
accounts for another five, and nonane accounts for nine. The compound contains
19 carbon atoms totai. Because of the "cyclo" in the name, there is one unit of
unsaturation. The one unit of unsaturation implies that there are 38 hydrogens
(two less then the 40 that would be present in an aliphatic, linear alkane). The
molecular mass is thus 19(12) + 38 = 266, choice B.

Copyright O by The Berkeley Review 259 Exclusive MCAT Preparation


Organic Chemistry llydrocarbons and Reactions Alkanes

Alkane Reactivity
Alkanes undergo a minimal number of reactions, and the few they do undergo
involve free radical chemistry. The two reactions of concern are free radi&l
halogenation (more specifically bromination, using Br2, and chlorination, using
C12) and combustion. Reaction 4.1 is the free radical chlorination of methane.

CH+(g) + Cl2(g) -il* CH3CI(g) + CHrCtr(1) + CHC13(I) + CCla(t) + C2Cl6(s)


80% 70% minor minor minor

Reaction 4.1

Free Radical Halogenation of Alkanes


A free radical halogenation reaction starts with the addition of activation energ\-
to cleave a halogen-halogen bond (the weakest bond in the reactants) to form two
free radical halogen atoms. A free radical is an atom, such as a halogen or
carbon, with one unpaired electron. According to the octet rule, most atoms wish
to have eight valence electrons. In a free radical, there are only seven, so free
radicals are highiy reactive. Figure 4-2 shows a 3-D perspective of four alkyl free
radicals with the p-orbital filled with one electron. The three substituents are
drawn slightly below the carbon atom, because the free radical molecule is
slightly trigonal planar due the electrostatic repulsion from the single electron.
Because the electron can exist in either lobe of the p-orbital, the substituents carr
be angied up or down, so the average of the two trigonal pyramidal forms is
planar' The stability of a free radical depends on its substitution. Tertiary free
radicals are more stable than secondary free radicals, which in turn are more
stabie than primary free radicals. Figure 4-2 shows the relative stability of the
alkyl free radicals.

o-6Xii.* '
"-6*gF ' o-6*9,# ' "Gx'+ >H.

3" Free radical 2' Free radical 1' Free radical Methyl free radical
Figure 4-2

Once a free radicai halogenation reaction has been initiated by the addition cr
light (activation energy), a free radical halogen atom then attacks an alkane anc
abstracts a hydrogen from the alkane to leave behind an alkyl free radical. The
halogen free radical abstracts the first hydrogen it encounters, but because alkr.
free radicals can react with other alkanes, over time the distribution favors the
formation of the more stable tertiary free radicals. The conversion from a
primary free radical into a tertiary free radical is shown in Figure 4-3 below.

t'.\ \-^*i n,\ n,.\


H*f cH.cH,.
JQ_
cH"cH1----*
.,,,| cH"cH^ . cH^cH
H:C HsC
.|
HaC HsC
Primarv Free Radical Tertiarv Free Radical

Figure 4-3

The reaction in Figure 4-3 heavily favors the formation of product, because the
tertiary free radical is substantially more stable than the primary free radical.

Copyright @ by The Berkeley Review The Berkeley Revieu


Organic Chemistry flydrocarbons and Reactions Alkanes

Free Radical Mechanrsm


The mechanism for a free radical halogenation is a chain reaction process
involving an initiation reaction, followed by propagation reactions, and
ultimately a termination reaction. An initiaiion reaction breaks a covaient bond
in a homolytic fashion to form two free radicals, so an initiation reaction goes
from no free radicals to two free radicais. Homolytic cleavage is djfferent from
heterolytic cleavage in that each atom gets a single eiectron (resulting in free
radicals) as opposed to the more electronegative atom getting both bonding
electrons (resulting in a cation and an anion). In halogenation, it is the haiogen-
halogen bond that is broken. Propagation steps involve the abstraction of an
atom from a neutral molecule by a free radical to form a new free radical.
Propagation reactions include the consumption and formation of a free radical,
so a propagation reaction goes from one free radical to one free radical. In free
radical halogenation, there are two propagation steps. Termination occurs when
two free radicals combine to form a neutral, stable molecule. Termination steps
involve the consumption of two free radicals, so a termination reaction goes from
two free radicals to no free radicals. There are several possible termination
reactions in a free radical halogenation reaction, most of which form minor side
products. The steps for a generic halogenation of an alkane are shown below as
Reactions 4.2,4.3, and 4.4.

Tnitiation: ila ---------> X.+.X


Reaction 4.2

Propagation: tGi* ----->


Reaction 4.3a
R. + H-X

-c?a -------> R-X + .X


Reaction 4.3b

Termination: n! * '.R -------> R-R


Reaction 4.4a

nli.x -------> R-X


Reaction 4.4b

*li.* --------> x-x


Reaction 4.4c

Overall Reaction: R-H + X-X --------+ R-X + H-X


Reaction 4.5
The sum of the propagation steps for a free radical halogenation reaction gives
the overall reactiort, Reaction 4.5. The initiation step is brief, for just a split
second at the start of a free radical reaction. However, the propagation steps
continue until the reaction is quencired or the free radicals are completely
consumed in termination reactions. There are always multiple termination steps
possible. One of the possible termination steps is the reverse of the initiation
reaction. A termination step is any reaction that combines two free radicals to
generate a stable compound. Termination steps are responsible for several minor
side products.

Copyright O by The Berkelev Revieu 26r Exclusive MCAT Preparation


Organic Chemistry tlydrocarbons and Reactions Alkanes

Example 4.2
Which of the following reactions represents an initiation step?
A. H3CCHzCHz. + Br. -) H3CCH2CH2BT
B. H3CCH2CH2. + H3CCH2CH3 -+ H3CCH2CH3 + @3C)2CH.
C. (H3C)2CH. +Br2 -) H3CCHBTCH3 + Br.
D. HOBr --> HO. + Br.
Solution
In an initiation reaction, free radicais are generated, so the product side has more
radicals than the reactant side. Choice A is eliminated, because there are fewer
radicals on the product side, making it a termination reaction. In choices B and
C, there is the same number of radicals on both sides of the reaction, so they are
propagation reactions. This elimrnates choices B and C. In choice D, the reaction
goes from zero free radicals on the reactant side to two free radicals on the
product side, so it is an initiation reaction.

Example 4.3
In the free radical chlorination of ethane, butane is a minor side product. Horr'
can this best be explained?
A. An ethane molecule attacked an ethyl chloride in a nucleophilic substitutior,
reaction to form butane.
B. An ethyl free radical removed a hydrogen from an ethane molecule.
C. Two ethyl free radicals combined to form a new sigma bond.
D. The carbon-carbon bond of ethane was cleaved during initiation to forrn
methyl free radicals, which rapidly combine to form iong-chain alkanes.

Solution
An ethane molecule has neither a lone pair of electrons nor an available pair o:
bonding electrons to share (like a n-bond), so it is definitely not going to act as a
nucleophile. This eliminates choice A. An ethyl free radical can definitel,,'
remove a hydrogen from an ethane molecule. However, that does not resuit ir.
the formation of butane, it simply regenerates the same molecules.

H3CCH2. + H3CCH3 -+ H3CCH3 + H3CCH2.


Choice B is eliminated. When two ethyl free radicals combine, they form a ne\^.'
sigma bond between the two free radical carbons. Combining the two two-
carbon fragments results in the formation of a four-carbon fragment. The resuL:
is that butane is formed from the termination reaction of two ethyl free radicals,
The best answer is choice C. Carbon-carbon bonds are not easily cleaved, so the
activation energy added in the initiation step is not high enough to cleave a
carbon-carbon bond. Even if it could cleave the carbon-carbon bond to forr.
methyl free radicals, there is iittle likelihood that four CH3 groups woulc
combine to form C+H1O. The loss of two hydrogen atoms would not occur.

Copyright O by The Berkeley Review 262 The Berkeley Kevieu


Organic Chemistry tlydrocarbons and Reactions Alkanes

Free Radical Halogenation Energetics


with each reaction, we consider how much product is formed (thermodynamics)
and how fast the reaction proceeds (kinetics). AH and AG vaiues determine how
much is formed while the activation energy (Eo.1) dictates the speed. For the
chlorination of methane, Reaction 4.1, the foilowing enthalpy valuel apply:
Ci-Cl -) Cl.+ .Cl AHi.itiatic,n = 58 kcal
H3C-H + .Cl -+ H3C. + H-CI AH1 = I ks.1
H3C. + Cl-Cl -) H3C-CI + .Cl AH2 = -26kcaI
Figure 4-4 shows the energy diagram associated with propagation steps in
Reaction 4.1.

HCl + CH3

fr
CJ overall AH = AHr
tr.l

OJ
CH3C1 +
o
A
Reaction Coordinate

Figure 4-4
The energetics of each free radical halogenation reaction is different. For
instance, bromine has a lower bond dissociation energy than chlorine, so
bromination requires less activation energy than chlorination. This is why
chlorination requires light for initiation, while either light or heat can initiate a
bromination reaction. F{owever, this does not mean that bromination proceeds
faster than chlorination. The reaction rate is determined from the aitivation
energy of the rate-determining step in propagation. Table 4-3 lists the energies
for the halogenation of methane. From Table 4-3, you can determine which
reaction is fastest and which reaction generates the most heat. The data shows
that free radical iodination of methane is unfavorable and free radical
fluorination of methane is too favorable, generatrng enough energy to explode.

AH values {Kcal/*o1")
Halogenation Reaction X=F X=Cl X=Br X=I
kritiation; X2 -+ 2X. 38 58 46 36
X. + CHa -+ HX + H3C. -32 +1 +76 +33
H3C" + X2 --+ H3CX + X. - /t) -26 -z+ -20
CH4 + X2 -+ H3CX + HX -102 -25 -8 +13

Table 4-3

Copyright @ by The Berkeley Review Exclusive MCAT Preparation


Organic Chemistry Ilydrocarbons and Reactions Alkanes

Free Radical Halogenation Selectivity


Because tertiary free radicals are more stable than other free radicals,
halogenation occurs preferentially at tertiary carbons. When there is more than
one unique carbon in the alkane reactant, the product distribution is a result of
the relative free radical intermediate stability, the relative abundance of
equivalent hydrogens, and the rate of the reaction. For the chlorination reaction,
the relative reactivity of 3" :2' : 1" carbons is 5 : 3.g : 1 at room temperature. At
temperatures around 100"C, chlorination selects for tertiary over seiondary over
primary by a ratio of roughly 4 : 2.5 : r. As the temperature increases, the
reaction proceeds faster, and is therefore less selective. For the bromination
reaction, the relative reactivity of 3' : 2" : 7" carbons is 1600 : 62 : r at room
temperature. This means that the product of free radical bromination is almost
exclusively tertiary, while the free radical chlorination reaction gives a more
balanced product mixture. Figure 4-5 shows the product disiribution for
comparable chlorination and bromination reactions.

cl"
__>
c+Hro
hv $.,
77.7"/o Secondary 28.3% Primary
Br
Br, I
.t\-/ $Br
hv
C+Hro 97.6"/o Seconda ty 2.4"h Prirnary

Figure 4-5

As a general rule, slower reactions are more selective than faster reactions,
because reactants have more time to select the best site for reacting. Chlorination
reactions are faster than bromination reactions, so bromination is more selective
than chlorination. In the slower bromination reaction, if a primary free radicai js
formed, it has time to abstract a hydrogen from another alkane and form a ner!.,
possibly more stable free radical. If the new free radical is tertiary, it is likely to
only react with a dihalogen molecule. Consider Reaction 4.6 and, the data in
Table 4-4 corresponding to the distribution of mono-halogenated products.

>\+X2grb.hx
Reaction 4.6

Trial Halogen Temp (K) Product A Product B Product C Product D


I Br2 298 03% 89.40/" 70.1% 0.2%
II Clz 298 29.1% 243% 32.00/" t4.6%
III Btz J/J 0.4% 88.3% 1,L.1% 0.2%
IV Clz J/J JJ.J 70
aa ao/
LL.L /O 27.8% T6]%
V 12 373 No Rxn No Rxn No Rxn No Rxn

Table 4-4

Copyright @ by The Berkeley Review 264 The Berkeley Revie*-


Organic Chemistry tlydrocarbons and Reactions Alkanes

The data in Table 4-4 confirms that bromination is more selective than
chlorination for tertiary over secondary over primary. Bromination is slower
than chlorination, but it should also be noted that bromination is more reversible
than chlorination, so it is more likely undergo a reverse reaction from an unstable
product to ultimately form the most stable product. The data in Table 4-4 also
shows that temperature has an effect on the reaction rate and therefore on the
selectivity. As the reaction temperature is increased, the reaction proceeds at a
faster rate, resulting in the formation of products based more on random
probability rather than selection for the most stable intermediate. In addition,
Boltzmann's law states that as energy is added to a system, the distribution of
compounds is shifted to the less stable compounds, to absorb the energy.
Actually, who reaily knows if Boltzmann said it, or if it's even a law. The key
thing is that as energy is added, less stabie compounds are formed.

Example 4.4
\44ry in Trial II of Table 4-4 is Product A formed to a greater extent than product
B?

A. Product A results from the more stable free radical, thus it is selected for.
B. Product B rearranges to form product A.
c. Product B is more stable, but there are more hydrogens that lead to product
A, so overall less Product B is formed.
D. Product A is more stable, but there are more hydrogens that lead to product
B, so overall less Product B is formed.

Solution
Product A results from a reaction at a primary carbon, so it proceeds via a
primary free radical. Product B results from reaction at a tertiary carbon, so it
proceeds via a tertiary free radical. This eliminates choice A. Rearrangement is
seen with carbocations, but not with free radicals or carbanions, so choice B is
eliminated. The best answer is choice C. It is often possible to answer a question
without fuil analysis. The reason for the substantial amount of Product A is
because there are six hydrogens that lead to Product A while there is only one
hydrogen that leads to Product B. Although tertiary reactivity with chlorination
is roughly four to five times greater than primary reactivity, the six-to-one
abundance ratio outweighs the four or five-to-one reactivity preference, making
the probability of forming Product A greater than the probability of forming
Product B. This is makes choice C the best answer. Choice D can be eliminated,
because there are more hydrogens available to form Product A than Product B.

Copyright @ by The Berkeley Review Exclusive MCAT Preparation


Organic Chemistry tlydrocarbons and Reactions Alkanes

Example 4.5
Why is there no reaction observed when iodine is used?
A. Iodine cannot form a free radical.
B. The iodine-iodine bond is too strong to cleave.
C. Free-radical iodination of an alkane is too reactive.
D. Free-radical iodination of an alkane is unfavorable.

Solution
Because chlorine and bromine form free radicals, we can be assume that another
halogens, such as iodine, can also form a free radical. This eliminates choice A.
Iodine is lower in the periodic table than chlorine and bromine, so the iodine-
iodine bond is weaker than the chiorine-chlorine bond and the bromine-bromine
bond. Because Cl2 and Br2 are cleaved, it is safe to assume that 12 is even easier
to cleave. This eiiminates choice B. Iodine forms weak bonds to carbon and
hydrogen, so the products are less stable than the reactants. Because the
products of free radical iodination are less stable than the reactants, the reaction
is unfavorable, so there is no reaction observed with iodine. This makes
choice D
the best answer and eliminates choice C. Fluorine is not used for completely
opposite reasons. Fluorine forms strong bonds to carbon and hydrogen, and
th!
fluorine-fluorine bond is weak. The products are so much more stable than the

:j'"",,^" t'""
",
Example 4.6
Why are there minimal di-halogenated products formed in the free-radical
chlorination of an alkane?
A. The acldition of the halogen makes the alkyl halide less acidic than the j
alkane, so it is less reactive to subsequent halogenation reactions.
B. The addition of the halogen makes the alkyl halide more acidic than the
alkane, so it is more reactive to subsequent halogenation reactions. -1.

c. After the first halogen is added to the alkane, the carbon-hydrogen bonds
grow weaker and thus more reactive.
D. After the first haiogen is added to the alkane, the weakest bond is a carbon- -j
halogen bond and not the carbon-hydrogen bond. As a result, it is easier to
remove the halogen rather than the hydrogen from the mono-substituted
alkyl halide. E
.
Solution
Halogens are electron-withdrawrng, so their presence on a molecule increases its
acidity. This eliminates choice A. Choice B is invalid, because an increase in
-,1L

ts
reactivity would imply that more poly-halogenated products would form, not (-
less. Choice C can be eliminated for almost the same reason. If the carbon- D
hydrogen bond is weaker, and thus more reactive, then it would be easier to add
a_second halogen than the first, making poly-haiogenation preferable. once an
5-c
alkane is haiogenated, the weakest bond is the carbon-halogen bond, not a
carbon-hydrogen bond. If a second halogen free radical ruuit, with an alkvl :._
halide (rather than an alkane), it preferentiully r"rr,orr"s the halogen (breaking the :_
weakest bond), forming a non-halogenated alkyi free radical. this is because the -:
reverse halogenation reaction (in propagation) is more favorable than the
removal of a hydrogen and subsequent additional halogenation reaction. Choice

"'*.

Iopyright O by The Berkeley Review The Berkeley Review :f


Organic Chemistry Ilydrocarbons and Reactions Alkanes

Example 4.7
To synthesize a primary alkyl halide from an alkane in highest yield, what
should be done?
A. Bromination at25"C
B. Chlorination at25'C
C. Bromination at 100"C
D. Chlorination at 100'C
Solution
A primary alkyl halide is the least favorable product, so the best free radical
halogenation reaction is the one with lowest selectivity. According to the data in
Table 4-3, chlorination is less selective than bromination and selectivity is
reduced at higher temperatures. This means that choice D, chlorination at the
highest listed temperature, is best.

Example 4.8
Using the data listed in Table 4.3, what percent of the mono-halogenated
products is 2-bromo-2-methylbutane following the free-radical bromination of 2-
methylbutane at75"C?
A. 88.1%
B. 88.7%
c. 89.1%
D. 89.7%
Solution
To answer this question, you need to read Table 4-3. In trials I and III, product B
is 2-bromo-2-methylbutane. Therefore, we need to estimate how much Product B
is formed at75"C. At25'C, there is 89.4% product B formed, while at 100"C there
is 88.3% product B formed. This means that the amount of product B formed at
75'C should be between 88.3% and 89.4o/o, which eliminates choices A and D.
The amount formed at75'C should be closer to 88.3% than89.4"h, so choice B is
the best answer.

Example 4.9
How many mono-chlorinated structural isomer products are possible when 2,5-
dimethylhexane undergoes free radical chlorination?
A.3
8.5
c.6
D.8
Solution
This question is asking for how many structural isomers there are for chloro-2,S-
dimethylhexane. Because of the mirror plane through the carbon 3-to-carbon 4,
bond, there are three unique carbons on 2,5-dimethylhexane. This means that
there are just three carbons that can be chlorinated, so there are only three mono-
chlorinated structural isomers. If stereoisomers were included, the value would
increase to four, given that chlorination of the secondary carbon yields a
stereogenic center. The best answer is choice A.

Copyright @ by The Berkeley Review 267 Exclusive MCAT Preparation


Organic Chemistry tlydrocarbons and Reactions tlydrocarbon Keactions

Aydroiberbdh Radtiddb , ,,-,,.,*,,,,,,r',i,,,t,,,,,,,f,,,,

Elimination Reactions
The reaction that forms an alkene from a substituted alkane is elimination. It is
named from the fact that a functionai group and a hydrogen on adjacent carbons
are eliminated in order to form a n-bond. The-reac-tion requires elevated
temperatures to help overcome the activation energy and to p.rsh the reaction in
the forward direction. Like the nucleophilic substiiution ,*u.tior,r, there are two
reaction mechanisms, appropriately named E1 and E2. As with nucleophilic
substitution, the two versions are named also for theiireaction orders (kinetic
rate dependence). E1 is similar to sry1 and E2 is similar to slr]2, except that the
product is an alkene. In an E1 elimination, the leaving gto.rp first ieaves and
there is a carbocation formed. The empty p-orbital of thi c"arbocation eventuallr-
becomes one of the two p-orbitais in the new n-bond. In an E2 elimination,
a base
removes an alpha hydrogen to force the leaving group off of the neighboring
carbon. Elimination converts a functionarized alkyigroup into an alkenJ.
E2 Reaction (Carried out under Basic Conditions at High Temperature)
The E2 reaction is corrcerted (one-step) rike the sNI2 r;action, with one major
exception being that the E2 reaction occurs at higher temperatures (temperatuies
above ambient temperature) than the S52 reaction. An E2 reaction also requires
that the base be bulky. Because of the steric hindrance aisociated with a bulky
base, it is.less apt to act as a nucleophile and thereby minimlze in;;;;fi;;
Sp2 reaction. An important feature of the E2 reacti-on is that the substituenti
being eliminated must be onti to one another (have a dihedral angle of 1g0.). The
mechanism for an E2 reaction is shown in Figure 4-6.

! oR-
(^ R:
H o
I

H I

-----+
A
H
t H
H
H,CV X-
H

H3Ctt,,) (trrH
HgC
H H

Figure 4-6
In an E2 reaction, the compound may have to rotate to proper conformation
before the reaction can proceed to form the alkene. This is to siy that the leaving
group must be oriented anti to the hydrogen being eliminated before the base
attacks the proton to start the reaction. whenever there are two alpha hydrogen,.
that can be eliminated from the starting reagent, there are two posiible prodicts.
The consequence is that when the starting material has a specific stereoChemistl'
at one of the reactive carbons, a corresponding specific geometrical isomer
product (either cis or trans) forms. Figure 4-7 shows an exampte where there are
two alpha protons to choose between. One of the two protoni has been replacecl
by deuterium, so that the structure can be monitorecl and the product can be
used to support the idea that the elimination reaction o.c.rrr"d from the anti
orientation. Figure 4-7 shows the rotation to anti and formation of both products.

Copyright @ by The Berkeley Review 268 The Berkeley Revien' r.:


Organic Chemistry Ilydrocarbons and Reactions llydrocarbon Reactions

DH
LINR^ \ /
----1- EJ
^/\ H:C CH:

H.C H
Bt LiNR,

H CH:
A
--+
H
F< cHs

Figure 4-7
If the reaction were not carried out through a mechanism that required anti
orientation, then there would have been four elimination products total,
rather
than just two. The four products wourd include the cis and trans alkenes
featuring the deuterium, and the cis and trans alkenes with no deuterium.
Figure
4-B shows the two alkenes not formed,in the reaction in
Figure 4_7.
HH

><
HsC CHs CH:
Not formed in the elimination reaction of (2s, 3R)-3d-2-bromobutane,
therefore the reaction must have proceeded by the anti orientation.

Figure 4-8
Multiple products can result when their are multiple hydrogens available for
deprotonation. In Figure 4-7, onry the products with in iniernai n-bond are
drawn. The internal n-bond is more favorable then the terminal n-bond by about
1.5 kcals, making 1-butene a minor side product in the reaction
in Figure [-2. rn"
relative stability of substituted alkenes ii shown in Figure 4_9.

R R'''
\_J R

R' R''
Tetrasubstituted
R' R''
n
\_-/
HR H
\--/

Trisubstituted Disubstituted Monosubstituted

Figure 4-9

Terms such as anti-periplanar orientatiott should sound familiar and immediately


make you think of an F2 reaction. Any time you see a strong bulky base, a
hydrocarbon with a leaving group, and high temperature, you should think
of an
E2 reaction.

Copyright @ by The Berkeley Revierv Exclusive MCAT Preparation


Organic Chemistry tlydrocarbons and Reactions tlydrocarbon Reactions

E1 Reaction (Carried out Under Acidic Conditions at High Temperature)


E-t reaction also yields an alkene, but it goes through a different mechanism
Tu
than the E2 reaction. The mechanism for the t1 reactioi is quite similar to that of
the sI.J1 reaction. In both the E1 and sNJ1 reactions, there is (1) a carbocation
intermediate formed after the leaving group leaves in the rate determining step
of the reaction and (2) the possibility of rearrangement with the carbocation
intermediate. A schematic for a typical E1 reaction is shown in Figure 4-10.

+ Hro
HsQ
OH --+
o
oHz
,rg,
-n
------>
A
T'.';*1f\'" '
. Hb\
-HA HJ
i
H"C CH,

Hb CHs
><J
Ha H

Figure 4-10

To recognize E1, as opposed io E2, it is easiest to look at the reaction conditions


E1 reactions are best carried out under acidic conditions while E2 reactions
require basic conditions. Both reactions proceed under thermal conditions
(elevated temperature) and yield the most substituted alkene as the major
product. There may be some of the less substituted alkene formed in smal1
quantity. The reaction in Figure 4-11 shows the formation of both the 2-alkene
major product and the 1-alkene side product.

.atll* H-'-'--.-H
HO:
."O
'\) s- ::.
..S"1:.t,
*r"au"R H3c$\'7-l -:
HsC H
- HrC /, H

\SH2CH3
7
CHs

1t

l,,rt\\ CH, cH2cH3


r.3
Urclrr,,rp-W, rrrttl CH2C H.
,? H,c7\ts-{./
/F.",
_-H
t
CH:

Figure 4-11

Copyright @ by The Berkeley Review The Berkeley Review


Organic Chemistry Ilydrocarbons and Reactions Ilydrocarbon Reactions

Example 4.10
What is the major product when 2-methylcyclopentanol is treated with
concentrated sulfuric acid at 50"C?
A. B.

CHz

cHs

Solution
At an elevated temperature in the presence of a strong acid, the reaction proceeds
by an E1 mechanism, so the product is a highly substituted aikene. Choice A is
eliminated, because it is a terminal alkene. Choice B is eliminated, because it is
an alkane and not an alkene. Choice D is eliminated, because it is not the most
substituted alkene and it formed via a secondary carbocation, a less stable
intermediate than the tertiary carbocation. The best answer is choice C, because
the n-bond is highly substituted and it includes the tertiary carbon, implying that
it was formed via a tertiary intermediate.

Rearrangement is possible with carbocation intermediates, so E1 reactions are


susceptible to forming rearranged products. Hydride and alkyl shifts are rapid,
intramolecular processes that occur before the soivent can remove the
neighboring proton to form an alkene.

Carbocation Stability
Because carbocations are common intermediates, it is important to know the
relative stability of the various carbocations. The relative stability of alkyl
carbocations is 3' > 2' > 1' > Methyl. For carbocations conjugated to a t-bond,
vinylic and benzylic carbocations, there is additional stability because of
resonance. Carbocations can undergo rearrangement by having hydrides or
alkyl groups shift, resulting in a different carbocation. For instance, if a
secondary carbocation (R2CH+) is formed, it can rearrange to a tertiary
carbocation (R2R'C+), if a tertiary carbocation is possible. Figure 4-12 shows the
relative stabiiity of alkyl carbocations.

'"{+H: '"" *= '" :6"'"


o:.
)@..,,,trtu
".. 'slx:
3" Carbocation 2'Carbocation 1'Carbocation MethylCarbocation

Figure 4-12

The conclusion that can be drawn from the relative stability of alkyl carbocations
is that methyl groups are electron donating to electron poor carbons, which can
be extrapolated to say that alkyl groups are electron donating. Figure 4-13 shows
three rearrangements where a less stable carbocation is converted into a more

Copyright @ by The Berkeley Review 277 Exclusive MCAT Preparation


Organic Chemistry tlydrocarbons and Reactions llydrocarbon Reactions

stable carbocation via a hydride shift. The third example forms an allylic
carbocation, which is more stable than alkyl carbocations, beiause of resonance.

H H

HI
I

"\_ o_,/cylH
)'w;* 1'
----->
"Yt-t\r,
H2"
'H

CH"

H\ l''
--------> .1--.n,
nf-.vn
J

H
3"H

HHHH
\/\/
/c-c \
H ------> c-c-\ H
H o/ /
H
o /'

"YJt" l"
,,f-'Y#.,
2" Allvlic

Figure 4-13

which of the following carbocations is apt to undergo rearrangement?


A. (H3C)3C+
B. (H3C)2CH+
C. H3CH2C+
D. (H3C)2CHCH2+

Solution
Choice A is already a tertiary carbocation, so it has no reason to rearrange, least
of all to a primary carbocation. Choice A is eliminated. Choice B is a secondarr-
carbocation, but can only rearrange to form a primary carbocation, so it is
eliminated. choice C is a primary carbocation that only has primary carbons, so
it is eliminated. In choice D, a hydride shift can covert a primary carbocation
into a tertiary carbocation, making choice D the best answer.

Copyright O by The Berkeley Review 272 The Berkeley Revieu'


Organic Chemistry tlydrocarbons and Reactions flydrocarbon Reactions

Electrophilic Addition Reactions


Although isolated alkenes are not a topic tested on the MCAT, polyenes are a
topic, so generic electrophilic addition reactions are viable. Electrophilic addition
reactions involve a n-bond actrng as nucleophile by attacking an electrophile to
form a substituted alkane product. In the first step of electrophilic addition
reactions, the n-bond of the alkene donates its eiectron density to an electrophile.
The first step of a generic electrophilic addition reaction is shown in Figure 4-14.

----------t-

Step I
E* represents any electrophile (lone pair acceptor)

Figure 4-14

After the electrons from the n-bond are donated to the electrophile, a positive
charge is situated on the most substituted carbon of the original alkene in the
carbocation intermediate. This is the first step in almost all electrophilic addition
mechanisms. In the second step of the mechanism, a nucleophile attacks the
carbocation formed in the first step of the electrophilic addition reaction. The
second step of a generic electrophilic addition reaction is shown in Figure 4-15.

\-/' -----------.r>
H-Nu:J \ H-Nu
SteP II
H-Nuc represents any nucleophile (lone pair donor)

Figure 4-15

It is important to recognize that when there is a carbocation intermediate, there is


potential for rearrangement. If the carbocation is unstable, then prior to the
attack by a nucleophile, the carbocation can rearrange by way of a hydride shift
or alkyl shift to form a more stable carbocation. Rearrangement is not shown in
this example, but it can occur between steps one and two of the mechanism. The
nucleophile should be a weak base, otherwise it can deprotonate the intermediate
to carry out the reverse reaction (elimrnation) and regenerate an alkene. To avoid
this, eiectrophilic addition reactions are carried out under acidic conditions. The
final step is the neutralization of the cationic product, which is carried out by a
solvent basic enough to deprotonate the cationic species. The final step of a
generic electrophilic addition reaction, a workup step, is shown in Figure 4-15.

! -\
\lll\-,/t
sorv

L-#
\-/ Step (workup)
----------.>

Solv represents any polar/protic solvent

Figure 4-16

A protic solvent is capable of forming hydrogen bonds and transferring protons.

Copyright @ by The Berkeley Review 273 Exclusive MCAT Preparation


Organic Chemistry Ilydrocarbons and Reactions tlydrocarbon Reactions

It is a good idea to understand basic mechanism of electrophilic addition to


dienes and to recognize the class of reaction. Know how to diaw the reaction if
they describe the mechanism for the reaction in words. The MCAT doesn't focus
much on memorization. The focus is on conceptual understanding, so be certain
that the corresponding terms such as Markovnikov, syn and anti ire completeiy
understood. Later in this section we will look at a variations on this same
mechanism with 1,2 versus 1,4 addition reactions of conjugated alkenes.

Regioselectivity (Markovnikov versus Anti-Markovnikov Addition)


The concept of regioselectivity occurs when the reactant electrophile and the
alkene both lack mirror plane symmetry. This is to say that the double-bond
carbons are not equaily substituted. Because the two carbons are not equally
substituted, one is less sterically hindered than the other. As a mechanistii rule
in electrophilic addition reactions, the first substituent attacks the less hindered
carbon of the alkene, leaving the second substituent to add to the other carbon.
Cenerally, the less hindered carbon of the intermediate is the aikene carbon that
wasn't attacked by the first substituent. A Marlcoanikoa addition product results
from the addition of the electrophile to the less substituted carbon of the alkene
and the nucleophile to the more substituted carbon of the alkene. In the case
where a strong acid reacts with an alkene the electrophile is a proton, you can
simply say, "H goes where H is." An onti-Markoanikoa ad,dition prod,rct is the
opposite of a Markovnikov addition.

Stereoselectivity (Syn versus Anti Addition)


The concept of stereoselectivity occurs when the alkene reactant or intermediate
has asymmetric faces, and thus non-uniform steric hindrance. syn additiol
refers to a reaction where the two new substituents add to the same side (face) of
the alkene reactant or intermedrate. Anti addition refers to a reaction where the
two new substituents add to opposite sides (face) of the alkene reactant or
intermediate. As a mechanistic rule, a substituent attacks at (and adds to) the
less hindered face of the alkene (or intermediate). If the two substituents add at
the same time, they add in a syn fashion to the least crowded face of the alkene.
If the two substituents add at different times, they add in an anti fashion to
opposite faces of the alkene. once the first substituent adds to an alkene, it
makes one side of the intermediate more crowded than the other side. This is
referred to as stereoselectivity, because the face at which the substituent attacks
dictates the stereclchemistry of any newly formed chiral centers.

Stereoisomer Formation
You should always consider if stereochemistry is involved in a reaction; whether
it forms a racemic mixture of enantiomers or a major/minor mixture of
diastereomers. When no chiral center is present on an alkene reactant, there is no
asymmetry to influence the reaction. There is an equally likeiy chance to attack
the alkene from either face. The result is the formation of a racemic mixture of
enantiomers, assuming that new chiral centers are formed. When a chiral center
is present on an alkene reactant, its asymmetrv influences the stereoselectivity of
the reaction. There is a greater chance to attack the alkene from the less hinclered
face than the more hindered face. The result is the formation of a major/minor
mixture of diastereomers. A good rule to follow is that if there is no optical
activity in the reactants, then there can be no optical activity in the product
mixture. This means that the presence of chirality influences further chirality.

Copyright @ by The Berkeley Review 274 The Berkeley Review


Organic Chemistry Hydrocarbons and Reactions llydrocarbon Reactions

1'2-Addition to a Conjugated Diene vs. 1,4-Addition to a Conjugated


Diene
Electrophilic addition reactions are not iimited to alkenes with
isolated n-bonds,
as these reactions also take_place on conjugated dienes.
with conjugated ciienes,
there are multiple potential products. t,z--aaaition refers to
an addition reaction
that adds substituents to the first and second atoms (usually carbon)
in the
conjugated n-network. 1,4-addition refers to an addition reaction
that adds
substituents to the first and fourth atoms in the n-network. As a general
rule, r,4_
addition is favored at higher temperatures, because the more s-table product
is
formed (thermodynamic control). At lower temperatures, the more
stable
intermediate dictates the product, so r,z-ad.d.Ltion is favored. This is known
as
kinetic control. There is always the option to add either 7,2 or 1,4 when
the
has conjugation. Figure 4-17 shows the product distribution for the
fysJem
l-rydration of 1,3-butadiene at two different temperatures.

H2C:
z- CH
-\ L
H:SOq{aq)- H,c-.')ot
"r.- ttr.-
o"c \ \
HC_ CH2 HC- CH2 HC- CH2OH
62% 38%
1,2-AdditonProduct 1,4-Additonproduct

H.C-
.\ CH
H2SO,1(aq)
l-hL-
'
c)ot H,c-
'r-\ cH
50"c \
HC- CHz HC- CH2 HC- CH2OH
16% 84%
1,2-Additonproduct 1,4_Additonproduct

Figure 4-12

In the example in Figure 4-77, there are two possible carbocation intermediates.
The mechanism and both intermediates ur" ,ho*r, in Figure 4-1g.
To start the
reaction, the proton adds to the terminal carbon (least hiidered carbon)
leavi'g
the carbocation on carbon two (a secondary carbocation). The carbocation
is
allylic meaning it can resonate through tie n-network. It is in essence a
propylene cation that can have cationic density at either end of the rc-network.
What this means is that the carbocation can resonate to the terminal carbon
(carbon number four). This forms a primary carbocation, whicl-r is
not as stable
as a secondary carbocation. According to the Boltzmann distribution law,
as
temperature increases, the higher energy levels become more populated to
absorb this increase in energy. This means that at higher temperatures
there are
more primary carbocations than at lower temperatuies. This is why we see
the
product distribution favoring substitution of the alcohol at carbon two at lower
temperatures while we at higher temperatures we observe the substitution
of the
alcohol at the fourth carbon. Figure 4-18 shows the energy diagram and
mechanism associated with this reaction, which includes t*o poisiute
iuth*uyr.

Copyright O by The Berkelev Review 275 Exclusive MCAT Preparation


Organic Chemistry Ilydrocarbons and Reactions Ilydrocarbon Reactions

Kinetic Pathway Thermodynamic Pathway


(favorable at lower temperatures) (favorable at higher temperatures)

HO
+
2" carbocation is most stable
s
1" carbocation is least stable

I H H3C-CH
-J
rirC=Cri :HrC-C^\
o/, .e \HC-C@/H
' '
\ )^)^..
HC:CH2 \
HC:CH2 H

1f.""
H@
1f.""
H?C-
"\o
HsC-.i""' CH

HC- CH2OH2
HC: CH2

H*
1r
H
1f".
HeC- C-
>oH H"C-
"\ CH
"\
\ HC- CH2OH
HC: CH2
Monosubstituted alkene Disubstituted alkene
is least stable is most stable

1" carbocation

Lower energy intermediate

I
2' carboca tion
leads to the kinetic product.

Higher energy intermediate leads


to the thermodynamic product.

Reaction Coordinate *--*-+


Figure 4-18

The energy diagram shows the two possible pathways for the reaction and their
relative energetics. The dashed pathway represents L,2-addition to the
conjugated alkene, while the solid pathway represents 1,4-addition to the
conjugated alkene. From this data, you should be able to predict the more
abundant product at a given reaction temperature.

Copyright O by The Berkeley Review The Berkeley Review


Organic Chemistry flydrocarbons and Reactions Ilydrocarbon Reactions

Pericyclic Reactions
Pericyclic reactions involve the repositioning of both sigma-bonds and pi-bonds
through a cyclic transition state. These reactions are believed to be concerted,
meaning that the formation and breaking of all bonds occur simultaneously.
Pericyclic reactions include cycloaddition reictions,which most notably include the
Diels-Alder reaction, sigmatropic rearrnngement, and electrocyclic reactiors. We will
address only cycloadditions and sigrnatropic rearrangement. The significant
difference between a cycloaddition reaction and sigi-Latropic rearrangement
involves the number of molecules. In cycloaddition, two separate compounds
come together, resulting in a single new compound. In sigmitropic
rearrangement reactions, it is an intramolecular rearrangement that takes place.
Paramount to understanding these reactions is having a good idea about orbital
overlap in both sigma-bonds and pi-bonds. In everything we'll address in terms
of the MCAT, we shall only consider the positioning oith" atoms and not the
spin of the electrons within the moleculai orbitals. the first reaction we shall
consider is the Diels-Alder reaction.
Diels-Alder Reaction
The Diels-Alder reaction, an electrocyclic addition reaction, involves the addition
of a conjugated diene (4 n-electrons) to an alkene (2 n-electrons) to from a six
membered cyclohexene ring. The transition state for a Diels-Alder reaction is
similar to the resonance of benzene, as shown in Figure 4-19.

_->
49 Die ls-Alder Reaction

Figure 4-19
six n-electrons in a cyclic n-network make benzene aromatic, so we refer to the
transition state of a Diels-Alder reaction as aromatic (containing 6 n-electrons in a
ring). Diels-Alder reactions involve the addition of a 1,3-diene to a dieneophile.
The diene must have cis orientation about the central sigma bond to undergo a
Diels-Alder reaction. A sample Diels-Alder reaction is driwn in Figure 4-20.

diene dieneophile cyclohexene derivative


o o
22"
1

.A 1
2//\6

,(+
4
t
( CHs

cH2cH2
^*ll3\-/s l+ 4
CHg

cH2cH3
First: draw the Second: draw the rest of
cyclohexene ring the molecule connected
to the numbered carbons

Figure 4-20

Copyright @ by The Berkeley Review Exclusive MCAT Preparation


Organic Chemistry llydrocarbons and Reactions Hydrocarbon Reactions

The six membered ring that is formed is cyclohexene. The carbons are numbered
to help identify the product, which will make large, polycyclic products easier to
evaluate. Figure 4-21 shows a more complex Diels-Alder reaction. Because both
the diene and dienophile have substituents, there is a chance that stereoisomers
can form. The stereoselectivity is driven by orbital overlap in the transition state.
The two stereoisomers products, diastereomers, are drawn for the reaction.

o
ro

o
1

4 \
^*:o:::;1"
no
o
Endo Product Exo Product

Figure 4-21.

The last thing for us to consider is regioselectivity. when the diene and
dienophile have substituents, there exists the potential for different structural
isomers. Regioselectivity can be predicted using resonance, where the most
electron rich terminal carbon of the diene attacks the electron poor carbon of the
dienophile. The reaction is optimized when the dienophile has electron
withdrawing groups. Figure 4-22 shows the effect of electron donating and
electron withdrawing groups on the diene through resonance.

o o
EWG EW G EDG EDG

)
t)
4
/) <:+ l-/
\
l_-.-. <)
EWG = electron withdrawing group

Figure 4-22
'a:
EDG = electron donating group

when it comes to predicting regiochemistry, it's as simple as plus attracts minus.


Figure 4-23 shows a Diels-Alder reaction where regioselectivity is an issue.

zl. H
--+
A

H?co .s- 6*
* H3CO CHs
Major

Figure 4-23

Copyright @ by The Berkeley Review The Berkeley Review


Organic Chemistry Ilydrocarbons and Reactions flydrocarbon Reactions

Example 4.12
\A/hich of the following Diels-Aider reactions is
fastest?
A. o
2
l* 4.", --+ CHe
ll\A
HrC\ CHa

B. o
2
l* 4.". -^-
tl- CHa

nrao\ CHa H3CO

C. o

(+ ,4.',,
H.c^ H:C Ar- -CHs
cHs
CH:

D.

2
I,
-^>
"r.o\ H3CO CHs

Solution
The rate of a Diels-Arder reaction is increaseci by
the presence of an electron
donating_ group on the diene and an erectron
withdrawing group on the
dienophile. The rate of a Diels-Alder reaction is decreased
by the presence of
bulky groups in the transition state. Choices C and D are
eliminated because of
the two methyl groups on the dienophile. No matter
how the molecure aligns
entering the transition state, one of the methyl groups wiil
be in the way. In
choice B, there is a methoxy group on the diene, *-hil" ,^
choice A is just a methyr
Methoxy groups donate electron density through resonance, so choice
,grou.p. B
has the more electron rich diene, resulting in a faster
reaction than choice A.

Cope Rearrangement
The Cope rearrangement is a sigmatropic rearrangement
i.vorving two pi-bonds
and one sigma-bond. Figure 4-24 shows a simpl"e Cope rearrangement
carried
out on a 1,S-diene. In more comprex examples, stereo.i-rerr-ristry
rnay be an issue,
because stereocenters can be both formed and rost
as hybridizaiior-, changes.

'G 4
HaC -\g --+
A
,-,,.4,
Figure 4-24

Copyright @ by The Berkeley Review 279 Exclusive MCAT preparation


Organic Chemistry Hydrocarbons and Reactions Hydrocarbon Reactions

The Cope rearrangement involves two pi-bonds and one sigma-bond aligned in
such a way that the terminal p-orbitals of the two pi-bonds are close enough to
overlap. The reaction requires the input of energy to overcome the activation
barrier. The structure of the product is dictated by the orbital overlap in the
transition state.

Claisen Rearrangement
The Claisen rearrangement is similar to Cope rearrangement, except that in a
Claisen rearrangement, the reactant is a vinylic allyi ether. The rearrangement
involves two pi-bonds and one sigma-bond and has a transition state that is
similar to the one observed with the Cope rearrangement. The difference is the
presence of an oxygen. Figure 4-25 shows a Claisen rearrangement.

as o^
w ---l>
A
\,
Figure -l-25

when the ether is benzylic instead of vinylic, the cyclic ketone can quickly
tautomerize to form a phenol. The preference of a phenol over the cyclic ketone
is due to the aromaticity of the benzene ring.

Example 4.13
\z\4rich statement is valid in terms of the sigmatropic rearrangement reactions?
A. Aldehydes are formed from a Claisen rearrangement of a vinylic allyl ether
when the allylic ether carbon is unsubstituted.
B. Aldehydes are formed from a Claisen rearrangement of a vinylic allyl ether
when the vinylic ether carbon is unsubstituted.
c. Aldehydes are formed from a Cope rearrangement of a vinylic allyl ether
when the allylic ether carbon is unsubstituted.
D. Aldehydes are formed from a Cope rearrangement of a vinylic aliyl ether
when the vinyiic ether carbon is unsubstituted.

Solution
Choices C and D are eliminated immediately, because Cope rearrangement
results in the conversion of one 1,5-diene into another 1,5-diene, not an aidehyde.
The question is reduced to determining which carbon in the reactant forms the
carbonyl group following Claisen rearrangement. The reaction is shown be1ow.

H
vinylicT\ I
aidehyde

---l>
l
altvtic etner
It is the vinylic ether carbon that becomes the carbonyl carbon, not the allylic
ether carbon. This means that the vinylic ether carbon must only have a
hydrogen, and no carbons, in order to form an aldehyde and not a ketone. The
best answer is choice B.

Copyright @ by The Berkeley Review The Berkeley Review


Organic Chemistry tlydrocarbons and Reactions Terpenes

Terp,he$
Classification
Terpenes and terpenoids, btological molecules derived from terpenes, are
natural hydrocarbons found in plants and animals that are made from S-carbon
isoprene (2-methyl-1,3-butadiene) units. The five-carbon skeleton of isoprene can
be found in terpenes. Terpenes are classified by their number of carbon atoms.
Monoterpenes have ten carbon atoms, sesquiterpenes have fifteen carbons,
diterpenes have twenty carbons, sesterterpenes have twenty-five carbons and so
on, The in aitro synthesis of terpenes and terpenoids is called nstural product
synthesis. Some naturally occurring monoterpenes are shown in Figure 4-26.

r CH,O
gH
Geraniol Myrcene Citronellol Citronellal
(Oil of Germanium) (Oil of Bay)

Limonene o,-Terpinene y-Terpinene


N" r CH.O

Citral
(or Limin) (Oil of Coriander) (Oil of Coriander) (Oil of Lemongrass)
o H"C. -CH"
I t' CH"
"XJ
tt ."-\1./
u? lS
LR
Y\oH
^
i CH: HeC o
Carvone . - cr-Pinene Camphor
(Oil of Spearmint) Menthol (Oil of Turpentine)
(Oil of Peppermint)

Figure 4-26

Studies in biogenesis show that the large terpenes are synthesized using
isopentenyi pyrophosphate rather than isoprene. Pyrophosphate adds across the
diene of isoprene to form either isopentenyl pyrophosphate or dimethylallyl
pyrophosphate, which are interconverted by isomerization. Figure 4-27 shows
isoprene, isopentenyl pyrophosphate, and dimethylallyl pyrophosphate.

L,Isoprene
o-
oo
ilil
P-
ll
oo-
Isopentyl pyrophosphate
o- P- oH o-
oo
illt
P-
tl
o-
o-
Dimethylallyl pyrophosphate
o- P- oH

Figure 4-27

These molecuies add to one another in a way where the n-bond of isopentyl
pyrophosphate is the nucleophile and pyrophosphate of another molecule is the
leaving group. A proton is lost from the nucleophilic moiety to regenerate a ru-
bond. The reaction involves head-to-taii addition. When cyclizing, the bond that
is formed to complete the ring is rarely connected head-to-tai1. Figure 4-28 shows
the reaction of isopentenyl pyrophosphate and dimethylallyl pyrophosphate.

Copyright O by The Berkeley Review 281 Dxclusive MCAT Preparation


Organic Chemistry Hydrocarbons and Reactions Terpenes

oo
ll il
Geranyl pyrophosphate
Isopentyl pyrophosphate
oo
ll ll
oo
ilil
- r
HO- P- O- P- O- Y HO- P- O- P- O
tt
o- o-
Figure 4-28

Both plants and animals synthesize terpenes. Larger terpenes are built from
multiple additions of isoprene units, including both isopentenyl pyrophosphate
and dimethylallyl pyrophosphate. Geranyl pyrophosphate (a C-10 terpene
derived from the head-to-tail connection of two isopentenyl pyrophosphate
molecules) is the first monoterpene in many natural synthetic pathways.
Another isoprene unit can be added to geranyl pyrophosphate to form farnesyl
pyrophosphate (a C-15 terpene). These molecules can undergo further addition,
dimerization, or modification into other terpenes and terpenoids. Figure 4-29
shows a generic pathway for the biosynthesis of larger terpenes.

loo
I

4./\o-li-o-ll-o.,
l"o
,.^^o-ll_o_ll_on
tt
o- o-
Isopentyl pyrophosphate
(3-Methyl-3-butenyl pyrophosphate)
or Dimethylallr, Orr3pn"rf,i","
(3-Methyl-2-butenyl pyrophosphate)
,
I Isopentyl

{PFophosthate

oo
illt
o-
tt o- P- oH
P-

Monoterpenes (C1s) +_ Geranyl pyrophosphate O- O-


(C1o-pyrophosphate)

tt
(C15) Farnesyl pyrophosphlt" o- o-
(C,5-pyrophosphate)\
sesquiterpenes
- \ squalene ,
(c36)
I tsoPentYl
I pyrophosphate
rr-"r--""r-."".- I

j t
Diterpenes(C2s) # C2g-pyrophosphate Lanesterol (C36)

V {
Tetraterpenes (Ca6) Chloesterol (C27)

Figure 4-29

Copyright @ bv The Berkeley Review 282 The Berkeley Review


Organic Chemistry ffydrocarbons and Reactions Terpenes

As Figure 4-29 shows, the triterpene squarene can undergo


further reactivity
to generate cholesterol, which does not have a number
of carb'ons that is divisible
by five. so while cholesterol may not be a terpene, its
synthesis involves terpenes
and terpenoids. The basic schematic for the biosynthesis
of cholesterol startinj
from isopentenyl pyrophosphate is shown in Figure 4_30.

oo
ilil O- P-
o
j.
-P-
f"
Isopentenyl pyrophosphate
OH

Figure 4-30
As a general rule, smaller terpenes are found primarily
,larger terpenes,
in plants, while some
such as lanesterol (a C-30 precursor to steroid hormones)
and B-
carotene (a C-40 source of vitamin A), aie found in plants
and animals. For
instance, the monoterpene pinene is found only in plants while
Vitamin 41, a
diterpene, is found in both plants and animals. Frg.rr" 4-31 shows
some selected
larger terpenes.

Elemene Farnesol
(oiiofCoral) (Oil of Lemon)
Z+

?or "

W
G-Selinene Vitamin .A,1
(Oil of Celery)

HeC CHg CH: CHs OH

Figure 4-31

Copyright @ by The Berkeley Review 283 Exclusive MCAT preparation


Organic Chemistry flydrocarbons and Reactions Terpenes

One of the skills that you must deveiop to do well on terpene related
questions on the MCAT is to recognize terpenes and be able to identify the
isoprene subunits in the carbon skeleton. Figure 4-32 shows the analysis of some
terpenes for the isoprene units in the skeletal fragments.

Me

+Me -
^4{ ll t
Me

^'r ,-
o,-Terpinene ,ll

Me t,,-=A' "*A
y-Bisabolene
M

Zingerberine
+ix ^4;
Patchouli Oil

Figure 4-32

A common lab technique employed to isolate terpenes is steam distillation,


where the pulp of some natural material is placed into water and boiled so that
the natural oiis are distilled from the pulp. Steam distillation allows the essential
oils to vaporize at a temperature lower than their boiling point, so they do not
degrade. The distillate is a mixture of water and terpenes, which are easily
separated using extraction techniques. Terpenes can also be extracted from pulp.
When isolating terpenes, it is a mixture of geometrical isomers that is collected.
The different geometrical isomers of a terpene are given the prefixes a"-, B-, y- and
so on. The different geometrical isomers have similar physical properties, but
because of differences in conjugation, they exhibit differences in the absorption of
photons in the ultraviolet (UV) and visible range of the EM spectrum.
Terpenes are UV active, because of their n-bonds. An isolated alkene has a
UV absorbance around 180 nm. A conjugated diene has a UV absorbance around
225 nrrl, which is significantly more intense than the absorbance of an isolated
alkene. As the conjugation of a n-network increases, the wavelength of
maximum absorbance, l,*u", and the intensity of absorbance, t, increase. Some
terpenes contain oxygen, which is added in a way that does not alter the carbon
skeleton. Carbonyls exhibit absorbances of greater wavelength than alkenes of
the same conjugation, For instance, carvone (shown in Figure 4-26) is evident b,v
a carbonyl absorption at 7744 cm-1 in its IR spectrum and a strong UV (e >
10,000) absorption at L,p31 = 242 nrrr. Terpenes are often isolated in educational
laboratory experiments. Because of their biological significance and the fact thev
are isolated in lab experiments, they are highly represented on the MCAT. If you
have a fundamental understandrng of terpenes, then you should be fine.

Copyright @ by The Berkeley Review 284 The Berkeley Review


Organic Chemistry flydrocarbons and Reactions Section Summary

Key Points for Hydrocarbons and Reactions (Section 4)


Alkanes
1' Hydrocarbon compounds with only carbons, hydrogens, and sigma bonds
a) Only contain C-C and C-H single bonds
i. Can be aliphatic (straight chain) or cyclic
ii. Low water soiubility, row boiling point, and row melting point
iii. Relatively inert compounds that are used as solvents
b) Undergo free radical halogenation reactions with chlorine and bromine
i. Involves initiation, propagation, and termination in that order
ii.Bromination is more selective than chlorination
iii.
The relative stability for alkyl free radicals is: 3" > 2' > l" > methyl

Hydrocarbon Reactions (Reactions involving ru-bonds)


1. Elimination
a) Loss of an H and a leaving group to form a ri-bond
i. Requires high temperature
ii. Competes with nucleophilic substitution reactions
iii. Goes by way of one of two mechanisms: E1 or E2
b) E1 reactions are similar to 5511 reactions
i. Requires a strong acid (Bronsted-Lowry or Lewis)
ii. Forms carbocation intermediate so rearrangement is possible
iii' Forms most substituted and least sterically hindered alkene
c) E2 reactions are similar to 5512 reactions
i. Concerted reaction that requires a strong, bulky base
ii. Proton to be lost and the leaving group must be anti to one another
iii. No intermediate formed, only a transition state

2. ElectrophilicAddition
a) An electrophile can be added to a n-bond followed by nucleophilic attack
i. The n,bond is a weak nucleophile
ii. The reaction is driven by the strength of the electrophile
b) Electrophilic addition reactions exhibit regioselectivity
i' Steric hindrance and carbocation stability influence the site of attack
ii. when the electrophile attacks the ress substituted carbon, the
reaction is said to be a Markovnikov addition
iii. when the electrophile attacks the more substituted carbon, the
reaction is said to be an anti-Markovnikov addition
c) Electrophilic addition reactions exhibit stereoselectivity
i. Steric hindrance is the most influential factor in stereoselectivity
ii. when substituents add one at a time (the first is added before the
second on attacks) the product exhibits anti addition stereochemistry
iii. when substituents add simultaneously (both add at the same time)
the product exhibits syn addition stereochemistry

Copyright @ by The Berkeley Review Exclusive MCAT Preparation


Organic Chemistry Ilydrocarbons and Reactions Section Summary

3. 1,2-Additionversus 1,4-Addition
a) Electrophilic addition reaction can proceed in multiple ways with
conjugated n-systems
i. Typically, \,2-addition is favored at lower temperatures (kinetic
control)
ii. Typically, 1,4-addition is favored at higher temperatures
(thermodynamic controi)

4. Diels-AlderReaction
a) Reaction of a diene and a dienophile (alkene)
i. An electrocyclic reaction carried out with either light or heat
ii. Forms a cyclohexene product
iii. Stereoselectivity: Endo product is preferred over exo product
iv. Regioselectivity: Depends on the resonance nature of the groups on
the reactants

5. Sigmatropic Rearrangement (Cope and Claisen Rearrangements)


a) Both sigma-bonds and pi-bonds are broken and formed via a cyclic
transition state
i. Cope rearrangement converts a y,8-unsaturated alkene into another
y,8-unsaturated alkene via a realignment of molecular orbitals
ii. Claisen rearrangement converts a vinylic allyl ether into another y,5-
unsaturated carbonyl via a realignment of molecular orbitals
iii. Sigmatropic rearrangement requires heat
Terpenes
1. Natural products derived from the connecting of five-carbon units
a) Derived via biosynthesis involving either isopentyl pyrophosphate or
dimethylallyl pyrophosphate
b) Can be cleaved into isoprene subunits
i. Terpenes are named for their carbon count: 10 C = monoterpene, etc.
ii. Isolated by steam distillation or extraction as natural oils
iii. Presence of n-bonds results in UV absorbances. As conjugation
increases, intensity and l"*u* both increase.

Copyright @ by The Berkeley Review 286 The Berkeley Review


Alkanes : and,, ftr5rdroctrrbon' Keaction:s

I. Free Radical Halogenation Selectivity (r -7)


II. Free Radical Reactions (B - 14)

III. Elimination and Stereochemistry (r5 - 2t)


IV. Elimination Study (22 - 28)

V. Phermones (2e - 35)

VI. Creen Synthesis (36 - 4t)


VII. Conjugated n-Networks (42 - 48)

VIII. Diels-Alder Reaction Rate Study (4e - 54)

IX. Diels-Alder Reaction (s5 - 60)


X. Claisen and Cope Rearrangements (6r - 67)
XI. Isoprene Units (68 - 73)

XII. Terpenes (74 - BO)

XIII. Fatty Acids and Oils (81 - 86)

XIV. Occidentalol Synthesis (87 - e2)

Questions not Based on a Descriptive Passage (e5 - loo)

Alkanes and Hydrocarbon Reactions Scoring Scale


Kaw Score MCAT Score
84 - 100 l5-15
66-85 10-12
47 -65 7 -9
34-46 4-6
t-33 t-5
Passage I (Questions 1 - 7) 3 . The chlorination of methylcyclopentane would yield
how many different structural isomers?
Alkyl halides can be lormed tiom a halogen reacting with
an alkane. Alkanes can be treated riith either chlorine gas and
4.2
UV radiation to form chloroalkanes. or rvith bromine liquid B. 3
and UV radiation to form bromoalkanes. The bromination of c. 4
an alkane is a slower and more selective reaction than the D. 5
chlorination of an alkane. Because of this greater selectivity,
bromination is preferred over chlorination in the synthesis of
highly substituted haloalkanes.
For the chlorination of an alkane, the reactivity
preference for carbon substitution through liee radicals 4. What is the most abundant product in the bromination
of 2-methylbutane?
follows the trend 3'carbon > 2" carbon > 1'carbon by a
factor of 4 : 2.5 : I at a given temperature. This means that A. 1-bromo-2-methylbutane
for a compound like butane with four secondary hydrogens B. 2-bromo,2-methylbutane
and six primary hydrogens, the ratio of chlorination products C. 2-bromo-3-methylbutane
is not based on random probability aione. Random D. 1-bromo-3-methvlbutane
probability predicts the formation of two 2-chlorobutanes for
every three 1-chlorobutanes. Because of the reactivity
preference of secondary carbons over primary carbons by a
ratio of 2.5 : l, the product distribution instead is five 2-
chlorobutanes to three 1-chlorobutanes. This implies that the 5 . Following the free radical monochlorination of pentane
percentage of secondary products is 62.5Vo (rather than 40Vo what is the ratio of2-chloropentane to 1-chloropentane?
which is expected when there is no site preference).
A. 2:3
The calculation of the number of products is based on the
reactivity coefficient times the number of unique hydrogens.
B. 5:3
In a molecule like pentane, there are three types of hydrogens c.2.5:1
in a 3 : 2'. I ratio. Six hydrogens are secondary and six are D. 3: 1

primary. The abundance of the primary hydrogens is 6


hydrogens x 1 reactivity for 6. The abundance for carbon two
and carbon four is 4 hydrogens x 2.5 reactivity for 10. The
abundance for carbon three is 2 hydrogens x 2.5 reactivity for
5. This means that the chlorination product ratio is six l- 6 . In the monochlorination of n-hexane, how can a ratio of

chloropentane to ten 2-chloropentane to five 3-chloropentane.


1.07 2-chlorohexane to one 3-chlorohexane be
explarned?
1 . How many degrees of unsaturation are there in the A. Based on the relative reactivity of the carbons and
compound C5H9C1? the abundance of hydrogens, a ratio of 1.07:1 is
A. 0 expected.

B. 1
B. The second carbon of n-hexane is less sterically
hindered than the third carbon.
C,2
D. 3 C. The second carbon ofn-hexane can better stabilize a
free radical due to resonance.
D. The third carbon of n-hexane can better stabilize a
free radical due to the inductive effect.

2 . How many structural isomers of C5H11Cl are there?


A. 5 7 . If the alkane reactant exhibits ring strain, the instability
of the free radical intermediate may cause the ring to
B. 6 break. Which free radical alkane is LEAST stable?
c.1 A. Propyl free radical (C:HZ.)
D. 8
B. Cyclopropyl free radical (CSHS.)
C. Butyl free radical (C+HS.)
D. Cyclobutyl free radical (CqHl.)

Copyright @ by The Berkeley Review@ 289 GO ON TO THE NEXT PAGE


Passage ll (Questions 8 - 14) 9. Which of the following energy diagrams corresponds to
reaction of 12 with an alkane?
A halide can be substituted onto an alkane by way of a
free radical mechanism. Halogenation of an alkane is B.
initiated by the homolytic cleavage of a diatomic halogen
molecule into free radical halogen atoms. During subsequent
steps in the reaction, an alkane reacts with the halogen free I
radicals to form an alkyl halide. The reaction requires some bo
source of activation energy to cleave the halogen-halogen
bond. Depending on the halogen-halogen bond strength, the nE
Reaction Coordinate Reaction Coordinate
amount of activation energy necessary varies from thermal to
ultraviolet radiation for the initiation step.
The mechanism is a sequence broken down into steps
that fit into one of three categories: initiation, propagation,
and termination, in that order. The initiation step involves
homolytic cleavage of a halogen-halogen bond to form two
free radicals. The second phase of the reaction sequence is
propagation where the free radical is transferred through a set
of abstraction reactions. The last phase of the sequence is the
Reaction Coordinate Reaction Coordinate
termination step where two free radicals combine to fbrm a
sigma bond. The reaction involves two transition states. in
which the second is of higher energy than the first. Table 1
shows the bond energies of the halogens and the reaction
enthalpies for the various halogenation reactions:
1 0. The first propagation step in a free radical reaction is
Compound B.D.E. AH.t which of the following?

154 kJ -483 kJ A.XZ + R. RX + X.


F2
mole B. RH + X. ->
-> HX + R.
mole
Clz n9 kI 114 kJ C.Xz 2X.
mole mole ->
D. X. + R.
Br2 193 kJ -33 kJ
-> RX
mole mole
I2 149 kJ t1'.. kJ
mole mole
Table I 1 1. The strongest halogen-halogen bond corresponds to
which of the following?
The enthalpy of a chemical reaction can be found by
using Equation l. A. The shortest halogen-halogen bond.
B. The second shortest halogen-halogen bond.
AHr^n = I Energy6onds broken - Energy6on6s fbrmed
C. The longest halogen-halogen bond.
Equation 1
D. The second longest halogen-halogen bond.
The average bond dissociation energy {br a sigma bond
between an spJ-hybridized carbon and ahydrogen is 413 kJ
per mole. A hydrogen free radical cannot be formed in this
reaction mechanism.
12. Which of the following steps is NOT found in a free
8 . The MOST stable type of carbon fr-ee radical fonned in
radical halogenation reaction ?
the monobromination of (R)-3-methylhexane is best
described as: A. RH + X. RX + H.
A. B. XZ + R. -> RX + X.
primary. ->
B. seconclary.
C. RH + X.-> HX + R.
C. D. X. + R. RX
tertiary. ->
D. quatefnary.

Copyright @ by The Berkeley Review@ 290 GO ON TO THE NEXT PAGE


13. Which of the following conclusions can be inferred Passage lll (Questions 1S- 21)
from the observation that usually only one halide reacts
with the alkane and minimai multiple halogenated Due to the usefulness of alkenes in synthesis, they are
products ar.e isolated from the product mixture? often a starting material in many synthetic transformations.
I. Halogens do not help to stabilize free radical This can be attributed to the numerous addition reactions
intermediates. alkenes undergo. Because of their synthetic usefulness, it is
II. Halogens, once on an alkane, increase the C-H important to be able to synthesize alkenes in a geometrically
bond strength. specific manner. This is to say that it is beneficial to be able
to create predominantly the trans (E) geometrical isomer or
III. Halogens, once on an alkane, decrease the C_H
bond strength.
predominantly the cis (Z) geometrical isomer. Reactions
proceeding by the E1 mechanism result in the formation of
A. II only mostly the trans geometrical isomer with some cis
B. III only geometrical isomer formed. The E2 mechanism, on the other
C. I and II only hand, allows for the formation of either the trans or cis
D. I and III only geometrical isomers in high purity, if there are chiral centers
present in the reactant. Reaction 1 is an E2 reaction carried
out in the hopes of synthesizing Z-3-methylpentene, to be
used in subsequent steps ofg total synthesis process:

Br
CH"
HqC CHrCH'
4. Given that a C-F bond energy js 462 kJ per mole and a '
><
1 rbutOK
H-F bond energy is 588 kJ per mole, what is the heat of -->
t-butOH
reaction associated with fluorination of an alkane?
H:C CH2CHj H CHr
A. - 483 kJ
mole Reaction I
B. -lt6 kJ Reaction 1 proceeds by an E2 mechanism at elevated
rnole
temperature, so the bromine leaving group and the hydrogen
C. +116 kJ
mole on carbon 3 must be aligned in the anti orientation. As
drawn in Reaction 1, the bromine leaving group and the
D. + +83 kJ hydrogen on carbon 3 are not correctly-al-igned for f2
mole
elimination, nor is the structure drawn in its most stable
conformation. Counting the conformer shown in Reaction l,
there are three staggered conformations total for the reactant.
Only one of the three orientations has the anti orientation
necessary for the E2 reaction.

15. What conclusion can be made about the elimination


reaction that generates the following data?

Trial (H3C)3CBr t-ButOK Rate


I 0.25 M 0.25M 4.61 x t0-3 M/s
il 0.50 M 0.25M 9.29 x t0-3 Mls
III 0.40 M 0.50M 1.47 x 10-2 Mts
A. The reaction proceeds by an E1-mechanism,
because the data indicate that the rate-determining
step is unimolecular.
B. The reaction proceeds by an E1-mechanism,
because the data indicate that the rate-determining
step is bimolecular.
C , The reaction proceeds by an E2-mechanism,
because the data indicate that the rate-determining
step is unimolecular.
D. The reaction proceeds by an E2-mechanism,
because the data indicate that the rate-determining
step is bimolecular.

Copyright @ by The Berkeley Review@ 291 GO ON TO THE NEXT PAGE


16. Which of the staggered complexes is MOST stable for 20. What is the role of concentrated sulfuric acid in an E1
the reactant? elimination reaction?
A. The conformer drawn in the example. A. Sulfuric acid protonates the leaving group making
B. The conformer fiom which the E2 elimination it a better leaving group.
takes place. B. Sulfuric acid serves to dehydrate the solvent
C . The conformer with carbons I and 4 anti to one preventing back reaction.
another. C. Sulfuric acid dissociates into sulfate which helps
D. All three staggered conformers all equal in stability. remove the proton allowing the leaving group to
leave.
D . Sulfuric acid stabilizes the carbocation intermediate
by protonating the cationic carbon.

17. The loss of optical rotation in the reaction can be


explained by which of the following statements?
A. Ez eliminates the carbon 2 chiral center.
B. EZeliminates the carbon 3 chiral center.
21 . How many units of unsaturation are present in the
product of chlorocyclohexane and strong base and heat?
C. Ez eliminates both the carbon 2 chiral center and
the carbon 3 chiral center.
A. 0
D . The product is a pair of enantiomers.
B. I
c.2
D.3

I 8. Which of the following structures (from the perspective


of the eye) is the Newman projection for the reactant?
Br
CH:
H$t),
H:C CHzCHT

A. B.

",$.".
Br
CH:

H
CH2CHj CH2CH3

c. D.
Br Br

;"4; ;"V: CH2CH3 CH2CH3


pru
rea{
reag

)'t
19. The ENANTIOMER of the reactant has which of the
followin g stereochemical orientations ?

A. 2R, 3R
B. 2R, 35
C. 25,3R
D. 25, 35

Copyright @ by The Berkeley Review@ 292 GO ON TO THE NEXT PAGE copl:


Passage lV (Questions 22 - 28) 23. Which of the reactions results in an optically active
product mixture?
Elimination results in the formation of a new n-bond
following the loss of two groups from the molecule. The
A. Reaction I only
reaction can proceed by one of two possible mechanisms: E1 B. Reactions I and II only
or 82. C. Reactions II and III only
In E1 reactions, the leaving group first leaves to form a D. Reaction III only
carbocation intermediate. The carbocation has the potential
to undergo rearrangement. The solvent serves as a base and
deprotonates a hydrogen off of the carbon adjacent to the
cationic carbon. The result is the formation of a new fi-bond.
24. When the following reaction is carried out, why does
In E2 reactions, the leaving group leaves simultaneously the optical activity disappear?
as the proton is removed by a strong base. The proton and
leaving group must be anti to one another in a staggered
conformation to undergo an E2 reaction. An E2 reaction conc. HrSOa Major Product
requires that the base be strong, to remove a weakly acidic cH3 60- (crp=6';
hydrogen, and bulky to reduce the amount of side products
formed from nucleophilic substitution. Figure I shows three CHr
elimination reactions carried out concurrently. A. After the sulfate group substitutes for the hydroxyl
Reaction I: group, the chiral centers cancel one another.
CI B. The product is meso.
Yg C.

@
The major product is an achiral alkene, resulting
from rearrangement.
t-butOK
-----'+ D. The product is an achiral alcohol.
t-butOH,50"C

II:

&
Reaction
CI
25 . Which of the following observations are consistent with
the mechanisms discussed in the passage?
t-butOK
-..+
rbutOH,50'C
I. Increasing the base concentration in Reaction I
increases the reaction rate.
II. The amount of alkene product is maximized at
lower temperatures.
III:

&
Reaction
OH III. Reaction II has a competing an Sp2 reaction.
A. I only
conc. HrSOa
B. II only
_->
C. I andll only
60'c D. I and III only
Figure L. Three Elimination Reactions

The product shown in each of the reactions is the major


product. Among the minor side products for each of the three 26. Why do Reactions I and II yield different major
products?
reactions is the alkene products from one of the other two
reactions. A. Et reactions can undergo rearrangement if the
hydrogen is on the correct side of the plane.
2 2. Which of the elimination reactions in Figure 1 involves B. Et reactions are influenced by steric hindrance.
rearrangement? C. EZ reactions require that the leaving group and
A. Reaction I only proton are anti to one another.
B. Reactions I and II only D . EZ reactions require that the leaving group and
C. Reactions II and III only proton are syn to one another.
D. Reaction III only

Copyright @ by The Berkeley Review@ 293 GO ON TO THE NEXT PAGE


27 . How does the hybridization change fbr the carbons Passage V (Questions 29 - 35)
common to both rings in Reaction III?
A, sp2 to sp3 Phermones are chemicals secreted by animals (most
commonly insects) that elicit a specific behavioral reaction in
B. spz to sp other members of their same species. They are effective in
C. sp3 to sp2 low concentration in sending signals between members of the
D. spi to sp same species for such things as reproduction, danger
warnings, and aggregation (in the case of a food supply.)
Many phermones are simple hydrocarbons. For instance,
when in danger, ants secrete undecane (.Cy1H2$ or tridecane
(Cf :HZS) to inform other ants of the trouble. Many of the
traps and sprays we use to capture and kill insects take
28. If diethyl amine, (H3CCH2)2NH, is used in Reacrion I
advantage of sex attractants. The structures of four sex
instead of t-butOK, what is the major product? phermones are shown in Figure 1.
a'E,rN

Tiger Moth sex attractant


(2-Methyl heptadecane)

o4.",
Oriental Fruit Moth sex attractant
[(E)-8-Dodecen- I -yl acetare]

HpCa
\J
/\
.CttHzt

HH
House Fly sex attractant
(Muscalure [(9Z)-Tricosenel)

Silkworm Moth sex attractant


(Bombykol)
Figure 1 Four random phermones
Phermones are specific to each species, because receptor
proteins are highly selective in what they bind. In one of the
rare cases where two geometrical isomers both elicit the same
response, the Oriental Fruit Moth responds to both the E-
isomer, shown in Figure 1, and the Z-isomer. There are
cases where two similar species to a phermone that is similar
in structure, but not exactly the same. For instance, the
Grape Berry Moth uses (Z)-9-dodecen-1-yl acetate as a sex
attractant in roughly the same concentration that the Oriental
Fruit Moth uses (Z)-8-dodecen-1-yl acatate.

29. The Silkworm Moth sex phermone has all of the


following structural features EXCEPT:
A. one stereogenic center.
B. one cis double bond.
C. no tertiary carbons.
D. conjugation.

Copyright @ by The Berkeley Review@ GO ON TO THE NEXT PAGE


30. Relative to the Oriental Fruit Moth sex phermone 3 5. Which spectroscopic observation does NOT correlate
shown in Figure 1, the follorving compound is: with the conesponding compound?
o A. The Oriental Fruit Moth sex phermone: an IR
il absorbance at 1141 cm-l
oA cH. B. Bombykol: a UV-visible absorbance at22'l nm
C. Muscalure: two signals around 5.00 ppm in its
A. a confbrmational isomer. luNIrvlR
B. a geometrical isomer.
D . The Tiger Moth sex phermone: 14 signals in its
C. an optical isomer. l3cNltR spectrum
D. a structlu'al isomer.

31.. Which of the following statements accuraiely relates the


four structures shown in Figure 1?

I. Muscalure has a shorter wavelength of rnaximum


absorbance in UV-visible spectroscopy bombykol.
II. The Tiger Moth sex phelmone has the more units
of unsaturation than undecane.
m. The Oriental Fruit Moth sex phermone can be
classifled as a terpene.
A. I only
B. iII only
C. I and II only
D. i and III only

3 2. What physical property is NOT expected for muscalure?


A. Low miscibility in water
B. A boiling point above room temperature
C. High lipid solubility
D. The ability to rotate plane-polarized light

33. Which of the phermones in Figure t has the greatest


number of primary carbons?
A. The Tiger Moth sex phermone
B. The House Fly sex phermone
C. The Oriental Fruit Moth sex phermones
D. The Silkworm Moth sex phermone

34. The Green Peach Aphid def'ense phermone is shown


below.

What is NOT true of the structure?


A. It can be synthesized from isoprene units.
B . It has *o.e sp2-hybridized carbons than spj-
hybridized carbons.
C . It can potentially undergo 1,4-addition.
D. lt is highly flexible.

Copyright O by The Berkeley Review@ 295 GO ON TO THE NEXT PAGE


Passage VI (Questions 36 - 41) -l 6 . In Reaction I, the alkyne is besr described as:

In the recent years, many chemists around the world have


A. a dienophile.
shifted their focus to so called green chemistry. Green B. an electrophile.
chemistry, also called sustainable chemistry, aims to develop C. a nucleophile.
chemical reactions and processes that are environmentally D. an oxidant.
safe. The goal is to reduce waste generation rather than
ernploying waste management--the "end of the pipe"
solution. The most significant alteration to traditional
chemistry is the recycling of solvent, or in optimal cases, the 3 7. Which of the following intermediates is consistent with
elimination of solvent. This is achieved in many ways, the two structural isomers formed in Reaction II?
including doing reactions under high pressure to make the
system act like a supercritical f'luid.
A.o B.o
Areas of current research in green chemistry include the il il
C
use of renewable raw materials, direct oxidations using
oxygen, improved separation during the course of a reaction, /\ ,zt\
and all forms of catalysts. The aim is to maximize atont- HzC- CHCH3
economy, the tracking of how many atoms used in the CHz
reaction end up in the product, by not using solvents or
protecting groups. Figure I lists three atom-economical
c' .cH,
D' CH.
reactions used in green synthesis.

Reaction I:
-tt
HrC- C/
tt
HC-C
,/

H2C-O
.+ i.sd'u.":.:.
H:C
COzCHT H2C- O

co2cHj 98%o vterd


38. Which of the reactions in Figure I involves the
fbrmation of new stereocenters?
A. Reaction III only
Reaction II: o B. Reactions I and II only

H
A CH:CHzCHr
C.
D.
Reactions I and III only
Reactions II and III only
H2C= CHCHj +CO+Ho-
- cat. Rh O

HJt CH(CHr)r 3 9. Which of the following changes does NOT fit with th;
philosophy of green chemistry?
A. Using supercritical fluid as a solvent.
B. Using protecting groups and not removing ther
until the very last step of the reaction.
Pd, H2
Supercritical CO2
C . Using solid-state catalysts built into the 1ar
equipment.
cHr l D. Running a constant stream of oxygen gas througi
HrC Hrc the reaction vessel fbr oxidation reactions.
> 997o yield

Figure 1. Three Atom-economical Green Syntheses


40. Reaction III can be described by all of the follorvin_.
Reaction I is a Diels-Alder reaction. Reaction 2 is a terms EXCEPT:
hydrofbrmylation reaction, and Reaction 3 is a hydrogenation
reaction. All o1' the reactions in Figure i start with alkenes, A. reduction.
a common starting material in the production ol plastics and B. hydrogenation.
polymers. Green chemistry is ideal fbr polymerization C. stereoselective.
reactions, which by design aims to minimize the material D. regioselective.
need to carry out the propagation reaction. Green synthesis
techniques can be applied to any reaction.

Copyright @ by The Berkeley Review@ GO ON TO THE NEXT PAGE


41 . What is the major product of the reaction below? Passage Vll (Questions 42 - 48)
o
lt A chemist set out to synthesize a series of conjugated

e(+ la" -:+


,\A
dienes. Starting with an allylic alcohol, generating a
conjugated diene involves an acid catalyzed elimination
reaction. Elimination by way of an E1 mechanism to form a
o conjugated diene is shown in Reaction 1.

OH
oB'o
tu
A.

conc. HrSO,
A

Compound I Compound II
o o
Reaction

ee("q"
1
C.
uo
o D.
Reaction 1 is monitored using UV spectroscopy. Over
the course of the reaction an intense UV absorban ce at 179
nm diminishes as a new peak at 222 nm appears.

o
When the product of Reaction I is treated with acidic
o water, two products of detectable quantity are formed. Figure
I shows the distribution of the two hydration products at
35'C, labeled Compound 3a and Compound 3b.

OH

@
I

OH
Compound III Compound IV
39.27n 60.\Vo

Figure I
The percentage ofthe secondary alcohol formed increases
as the temperature ofthe hydration reaction increases. This is
attributed to a shift from kinetic control to thermodvnamic
control.
It is found that if the allylic alcohol in Reaction I is
replaced by a new compound containing both an alcohol
group and a carbon-carbon n-bond, with the exception of a
vinylic alcohol, a conjugated diene is formed upon treatment
with concentrated strong acid at elevated temperatures.

4 2. Which of the following starements accurately reflect


Reaction I?

I. Rearrangement is possible during the reaction.


II. A vinylic carbocation is formed as an intermediate
in the reaction.
m. The first step of the reaction is the protonation of
the hydroxyl oxygen.
A. I only
B. II only
C. I and II only
D . I and III only

Copyright @ by The Berkeley Review@ GO ON TO THE NEXT PAGE


43. Which spectroscopy technique is MOST effective ln 4 8. What is the major product of the reaction below?
distinguishing Compound III from Compound IV?
a. IHNlaR H2O/H2SOa
B. Infraled
C. Ultraviolet O[.", 75'C

D. Visible
A. -..>B' oH

44. Cornpound
A.
B.
IV is best described
a single, achiral molecule
as:

d.,, d..,
"G;
a single, chiral molecule.
C. a pair of diastereomers.
D. a pair of enantiomers.

45. Which of the following

^er* "ff*
compounds is an allylic
alcohol?

'er-* 'er*
46. 1,2- and 1,4-addition is possible in all of the following
compounds EXCEPT:
A. 2,4-Hexadiene
B. 2-methylcyclopentadiene
C. 1,4-cycloheptadiene
D. 1,3-cyclohexadiene

4 7. Which of the following species is NOT an inrermediate


formed during the hydration of Compound II?
A'

A^
tgr,
'eD
t;,,\--

\-ry

Copyright @ by The Berkeley Review@ GO ON TO THE NEXT PAGE


Passage Vlll (Questions 49 - 54) The product of a Diels-Alder reaction is a cyclohexene
derivative. The Diels-Alder reaction is classified as an
The Diels-Alder reaction is quicklv becoming the premier
electrocyclic reaction and it is believed to proceed by a
way to synthesize polycyclic compounds. New carbon_
concerted mechanism that goes through a so called aromatic
carbon bonds are formed when a conju_sated diene reacts with
transition state, where the six n-electrons in the reactant
a compound containing a n-bond (dienophile) at elevated
interact to form new bonds.
temperatures.
In the presence of a Lewis acid, the rate of the reaction
The reaction is fastest when the conjugated diene has an
increases substantially, which implies that the Diels-Alder
electron-donating substituent and the dienophile has an reaction has an alternative mechanism, where the conjugated
electron-withdrawing substituent. Reaction 1 was carried out
diene acts as a nucleophile by attacking the dienophile. This
for a total of eight trials to determine the effects of electronics
infbrmation is in agreement with the rate shift associated
and steric hindrance on the reaction rate. Table I lists the
with the addition of electron-withdrawing groups to the
data fbr the eight trials.
dienophile.

49 . The product shown in Reaction I is one of two


enantiomers that is formed. Which of the following
pairs of molecules represents the product mixture
formed in Trial 6?
A. CH:

CH: CH:
Reaction I &
""1( CHr cHr

Relative o
Trial A B C D X
rate
CH: o
H H H H H t.0
"J( CH: &
cHs
CHr CHq
2 CH: H H H H 5.3

4
J CH:

CH: CH:
H H

H
cHr

H
H

H
8.1

0.00s0
"ff::, CH:
&
CH:
.

5 H H H H '\'. 82 o

6 cHr H H H
o

'Y'"',
o
4t1
'$f;: cHr
&
cHr H H CHr
T,"'
o
613

o
cHr

8 CH: CH: H H '1('"' 0.34


o
Table 1

Copyright @ by The Berkeley Revierv@ 299 GO ON TO THE NEXT PAGE


50. How would the relative rate change if ethyl groups were s4. Which of the following compounds is the WORST
used instead of methyl groups in Trial 4? dienophile?
A. The relative rate would be significantly lower than A.NB.
0.005.
B. The relative rate would be about 0.005.
C. The relative rate would be significantly higher than

D.
0.005.
The relative rate would be 200, the reciprocal of
Io'""''
COCHzCHT
0.005.

51. Addition of which of the following


c.o D' cHrCHe
species to Trial 7
would increase the rate?
/
A. AICI3
B. CCla (" ()
C. KH
D. LiAlHa

(t If Trial 2 proceeds by a nucleophilic mechanism, rather


than a concerted mechanism, the intermediate carries
charges. Which of the following structures best shows
the structure of the intermediate in Trial 2?

y*,(/*,
c' D'
cHr oo cHq oo

53. How can the significantly lower reaction rate in Trial 4


than Trial 2 best be explained?
A. The methyl groups on the conjugated diene exhibit
steric hindrance when the conjugated diene and
dienophile form the transition state.
B. The methyl group on the conjugated diene
withdrawals electron density from the conjugated
diene.
C. The two methyl groups are acting like
intramolecular Lewis acids on the dienophile.
D . The carbonyl group is more electron-withdrawing
on the dienophile in Trial 2 than it is in Trial4.

Copyright @ by The Berkeley Review@ GO ON TO THE NEXT PAGE


Passage lX (Questions 55 - 60) 5 6. Which of the following conclusions can be drawn from
the data in Table I
In a Diels-Alder reaction, the alignment of the diene and
dieneophile determines the structural orientation of the
A. OCH3 is more electron donating than CH3 because
OCH3 in the X position yields more producr A.
substituents in the final product. If both the diene and the
dieneophile are asymmetric, then there are two different B. OCH3 is more electron donating than CH3 because
orientations that the two reactants can assume when they OCH3 in the X position yields more producr B,
align to form the transition state. The preferred alignment C. CH3 is more electron donating than OCH3 because
can be predicted using resonance theory. The two reactants CH3 in the X position yields more producr A.
align in a manner so as to have a partially positive site D. CH3 is more electron donating than OCH3 because
attacking a partially negative site. A generic Diels-Alder CH3 in the X position yields more product B.
reaction of an asymmetric diene with an asymmetric

*ff..q.
dieneophile is shown in Figure 1.
X o X o X

.+. 5 7. Predict the major product for the following reaction:


H3CH2CO O

)^.
o l+L^*
Compound A CompoundB \ cHrcr
Figure I Asymmetric Diels-Alder reaction
A. B.
Ifthe reactants are asymmetric, the product distribution
of Product A to Product B is never 50-to-50. When X is
electron donating and Y is electron donating, Product A is the
''""44.
major product. When X is electron withdrawing and y is
electron donating, Product B is the major product. Table I
lists the product distributions for a series of reactions where
\,4 cH2cl CH2CI
the X and Y groups are varied. In both Product A and
Product B, the Y group is always cis to the carbonyl group.
c. D.
H3CH2CO H3CH2CO

eq: Oq:
X Y A B
OCH3 NHCH3 94Vo 6Vo

OCH3 CH: 88Vo l2%o

CH: NHCH3 87Vo l3Vo o o


cHr CH: 63Vo 37Vo

COCH3 NHCH3 lSVo 82Vo

COCH3 CH: 3lVo

,o
69Vo
5 8. What is the major product for the following reaction?
Table 1

The product distribution in Table 1 supports the ____>


prediction about the electron donating and withdrawing effects 150'C
based on resonance theory. A methyl substituent is
considered to be mildly electron donating.

55. If the Y-substituent is a second carbonyl functional


^O:")
group (-CR=O), making the alkene reactant
symmetric, what would be predicted for the distribution
between Product A and Product B? c.
A. >50Vo Product A;<S}VoProductB
B. <50Vo Product A:>50Vo ProductB
C. 507o Product A:50Vo Product B
D. Product A and Product B are the same compound

Copyright @ by The Berkeley Review@ 301 GO ON TO THE NEXT PAGE


59. Two structural isomers are formed fr.om Diels-Alder Passage X (Questions 61 - 67)
reactions that involve:
Pericyclic reactions are single-step reactions that involve
A. a symmetric diene with a symmetric dieneophile.
the movement of electrons through cyclic transition states
B. an asymmetric diene with symmetric dieneophile.
that involve pi and sigma orbitals. One class of pericyclic
C. a symmetric diene with asymmetric dieneophile. reactions is the sigmatropic rearrangement, which involves
D. an asymmetric diene with asymmetric dieneophile. the migration of a sigma-bonded group across a pi-electron
system. The two most common sigmatropic rearrangements
are the Cope rearrangement and the Claisen reatangement. In
the Cope rearrangement, one 1,5-hexadiene yields a new 1,5-
hexadiene. In the Claisen rearrangement, an allyl vinylic
ether yields an unsaturated carbonyl compound. Both
60. Counting stereoisomers, how many possible products reactions are shown in Figure 1 below.
can be formed from the followine reaction?
Cope rearrangement
H:Co
HrC
CH.
'----*
A

4.2 A 1,5-diene A new 1,5-diene


B.4
c.8 Claisen rearrangement
D. 16
o/N
v
l)\
An allyl vinylic ether
t3,31 _

J
A y,&unsaturated
carbonyl compound
Figure 1 Cope and Claisen rearrangements 64"

When the reactant in a Claisen rearrangement includes a


benzene ring, the ketone formed from the allyl phenylic ether
undergoes tautomerization and converts into a phenol. The
ultimate product is the one that is most stable. Figure 2
shows a synthesis pathway that involves a Claisen
rearrangement, a Cope rearrangement, and tautomerization.

CH:
Hrc
__>I
Step
A

Step II Ia
v
OH o
Hrc
Step III 65. rn
<-
A.
B.
CH: cHr C.
Figure 2 Synthesis using Claisen and Cope rearrangements
D.

Copyright @ by The Berkeley Review@ 302 GO ON TO THE NEXT PAGE Copyright


61. How can it be supported that the Cope rearrangement is 6 6. What spectroscopic evidence supports the formation of a
concerted rather than a multistep process involving product in a Claisen rearangement?
substitution?
A. Appearance of a signal at 9.5 ppm in the 1HNMR
A. All of the stereocenters are retained
B. Appearance ofa broad absorbance around 3400 cm-t
B. All of the stereocentersare inverted in infrared spectroscopy
C. Several cross products are formed C. Disappearance of an absorbance around 1700 cm-l
D. No cross products are formed in infrared spectroscopy
D . Disappearance of two signals between 5 and 6 ppm
in rhe IHNMR
62. Step III in the synthesis shown in Figure 2 occurs
because the product:

A. loses steric hindrance after oxidation.


B. gains aromaticity after reduction. 6 7. What is true of the Claisen and Cope rearrangements in
Figure 2?
C. loses resonance after tautomerization.
D. gains aromaticity after tautomerization. A . Step I is Claisen rearrangement and step II is Cope
rearrangement; the units of unsaturation decrease
from 5 to 4 during the Claisen rearrangement.
B. Step I is Cope rearrangement and step II is Clasien
63 . Heat serves what role in the Claisen rearrangement? rearranggment; the units of unsaturation decrease
A. To provide energy to overcome from 5 to 4 during the Cope rearrangement.
the activation
barrier C . Step I is Claisen rearrangement and step II is Cope
B. To drive the exothermic reaction rearrangement; the units of unsaturation remain 5
during the Claisen rearrangement.
C. To generate pi-bonds D. Step I is Cope rearrangement and step II is Claisen
D. To fbrm sigma-bonds rearrangement; the units of unsaturation remain 5
during the Cope rearrangement.

64 . Which of the following orbital arrangements represents


the transition state of a Cope rearrangement?
A. B.

6 5. The reactant in Figure 2 is best described as a:


A. allyl benzylic erher.
B. allyl phenylic ether.
C. vinyl benzylic ether.
D. vinyl phenylic ether.

Copyright @ by The Berkeley Review@ 303 GO ON TO THE NEXT PAGE


Passage Xl (Questions 68 - 73)

The isoprene unit is one of nature's favorite building


blocks. Isoprene (2-methyl-1,3-butadiene) reacts at carbons
one and two or one and four with other isoprene molecules to
i
ibrm terpenes, a class of bio-organic molecules. Terpenes are
il
found in such natural products as rubber and essential oils. 1l

1i

Nearly all of the naturally occurring terpenes result from the


head-to-tail connectivity of isoprene units. They connect by
undergoing either nucleophilic substitution or electrocyclic
addition, such as a Diels Alder reaction. Figure 1 shows
some common terpenes.

i"'
H:C
-s
CH:
cHr

cr-Pinene Caryophyllene

HSC CH: CH: cHr OH


Vitamin.\

t CHr

*o^ HO
Cholesterol
Citronellol
Figure 2. Biosynthesis of Cholesterol
Figure 1. Common Terpenes
6 8. Which of the following compounds is NOT a terpene?
Both plants and animals synthesize terpenes. Pinene, a A. Limonine (C16H16)
monoterpene, is fbund in plants and Vitamin A1, a diterpene,
B. Geraniol (C16H1gO)
is found in both plants and animals. Smaller terpenes are
found primarily in plants, while some larger terpenes, such as
C. Patchouli alcohol (C15H26O)
lanesterol (a C-30 precursor to steroid hormones) and B- D. Stearol (C1gH3gO)
carotene (a C-40 source of vitamin A), are found in plants and
animals. Terpenes can be modified into other compounds,
known as terpenoids.
Studies in biogenesis show that the large terpenes are 69. What irregularity in a sample of a sesquiterpene (1-<
synthesized starting from isopentenyl pyrophosphate. The carbon terpene) would indicate that the compound wa:
pyrophosphate adds across the diene of isoprene to form either synthesized in lab as opposed to extracted from a plant.'
isopentenyl pyrophosphate or dimethylallyl pyrophosphate.
These molecules then add to one another by way of A . The compound is not enantiomerically pure.
nucleophilic substitution reactions, where the pyrophosphate B. The compound is not a racemic mixture.
acts as a leaving group. Geranyl pyrophosphate (a name C. The compound had impurities with 16 carbons.
given to C-10 terpenes) is the first monoterpene in many D . The compound had impurities with 20 carbons.
natural synthetic pathways. More isoprene units are added to
the monoterpene to forfir other terpenes and terpenoids.
Figure 2 shows a basic schematic for the biosynthesis of
cholesterol starting from isopentenyl pyrophosphate.

Copyright @ by The Berkeley Review@ 304 GO ON TO THE NEXT PAGE


70. Which carbon is most susceptible to nucleophilic attack Passage Xll (Questions 74 - B0)
in isopentyl pyrophosphate?
Terpenes are natural organic molecules found in plants
and animals. They are formed from the basic subunit of
o o isoprene, a five-carbon conjugated diene. Terpenes and
il il terpenoids, biological molecules derived from terpenes, have a
o- P- o- P-
P oH total carbon count that is divisible by five. Terpenes are
I I classified according to the number of carbon atoms they
o- ()
contain. Monoterpenes have ten carbon atom, sesquiterpenes
A. Carbon 1
have filteen carbon atoms, diterpenes have twenty carbon
atoms, sesterterpene has twenty-five carbons and so on.
B. Carbon 2
C. Carbon 3
Because terpenes are natural products, they are common
in many household items, such as flavoring agents in various
D. Carbon 4
{bods and the active molecule in many drugs. Much of
organic chemistry research involves the development of in
uilro synthesis of terpenes and terpenoids. Some naturally
7 1. Isoprene units are believed to be formed from three
occurring monoterpenes are shown in Figure 1.
acetyl coenzyrne A molecules. What is a likely side
product from the reaction?
o
il
I

HrC---.\ - SCoA
Acetyl Coenzyme A
Limin
A. Carbon dioxide gas
Carvone
B. Ethanol
C. Acetic acid
D. Isopropanol

1'' The biosynthesis of which of the following molecules


likely involved a Diels Alder reaction with isoprene
units?
Myrcene
A"Camphor
A. Caryophyllene Figure 1. Four Common Monoterpenes
B. Citronellol
C. a-Pinene Some terpenes contain oxygen, which is added in a way
D. Vitamin A1 that does not alter the carbon skeleton. Carvone is evident by
a carbonyl absorption at 1750 cm-1 in the IR spectrum and a
strong UV (e > 10,000) absorption at L.'nu* = 242 nm.
73. Which of the labeled bonds in y-terpinene was formed
in the biological synthesis fiom isoprene units? 7 4. When limin is converted into carvone, what type of
reaction has to transpire?
Bond d
A. Oxidation of carbon
CH: B. Reduction ofcarbon
C. Hydrolysis of a n-bond
D. Nucleophilic substitution

A. Bonda 7 5. Which compound in Figure 1 is LEAST likely to


B. Bond b r.rndergo ozonolysis when treated with 03?
C. Bondc A. Camphor
D. Bondd B. Carvone
C. I-imin
D. Myrcene

Copyright @ by The Berkeley Review 305 GO ON TO THE NEXT PAGE


76. How many singlets does camphor show in its proron Passage Xlll (Questions B1- BG)
NMR spectrum?
A. Two
Many processed fbod products often contain partially
hydrogenated vegetable oil as one oftheir ingredients. Partial
B. Three
hydrogenation serves to reduce some of the n-bonds found in
C. Six natural oils. Naturally occurring fatty acids, such as
D. Nine vegetable oil, often have long carbon chains. They can be
hydrogenated to convert the alkyl chain, which may contain
multiple double bonds, into to an aliphatic R group.
Hydrogenation raises the compound's rnelting point, and
ofien converts a naturally occurring liquid into a solid. The
77 . If myrcene reacts with another isoprene unit, what kind fatty acids can be found as either the carboxylic acid or as pan
of terpene is fbrmed? of a fatty acid triglyceride. Figure I shows the enzymaticalll'
A. Diterpene controlled conversion of a fatty acid triglyceride into glycerol
B. Monoterpene and three fatty acids.

C. Sesquiterpene o o
D. Sesterterpene
-oA*, soAn,
o o
Li'ases
- oAo * HoAnu
[I
R2 + H2o
78. Camphor is likely to show which of rhe following o
physical and chemical proper ties'/
L
II.
High water solubility
A boiling point above 298K
-oA*.
1Figure Enzymatic Hydrolysis of a Triglyceride
HoAn,
m. No specific rotation of plane polarized light
The fatty acid is isoiated when a tatty acid triglyceride is
A. I only hydrolyzed. Three carboxylic acids are formed from the fatty
B. II only acid triglyceride. The R in Figure 1, represents any alkyl
C. I and II only group. In naturally occurring fatty acids, the R has an odd
D. I and III only number of carbons. Including the carbon of the carboxylic
acid lunctional group, naturally occurring fatty acids have an
even number of carbons. This is attributed to the fact thar
fatty acid biosynthesis occurs two carbons at a time, via
acetyi coenzyme A. Natural fats can be distinguished fiom
79. Does limin display a stlong (log e > 4) UV absorption? synthetic lats by their carbon chain length. Table 1 lists
some common fatty acids that are naturally found in animals:
A. Yes, because the n-bonds are spaced fal apart.
B. Yes, because of the six-membered ring, Acid Formula n
C. No, because there is no carbonyl group. Arachidic CH3(CH2)1sCO2H 0
D. No, because the n-bonds are not conjugated.
Arachidonic CH g (C Hz)+( CH =C HC H2) +(fH) zCO zH 4

Behenic CH3(CH2)2sCO2H 0

Lauric CH:(CHz)roCO2H 0

8 0. Which compound is the direct product of a Diels Alder Lignocaric CH3(CH2)22CO2H 0


condensation of two isoprene units? Linoleic CH j (CH 2 )a(C H=CHC Hz) z(CH) oCO zH 2
A. Carnphor
Linolenic CH3CH2(CH=CHCHz): (CH2)6CO2H 3
B. Carvone
Myristic CHj(CH2)12CO2H 0
C " Limin
D. Myrcene C)leic CH 3 (C H2 )7 CH =C H ( CHz )7 CO 2H I

Palmitic CH3(CH2)1aCO2H 0

Palmitoleic CH 3 (C H 2 ).5 CH =C H (CHz )1 CO 2H I

Stearic CH3(CH2)16CO2H 0

Table 1 Common Fatty Acids

Copyright O by The Berkeley Review@ GO ON TO THE NEXT PAGE


Vegetable oils generally have more unsaturation than 8 3. Addition of D2 with Pd catalysr reduces n-bonds by
animal fats. For instance, corn oil is 63Vo linoleic acid and adding deuterium to each n-bond carbon. Treatment of
26Vo oleic acid, with the rest being made of other saturated oleic acid with D2 and palladium yields a compound
fatty acids. Safflower o1I is'75Vc linoleic acid, l4To oleic with how many chiral centers?
acid, and 4Vo linolenic acid with the rest being made of other
A,7nro
saturated fatty acids. As the amount of unsaturation
increases, the melting point of the fatty acid decreases, B. One
assuming that the number of carbons remains constant. For C. Two
this reason, many animal fats are solids while many D. Four
vegetable oils are liquids at room temperature. Fatty acids
can play one of three roles in biological systems. They are
found as the building blocks of cell walls as phospholipids
and glycolipids. Fatty acids form derivatives that serve as
hormones (intercellular messengers). Fatty acids are also 8 4. Treatment of an alkene with potassium permanganate
used for fuel through fany acid metabolism. yields a vicinal diol at the alkene carbons. Where do the
hydroxyl groups add when the most unsaturated fatty
81. What is the structure for the MOST abundant fatty acid acid in safflower oil is treated with KMnO4 under basic
found in corn oil? conditions?
A. Carbons 8, 9, 11, 12,14, and 15
B. Carbons 1,9, ll, 13, 15, and 17
C. Carbons 9,10,12, and 13
D. Carbons 9,10, 12,13, 15, and 16
B.o
HO 85 . Bromine liquid is used as a quantitative test reagent to
determine the amount of n-bonds per molecule of a
compound. Which of the following acids consumes the
MOST Br2 per molecule?
A. Arachidic
HO B. Arachidonic
C. Linoleic
D. D. Linolenic

HO

86. Complete hydrogenation of palmitoleic acid yields


which of the following acids?
A. Myristic
82. The n-bond of a fatty acid can be reduced via
hydrogenation when treated with hydrogen gas and a
B. Palmitic
catalytic metal or by FADH2. Treatment of linoleic C. Stearic
acid with FADH2 yields a product: D. Arachidic
A. with lower molecular mass and a lower melting
point than the reactant.
B. with higher molecular mass but a lower melting
point than the reactant.
C. with a lower molecular mass but a higher melting
point than the reactant.
D. with a higher molecular mass and a higher melting
point than the reactant.

Copyright @ by The Berkeley Review@ GO ON TO THE NEXT PAGE


Passage XIV (Questions 87 - 92) 87 . How many chiral canters are present in occidentalol?

Terpenes and terpenoids are natural compounds found in


A. I
plants and animals that are built fiom 5-carbon reactants. B. 2
Figure I shows the sesquitetpenoid (+)-occidentalol. c. 3
D.4

'rr4< 88. Terpenes containing flfteen carbons are best described


AS:
loH A. monoterpenes.
Figure 1 (t)-Occidentalol
B. diterpenes.

A multistep synthesis leading to occidentalol begins C. triterpenes.


with the conversion of Compound 1 into Compound 6, D. sesquiterpenes.
which can further react to form occidentalol. Fisure 2 shows
the synthesis of Compound 6.
89. Which compound has the longest maximum wavelength

la" of absorbance, Imax, in ultraviolet spectroscopy?


A.
B.
Compound I
Compound 2
150'C,-Cq
---+ C. Compound 3
CO2Me Step I CO2Me
Compound 1 Compound 2
D. Compound 4

I l' HoaoH
Step 2 90. What intermediate compound forms in Step I before
| rrou. phH. A decarboxyl ation takes place'?
the
Y z. LeH, eroEt
A. A cyclohexadiene
SOj.Py, THF,0"C; B. A cyclohexene
<-
C. An cr,B-unsaturated ketone
D. Alactam

Compound 4
91. To synthesize occidentalol, Compound 6 is first
converted into a methyl ketone, which is then treated
with MeLi in Et2O at *70'C. What is the role of
I

Step 4ll NHCI. HOAc


I MeLi?
V
A. To act as an electrophile and accept electron density
EtO\ -j- form the alpha carbon.

_oEto- fr
SMe B. To act as a nucleophile and donate electron density
to the carbonyl carbon.
o C. To act as a base and deprotonate the alpha proton.
DME,62'C
---_->
HMPA,
D. To act as an acid and protonate the carbonyl
Compound 5 Step 5 Compound 6 oxygen.
Figure 2 Synthesis of Compound 6

Step 1 involves a Diels-Alder cycloaddition followed by 92. If Compound 4 were treated with strong acid, at which
decarboxylation of the polycyclic system. Step 2 involves carbon in the n-network is it most likely to gain H+?
the conversion of the ketone group of Compound 2 into a
A. Carbon a
ketal followed by the reduction of the ester into a primary
alcohol" Further reduction of the vinylic alcohol group in
B. Carbon b

Compound 3 forms a methyl group in Compound 4. The C " Carbon c


protecting group is removed in Step 4 to relorm the ketone. D. Carbon d
Compound 5 undergoes a variation on the Wittig reaction to
form an alkene in Compound 6.

Copyright O by The Berkeley Review@ GO ON TO THE NEXT PAGE


Questions 93 through 100 are NOT based on a
97, A conjugated diene is necessary in which of the
descriptive passage. following reactions?
A . Claisen rearrangement
93. Leukotriene A4, LTAa, is derived from arachadonic B. Clemmensenreduction
acid. Its structure is shown below: C . Cope rearangement
D . Diels-Alder cycloaddition

k
cora
9 8. All of the following observations are associated with an
E2 reaction EXCEPT:

What is NOT true of LTA4?


A. the base must be bulky and strong.
B. whether the major product has cis or trans geometry
A. LTAa has six units of unsaturation. depends on the stereochemistry of the reactant.
B. LTAa has six n-electrons in a conjugated system. C. that rearrangement is observed.
C. LTAa is capable of undergoin C 1,2-, 1,4-, 1,6-, or D. heat is required to drive the reaction.
1,8-addition when treated with an electrophile and
nucleophile.
D. LTAa has more sp2-hybridized carbons than it has 9 9. All of the following are physical properties of a terpene
spj-hyb.idir"d carbons. EXCEPT:

94. What is the major organic product of the reaction A. High lipid solubiliry
below?
B. High boiling point
C. Low volatility
n't\.0$1, D. High specific rotation
Br.
__=+
hv
t1cbHcH., I 0 0. Which of the following molecules have dipoles NOT
o."*,.:,'rlT, B.
H:C
equal to zero?
L E-butene
II. CzHq
I{TCCHCH3 HTCCHCH3 il. Z-butene
D' nrc A. I only
"'"r" ..,noTr B. II only
I C . iII only
FIlCCHCH3 I!CCBTCHT D. I and III only
95. Which of the following reactions is a propagation
reaction?
A. H3C. + H3CCH2CH3 -+ 2 H3C. + H3CCH2.
B. H3C. + H3CCH2. -+ H3CCH2CH3
1.B 2.D 3.C 4.8 5.8 6.B
1.8 8.C 9.D l0.B ll.B 12.A
C. H3CH2C. + H3CCH3 -+ 2 H3CCH 2, + | t2H2 13. c 14. A 15. D 16. C 1'.7. C 18. D
D. H3C' + H3CCH2CH3 -+ CH4 + (H3C)2CH. 19. c 20. A 21. c 22. D 23. B 24. C
2s. D 26. C 21. C 28. B 29. A 30. D
31. A 32. D 33. A 34. D 35. D 36. A
9 6. The terpene (+)-B-trans-Bergamontene shown below:
37. A 38. A 39. B 40. D 41. C 42. D
43. A 44. D 45. A 46. C 47. B 48. A
49. D 50. A 51. A 52. D 53. A 54. D
55. D 56. A 51. A 58. A 59. D 60. C
61. D 62. D 63. A 64. B 65. B 66. A
61. C 68. D 69. C 70. D ',71. A 12. C
13. B 14. A "/5. A 16. B 11. C 78. B
A. is generated from three isoprene units. 19. D 80. C 81. B 82. D 83. C 84. D
B. is a terpenoid and not a terpene. 8s. B 86. B 87. C 88. D 89. A 90. B
91. B 92. A 93. D 94. D 95. D 96. A
C. has three units of unsaturation.
91. D 98. C 99. D 100. C
D. has sixteen possible stereoisomers.

Copyright @ by The Berkeley Review@ ALL DONE, NO MORE!


Alkanes and Hydrocarbon Reactions passage Answers

Choice B is correct. Hahdes are like hydrogens when considering degrees of


unsaturation, because they, like
hydrogen, make only one bond with carbon. Plugging into the ronJ*itl[ formula
for units of unsaturation yields
an answer of 1. Pick B for best results.

Units of Unsturation = 2(#C) + 2- (#H) - (#cl) _ 2(s)+2-(e)-(1) _ 70 +2-9 -7 _12-10 _2_,


___ r
2 2 22
) Choice D is correct. According to the molecular formula, there are no units of unsaturation
within the molecule,
so there are three linear carbon backbones to be considered. No other carbon backbones
are possible, because
they would be either on9_of the following structures, but drawn differently, violaie the
units of unsaturation, or
not be structurally possible.

(-C-CC-C I

C-C
I

I I
C C
on each of the carbon backbones, the chlorine must be systematically placed to deduce the total number
of
structural isomers that are possible (stereoisomers do not count in this question).
There are three non-equivalent
carbons in the five carbon siraight chain, therefore there are three monochloro
isomers of pentane. There are
four non-equivalent carbons in the four carbon chain, therefore there are four monochloro
isomers of 2-
methylbutane. There are two non-equivalent carbons in the three carbon chain, but one
of the carbons (the
central carbon) already has for-rr bonds, therefore there is only one monochloro isomer
of 2,2-dimethylpropane.
This means that there are eight structural isomers total, so pict< o for greatest success.
C_C-C_C-C C_C-C-C-C C_C-C_C-C
tti
C1 C1
1-chloropentane 2-chloropentane 3-chloropentane

C-C-C-C C-C-C_C
rl /\
C-C-C_C C-C-C_C
lr
clc CCI CCi CCI
1 -chloro-2-methylbutane 2-chloro-2-methylbutane 2-chloro-3-methylbutane 1-chloro-3-methylbutane

C_ CI
I

C- C- C 7-chloro-2,2-dimethylpropane 8 total
I

J. Choice C is correct. By symmetly, there are four unique carbons on methylcyclopentane, therefore
chlorination
can occur at a total of four different sites (four different carbons). This yields u tot^1
of four structural isomers.
The correct answer is choice C.

Copyright @ by The Berkeley Review@


&.{ C1

HYDROCARI]ONS EXPLANATIONS
4. Choice B is correct. Bromination is highly selective for tertiary carbons over secondary and primary carbons, so
the exact quantity of each type of hydrogen need not be accounted for. The most abundant product from
bromination results from the bromination of a tertiary carbon. Carbon number two is tertiary, so the most
abundant product is 2-bromo-2-methylbutane. Pick B, and be a chemistry master.

5. Choice B is correct. 1-chloropentane and 2-chloropentane are formed from chlorination of a primary and
secondary carbon, respectively. There is a difference in reactivity between primary and secondary carbons in
free radical chlorination reactions. Secondary carbons are 2.5 times as reactive as primary carbons. This means
that the ratio of hydrogens (abundance) and the relative reactivity must both be accounted for when
estimating the final ratio. Drawn below is the application of quantity and reactivity. Hydrogens symbolized
by Hu lead to 1-chloropentane and hydrogens symbolized by HU lead to 2-chloropentane. They are multiplied
by their respective reactivity factor.

Hu-l
"\
L
/to
4-.^-c-.^-C\^,,
"\ l'
\-
H,
Ha
@'ffi
n*'"irr{--
Ha
6Hux7=6:4H6x2.5=10
H, i----
2-chloropentane

The ratio is 10: 6, which reduces to 5 : 3. This makes choice B the best answer.

6. Choice B is correct. Four secondary hydrogens lead to 2-chlorohexane and four secondary hydrogens lead to 3-
chlorohexane, sotheproductratio shouldbe 1:1. Becausethe ratio is7.07: l andnot 1: l,choiceAis
eliminated. There is no resonance in the molecules (because there are no lone pairs and no n-bonds), so choice C
is eliminated. The inductive effect wouid favor the formation of 3-chloropentane, so choice D is eliminated.
Steric hindrance is the best explanation why the carbon-2 is selected over carbon-3, so choice B is best.

7. Choice B is correct. The question addresses ring strain destabilizing not only the alkane, but the free radical
intermediate. Based on that, choices A and C are eliminated, because they are not ring structures. A three-
membered ring has more ring strain than a four membered ring, so the best answer is choice B.

8. Choice C is correct. The stability of the carbon free radical is attributed to the donation of electron density
from the alkyl substituents through hyperconjugation. Because hydrogens cannot donate electron density
through hyperconjugation, the stability of a free radical depends on the number of alkyl substituents attached.
As a result, the stability of free radicals is tertiary > secondary > primary (3' > 2" > 1"). Quaternary free
radicals cannot exist, because the presence of four bonds and a free electron on carbon would exceed the octet rule
for carbon. The best answer is therefore choice C.

9. Choice D is correct. According to Table 1, the enthalpy change is positive for the reaction of iodine with an
alkene, so it is an endothermic reaction. This means that the products are in a higher energy state than the
reactants, which eliminates choices A and B. In the next to the last sentence of the first paragraph, it is stated
that the second transition state is of higher energy than the first transition state, so the best answer is choice
D. If you find yourself asking "do they really expect me to know this?", it's probably in the passage... find it!
10. Choice B is correct. Initiation forms the halogen free radical (X.), so the first propagation step involves the
halogen free radical as a reactant. This eliminates choices A and C. In propagation step 1, the free radical
abstracts a hydrogen from an alkane, which is choice B. Eliminate choice D, because it is a termination step.

1.L" Choice B is correct. According to the data in Table 1, the strongest halogen-halogen bond is formed between two
chlorine atoms. Chlorine is the second halogen from the top in the column VII of the periodic table. Atomic
radius increases as you descend a column of the periodic table, so chlorine is the second smallest halogen.
Therefore, the strongest halogen-halogen bond is formed between the second smallest halogen. The fluorine-
fluorine bond is an exception to the shorter bond equals stronger bond rule, because of inter-nuclear repulsion and
the odd fact that the single bond is actually a pi-bond, rather than a sigma-bond. The best answer is choice B.

Copyright O by The Berkeley Review@ HYDROCARBONS EXPLANATIONS


12. Choice A is correct. Hydrogen free radical is unstable, and thus it cannot be formed
in the reaction mechanism.
Choice A produces a hydrogen free radical, therefore the best answer is choice
A.
13. Cl'roice C is correct. If the halogen were to stabilize the free radical when attached,
transition state and make the reaction pathway for adding a second halogen more favorable.
it would lower the
The halogen will
not stabilize the free radical, because halogens are election withdrawiig by the inductive
effect. Hilogens
therefore do not heip stabilize the free radical intermediate so statement I is true. Because
a second halide
does not add to an alkyl halide, but a halide does add to an alkane, we assume
that the alkane is more
reactive' The difference is either in the bond broken or the bond formed. It is safe to assume
that the
wili choose a pathway of lowest energy so by breaking the C-H bond of the alkane preferentially over reaction
the C-H
of the alkyl halide, it can be concluded that the bond broken is of lower energy. rhi, makes
statement II true.
If statement II is true, then statement III cannot be true. The best answer is therefore choice C. As
a note, an R-
H bond is stronger than an R-X bond, so if a halogen free radical (X.) were to react with an alkyl haiide,
it
would abstract the halide, not a hydrogen. This is the more logical explanation for the absence of multiple
substitution products with free radical halogenation.

14. Choice A is correct. The enthalpy of reaction for a reaction between an alkane
and fluorine is listed in Table 1.
By simply reading the chart, it can be seen that best answer is choice A. Sometimes
answers are this easy to
get, so don't be fooied into thinking every question on the MCAT will be difficult.

15 Choice D is correct'-. Wl"l comparing Trial I with Trial II, both the electrophile
concentration and reaction
rate have doubled. This behavior is expected no matter what mechanism
is employed. When comparing Trial
I with Trial III, the base concentration has doubled, the electrophiie concent.ation has increasea (bi7 sixty
percent), and the rate has more than doubled. If the reaction depended just on
the electrophile, the raie"would
be less thang'2 x 10-3 M/s. Because the rate in Trial III is morl than tire rate in Trial
I, ihe rate-determining
step depends on both reagents, making it bimolecular. E2-reactions are bimolecular,
so choice D is correct.

t6. Choice C is correct. The most stable conformation has the largest substituents
with anti orientation to one
another' The bulkiest group on carbon two is the methyl group comprised of carbon
1 and the bulkiest group on
carbon three is the ethyl group comprised of carbons + anJ s, so choiie C is
correct. The Newman projections for
the three staggered conformations are drawn below:
Br. .CH" Br. .*CH"CH3
\->cir.cH"
H\Y
"-;,-4i'"'
HsC cH2cH3 HsC
\ H "t,><
HsC CHs
Conformer from Conformer from which Conformer with
the example elimination takes place C1 and Cn anti

CHs HsC-r cH2cH3 H3CH2C H

cHs CHs CH:


CH2CH3 H CHs
'l'7' Choice C is correct" Carbons in the reactanthave sp3-hybridization while two carbons
i1 the alkene product
have sp2-hybridization, and thus those carbor-r, .ur-rr-roi have chirality. There are two
stereocenters in the
reactants (on carbon two and carbon three), and both stereocenters are lost in
the elimination reaction. Because
the alkene product has no chirai centers, it can have no optical activity. The best answer
for the question is
choice C.

18' Choice D is correct. From the perspective of the eye as shown, the bromine atom is bonded
to the rear (eclipsed)
carbon, therefore choices A and C are eliminated. From the perspective of the eye, the methyl
would stick out
to the left and the hydrogen would stick out to the right. The-besi choice is thus answer choice
D.
Copyright @ by The Berkeley Review@ 312 HYDROCARBONS EXPLANATIONS
't9. Choice C is correct. The enantiomer of the reactant is its mirror image, therefore both chiral
centers must be
opposite that of the reactant's chiral centers. The determination oiboth of the reactant's
chiral centers is
shown below. The chirai centers of the enantiomer are 23 and 3R. The best answer is choice
C.
1

Br
cHg
Compound is (2R,3S), so the H$)
cH2cH3 enantiomer must be (2S,3R) HsC
Carbon 2 = R Carbon 3 = S

20. Choice A is correct. The role of a strong acid in an elimination reaction is to protonate the leaving group and
increase its tendency to leave. Protonation occurs in the E1 reaction rather thin the E2 reaction.
T;hI acii will
not dehydrate the solvent, so choice B is eliminated. The sulfate ion is not a strong enough base to remove a
proton, so choice C is both wrong and impossible. Choice D should be eliminatia i*J"aiutely,
because a
carbocation cannot be protonated. The best answer is choice A.

21.. Choice C is correct. Chlorocyclohexane when treated with a strong base und.ergoes an elimination
reaction by
way of an E2 mechani-sm to yield cyclohexene. There are two units of unsaturatiion in cyclohexene,
one unit of
unsaturation for the n-bond and one unit of nnsaturation for the ring. The correct answer is
choice C.

)) Choice D is correct. This question is another way of asking, "Which reaction proceeds via a carbocation
intermediate in its mechanism?" The first two reactions proceed by an E2-mechanism, which is a one-step
process having no intermediate. The E2-mechanism is predictable, becaur" of th" presence of
the strong, bulky
base. The base must be strong enough to remove a proton from carbon and it must te bulky enough
to #inimize
competition with an Sp2-reaction. Reaction III proceeds by an E1-mechanism, because of the preJence of
strong
acid, which protonates the hydroxyl group. This makes a better leaving group, and ultimaiely
facilitates the
formation of a carbocation. In fact, the location of the doubie bond in theiiial product can only be explained
by
-
a hydride shift. The hydride shift results in the conversion from a ,".orliury carbocation into a tertiary
carbocation. Only Reaction iii exhibits rearrangement, so the best answer is choice D.

23. Choice B is correct. This question involves counting the chiral centers in each product in Figure 1
to see which
molecules are chiral, and then analyzing to see if chirality is involved in the reaction. The m-ajor
products from
Reaction I and Reaction II both have chiral centers present. In Reaction I, two of the original three
chiral
centers are lost , but that still leaves one remaining chiral center, so the mixture is optically uJtirr".
In Reaction
II, one of the original three chiral centers is lost , which leaves two remaining chiral centers, so the mixture is
optically active' In Reaction III, all three chiral centers are lost, so the fiial product mixture exhibits no
optical activity. The best answer is choice B.

,t Choice C is correct. The reaction involves the use of concentrated strong acid at high temperature, so the
reaction is apt to proceed via an E1-mechanism, An E1-mechanism entails the hydroxyl
[roup being protonated
and then leaving, producing a secondary carbocation. A hydride shift resuits in ihe conversion fropf secondary
carbocation into a tertiary carbocation, which is the intermediate from which deprotonation to form the
alkene product occurs. A11 three chiral centers are lost, which is also seen in Reaction III. This is best
explained in choice C. On the MCAT, test-takers are expected to be able to see the analogy between the
reaction in a question and a reaction in the passage.

25. Choice D is correct. Reaction I is an E2-elimination reaction. One of the characteristics of an E2-reaction is
that the reaction is concerted, so the rate of the reaction depends on both reactants. If you increase the
concentration of either reactant, in statement I the base, the reaction rate increases. This makes Statement I a
valid statement. Elimination reactions require heat, so a decrease in temperature decreases the amount of
elimination product that forms. You may recall that to maximize the subsiitution product and minimize the
competing elimination product, temperatures are reduced. This makes Statement Il invalid, which eliminates
choices B and C. Reaction II is an E2-reaction, so it has a competing Sp2-reaction occurring in the same flask.
This makes Statement IiI valid, and makes choice D the best u.,r*"r.

Copyright @ by The Berkeley Review@ JIJ HYDROCARBONS EXPLANATIONS


26. Choice C is correct' First and foremost, with,a strong, bulky base, the reaction
proceeds by an E2-mechanism.
This eliminates choices A and B' The E2-mechanism-requires that the hydrogeir
being lost be oriented anti to
the- leavrng group, in this case chlorine. In Reaction I, there
are two anti irydrigens, and the one that is chosen,
is the one that leads to the more substituted alkene product. In Reaction
Ii, the"re is only one anti hydrogen, so
only one-product may be. formed. The hydrog"n o., ii-," more substituted alpha
carbon has gauche orientation, so
it is not deprotonated. The best answer is choice C.

27 ' Choice C is correct. The two carbons common to both rings finish as doubie
bond carbons in Reaction III, so they
both finish with sp2-hybridization. Only choice C finish"es witn spi 1-rrAr1ii")ution,
so it is the best answer.
28. Choice B is correct. Diethyl amine is a weak base, while potassium tert-butoxide,
t-butoK, is a strong bulky
base' For elimination by an E2-mechanism, a strong, bulky base is necessary. This means
that when using
diethyl amine, there can be no elimination reaction, which eliminates choices C and D.
The reaction with
diethyl amine is nucleophilic substitution. The electrophiie is a secondary alkyl chioride,
which could go by
either an 5511 or S52 mechanism. Because the nucleophile is a good ,,rr.l"ophil", the
best choice is aJ sNz
reaction' An 5512 reaction results in the inversion of the reactive iite, which puts tne
amine group behind the
plane of the molecule and makes choice B the best answer.

29. Choice A is correct' The Silkworm Moth sex phermone has two double
bonds, one that is cis substituted and the
other that is trans substituted. This eliminates choice B. All of the carbons
are part of a straight chain with no
branching, so it has two terminal carbons that are primary and all the internal
.uiborr, are secondary. There are
C.is eliminar:d sinsle bond separates the two doubie bonds, so they are in
l"^:"1:::I_:11,,""?,::
fact coni t:tu:
eliminates choice D. Ali91y:ne
of the carbons have at least two identical substituents br they
l1::^t"T1q111,_lllt
nave sp'-hybridization, so there are no stereogenic centers. This makes choice A
the best answer.
30' Choice D is correct. The Oriental Fruit Moth sex phermone differs from the structure
in the question at the
position of the double bond. The structure in the question ha9 cis geometry,
while the phermone in Figure t has
trans geometry. This can be misleadingand tempt you to pick choice g. gut,
the conneitivity is not the same, so
the structures are structural isomers. The n-boncl is between carbons B ancl
9 in the Oriental Fruit Moth sex
phermone, but it is between carbons 9 and 10 in the compound that is shown.
This makes choice D the best
answer' The two structures are not interchangeable by a rotation about a bond, so
they are not conformational
isomers' This eliminates choice A. There is no itereogenic carbon, so the two structures
cannot be optical isomers.
This eliminates choice C. The two structures are structural isomers, making choice
D the best answer.
31' Choice A is correct. UV-visible spectroscopy is used to detect n-bonds. Bombykol
has conjugated n-bonds, while
muscalure just one.n-bond. Conjugation tednces the transition energy, so the
#avelength oi ria*i^.rm absorbance
increases with conjugation. This means, that bombykol has a greatJi l,*u*
than making Statement I a
valid statement. Choice B is eliminated. The Tiger Moth r"* ho.-on"1s an aliphatic
^1rr."ulrrr",
alkane, roit hu, no units
of unsaturation. Undecane is an 11-carbon aliphalic hydrocarbon, so it too has no units of
unsaturation. The two
compounds have the same units of unsaturation, zero, so Statement II is invalid. This eliminates
choice C.
Terpenes and terpenoids contain a number of carbons that is divisible by five and a
predictable connectivity that
can be partitioned into isoprene subunits. The oriental Fruit Moth sex
ih"r-o1" has twelve carbons in its chain
and two more for the acetate group. Fourteen is not divisible by five nor is the structure
one that can be broken
into isoprene subunits. This makes statement III invalid and makes the choice A the best
answer.
32' Choice D is correct. Muscalure is a cis alkene made of 23 carbons and 46 hydrogens. It is a long chain
hydrocarbon, so it has low water miscibitity. Choice A is a valid statement, and ttereby
eliminated. It is
excreted, so it must be a liquid under ambient conditions. This makes choice
B a valid statement, which
eliminates it. Because it is a long chain-hydrocarbon, lipids can dissolve into it.
It has high lipid solubility,
making choice C a valid statement, thereby eliminating it. There are no stereogenic
centers on muscalure, so it
will not rotate plane-poiarized light. This makes choiJe D an ir-rvalid statement] so choice D is the best answer.

33' Choice A is correct. All of the compounds in Figure t have straight chains (no rings),
so each has at least two
primary carbons' only 2-methylheptadecane his branching, ro lt hu, an extra primary carbon.
No other
str.ucture in Figure t has any branching, so 2-methylheptadeiane, the Tiger Moth
sex phermone, has the rnost
primary carbons. Choice A is the best answer.
Copyright O by The Berkeley Review@ 314 HYDROCARBONS EXPLANATIONS
34. Choice D is correct. The Green Peach Aphid defense phermone has 15 carbons connected in such a
way that it can
be partitioned into three isoprene subunits. Choice A is a valid statement, so it is eliminated.
The Green peach
Aphid defense phermone has four double boncls, so it has eight sp2-hybridized carbons. With fifteen
carbons
total, more than half of the carbons are sp2-hybridized. Choiie B is a valid statement, so it is eliminated.
The
Green Peach Aphid defense phermone has a conjugated diene, so it can undergo 1,4-addition
of an electrophile.
Choice C is a valid statement, which eliminates it. Because of all of the n-donds in the Green peach
Aphid
defense phermone, it is not very flexible. Choice D is an invalid statement, which makes it the best
answer.
35. Choice D is correct. The Oriental Fruit Moth sex phermone has an ester group. A carbonyl has an IR absorbance
above 1700 crn-l , so an ester accounts for the IR absorbance at1741 .i',-r.'Ch"i;; A is a valid correlation of
structure to spectroscopic observation, so it is eliminated. Bombykol has two n-bonds in a conjugated network.
The presence of n-bonds in a structure results in an absorbance in the ultraviolet-visible tuig" of the EM
spectrum, so an absorbance of 227 nm seems viable for a conjugated diene. Choice B is a valid iorrelation of
structure to spectroscopic observation, so it is eliminated. Muscalure has two carbons involved in double bonds,
each of which has a hydrogen attached. This means that the hydrogens on those carbons will be found
downfield, resuiting in two signals with values around 5.00 ppm in-lUx"traR. Choice C is a valid correlation of
structure to spectroscopic observation, so it is eliminated. The Tiger Moth sex phermone has eighteen carbons
oj.srynmetry. There are two equivalent methyl gtorrp!, so there arJseventeen uniqui carbons. The
?*.i:-{""e
TTCNMR would show 17 signals
if it were of high enough iesolution. The reality is that ,.uny of the signals
would overlap, so it would likely show less. It will not show fourteen signals in its 13CNMR spectrum,
so choice
D is an invalid correlation of structure to spectroscopic observation, makLg it the best answer.

36. Choice A is correct. Reaction I is a Diels-Alder reaction. Diels-Alder reactions involve the reaction of a
conjugated diene and a dienophile. Normally we think of the dienophile as an alkene, but the only
requirement is that it has a r-bond. The alkyne meets this requirement, so it is a dienophile. The best answer
is choice A. There is no nucleophilic substitution gorng on, so choices B and C are eliminated. There is
no change
in oxidation state, so the compound is not an oxidant or reductant, eliminating choice D.

J/. Choice A is correct. Each of the answer choices is lacking two hydrogens from the formula of the final product,
CaHgO. This means that hydrogenation converts the intermediate to tne final product. The final product is an
aldehyde and hydrogenation cannot convert a cyclic ether into a carbonyl, so choices C and D can be eliminated.
Hydrogenation adds two hydrogen atoms to neighboring carbons. Because an aldehyde is generated, we know
that one of the hydrogen atoms is added to the carbonyl carbon. The ring is cleaved, so iydrogenation must
break the strained ring by adding a hydrogen to the carbonyl carbon and a h"ydrogen to the neighbJring atom.
In
choice A, the hydrogen atoms are correctly displaced on the intermediate-(CH2-CH-CH3) to lorm both
aidehyde products (CH2-CH2-CH3 and CH3-CU-CH3) when a single hydrogen is added to tne akyl chain.
This is not the case in choice B (CH2-CHZ-CHZ), so it is eliminated.

38. Choice A is correct. On the product of Reaction I, ail of the functional groups are bonded to sp2-hybridized
carbons, so there are no stereocenters. This eliminates choices B and C. Bised bn the remaini.,g ur,r*", choices,
Reaction III must have formed a new stereocenter. The new stereocenter is located on the rin{ carbon with the
methyl substituent. Hydrogenation can occur from above or below the ring, so the methyl is albove the plane in
fifty percent of the product mixture and below the plane in fifty percent of tnu mixture. Reaction III forms a
racemic mixture. In Reaction II, the products have no chiral centers, so no new stereocenters were formed during
the reaction. This eliminates choice D and makes choice A the best answer.

39' Choice B is correct. The basic tenet of green chemistry is to minimize waste and side products and maximize
atom-economy. Atom-economy aims to get every atom added to the reaction containe. et-rait-rg up in the product.
Using a supercritical fluid as a solvent makes for easy recovery and reuse of the solventl Iieaction III uses
supercritical CO2, so choice A is valid and thus eliminated. Using protecting groups adds extra atoms to the
solution that are not destined to be part of the product, so it violatei ine princfof, of atom-ecor-romy. protecting
groups are difficult to recycle without spending a great deal of solvent, so they do not fit the green chemistr!
philosophy. Choice B is an exception. If catalysts are part of the lab equipment, such as cataljrtic beads, then
they are easily recovered and reused. This makes choice C in philosopnicat agreement with ti-re principles of
green chemistry. This eliminates choice C. It is stated in the passage that direcl oxidation using oxygen fits in
the philosophy of green chemistry, so choice D is eliminated. Choose B and be on top of your gui".

Copyright O by The Berkeley Review@ 315 HYDROCARBONS EXPLANATIONS


40. Choice D is correct. Reaction III is described in the passage as hydrogenation, the term applied
to a reaction
that adds hydrogen atoms. This eliminates choice g. rne gain of Uoias to hydrogen, u i"r,
electronegative
atom than carbon, is defined as reduction, so choice A is eliminated. Although it is not specified
in the
passage, when hydrogenating with a metal catalyst, the process adds the hydrJgen atoms in
a syn fashion,
which means that the reaction is stereoselective. This eiiminates choice C.- Both carbons gain a hydrogen
atom, so there is no regioselectivity. This makes choice D the best answer. Choose D for the
safe of correctness.
41.. Choice C is correct. The reaction is a Diels-Alder reaction, similar to Reaction I, except an alkene
is serving as
the dienophile, rather than an alkyne. A Diels-Alder reaction results in the formation of a new six-membered
ring, which is observed in all of the choices. When the reaction involves a conjugated cliene and an alkene,
the
product is a cyclohexene ring, so choices B and D are eliminated. To form choice"B, the dienophile would have
needed to be an alkyne, like Reaction I. The n-bond in the product is located between the two internal
carbons
of the-original conjugated diene, so they should be found otr the left side of the central ring. This makes choice
C the best arswer. Choose C for a brighter smile when scoring iike you did.

_^*
eK-r+ e[("
42. Choice D is correct. Reaction 1 is an elimination reaction by way of an E1 mechanism. In an E1 reaction,
a
carbocation is formed, so rearrangement is possible. Whether lt is observed iepends on the compo,ind, but it is
possible. Statement I is a valid statement. The intermediate is an allylic carbocation, where the cationic
carbon is bonded to one of the carbons in the double bond, not a vinylic carbocation, where the
cation carbon is
one of the carbons in the double bond. Statement II is invalid, which eliminates choices B and
C. The first step
in acid catalyzed reactions is the protonation of some functional group on the reactant. In this particular case,
the hydroxyl group is the most basic site, so it is protonated. Thii generates a good leaving
grorrp, which then
ieaves in the second step. Because the hydroxyl group is protonated to stait the reaction, Statement
III is
vaiid. This makes choice D the best answer.

{). Choice A is correct. Compound III and Compound IV are structural isomers of one another. They
each have a
hydroxyl
.group and an aikene functionality, so infrared spectroscopy yields the same key absorbances. This
makes infrared spectroscopy ineffective at distinguishing the two iilyiic alcoho1s, so choice B is eliminated.
Ultraviolet spectroscopy is great for determining the amount of conjugation in a system. However, both
compounds have the same number of rc-bonds, one, so ultraviolet spectroscopy yields essentially the same
spectrum for both compounds. Choice C is eliminated. Neither structure absorbs light in the visible range,
given that neither structure has extensive conjugation. This can be inferred from the passage when thly
mention that the peak at 779 nm disappears. That peak is associated with Compound I, which tappens to be
one of the two enantiomers represented by Compound III. Choice D is eliminatei, because 1,79 nmis not in the
visible range of the EM spectrum. The best method is 1HNMR, which can distinguish structurai isomers by
their equivalent hydrogens. Choice A is the best answer.

44. Choice D is correct' The tertiary carbon with the hydroxyl group has four unique substituents, so it is chiral.
This eliminates choice A. There is only one chiral center, so diastereomers are not possible (there must be at
least two chiral centers for diastereomers to be possible). This eliminates choice b. Because the hydroxyl
group can be above the plane or below the plane, there is more than one structure possible for Compound IV.
This eliminates choice B and makes choice D the best answer.

OH
Compound IV
m
Enantiomers
OH

Copyright O by The Berkeley Review@ HYDROCARBONS EXPLANATIONS


45. Choice A is correct. The first paragraph of the passage implies that Compound I is an allylic alcohol. If you
recall your general nomenclature, then you should know that when a functional group, in this case the hydroxyl
group, is on the carbon bonded to the alkene carbon, it is said to be allylic. Choice A fits this description.
Choice D should have been eliminated early, because it does not contains a n-bond. Choice B is eliminated,
because it is a vinylic alcohol (hydroxyl group directly bonded to the alkene carbon). Choice C has the double
bond too far from the alcohol group to be allylic, so it is eliminated as well.

46. Choice C is correct. The option for either 1,2-addition or 1,4-addition occurs when the reactant has conjugated
n-bonds. Choices A and D should be eliminated immediately, because when the two numbers describing the n-
bonds differ by 2, then the n-bonds are conjugated. Cyclopentadiene has only five carbons, so one n-bond must be
between carbons one and two. The second ru-bond must be between carbons three and four, because in a five-carbon
ring, no matter how you place two double bonds, for the ring to not be so strained it can't exist, they must be
conjugated. Only in choice C are the n-bonds not conjugated, so choice C is the best answer.

f'*
Choice A Choice B Choice C Choice D

.A/
47. Choice B is correct. The hydration of Compound II starts with the addition of a proton to the conjugated n-
network. The easiest carbon to protonate, because of steric hindrance and resonance stability, is the secondary,
terminal carbon of the system. This generates the structure in choice C, so choice C is eliminated. That
structure can undergo resonance to generate the structure shown in choice D. This eliminates choice D. If water
were to attack the structure shown in choice C, the structure in choice A, a new intermediate, forms. This
eliminates choice A. By default, the best answer is choice B. Choice B is not possible, because the structure
would have to gain a proton at the more sterically hindered terminal carbon of the n-system.
, .:--\
Ha H,o.
-' \
) lra

eI).\eD
Choice C

48.
m
Choice A is correct. At 35"C, the hydration of Compound II using sulfuric acid and water yields abottt 40"h
secondary alcohol and 60"h tertiary alcohol. It is stated in the passage that "the percentage of the secondary
alcohol formed increases as the temperature of the hydration reaction increases. This is attributed to a shift
from kinetic control to thermodynamic control." This means that at 75'C,it is reasonable to suspect that the
secondary, allylic alcohol is the major product. This eliminates choice C. The product is not a vinylic alcohol,
so choices B and D are eliminated. In choice A, a secondary allylic alcohol is formed, so it is the best answer.
Taking the information in the third paragraph of the passage and Figure 1, and erasing the cyclopentane ring
could aiso solve this question.

49. Choice D is correct. It is easiest to start by evaluating which pair represents enantiomers. Enantiomers, you
recall, are non-superimposable mirror images. In this case, it is easier to compare the chiral centers rather
than reorient the structures to see if they are mirror images. If all of the chiral centers differ, then the two
structures are enantiomers. In choices A and C, only one of the three chiral centers differs between the pair, so
they are diastereomers, not enantiomers. The next factor to consider is the alignment of the carbonyl
substituents. They are cis to begin with (on the alkene reactant), so they should finish cis. In choice B, the two
carbonyl substituents are trans, so choice B is eliminated. In choice D, the groups are cis, so it is the best answer.
Copyright @ by The Berkeley Revierv@ 31'/ HYDROCARBONS EXPLANATIONS
50. Choice A is correct. In Triai 4, with methyi groups in positions A and B of the
conjugated diene, the rate is
roughly 0'005 that of 1,3-butadiene. The reason for thli reduced rate is the steric hindrance
associated with
the methyl qrgup pointing to the middle of the transition state. No matter how
the molecule contorts, one of
the two methyl groups is aiways pointing inwards, where the transition state forms.
If ethyl groups were used
in lieu of methyl groups, then the steric hindrance would be even more substantial and
the reaction rate would
be even slower. Only choice A presents a slower reaction rate, so choice
A is the best answer.
51. Choice A is correct' According to the passage, the addition of a Lewis acid to the
system increases the rate of
the Diels-Alder reaction. Choice A, AlC13, is a Lewis acid, because the aluminum lacks
a complete octet.
Choice B, CCl4, is a common organic solvent where each atom has a satisfied octet. Because
CCI4 acts like a
solvent and not a Lewis acid, choice B is eliminated. Choice C, KH, is a strong base that
reaaity donates
electrons. Because KH acts like a base and not a Lewis acid, choice C is eliminaied. Choice
D, LiAiH4, is a
strong reducing agent where each atom has a satisfied octet. Because LiAIH4 acts like a reducing
agent and not
a Lewis acid, choice D is eiiminated. The best answer is choice A.

52. Choice D is correct. If the reaction proceeds by a nucleophilic mechanism, then we must
determine which
molecule is acting as the nucleophile and which is acting as the electrophile. The
reaction is best when the
dienopirile has an electron-withdrawing substituent, so lei us assume that the diene is
the nucleophile and the
dienophile is the electrophile. Based on this mechanism, the part of the intermediate
that originally came
from the diene should carry a positive charge (because it donated electrons) and the
part of the intermediate
that originally came from the dienophiie should carry a negative charge (because
it accepted electrons). This
is observed in each answer, choice eicept choice A, so choicJ A is eliminatecl.
on the bright side, based on the
answer choices, we know that our assumption about the diene being the nucleophiie
anJdienophiie being the
electrophiie is valid. The drawing belolt' shows the forination o? th" first intermediate after nucleophilic
a ttack.

cHe

The second intermediate drawn matches choice D, so choice D is the best answer.
Choices B and C could have
been elimi'ated by the incorrect location of the positive charge.

53' Choice A is correct. The dienophile is the same in Trial 2 and Trial 4, so the difference in
reactivity must be
attributed to the conjugated diene' This eiiminates choice D. Methyl groups, when bonded
to a structure, do not
act as Lewis acids, because their octets are complete. This eliminates choice C. The
methyl groups are mildiy
electron-donating, not eiectron-withdrawing, to .hoi." B is incorrect. The most significant
iacior in the reaction
rate is that the extra methyl group on the diene in Trial 4 causes steric hindrance in the
transition state. No
matter how the diene contorts, one of the two methyl groLlps interferes with the incoming dienophiie.
The best
answer is choice A.

a ,f. U
r.3

Minimal steric hindlance


__>
&ilf*'--->U;
Signi licant steric hindrance

Choice D is correct. It is stated in the passage that a dienophile is enhanced when it has
an electron-
withdrawing group conjugated to the alkene. Choice A is enhanied by the carbonyl groups conjugated
to the
alkene, so choice A is eliminated. Choice B, albeit an alkyne and not an alkene, has el-ectronlwlthdrawing
ester groups conjugated with the n-bond, so it is enhanced as a dienophile. Choice B is
eliminated. Choice C is
enhanced by the carhonyl groups conjugated to the aikene, so choice C is eliminated.
In choice D, the amine
group is an electron-donating group that lessens the reactivity of the dienophile. Choice D is the best
answer.

Copyright @ by The Berkeley Review@ 3r8 HYDROCARBONS EXPLANATIONS


55. Choice D is correct. If the Y-group is a carbonyl group, then it is the exact same substituent as the other
carbonyl group (on the adjacent carbon), making the compound symmetric and thus indistinguishabie. Both
Product A and Product B are the same compound, if the reactant is symmetric, so choice D is the best answer.

5b. Choice A is correct. In the second paragraph of the passage it is stated that when X is an electron-donating
group/ product A is the major product. Because the OCH3 reactant yields more Product A than the CH3 reactant
in comparable reactions, it can be concluded that an OCH3 group is more electron donating than a CH3 group.
The best answer is choice A. Choices B and D should have been eliminated, because tie major prlJuct is
product A, not product B.

57. Choice A is correct. By analogy, OCH2CH3 (ethoxy) is an electron donating group like OCH3 (methoxy). The
presence of the eiectron-donating group makes Product A the more favorable product. Product A fiom the
generic reaction of the passage is choice A. Be careful not to choose B without paying attention to the location
of the double bond. The double bond in choice B is on the side opposite from where it should be.

58. Choice A is correct. Two five-carbon species are combined, so the final product can have only ten carbons
altogether. Choice D is eliminated for having twelve carbons total. One o] the double bonds is in the wrong
location in both choice B and choice C. The best answer is therefore choice A. The stereochemistry with the
new cyclopentyl ring trans to the bridging carbon is what is referred to as the "endo product." The arrow-
pushing schematic from the reactant to the product is drawn below:

ryO->OrO
..rrrl\\

.,,tttlll

59. Choice D is correct. Structural isomers have different bonds (connectivity of atoms). Product A and Product B in
the sample reaction in Figure 1 are structural isomers. Structural isomers result when both reactants are
asymmetric. The best answer is choice D.

50. Choice C is correct. For the reaction as drawn in the question, with two asymmetric reactants, there are two
possible structural isomers (corresponding to product A and product B in the generic reaction) that can form. In
both structural isomers, there are two new chiral centers formed. For a compound with two chiral centers, there
are four (22) possible stereoisomers, meaning that there are four possible stereoisomers for each structural
isomer. The result is that there are eight possible isomers total, so the best answer is choice C. In reality, not
all eight isomers are observed to any measurable level in a Diels-Alder reaction. The major product results
from the transition state of least steric hindrance. In a typical Diels-Alder reaction suclt as thir, th" major
products are an enantiomeric pair of one of the two possible structural isomers. The less favorable structural
isomer may also be formed, resulting in an enantiomeric pair, but it is generally in much lower concentration
than the more favorable structural isomer.

6't. Choice D is correct. A concerted reaction occurs in one step. Given that a sigmatropic rearrangement involves
just one molecule, if it occurs in just one step, then only one product can be formed. This eliminates choice C,
because there are not multiple products, let alone cross products. The stereochemistry can be lost at centers that
go from sp3-hybridizationl, sp2-hybridization and it can be gained at centers that go from sp2-hybridization
to spr-hybridization. Carbons that do not change hybridization cannot experience a change in stereochemistry.
This means that there is no set rule about the complete retention or the complete inversion of all stereocenters.
This eliminates choices A and B. The only possible answer is the one that supports no cross products being
formed, because the molecule only reacts one way. Choice D is the best answer.

62. Choice D is correct. Step III converts a cyclic ketone into a phenol, so the product has aromaticity that the
reactant does not. The gain of aromaticity drives the reaction, so choices A and C are eliminated. The
conversion from a ketone to phenol shifts the n-bond from the carbonyl to the benzene ring, so it is the result of
tautomerization, not reduction. The best answer is choice D.
Copyright @ by The Berkeley Reviervo 319 HYDROCARBONS EXPLANATIONS
63. Choice A is correct. The role of heat in any pericyclic reaction is to provide energy for the
reactant to realign
its orbitals to achieve the transition state. The best answer is choice A. c-hoice B should have been
eliminated, because exothermic reactions generate heat, so no heat must be added to drive
them. Heat is
released when bonds are formed, sigma or pi, so choices C and D are elimilated.

64. Choice B is correct. The Cope rearrangement involves a 1,S-diene, so there are six carbons within
the molecular
orbital of the transition state. Choice A is elimrnated because it has only four carbons. Choice C is eliminated
because the orbitals show no fi-overiap between adjacent carbons. Choice D is eliminated because
there is no
overlap across the complete cycle. The best overlap is choice B, where the sigma-bond is present o1 the
left and
the terminal orbitals are aligned correctly to form i pi-bond.

65. Choice B is correct. The oxygen is directly bonded to the benzene ring in the reactant, so it is phenylic a'd not
benzylic. This eliminates choices A and C. The oxygen is also bonded to the carbon alpha to ihe aikene. This
makes the carbon allylic, so choice B is the best answer.

66. Choice A is correct. The Claisen rearrangement converts an ether into a carbonyl, so the spectroscopic evidence
must depict either the loss of an ether or gain of a carbonyl group. Aldehyde protons shor,n, a signai
around 9.5
ppm in the 1HNMR, so the formation of in aldehyde wouliin fact correspond with the appearince of a
signal
around 9.5 ppm. Choice A is the best answer. Infrared absorbances uro.r,-rd 1700 cm-l inaicate the presence
of a
carbonyl grouP and broad infrared absorbances around 3400 cm-1 indicate the presence of an alcohol
group. No
hydroxyl group appears in either the reactant or product, so choice B is eliminated. The reaction would
be
supported by the appearance of an absorbance around 1700 cm-1 in infrared spectroscopy, not a disappearance,
so
choice C is eliminated' Signals between 5 and 6 ppm in the 1HNMR co.respond to vinylic hydroge-ns
bonded to
aikene carbons, which are present before and afier the reaction, so choice D is eliminated.

67. Choice C is correct- The first reaction in the synthesis in Figure 2, Step I, invoives the oxygen. The Claisen
rearrangement involves oxygen as the ether is converted into a ketone. This eliminates ihoices B and D.
According to the remaining choices, Step II is a Cope rearrangement. To determine the best answer, we must
decide if the units of unsaturation decrease by one cluring the Claisen rearrangement or r,r.hether they remail
constant at five. In all compounds in Figure 2, there are four n-bonds and one .lig, ,o there are aiways iive
units
of unsaturation. This makes choice C the best answer. You could also conclud"J thut the units of unsaturation
do not change by looking at the Claisen rearrangement in Figure 1, where there are two n-bonds in both the
redctant and product.

68. Choice D is correct. Terpenes are composed of isoprene subunits which are made of five carbons. To be a terpene,
a molecule must have a nurnber of carbons that is divisible byfive. Stearol has eighteen carbons, so it cannot be
a terpene. The correct choice is D.

69. Choice C is correct- If the sesquiterpene were derived from a natural source (such as extraction or distillation
from a plant), then any impurities would be naturally occurring impurities. If there were two enantiomers
present, that would be explained by attack at a planar site from two sides. This can occur in nature although
enzymes strongly favor synthesis of one enantiomer over another. Choices A and B are eliminated, because
chiral impurities can occur in nature. The dead give-away would be an impurity with sixteen carbons. Terpenes
have muitiples of five for their carbon values. Because sixteen carbons is not possible, choice C is the best
choice. A twenty-carbon impurity is a terpene, thurs it is naturally occurring.

v0. Choice D is correct. The carbon that is most susceptible to nucleophilic attack is the carbon with a leaving
group attached" Carbon four, with the pyropl-rosphate leaving group; is the most electrophilic. Alkene carboni
do,act as electrophiles oir occasion, but in this iornpound, carbon four is more electrophilic than an alkene
carbon" The best answer is choice D.

71" Choice A is correct. Combining three acetyl coenzyme A molecules result in six carbons total. Isoprene units
have only five carbons, so one carbon must be in a side product. Carbon dioxide contains only one carbon, so
choice A is the best choice. Ethanol and acetic acid each contain two carbons ancl isopropanol contains three
carbons. Choices B, C, and D are all eliminated.

Copl,right @ by The Berkeley Review@ HYDROCARBONS EXPLANATIONS


nt Choice C is correct. A Diels-Alder reaction forms cyclohexene, so caryophyllene and citronellol cannot have
been formed from a Diels-Alder reaction. This eliminates choices A and B. Both c,-pinene and Vitamin 41
have a cyclohexene moiety, so we must look closer. Diels-Alder reactions involve a di"ne and dienophile, so
we can look at the compounds in a retrosynthetic fashion. In Vitamin ,A.1, the retro Diels-Alder reaction does
not generate terpene fragments, so choice D is eliminated. Choice C is the best answer by default.

,/ J. Choice B is correct. Bond a can be eliminated immediately, because the fragments formed from the break are
three carbons and seven carbons. Bond c can be also eliminated immediately, because the fragments formed from
the break are nine carbons and one carbon. These are not multiples of five, therefore the two fragments cannot
be involved in the synthesis. This eliminates choices A and C. Bond b and Bond d when broken can leave a ten
carbon molecule, so neither can be eliminated. The trouble with bond d is that the fragment to the right of the
break cannot form a 2-methylbutene, because it loses the tertiary carbon. Choice D is eliminated. Isoprene
units must be isopentenyl, not straight chain pentenyl, thus the break is not allowed. The two possible retro
synthesis pathways are shown below, and only Bond b is involved. Choice B is the best answer.

Bond d Bond d
CFI? CIT

Bond b must have been formed None of the labeled bonds were
to connect the isoprene units. formed to connect the isoprene units.

74. Choice A is correct. Carvone differs from limin by a carbonyl group. To go from iimin to carvone, a carbon must
lose two bonds to hydrogen and gain a double bond to oxygen. This is oxid.ation, so choice A is the best answer.

75. Choice A is correct. Ozonolysis is the oxidative cleavage of a double bond between two carbons. The resulting
products are carbonyl compounds that vary from aldehydes to ketones to carboxylic acids, depending on the
work up step. To undergo ozonolysis, the reactant must contain an alkene functional group. All of the compounds
have an alkene functionality except for camphor. This makes choice A, camphor, the best answer.

76. Choice B is correct. Singlets in the proton NMR are caused by unique hydrogen atoms in an environment where
the adjacent atoms have no bonds to hydrogen, and thus there are no neighboring hydrogens with which
coupiing can take place. In camphor, ali of the methyl groups are bonded to quaternary carbons, so they all fit
this description. Because the cyciic structure is incapable of rotation, like an alkene, the two methyl groups
bonded to the bridge carbon are not equivalent, causing them to express different NMR signals. The result is
that each of the methyl groups are represented by a singlet in the proton NMR. All of the remaining hydrogens
on camphor are on carbons adjacent to neighboring carbons with hydrogens, so there are no other singlets than
the ones from the methyl groups. This generates three proton NMR singlets, so the best answer is choice B.

No Hs on neighbor, cannot be
rotated to be equivalentwith No Hs on neighbor, cannot be
other bridge methyl group. rotated to be equivalent with
other bridge methyl group.
(3H singlet)
H

)g-- No Hs on neighbor, isolated


g H3cs methyl grorrpl (3H singlet)

77. Choice C is correct. Myrcene contains ten carbon atoms, so the addition of another isoprene unit would result in a
product with fifteen carbons total. According to the first paragraph of the passage, terpenes having fifteen
carbons are referred to as sesquiterpene, making choice C the best answer.

Copyright @ by The Berkelel, Revie*'3 321 HYDROCARBONS EXPLANATIONS


Choice B is correct. Can-rphor has a carbonyl group (water soluble) and a iarge alkyl ring system (not water
soluble). It is hard to decide based on the structure. It happens that the compound is watei soluble, which you
may know first hand from using camphor-containing cleaning agents for skin. The question is whether or not it
is highly water soluble. Because there is some ambiguity, let's say for now thai it 1ikely not highly water
soluble, and consider statement I to be invalid. Camphor is a liquid at room temperature, as you ti-tignt n^rr"
seen if you synthesized it in a lab experiment. Being a liquid at standard temperature, its boiling point"is above
298 K. Statement II is valid. Camphor has two chiral carbons, so it rotates plane-polarized lighi. This makes
statement iII invalid. Choice B is the best answer, but not with one hundred percent certainty.

79. Choice D is correct. It is stated in the passage that carvone has a strong UV absorbance (e > 10,000). Carvone
has conjugation, which causes its intense UV absorbance. On the other hand, limin has no conjugation, so its UV
absorbance is not as intense as that of carvone. This means that the UV absorbance for limin his an e less than
10,000 (therefore, log e < 1og 104 = 4). The best answer is choice D. This question required some background
information on UV spectroscopy. The minimum you should know is that n-bonds ire UV active, and with
conjugation, the intensity of the absorbance increases and the energy of the absorbance decreases.

80. Choice C is correct. A Diels-Alder reaction is a cyclization reaction that involves the addition of a diene to a
dienophile (alkene) to form a cyclohexene product. Both limin and carvone are cyclohexene compounds,
eliminating choices A and D, but carvone has a carbonyl group and isoprene contains only Cs and Hs. The best
answer is limin, choice C.

81. Choice B is correct. It is stated in the passage that corn oil is 63'/, iinoleic acid. Looking at table 1 shows that
linoleic acid is made up of eighteen carbons and has two n-bonds. Choices C and D are eliminated, because they
only have sixteen carbons. Choice A is eliminated, because it l-ras three n-bonds. The best answer is choice B.
You can try to match the exact location of each n-bond from the formula in Table 1 to the drawing in the answer
choices, but doing so is not time efficient.

82. Choice D is correct. Linoleic acid contains two n-bonds, both with cis geometry. When linoleic acid is treated
with FADH2, the result is hydrogenation of the diene and the formation of the aliphatic carboxylic acid (of
eighteen carbons) stearic acid. The gain of four hydrogen atoms increases the moleCular mass of tne acid (by
four), and the loss of unsaturation results in more molecular flexibility, which results in a higher melting poini.
Unsaturated fats, with less flexibility and therefore less ability to engage in intermoleculaiinteractioni, have
lower melting points than saturated fats of comparable mass. This is Co**orl organic chemistry knowledge
that you should have addressed when comparing vegetable and animal fats. The coirect answer is choice D.

83. Choice C is correct. Oleic acid is an eighteen-carbon acid with one n-bond between the eighth and ninth
carbons. The a-bond in oleic acid has cis orientation. Treating oleic acid with deuterium (D2) and a catalytic
metal like pailadium adds two deuterium atoms across the n-bond of the alkene molecule. The two deuterium
atoms add syn to one another at carbons eight and nine. The result is the formation of two new chiral centers.
There are no chiral centers to begin with, so the product has tr.tro chiral centers. The best answer is choice C.

84. Choice D is correct. Potassium permanganate reacts with alkenes to form diols by adding two hydroxyl groups
is a syn addition fashion to the carbons of the n-bond. Linolenic acid has three n-bonds, located between
carbons 9 and 10, carbons 12 and L3, and carbons 15 and 1,6 when the carboxylic acid carbon is considered to be
carbon one (IUPAC convention). Hydroxyl groups form at all sp2-hybridized carbon sites. This results in a
product with hydroxyl groups at carbons 9,70, 12,13, 15, and 16, as listed in choice D.

85^ Choice B is correct. The most bromine per moiecule is consumed by the fatty acid with the greatest number of n-
bonds present. For every n-bond present, one molecule of bromine tiquid will be consumed. Arachidonic acid has
four n-bonds. Arachidic acid has no n-bonds present in its structure, linoleic acid has two n-bonds present in its
structure, and linolenic acid has three n-bonds present in its structure. Arachidonic acid is the mosiunsaturated
of the choices. The correct answer is thus choice B.

86. Choice B is correct. Paimitoleic acid has sixteen carbons and one n-bond. When palmitoleic acid is fuliy
hydrogenated, it forms the aliphatic acid of sixteen carbons (listed in Table t as pilmltic acid). The best
answer choice is B.

Copyright O by The Berkeley Review@ 322 HYDROCAR.BONS EXPLANATIONS


87. Choice C is correct' Figure 1 shows occidentaiol
with three specified chiral centers, so choices A and B.ffi
::,il;lXl*"1'fli,*:lr":::,-".11",::lE_::ly:lt1:
(cHz) "itn".iul"
sroups' rhe methvr group and tertiirv arcohol carbon
,;a_i;;fi;ii;" or they are methyrene
o"
chiral centers on the structure. The best answer is choice
C. ""inl.i#J:T:.ffi:ffil":J""f:tJ:ii::
88. Choice D is correct' The first paragraph states that occidentalol
is a sesquiterpenoid. occidentalol has fifteen
carbons total' so it is a tuatotlibl" conclus.ion that sesquiterpenes
hurr" iift""r-r carbons. Choice D is the best
answer' Monoterpenes have ten carbons, diterpenes trave
twenty carbons, and triterpenes have thirty carbons.
89. choice A is correct' The maximum wavelength of absorbance,
network increases' All four choices are conjugaied dienes,
r*u*, increases as the conjugation of the rc-
but Compound 1 also has a carbonyl in the conjugated
networks' As such, the compound with the ioigest
L'u* is Compound 1, choice A.
90. Cl'roice B is correct' step 1 involves a Diels-Alder reaction
followed by decarboxylation. The intermediate
compound is the Diels-Alder product' There is. no nitrogen
present in either reactant, so the compound cannot
a lactam (cyclic amide)' Thii eliminates choice be
? Tie p.oar.t of a Diels-Alder reaction involving a diene
and an alkene (dienophile) is cyclohexene, so choice s
is the best answer. The diene in Compound 2 is
regenerated from cyclohexene after decarboxylation,
so choice A is eliminated. The reaction is shown
below:

150'C
____*>

o- Cyclohexene
Lactone (not lactam)
C02Me CO2Me
CO2Me Carbonyl is NOT conjugated to alkene
Compound 1 Compound 1.5 ( intermediate compound in Step 1)
The compound is a cyclohexene with a lactone that
is not conjugated to the alkene.

9"t_. Choice B is correct' occidentalol has one. more me-thyl


group than compound 6, so the role of methyl lithium,
MeLi' must be to add a methyl group to compouna o. int
eliminates choices C and D. Methyt lithium has an
anionic carbon' so it acts as a nucleophile rither than
an electrophile. This eliminates choice A and makes
choice B the best answer' The methyi anion attacks
the carbonyl carbon in the same fashion as an alkyl
magnesium bromide anion attacks a caibonyl in a Grignard
reaction.
92. Choice A is correct' A conjugated diene can be protonated
at either terminal carbon of the ru-network, because
the carbocation that results is resonance stabilized. This
eiiminates choices B and C. Carbon a is a secondary
carbon while carbon d jgrtiary carbon. It is easier to protonate the less hindered site, so carbon a is the site
1s l
that is most likely to gain H+. The best answer is choice A.

93' Choice D is correct' Leukotriene 44, LTA4, has four alkene


n-bonds, one carbonyi n-bond, and an epoxide ring.
choice,A is a valid sratement, which eliminates it. LrAa has three
Il':-:"::i:i::::i::^:l,T'"*'l.lion.
alkene n-bonds in conjugation, resulting in six n-electrons in a
conjugated system. cnrr.?i:r, Jtffnri"1"""."",,
which eliminates it' Because of the extensive conjugation, there
are several sites at which a" nucleophile and
electrophile may add' For instance, if the epoxide o*yg"n
were protonated, a nucleophile could attack the ring
or the left carbon of any n-bond to add u"ros the slstem,
Tilis means that r,2-addition, r,4-addition, 1,6-
addition, and 1,8-addition are all possible. Choice C is a
valid statement, which eliminates it. In all
likelihood, this answer choice uurrr"d the coveted "huh?", meaning
you can,t eliminate it, because you're just
not sure" on a multiple choice exam, this is not a problem.
You just need to look at choice D and use your testing
logic' LTAa has four alkene n-bonds and one carbonyt n-bond, so
tn"." ur" carbons. There
are twentv carbons total in LTA4, so eleven of them.are 'ri'r"-rp'r+yrrialzed
sp3-hybridlzed. There
than sp2-iybridized carbons, ,o .hoi." D is an invalid statement,
_.1",pl;;,[;;""urUo.*
which makes"r" it the best answer.

Copyright @ by The Berkeley Revierv@ .11i HYDROCARBONS EXPLANATIONS


94. Choice D is correct. in the presence of light, adds to the most substituted carbon of an alkane by way
,Bromine,
of a free radical mechanism. The most substituted carbon is tertiary, so the choice with bromine added to the
tertiary carbon (most substituted) is the best answer. This makes choice D the correct choice. T'he product
shown, as well as its enantiomer, are both formed.

95. Choice D is correct., Free radical propagation reactions keep the free radical reaction going, so to be a
propagation reaction, there must be the same number of free radicals on each side of the equation. In choice A,
there is one free radical on the reactant side and three free radicals on the product side, so it is not a
propagation step. Choice A is eliminated. In choice B, there are two free radicals on the reactant side and no
free radicals on the product side, so it is a termination step and not a propagation step. Choice B is eliminated.
In choice C, there is one free radical on the reactant side and two free radicals on the product side, so it is not a
propagation step. Choice C is eliminated. In choice D, there is one free radical on the reactant side and one
free radical on the product side, so it is a propagation step. Choice D is the best answer. You may not recognize
the reaction from the overall mechanism, but it converts a less stable free radical into a more stable free
radical, which ultimately impacts the product distribution.

Choice A is correct. Bergamontene contains fifteen carbons, so it is likely made from three S-carbon isoprene
units. We can't be sure without analyzing the structtire to find the isoprene fragments, but that is not time
efficient. Choice A is the best answer so far, and shall remain our choice until a better one comes.
Bergamontene is a hydrocarbon with no heteroatoms, so it is a terpene and not a terpenoid. This eliminates
choice B. Bergamontene has two n-bonds and a cyclohexane ring, so at first look choice C is tempting. But the
molecule is bicyclic, meaning it has a second ring, the four-membered ring connected to the cycioheiane ring.
Bergamontene has four units of unsaturation, not three. This eliminates choice C. To verify this, bergamontene
has 24 hydrogens and therefore a formula of C15H24. The units of unsaturation are
1z1iS; + Z -24\/2 = 4, so
choice C is eliminated. There are three chiral centers on bergamontene, so 8 (23) is the maximum number of
stereoisomers, not 16. This elimir-rates choice D and secures choice A as the best answer.

97 Choice D is correct. Both the Claisen rearrangement and the Cope rearrangement require dienes, but they need
not be conjugated. The two n-bonds must be separated by three sigma bonds, so choices A and C are eliminated.
Clemmensen reduction converts a carbonyl into an alkane, so no diene of any kind is required. Choice B is
eliminated. A Diels-Alder reaction involves the cyclization of a conjugated diene and a dienophile, so it must
l-rave a conjugated diene. This makes choice D the best answer.

Choice C is correct. For an E2 reaction, the base must be strong enough to remove a proton from carbon and bulky
enough to not undergo substitution. This makes choice A a valid statement and thereby eliminates it. For an E2
reaction, the leaving group and proton being lost from carbon must be positior-red anti to one another, so the
geometry of the product is dependent upon the alignment of the reactant. Cis versus trans results from the
orientation and stereochemistry, so choice B is a valid statement and thereby eliminated. For an E1 reaction, a
leavilg group first leaves, resulting in a carbocation. With carbocations, rearrangement can be observed, so it is
with E1 reactions that we see rearrangement, not E2 reactions. Because E2 reactions are concerted, there is no
rearrangement, so choice C is an invalid statement and thereby the correct answer. Heat is required to drive
both El and E2 reactions, so choice D is a valid statement. It is eliminated, leaving choice C as our choice.

99. Choice D is correct. Terpenes are hydrocarbons of 70,15,20, etc... carbons, so they are somewhat massive lipids.
Because they are hydrocarbons, they are lipid soluble, so choice A is a valid statement. Choice A is
consequently eliminated. Terpenes have molecular masses of about 140 g/mole, about 210 g/mole, about 280
g/moie, etc..., so they have somewhat high boiling points. High is a relative term, so we ian't be certain in
eliminating choice B. However, choices B and C are the same concept, so they mutually exclude one another
from consideration. It is important that you use all of your test taking stcllls. The specific rotation of a
compor-rnd is dictatecl by its chiral centers, which a terpene may or may not have. Given that there is no
general rule about the chirality of terpenes, we cannot conclude that they have high specific rotations" Choice
D is the best answer.

100. Choice C is correct. To have a dipoie nol eqr-ral to zero is to have a dipole. To have a dipole is to be polar. Cis
compouuds are always polar so Compound III is polar. Ethvlene is perfectly symmetric, io choice Compound II
is nonpolar. The question comes down to: "Is Compound I polar?" Compound I is not polar, because thl methyl
groups on t1-ie alkene cancel one another and sum to a resultant vector of 0. Choose C for best results.

Copyright @ by The Berkeley Review@ )1+ HYDROCARI}ONS EXPLANATIONS

Vous aimerez peut-être aussi